You are on page 1of 522

',";!

:;~

~~f'
.

.~

.,-: . i

NEW SENIOR
MATHEMATICS

Two Unit Course for Years 11 &12


J. B. Fitzpatrick

,,~

"-'

;!*:~~ '.

!;. .

,f

, !.
!

Rigby Heinemann
A division of Reed International Books Australia Pty Ltd
22 Salmon Street, Port Melbourne, Victoria 3207
World Wide Web http://www.reedbooks.com.au
Email heinemann@reedbooks.com.au
Offices in Sydney, Brisbane, Adelaide and Perth. Associated companies,
branches and representatives throughout the world.
Bernard Ftizpatrick
First published 1984

2000 1999
20 19 18

1998 1997
17 16 15

1996
14 13

12

All rights reserved. No part of this publication may be reproduced, stored


in a retrieval system or transmitted in any form or by any means whatsoever
without the prior permission of the copyright owner. Apply in writing to
the publisher.
Technical drawings by James Mackie (technimac)
Cover design by Judith Groenendijk
Typeset in Hong Kong by Setrite Typesetters
Printed by Chong Moh Offset Printing. Singapore
National Library of Australia
cataloguing-in-publication data:
Fitzpatrick, J.B. (John Bernard).
New senior mathematics: two unit course for

years 11 and 12.

For senior secondary school students.

Includes index.

ISBN 0 85859 352 1.

1. Mathematics. I. Title.
510

The author has taught senior mathematics for many years at


Xavier College, Melbourne, and has written a number of
mathematics text books for the years 11 and 12 syllabuses in
Victoria.

CONTENTS

2
3
<4

5
6
7
8
9,
10
11
12
13
14
15
16
17
18
19

Introduction
Basic Arithmetic
Basic Algebra
Equations and Inequations
Plane Geometry
Probability
Relations and Functions
Trigonometric Ratios
Revision Exercises A
Applied Trigonometry
Coordinate Geometry - Straight Line
Locus and Subsets of the Plane
Sequences and Series
Quadratic Functions
Indices and Logarithms
Revision Exercises B
Differential Calculus
Geometrical Applications of Differentiation
Integral Calculus
Trigonometric (Circular) Functions
Exponential and Logarithmic Functions
Applications of Calculus to the Physical World
Revision Exercises C
Test Papers
Summary
Answers
Index

vii

34

54

85

121

152

162

175

179

198

222

233

254

280

299

303

327

356

379

406

426

446

452

462

471

517

INTRODUCTION

.1

New Senior Mathematics, Two Unit Course/or Years 11 and 12 has


been written to meet the requirements of the syllabus for the two
unit course for Years 11 and 12 as approved by the New South
Wales Board of Senior School Studies. It covers all aspects of the
prescribed syllabus for the two year course.
It is assumed that students taking this course have completed the
School Certificate mathematics course and have shown a degree of
competence in the necessary skills. This two-unit course is intended
to give students an understanding of those aspects of elementary
mathematics which are applicable to real life situations.
Mathematics, like many other things, is best learnt by doing. A
student begins to appreciate the power of mathematics when he or
she has achieved a mastery of basic techniques, not after reading
lengthy explanations. To perform well, a student must endeavour
to understand mathematical concepts and must also acquire certain
manipulative skills in operations. The understanding of concepts
helps a student to master these skills, just as mastering operation
skills can help the student to understand concepts. Proper
motivation is necessary, however, if the acquisition of these skills is
not to be a boring and frustrating process.
The emphasis throughout the book is on the understanding of
mathematical concepts: to this end, worked examples have been
included to assist students to comprehend the subject matter.
However, understanding cannot be complete without competence
in certain skills. This competence must be acquired through
practice; therefore the text contains many graded exercises-more
than sufficient for a complete course. "Average" students will gain
competence from the early and middle examples in the sets of
exercises, while enthusiastic students may wish to proceed to the
more demanding problems near the end of the sets. To assist the
appreciation of the course as a whole, comprehensive sets of
revision exercises are included at the end of Chapters 7, 13 and 19
as well as Test Papers at the end of Chapter 19.
The first three chapters are devoted to basic arithmetic and
algebra. It is hoped that a large number of students will already

vii

have a sound knowledge of the work in these chapters so that much


of the contents will be familiar and considered only as revision. If
students hope to be successful at Years 11 and 12, it is essential to
have a firm foundation in algebra. Good algebra is central to
success in mathematics. Thus, in these chapters much time is
devoted to the basic skills of indices, factorization, equation
solving, substitution in algebraic expressions, etc.
The use of electronic calculators has been recommended in
sections where tables and 10garithmS"-weie---Tormerly used. The
calculator has superseded the use of logarithms as an aid to
computation. Emphasis has been placed on the importance of
checking the feasibility of a calculator result by using
approximations and reverse operations.
The author wishes to express his thanks to Mr John Sattler of
St Ives High School for reading the manuscript and offering many
helpful suggestions.
J .B. FITZPATRICK

viii

CHAPTER 1

Basic
Arithmetic
1.1 Fractions, decimals, percentages _ _ _ __
John ate

~ of the pizza.

The cost of living increased by 1O~o last year.

1 hectare = 2'5 acres (approximately).

The agent receives a 5070 commission.

We often read and speak sentences like these. Common fractions, decimals and
percentages are very much part of our lives. In fact, if we are to survive in a world of
buying and selling, it is essential that we be able to operate freely with them and also be
able to convert one to the other, e.g.
1
4
-83
19

025

25070

= 0'375 = 37'5070
= 076 = 76070
To convert a common fractio~ to a decimal, we divide the denominator
into the numerator.

EXAMPLE 1

(')
1 ~")
3 ("')
19 to d
' Is.
C onvert t h e common f ractions
I 4'
~11 8'
III 25
eClma

0) 4

11 '()20

(ii) 8

0-25
1

0-25

13 '06()40

(iii) 25 119'0150

0-375
3

-8 = 0375

076
19 = 0'76

25

2 NEW SENIOR MATHS: TWO UNIT COURSE

To convert decimals to common fractions, put 1 on the denominator for


the decimal point and 0 for each digit to the right of the decimal point.
Then express the fraction in simplest form.

EXAMPLE 2
Convert (j) 0-25, (ij) 0'375, (iii) 076 to common fractions.

25
100
375
(ij) 0375
1000
76
(iii) 0-76 =
100

1
4
15
40

5
20
75
200
19
25

(i) 0-25

3
8

To convert a common fraction to a percentage, multiply the fraction by


100.

EXAMPLE 3
1 (II.. ) 8'
3 ("')
19
t
E xpress (I.) 4'
III 25 as percen ages,

(i)!4 =!4
(ii)

i =i

x 100070

25070

= 37!OJo

x 100070

19 x 100m
= 76 mVIIo
25
VIO

..') 25
19
(III

Actually, the word 'percentage' means per 100. Thus:

lout of 4 is equivalent to 25 out of 100.

3 out of 8 is equivalent to 37! out of 100.

19 out of 25 is equivalent to 76 out of 100.

To convert a percentage to a common fraction, divide by 100.

EXAMPLE 4
Express (0 25%, (ii) 37!OJo, (iii) 76070, (iv) 3iOJo as common fractions.
(i) 25070 =

(II) 371m
:pO

1~
37!

= 100 = 2
76

(iii) 76070 = 100


(iv) 3kOJo

3i
100

1
4

10
75

x 100

19

= 25

3i X 8
100 X 8

25
800

5
1
160 = 32

I/BASIC ARITHMETIC 3

Notice the connection between decimals and percentages:


25070 = 025
37'5070 = 0375
76070 = 076
Note: If the denominator of a common fraction is divisible by 3, 7 or 11, it is advisable
not to try to convert it to a decimal, e.g.

j = 3 12

.()2()2() _..

0'666 _._
0'666 _.

This is called a recurring decimal or a repeating decimal. However, as a percentage,


2

3" = 3" x 100070

200
3070 = 66jOJo

EXERCISES 1(a)
1. Express each of the following fractions as (i) a decimal, (ii) a percentage.
3
5
5
(e)
16
7
(i)
40

(b)

(a) -

18
(c) 25

Z
8

13
20
29
(1)
50

1
(d) 40

(f)

(g) 16

9
(h) 100

(k) 18

(I)

17
4

2. Express each of the following decimals as (i) a common fraction, (ii) a percentage.
(a) 0-25
(e) 1-75
(i) 0-875

(b) 185
(f) 0-325
(1) 0-6125

3. Express each of the


(a) 40070
(e) 15070
(i) 125070

(c) 0375
(g) 0-84

(k) 2-65

(d) 0-23
(h) 0025
(I) 1-82

following percentages as (i) a decimal, (ii) a common fraction.


(b) 1070
(c) 10070
(d) 32-5070
(f) 65070
(g) 7iOJo
(h) 82iOJo
(1) 116070
(k) 14~OJo
(I) 112!OJo

4. Complete the following table.


Fraction

Decimal

Percentage

Fraction

Decimal

Percentage

3
10

0'35
17
8

13-5070
32070

0-45
19
40

0'7
7
25

8!OJo
11

12!OJo
0-08

14
35

4 NEW SENIOR MATHS: TWO UNIT COURSE

1.2 Repeating decimals _ _ _ _ _ _ _ _ __


All common fractions ~ where a and b are whole numbers are called rational numbers and
can be expressed as decimals by dividing the numerator by the denominator, e.g.

~ = 075

!8

1
3

6=

0125

03333.. .

016666 ...

O'c06
0-4545 ...

All of these decimals either terminate, e.g. 0~75, 0'125, 0'06, or have an infinite number
of digits to the right of the decimal point and one or more of these digits are,repeated
without ending, e.g.
0-3333 ... (the 3 is repeated)
016666 ... (the 6 is repeated)
04545 ... (the 45 is repeated)
These decimals are called repeating decimals and the repeating digits are written with a
dot on top, e.g .
. 0'3333 _. _ = 03
016666 ... = 016
04545 ... = 0'4S
Note: All numbers whose decimal form does not terminate and in which there is no
repeating pattern in the digits are irrational numbers_Repeating decimals are rational
numbers .

.J2 141421...
3-141592653589793. _.
.J2 and can be expressed as non-terminating decimals but there is no repeating pattern in
1('

1('

the digits. They are irrational numbers_

EXAMPLE 5
Express (i)

o-i, (ii) 0-46, (iii) 0-S3,

(iv)

= 0-2222 . - .
Multiply both sides of (1) by 10.
10 x oi = 22222 ...
Subtract (1) from (2)_
9 x o-i
2

(i) 0-2

0-2

as common fractions.

(1)

(iii) 0- S3

= o 5353

...
Multiply both sides of (1) by 100.
100 x 0'S3 = 53-5353 . __
Subtract (1) from (2)_
99 x 0-S3 = 53

(2)

= 04666 . - .
Multiply both sides of (1) by 10_
lOx 0-46 = 4-6666 __ _
Subtract (1) from (2).
42
9 x 046
42
7
0.46 = 4.2
15
90

(ii) 0'46

4-2S3

0-S3
(1)

(iv)

(2)

(1)

(2)

53
99

0-2S3 =

0-25353 __ _
Multiply both sides of (1) by 100.
1'00 x 0'2S3
2535353 .. _
Subtract (1) from (2).
99 x 0'2S3 = 25-1
0-2S3 = 25'1 251
990
:_ 4-2S3 4l.li
990

Repeating (recurring) decimals will be considered again in Chapter 11_

(1)

(2)

I/BASIC ARITHMETIC 5

EXERCISES 1(b)
1. Which of the following fractions can be expressed as repeating decimals?

1572743294

2' 9' 16' 3'


TI' 10' 13' 25' 15

2. Express each of the following fractions as a .repeating decimal.

274259511

3' 9' TI' 15' 6'

12'

3. Express each of the following repeating decimals as a common fraction.


0'5, 0'1'7, 0'68, 0'8, 0'2i, 05 i, 0'583, 1'37, 3'58, 2'136
4. Show by division that

~=

'142857142857 ... = 0i42857.Write

down~,~,~,

?, ~ as repeating decimals. Do you observe any pattern in the digits that repeat?
5. Show by division that /3

= 0076923 and write down 2 13' 1~' ... ~i as repeating

decimals. Do you observe any pattern in the digits that repeat?


6. Express

1 2

17' 17' 17' ...

16

as repeating decimals. What do you observe?

7. To convert a repeating decimal to a fraction, you (i) subtract the non-repeating digits
from the number, (ii) then put this over 9 for each repeating digit and 0 for each nonrepeating digit, e.g.
.
0'37 = 37

3
34
17

90
90

Is this procedure correct?

1.3 Lowest common multiple _ _ _ _ _ _ __


36 is said to be a multiple of 6, because 6 divides evenly into 36 without leaving a remainder.
20 is said to be a multiple of 5, because 5 divides evenly into 20 without leaving a remainder.
36 is a common multiple of 6 and 9, because both 6 and 9 divide evenly into 36.
20 is a common multiple of 5 and 2, because both 5 and 2 divide evenly into 20.

jl

18 is the lowest common multiple of 6 and 9, because 18 is the lowest multiple of both 6 and 9.
lOis the lowest common multiple of 5 and 2, because lOis the lowest multiple of both 5 and 2.

The L. C.M. (lowest common multiple) oj two or more numbers is the


smallest number into which each oj the given numbers will divide
without leaving a remainder.

EXAMPLE 6

Find the L.C.M. of 12 and 16.


Multiples of 12 are 12, 24, 36, 48, 60,

Multiples of 16 are 16, 32, 48, 64, ...

:. L~C.M. is 48.

6 NEW SENIOR MATHS: TWO UNIT COURSE

EXAMPLE 7
Find the L.C.M. of 6, 8 and 12.
Multiples of 6 are 6, 12, 18, 24, 30, 36, 42, 48, ...

Multiples of 8 are 8, 16, 24, 32, 40, 48, ...

Multiples of 12 are 12, 24, 36, 48, ...

:. L.C.M. is 24.

Note that 48 is a common multiple but is "not the lowest common multiple.

EXERCISES 1(c)
Write down the L.C.M. of:
1. 2, 4, 6
2. 15, 20
5. 8, 12, 24
6. 3, 5, .10
9. 12, 14, 21
10. 2, 3, 4, 5
13. 5, 10, 15, 20 14. 5, 12, 30
17. 7, 14,21,28 18. 3,6, 14,42
22. 3,7, 14, 21
21. 6,8, 10, 12

3.
7.
11.
15.
19.
23.

14, 21
5, 15, 20
6, 8, 14
13, 26, 39
9, 11, 33
3, 4, 8, 12

4.
8.
12.
16.
20.
24.

5, 6, 12
11,22, 33
3, 5, 7
6, 10, 14, 15
4, 12, 20, 24
3, 5, 12, 15

1.4 Equivalent fractions _ _ _ _ _ _ _ _ _ __


Two or more fractions that have the same value are equivalent fractions, e.g.

1
2
3
4
5

10 ...

2
4
6
8

If we multiply the numerator and the denominator of a fraction by the


same number, the value of the fraction is not changed.
1

2x 2=4

133
3"
6
3
4
12
5 x 4 20

2
3

5=

2x

EXAMPLE 8
3 4 18.
. d
.
A rrange t h e f ractIons
4'
5' 25 III or d er 0 f magmtu
e.
It is necessary to express each fraction with the same denominator. This can be done by
getting the L.C.M. of the denominators 4, 5 and 25.
L.C.M. of 4, 5 and 25 is 100.
3
3
25
75
4 = 4 x 25
100
4
4
20
80
5 = 5 x 20 = 100
18
18
4
72
x 4 = 100
. d . 18 3 4
T he or d er 0 f "magmtu e IS 25' 4' 5'

I/BASIC ARITHMETIC 7

EXERCISES ltd)
Complete 1. to 5.

3
15
21

1. 4 = 12 = - = 40 =
2 ~ - 10 _
8
~

3 .
7

25

42

35
8

54

77

120

24
4. 60
10
5

=2
84

5 .
12
18
9
90

6. Reduce each of the following to its simplest form.

(.. ) 27 (... ) 21 (.) 120 () 52 (.) 72 ( .. ) 36


(I.) 32
80 II Sf III 35 IV 165 V 72 VI 84 VII 54

( ... ) 33

VIII

ill

In 7. to 15. arrange the fractions in order, beginning with the smallest.


7 13 17
7. 8' 16' 20
7 5 13 11
10. 9' 6' 18' 12
1 2 5 5
13. 6' IT' 33' 22

7 13 3

8.

9' 18' 4

13 17 9
10
2 7 4 19
14. 5' 15' 9' 45

11.

15' 20'

5 3 9

14

1 17 7 31

12. 2' 32' 16' 64

8 3 I 33 2

15.
10' 100' 5

9.

7' 4'

1.5 Addition and subtraction of fractions _ __


To add or subtract fractions:

1. Find the L.C.M. of the denominators.


2. Express each fraction with this L. C.M. as denominator.
3. Add or subtract the numerators.

EXAMPLE 9

2
4
'I
'
E xpress 5
- 7+
9 as a smg
e f
ractlon.
L.C.M. o f 5, 15, 9 is 45

274

5-15+9

18

21

= 45 - 45

18

21
45

+
+

20

45

20

17

45

A mixed number, e.g. 3i, is a number consisting of an integer and a fraction. In adding
or subtracting mixed numbers, the integers and fractions can be considered separately.

8 NEW SENIOR MATHS: TWO UNIT COURSE

EXAMPLE 10
Express

5~

3! -

~ as a single fraction.

L.C.M. o f 3, 6, 5 is 30
5-3\

~5

3! 6

5-3+!-!-~

365
10
5
24

2 + 30 - 30 - 30

19

30

Ij~

EXERCISES 1(e)
Express each of the following as a single fraction.

1.

4. 1~

7.

4 + 8"
+

2.

2~

111

2+3- 4

5. 5~ + 2i -

533

8 + 16 + 4

8.

- 2i - ~
1
13 !1 _ 7
10. 3k

. 16

3.

31

6.

17.

8~ + 1~

12.

3~

15.

15

It

5\25

- 2370

751

9.

14 . 11 _ 6~ _ ~
9

7
16. 10 + 6j - 2!

8- 4+ 6

273
30 + 20

15 -

11. 3 \70

731

8" -

- 2 \52

12 -

478
10 + 45

18.

1.6 Multiplication and division of fractions _ __


EXAMPLE 11
(a)

-3 x -5

4
X

(c)

(b)

6
5

x
4 J1;.

= -

-5 x -2 of 9
8
3

$ 'l
$I 3
- X - x
84 P #5 5
3

5
8

= 20

EXAMPLE 12
(a)

5 . 3

8~ 4
-5 x 4'-

,82

5
6

(b)

(c)

125
24
514

25

xT

I/BASIC ARITHMETIC 9

Note the use of 'of' in example l1(e) and the difference in use of 'x' and 'of' in
examples 12(b) and 12(e).
Mixed numbers are converted to vulgar fractions before performing the operations of
multiplication and division.

EXAMPLE 13
1~ x 2~

(a)

.82

5" x

2~ + 4! x 2A

(b)

11

14
9
9
=5+2 x 4

Jf

147

22
5

= 5

JJ'

x9"x~

7
5

=~
=

EXERCISES 1(f)
Simplify the following.

14

1.

4 x "7 x 15

4.

9 x IT

15
+ 22

7. 1~ + 1~
10. ;2 +
13.

2~

16.

~ of

3" x 8 of 5"

5.

7~

1~

3 11

i x 51

2 X

x 16

8. 5i + 2!

~ of 2A

+ 2! x

2.

5 f 14

"7 0 15
7

6. 6 x 16 x

6 X 11. 2 3
9'IT
9-:
IT

3.-: 153 X 1IT


1
11 . 4

5
12. 2! + 12 x

n1 13
5
14. I V2 -: 4: X 12

15.

2x 2+ 4

1
7 f 3
17. 79 x 16 0 5"

18.

2l

8l

x 1~ + Ii

1.7 Order of operations - - - - - - - - - -


The order in which operations in arithmetic are performed is
(1) Brackets,

(2) OJ,

(3) Division and Multiplication, or Multiplication and Division, going


left to right.
(4) Addition and Subtraction, or Subtraction and Addition, going left
'to right.

EXAMPLE 14
(a)

4 of 12 +

6 x (8 - 4)

= 4 of 12 + 6
=9+6x4
9
= 33

+ 24

x 4

brackets first
then 'or
then x
then +

10 NEW SENIOR MATHS: TWO UNIT COURSE

(b)

3
1
4

10
4 of 5"

3
1

'of' fl'rst
10 - 5"

3
1

10 - 5"
then +

3+ 6+
2
S
= 3 -;"". 6 +
2

=3
=

431

5" + 10 - 5"

832

10 + 10 - 10

. i

6
x 5"

then

and

10

(c) 1

~ ~5 ~

This really means numerator


denominator and then divide.
2

6
IS

= 5" + 3

Numerator

Denominator = 1 -

10

+ 15 =

4
1 - IS

5" x 3"

Numerator
.. Denominator

denominator. Hence we simplify numerator and

16

IS

16

IT = IT =

16

IS'
11

IS'
5

lIT'

EXERCISES 1(g)
Simplify:

+ 21 of 10 x
S x (2 + 7) x

1. 6

(8

3.

S - 12 + 6
2
1

3
5. 4

+ 2 of 8 x 63 - 5"

2
3
3
13

7. 7 of 8 + 4 of 14

~ of ~
7~ - S!

11

. st - 4
9. 2j -

8.

3l 1

10. (2] -

(j + ~)
2
7)

97

14.

----=--;;

12

5"

18.

-;-;-f--h

13 + 5"

16.

"4 -

2. S X 2 + 7 X S - 12 + 6

4. 8 + 6 X (2 + 4) + 12

113
1
S

6. 2 + 4 + 8 - 5" of 8

13. 1 ~ ~ ~ i
3
12
4
S
15. 5" x 13 - 5" x 13
17

4)

+f

24
X 274

1.8 Addition and subtraction of decimals _ __


In addition and subtraction oj decimals, arrange the numbers with the
decimal points vertically below each other and then proceed as in
addition and subtraction oj whole numbers.

I/BASIC ARITHMETIC 11

EXAMPLE 15
Add 16'24, 8'371, 0'002.
16-24
8371
0002
24613

EXAMPLE 16
(a) Subtract 2725 from 8734.
87,34
-27'25
60,09
(b) Subtract 4'325 from 18'7.
Express both numbers to 3 decimal places by putting two zeros at the.end of 18'7.
187 has the same value as 18'700.
18'700
- 4325
14,375

1.9 Multiplication and division of decimals - -

To multiply two decimal fractions, proceed as in ordinary multiplication


of whole numbers and the number of decimal places in the product will
be equal to the sum of the decimal places of each decimal fraction .

"

EXAMPLE 17
(c) 465 x 100 = 465
46
4-65 x 10 = 46'5
x32
4-65 x 0-1 = 0465
92
4'65 x 001 = 00465
138
1472
Note in (c) that multiplication by a power of 10 involves simply moving the decimal
point to the right or the left.

(a)

57
x 8
45.6

(b)

For division of decimal fractions, multiply numerator and denominator


by the power of 10 that will make the denominator a whole number.

EXAMPLE 18
28 x 10
280
14
1-4 x 10
1862 x 100
(b) 18-62
002
002 x 100
28

(a)N

20
1862
2

931

12 NEW SENIOR MATHS: TWO UNIT COURSE

(c) 10-5 x 1-65


03

55

l(Y5 X

.l.H

X
I

(Multiplying numerator and denominator by 10)

0-3 x 5-5
1-65
Note: Only simple cases of multiplication and division will be considered here_ Thus
there will be no need for long multiplication or division and no need for calculators_
=

EXERCISES 1(h)
(Do not use a calculator)
1. Add
(i) 4-6, 5-23, 0-775
(ii) 3-88, 38-8, 388
(v) 0-042, 1-35,62-1
(iv) 6-57,7-42,19-1
(vii) 0-8, 0-05, 6-8, 2-54(viii) 3-65, 0-365, 0-0365

2. Evaluate
(i) 8 -3-2
(iv) 8-156 - 2-369
(vii) 0-7 - 0-56
3. Evaluate
(i) 6-25 x 4
(iv) 3-002 x 7

(ii) 5-62 - 2

(v) 0-882
0-033
(viii) 4-2
0-61

(ii) 8-76

(v) 301

x 03
x 0-1

(iii) 0-01, 0-001, I-I

(vi) 5-22, 0-06, 0-006

(iii) 17'5 - 4-86

(vi) 095 - 0006

x 100
(vi) 17-5 x 0-01

(iii) 816

4. Evaluate
(i) 8-7 x 0-003
(iv) 2-84 x 1000

(ii) 6

(v) III

(vi) 4'7 x 0-09

5. Evaluate
0) 17-6 + 0-2
(iv) 16 + 0'4
( .. ) 36

(ii) 84 + 6
(v) 7-7 + I-I

(iii) 8 + 0'5

VII

t:2

6. Evaluate
(i) 8-264 + 0-002
(iv) 5-46 + 1-2
._) 0005
(VII
7. Simplify
(i) 1'5 x 3-75
0-5 x 2-5
12-6 x 1-5 x 70
'IV) ~
(
006 x 45 x 4-9
.. ) 14-4 x 500 x 56
(
VII

1-8

2 -5 X 7

x 0-011
x 0,005

(iii) 125

x 0,02

(vi) 18 + 0-09

... ) 14-4
(Vlll
0'04

(ii) 0-08 + 04
(v) 81 + 0-9

(iii) 6 + 0-01

(vi) 346-2 + 1000

"') 9'15
(VIII

64 x 72
008 x 12
48 x 72
(v) 1-8 x 0-1 x 02
( "') 6-4 x 0-77 x 20
Vlll
4 x 0-8 x 88
(ii)

(iii) 9-6

x 8-1
x

132 x 5-5 x 108


(vi) 1.5 x 36 x 1'21

1/BASIC ARITHMETIC 13

1.10 Scientific notation (Standard form) _ _ __

.,

Scientific notation (standard form) is a very neat way of expressing large and small
numbers_
A number in scientific notation is expressed as a product of
(iJ a number between 1 and 10, and
(iO some power of 10.

EXAMPLE 19

(i) 540000

540000-

5-4

105

t 5'''1

+--

(ii) 426732

decimal places

4267-32
= 4-26732 x 103

t+3"'1
(iii) 00000068

decimal places
0-0000068 = 6-8

(iv) 0'007562

decimal places

0-007562 = 7-562 x 10-3

10-6

1+----6----1

/+J.. t
(v) 8 X 105 X 3 x 10-2

.) 1-5 x 1()6
(VI

decimal places
8 x 3 x 105 X 10-2

= 24 X 103

= 2-4 X 104

1-5
1()6

T x 10-4

= 03 X 10 10

109

EXERCISES 1(i)
1. Express in scientific notation:
(i) 3000
(ii) 468 -2
(iv) 12 million
(v) 752000
(vii) 380000
(viii) 42 x 105

2. Express in standard form:


(i) 00001
(ii) 0-0041
(iv) 0005002
(v) 0-00000022
(vii) 865 x 10-7
(viii) 0-62 x 10-4

(iii) 18 thousand
(vi) 380030

(ix) 862 x 104

(iii) 0-00000725

(vi) 27 thousandths
(ix) 0-0005

3. Express in scientific notation:


(i) the product of 3-2 x 1()6 and 4 x 10-2

(ii) the product of 4-82 x 103 and 5 x 105

(iii) the value of xY'where x = 3 million and y = 4 million


(iv) 126 x 108 divided by 7 x 102

(v) 7-5 X 10-6 divided by 15000

(vi) xyn where x = 5'5 x 103, y = 4-0 X 10-2 , n


(vii) 7217 divided by 700000

(viii) the value of ab where a = 1'4 X 10-3 and b = 8 X 10-5

14 NEW SENIOR MATHS: TWO UNIT COURSE

4. Express in standard form:


(i) the number of cm in 40 km
(ii) the number of grams in 3 tonnes
(iii) the distance from the Sun to Earth if light from the Sun, travelling at 300000
km/s, takes 8 minutes to reach Earth
(iv) the number of man-hours worked in a 40-hour week by 2 million workers
(v) the number of cm 2 in 5 hectares, given that 1 hectare = 104m 2
5. The mass of a molecule of hydrogen gas is about 334 x 10-24 gram. What is the mass,
in standard form, of an oxygen molecule, which is 16 times that of a hydrogen
molecule?
6. A light year is the distance light travels in a year. The speed of light is 3 x 105 km/s.
Taking 365 days in a year, find the distance from the Earth to the star Proxima
Centauri, 4'25 light years distant.
7. The Earth is 93 x 107 miles from the Sun. Express this distance in metres given that
1 mile
16 x 103 metres.
8. How long would it take for a rocket travelling at a constant speed of 1'5 x I04 km/h
to reach the Moon if its distance from Earth is 384 x 105 km?
9. Assuming that the planet Venus moves in a circular orbit of radius 112 million km
around the Sun and that it completes one revolution around the Sun in 200 days,
calculate the speed of Venus. Express your answer in scientific notation in km/h and
use 11" = 34.
10. An oil company had annual sales of 422 million kilolitres of its products. Express

this in litres in scientific notation.

1.11 Significant figures and decimal places _ __


Suppose that the exact number of people attending a concert was 2874. This number may
be rounded off as
2870 to the nearest 10, i.e. 287 x 103
2900 to the nearest 100, i.e. 2'9 x 103
3000 to the nearest 1000, i.e. 3 x 103
If asked how many people were at the concert we may say that there were about 3000.
We are using only one figure, namely 3, to signify the number present. If we say that
about 2900 were present, we are using two figures, namely 2 and 9, to signify the number
present. If we say that about 2870 were present, we are using three figures, namely 2, 8
and 7, to signify the number present.
The number of figures used is called the number of sign ificant jigures. This number can
be best determined by expressing the number in scientific notation first.
What is the difference in meaning between 28 and 28'00? Obviously in 2800 we are
interested not only in the 2 and the 8 but also in the first two decimal places. Hence 2800
has four significant figures, whereas 28 has only two. It is interesting to note that 6'02 has
three significant figures, whereas 002 has only one. Why?

\
I/BASIC ARITHMETIC 15

Number

Scientific
notation

Number of
significant figures

3000
2900
2870
2874
6-02
0-02
36-0052
00052
28
28-00

3 x 103
2'9 x 103
287 x 103
2874 x 103
6-02 x loo
2 x 10~2
3 60052 x 10 1
5'2 x 10-3
2-8 x 10 1
2'800 x 10 1

1
2
3
4
3
1
6
2
2

EXAMPLE 20
Express (i) 58-377, (ii) 4-061 to (a) two significant figures (S_F.), (b) two decimal places
(D.P.).
(i) (a)

58317 = 58'377
5838
= 584
58

(ii) (a) 4061


=
=

to
to
to
to

5 S.F.
4 S.F.
3 S.F.

2 S.F.

4061 to 4 S.F.
406 to 3 S.F.
4'1 to 2 S.F.

(b) 58377

=
(b) 4061
=

58377 to 3 D.P.

58'38 to 2 D.P.

4061 to 3 D.P.
406 to 2 D.P.

Significant figures are the number of figures (digits) about which we can be reasonably
sure. The greater the accuracy of our measuring device, the larger the number of
significant figures we can use.
A stopwatch that can measure accurately to 01 s might record the winning time for a
1oo-metre race as 108 s. We can be sure of only three figures or one decimal place in this
result. We could not write 108 as 10,80. Why?
Another timing device that can measure accurately to 0-01 s might record the time as
1082 s. We can be sure of four figures or two decimal places in this case.
The operations of addition, subtraction, multiplication and division based on measure
ments should not have answers with an accuracy greater than that of the least accurate
measurement involved.

EXAMPLE 21
Add 67 sand 3,58 s.
6'7 s
3'58s
10'28 s (incorrect)

6'7
36
103 s (correct)

If we pay no attention to significant figures, we would write the result as 1028 s. In this
case we are treating 67 as 670 but we do not know the value of 6'7 to two decimal places.
We cannot add a known quantity to an unknown quantity. Therefore, expressing 3'58 to
one decimal place, we add 3'6 to 67 to get 10-3.

i'

16 NEW SENIOR MATHS: TWO UNIT COURSE

EXERCISES 1(j)
1. How many significant figures are there in each of the following?
(i) 426
(ii) 000427
(iii) 9003
(iv) 00508
(viii) 1905
(v) 8
(vi) 8000
(vii) 271 x 1()4
2. To how many significant figures are ea.ch of the following stated?
(i) The distance from Melbourne to Sydney is 900 km.
(ii) The time for a 100 m race was 1008 s.
(iii) The length of a line was measured as 15 00 cm.
(iv) 1 hectare =:: 247 acres.
3. Round off the following to the number of significant figures indicated in brackets.
(i) 9726 (3)
(ii) 86493 (2)
(iii) 18077 (4)
(v) 50071 (1)
(vi) 3068 x 103 (2)
(iv) 00071 (1)
4. A rally was attended by 108562 people. Round off this figure to the nearest (i) 100,
(ii) 1000, (iii) 10000.
5. Round off the following to the number of decimal places indicated in brackets.
(i) 438 (1)
(ii) 432 (1)
(iii) 19066 (2)
(iv) 4004 (2)
(v) 8597 (2)
(vi) 06666 (3)
6. Round off each of the following to three significant figures.
(i) 0006042
(ii) 75378
(iii) 06095

(iv) 52380000

7. Write down the value of (i) 789, (ii) 0496, (iii) 157 to one significant figure. From
this, find approximately the value of

789 x 0496

(a) 789 x 0496 x 157, (b)


15.7.' (c) (789 + 157) x 0496.
8. Express each number to one significant figure and so write down the approximate
value of
(i) 3902 x 5 9
... ) 697 x 009
(ii) 3142 X 2.87 2
(
402 x 551
III 0.85 x 7001

1.12 Approximations _ _ _ _ _ _ _ _ _ _ __
Before performing any of the operations of addition, subtraction, multiplication,
division, finding powers and extracting roots, it is strongly advisable to get a rough
approximation of what the result might be.
For this purpose, we can express numbers correct to one significant figure, e.g.

Number

One significant figure

Number

One significant figure

8256
4862
6291
38729

8000
500
60
4

01172
00256
00061
000048

0.1
003
0006
00005

lIBASIC ARITHMETIC 17

EXAMPLE 22
By expressing each number correct to one significant figure, find approximately the value of
(.') 28-2 x 0-61

(") 4327
II 21-2

("') 716 x 18
III
0-042

x 0-61 :=::: 30 x 0-6 = 18


By long multi plication, the result is 17-202_

(i) 28-2

(") 4327 :=::: 4000 = 200


II 21-2
20
By long division, the result is 204-1
"') 716 x 18 _ 700 x 20
(III
0.042 0-04

700 x 20 x 100 = 350000

EXERCISES 1(k)
Express each of the following correct to one significant figure:
I, 4172
2. 0'685
3. 2748
4. 888
9, 0155
6. 0-0056
7, 00203
8. 1796
11. 181-5
12. 38
13. 007
14, 60

5, 50010
10. 3296
15. 411
By expressing each number correct to one significant figure, find approximately the value of
16. 78 x 6
19. 48-.2 x 139
22. 64'26 x 1800
736
25 . 4-8
28. 17 x 61

17, 66 x 31
20. 26-2 x 189
23. 0-026 x 0-062
18-23
26 . 49
117-6 x 0-002
29.
0-048

18. 196 x 0-42


21. 128 x 512
24. 99-2 x 1-006
56-2
27. 0-006
88-9 x 316
30
576

1.13 Use of calculators _ _ _ _ _ _ _ _ _ __


Throughout this book, frequent reference will be made to the use of calculators. In this
chapter, we will look at using them to perform the basic operations of multiplication
division
addition [] and subtraction
To make sure that the correct keys have been used and that the correct sequence of
operations has been followed, the answer obtained can be checked by
(i) using one significant figure,
(ii) using the reverse operation (addition and subtraction are reverse operations; so too
are multiplication and division).
before beginning operations_
You should get into the habit of pressing the clear key

0 '

B,

(a) Addition and subtraction

EXAMPLE 23
Find the value of
(i) 56-28 + 122

(ii) 894 - 2-037

(iii) 788'1 - 4-701

38'~2

.,

-----

----------~.~-~--~~--.--

18 NEW SENIOR MATHS: TWO UNIT COURSE

Operation sequence
(i)
(ii)

(iii)

Display

D 28 1202
[9 89 0 4
20037
I9 788
EJ 4 701 []

6848

56

87'363
38

821919

Check:
(i) One significant figure: 60 + 10
70
Reverse the operation of addition by subtracting 122 (or 56'28) from 68'48:

68

D 48

1202

5628 correct

(ii)

One significant figure: 90 - 2


88
Reverse the operation of subtraction by adding 2'037 to 87-363:

[9

87

363

[]

037

894 correct

5 + 40
835
Reverse the operations by adding 4'701 to 821919 and subtracting 38'52:

(iii) One significant figure: 800

[9

821
919 [] 4
701
38
52
7882 incorrect
An error has been made, either in the actual solution or in this checking. Can you
locate where the error may be?

(b) Multiplication and division


EXAMPLE 24
Find the value of
(i) 8'796 x 0325

(ii) 616-9 + 28-71

Operation sequence
(i)

80796

(ii) [961609

GJ
B

B
71 EJ

0325
28

Display
2-8587

21-487287

Check:
(i) One significant figure: 9 x 0-3 = 27
I
Reverse the oper~tion of multiplication by dividing 28587 by 0-325:

208587

EJ OJ 325

8-796 correct

(ii) One significant ftgure: 600 + 30

20
Reverse the operation of division by multiplying 21-487287 by 28-71:

21
487287
28071
616-9 correct

(c) Combining operations


What is the correct answer for 6 + 4 x 2? Is it 20, obtained by adding 6 and 4 and
multiplying the result by 2, or is it 14, obtained by mUltiplying 4 and 2 and then adding the
result to 6?
Which answer does your calculator provide when you perform these operations from
left to right?
6[]40 2
Some calculators give 20 and others give 14.

l/BASIC ARITHMETIC 19

To avoid confusion, it has been agreed that the operations of multiplication and
division are performed before addition and subtraction, e.g.
72 + 6 - 2
10 x 2
3 x 6 + 12 + 4
6 + 4 x 2
= 6 + 8
= 12 - 2
= 20
- 18
+ 3
14
10
=5
If addition and/or subtraction are to be performed before multiplication and/or
division, then the particular section is enclosed in brackets (parentheses), e.g.
(6 + 4) x 2
72 + (6 - 2)
10 x 2
3 x (6 + 12) + 4
= 10
x 2
= 72 + 4
= 20
3 x 18
+ 4
= 20
= 18
20 - 13!
~

EXAMPLE 25
Find the value of
(i) 172 + 1845 x 927 - 4266 + 18
(ii) (17'2 + 18'45) x (9'27
42'66) + 18
Operation sequence
(i)

18045 ~ 9027 ~

[g
[g
[g

42

[9

17

Display
1710315

D 66 BID 8 EJ

237

EJ

17 02 [] 17100315
23
Answer is 164'5 correct to 4 figures.
(ii)

7~

1645315

3565

180 45

EJ 42066 B
[9 35 0 65 0 33 0 39 ~
[990 27

-33'39
-661,3075

Answer is -661' 3 correct to 4 figures.


Note the use of ~ key. How does it differ from the

key?

EXAMPLE 26
Find the value of 1742
81
62 + 0'48
Operation sequence

[9

17

042EJ 8 01E]

[g6D2EJD48B

Display
932
668

El 0

1'3952096
[g 9 32
6
68 E]
Answer is 1395 correct to 4 figures.
Why did we perform the operations of addition and subtraction 'before division in
this case?

(d) Percentage key


Because so many practical problems involve percentage, the percentage key on the
calculator is very useful. However, some calculators do not have a Ufo key. In this case it is
necessary to convert Ufo to its decimal form, e.g. 20% = 0'2, 12!Ufo = 0125.

20 NEW SENIOR MATHS: TWO UNIT COURSE

EXAMPLE 27
Calculate 12~OJo of 80.

Operation sequence
12

[]

Display

10

80

125 ~ 80
10
Note: You will observe that many calculators cannot cater for vulgar fractions. Such
fractions must be converted to decimals, e.g. ! 0'5, 3i
3'75, 5~ = 516666 ...

or

EXERCISES 1(I)
Use a calculator to find the value of the following, correct to 4 figures. (It is essential that
you check the answers you get.)
1. 172 + 12 + 86 15
3. 283'5 - 7734 - 19-92
5. 14'6 x 8-07 x 0002
7. 442 + 1987
9. 18-2 x 3-6 + 7-7
11. 92 x 77 - 31 x 0'88
(8 x 32)
13. 25
414 x 18
15 .
005

12.
14.
16
.

17.

18.

283 x 9-5

19.
x
21. 14070 of 728
23. 17*OJ06f98'3

2.
4.
6.
8.

10.

0'381 + 54
lll8
881
0'326 + 137
83 x 36 x 5'18
56 x 64 + 36
44
13
16
144
8-6 + 21 + 95
177 x 59 x (17'5 - 8'3)
902 x 196~2
48 x 0089

386
1'529

8.8 x 33

20. (191

+ 62) x 189 - 038

22. 9'5% of 46
24. 22070 of 90 - 15% of 80

1.14 Powers and roots _ _ _ _ _ _ _ _ _ __


(a) Squares and exact square roots
Most mathematical tables contain the squares of all the natural numbers 1 to 100. Their
exact positive square roots can be obtained by reading these tables in reverse. The tables
have been inserted on page 24 for your reference. At this stage, only the columns headed
nand n 2 will be necessary.
However, the use of tables has now been superseded by the use of the calculator. When
using a calculator make sure you check your result. Since the operations of squaring and
extracting a positive square root are a pair of reverse operations, squaring can be checked
by taking the square root and taking the square root can be checked by squaring e.g.
42 = 16 and so -Jf6 = 4
--}004
02 because 02 x 02
004

EXAMPLE 28
(a) 15 2
(b) 242
(c) 65 2

225 and so --}225


15
576 and so --}576
24
4225 and so --}4225
65
The tables can also be used to find the squares of certain rational numbers and the
corresponding square roots.

1/BASIC ARITHMETIC 21

EXAMPLE 29
(a) 2-4' =.
..)5'76 =

J576
100

= ..)576

~
~64

r-:::I6!

vtJ~

--

it

036 and so, in reverse,

v'Ioo =

.J9

(d) (2!)2 =

/;0

-J36

3'

576 and so, in reverse,

24 = 2'4
10

~100

~
.)0'36 =~ 100 =

(~),

ib~

(I~)' = I~

(b) 06'

(e)

;6:

(;~), =

10 = 0-6

and so, in reverse,

~
5 5

2 x 2=

J254

25

6~ and so, in reverse,

~
..J4 - 2~

-- 21

Squares and square roots using a calculator


Refer to (a) and (c) in example 29
Operation sequence
2.42

2040

or

2'42

20 4 0

or

2.42

2[J4 ~

(a)

Display
5-76

B
20

576
576

or 2.42
2
4
2
In reverse,
~5-76 = 50 76i.JX1
or .)5'76 = 5

76

(~), = 3 B

(c)

576
24

@l

24

0'140625

i.e. 0-375 squared


You will observe that the calculator will not display :4; vulgar fractions are expressed
as decimal fractions. Of course,

~ can be expressed as 0375 first and then any of the four

sequences as in (a) above can be performed.


In reverse,

J~=9

orJi4 =

64ElG
B

~D5

64

O'375

(not ~)

0375

EXAMPLE 30

+ 16 =
Note: ~9 + 16

(a) .)9

5.

*'

~9 + ~16 = 3 + 4 = 7.
~ is the symbol for positive square root and the bar across the top is equivalent to

brackets to indicate that we evaluate 9 + 16 first and then extract the positive square
root.

22 NEW SENIOR MATHS: TWO UNIT COURSE

(b) "'/(0'5 2

+ 1'44)
+ 36) = .../52

1.22) = "'/(0'25

"'/1'69

13

62) = .../(16
~ 7211 to 3 decimal places using tables of squares
and square roots.
Using a calculator to evaluate .../52:
Operation sequence
Display
7 2111
[g52 ~

(c) .../(42

B . B

or lQ 52
5
2
Since 7 = 49 and 82 = 64,

.J52 lies between 7 and 8.


x-!, we can use either JFxJ key or the ~ key where y = 0'5.

Fx

Since

72111

Cubes and cube roots

EXAMPLE 31
(a) (0'3)3

0-3 x 03 x 03

125
8

5 . 5
5
222

=-x-x

10
TX

10

0'027

10

1000 _ 37-L
-

For calculators, use

27

key where y = 3 or multiply the number by itself twice.


Display

7.8

(d)

D8
7D 8
7

or78 3

474'552
474-552

EXAMPLE 32
(a)

(b)

<t3I

3 because 3 x 3 x 3

(c) ~O'OO8 = ~8 x

Since

= 27

~
10-

10- 1 = 02

x!, we can use the ~

~=

key on the calculator where y

O' 333 _..

Display
(d)

~846'6

846

6 ~ 0'333 ... ~

Check: ~846'6 ~ ~800


1000.

a number between 9 and 10 because 93 =

9'460
729 and 103 =

EXERCISES 1(m)
(Answer 1 to 9 without the use of a calculator).
1. Evaluate
(a)
(e)

IF
42

(b) 172
(f) 122

(c) 202
(g) 402

(d) 15 2
(h) 25 2

2. Find the exact value of


(a)

.J64

(b)

-J9

(c)

.J49

(d)

.JITf

I/BASIC ARITHMETIC 23

(e) -JI00

(f) "';900

3. Write down the square of


(a) 02 .
(b) 1'5
(e) 004
(f) 16

(g) .../1600

(h) .../169

(c) 32
(g) 14

(d) 03
(h) I-I

(c) "'/0'04
(g) "';0'49

(d) .../0-36
(h) "';4-41

4. Find the exact value of


(a) -Jf.2I
(e) -JI024

(b) "';0'09
(f) "';30'25

5. Evaluate

(~y

(a)

(c)

(;5Y

(d) (2lf

(g) (3~)2

(e) (2t>2

(h) (2~)2

6. Find the exact value of

(a)j[

'[16
Sf

(e) ~

7. Evaluate
(a) (0'1)2
(e) 1.72

(b)~

J[
36

(d)

.J3-h

(h)

Ii

(c)

(T

(f) ~TOO

(g)

(b) l' F

(c) (2D2
(g) (0-07)2

(d) (3})2
(h) (0-03)2

(c) ';20-25

(d) ';0-01

(f) 3 F

8. Write down the exact value of


(a) "'/2'25
(b) ';i2~
(e) ~f
(f) ';0'04

(g)

49

v211

(h)

169

.Jl2I

9. Evaluate:
(a) -J(36 + 64)
(d) -J(0-09 + 0'16)

(b) -J(5 2 + 122)


(e) .../(1~)2 + 22

(c) "';(0-7)2 + (2-4f


(f) -J(1-2)2 + (1-6f

10. With the aid of a calculator, evaluate to 2 decimal places


(a) -J5'17
(d) .,[52 + 62

(b) -J86
(e) -J(3 2 + 82)

(c) -JO-03
(f) v.-c-:(l:--2-::-:-)-=-2-+-(-:-:::2--='1-:-:0)2

11. Without the aid of a calculator, find the exact value of:
(a) (1'1)3
(b) (0'5)3
(c) (0'1)3
3
(d) (l~)3
(e) (2t)
(f) (3D3
(g) (0-01)3

(h) (0-4)3

(i)

(~)'

12. Without the aid of a calculator, write down the exact value of:
(a)
(b) ~0'064
(c) ~

ms

3Is

(d) ~ill
(g)

</0001

3{343

(e) ~27
3

(h) -\10216

3fT

(f)

(I)

\/64

VlOOO
3{27

13. With the aid of a calculator, evaluate to four significant figures.


(a) (5'2)3
(b) (4-01)3
(c) (0'32)3
(d) (0-015)3
(g)

!./99

(e) ~81'6

(f) ~0'526

(h)

(i)

Jf

~O-l

24 NEW SENIOR MATHS: TWO UNIT COURSE

Squares and square roots


n

n2

n2

-In

n2

-In

n2

-In

1
2
3
4
5

1
4
9
16
25

1
1414
1732
2
2236

26
27
28
29
30

676
729
784
841
900

5099
5196
5292
5385
5477

51
52
53
54
55

2601
2704
2809
2916
3025

7141
7211
7280
7348
7416

76
77
78
79
80

5776
5929
6084
6241
6400

8718
8775
8832
8888
8944

6
7
8
9
10

36
49
64
81
100

2449
2646
2828
3
3162

31
32
33
34
35

961
1024
1089
1156
1225

5568
5657
5745
5831
5916

56
57
58
59
60

3136
3249
3364
3481
3600

7483
7550
7616
7681
7746

81
82
83
84
85

6561
6724
6889
7056
7225

9
9055
9110
9165
9220

11
12
13
14
15

121
144
169
196
225

3317
3464
3606
3742
3873

36
37
38
39
40

1296
1369
1444
1521
1600

6
6083
6164
6245
6325

61
62
63
64
65

3721
3844
3969
4096
4225

7810
7874
7937
8
8062

86
87
88
89
90

7396
7569
7744
7921
8100

9274
9327
9381
9434
9487

16
17
18
19
20

256
289
324
361
400

4
4123
4243
4359
4472

41
42
43
44
45

1681
1764
1849
1936
2025

6403
6481
6557
6633
6708

66
67
68
69
70

4356
4489
4624
4761
4900

8124
8185
8246
8307
8367

91
92
93
94
95

8281
8464
8649
8836
9025

9539
9592
9644
9695
9747

21
22
23
24
25

441
484
529
576
625

4583
4690
4796
4899
5

46
47
48
49
50

2116
2209
2304
2401
2500

6782
6856
6928
7
7071

71
72
73
74
75

5041
5184
5329
5476
5625

8426
8485
8544
8602
8660

96 9216 9798
97 9409 9849
98 9604 9899
99 9801 9950
100 10000 10

1.15 Irrational numbers _ _ _ _ _ _ _ _ _ __


Not all numbers can be expressed in the form ~ where a and b are integers.
Consider the right-angled triangles below in which, as you have seen from your study of
the Theorem of Pythagoras, the hypotenuse has lengths -J2 units and .J5 units.

Ld'

I/BASIC ARITHMETIC 25

What are the values of numbers such as .J2, -J), -15, ~?


The following set-square construction shows how to draw lines of length .J2, ..[3,
-15, ~, ... units.
,.....

..J4,

....--- ..........
........,

-------- ............ ,

//,"""",- ---1----- . . . . . . . "

,,-

....

/
/

/.1

I'.

.... 1

...... , "

.... ,

....' , ,

1 ....

""',

""
,

\
\

o
-15, -J6 obviously

'

,
,\,,' \
\

,,

\ \

\'\
"
"

,~
3

Numbers such as .J2, -J),


have some exact value because they can
measure lengths of lines as shown in the set-square construction and there are points on
the number line corresponding to them, as can be seen in the above figure.
We know that
12 = 1 and so vT = 1,
22
4 and so ..J4 = 2,
32 = 9 and so -J9
3, but
What number squared is equal to 2?
What number squared is equal to 3?
What number squared is equal to 5?
1.42
196 and 1.5 2 = 225 and so 14 < .J2 < 15
This means that .J2 lies between 1'4 and 1'5 on the number line.
1412
19881 and 1.422
20164 and so 141 < .J2 < 1-42.
This means that .J2lies between 141 and 142 on the number line.
Your four-figure tables give 1'414 as.J2 but if you square 1414 you don't get exactly 2;
thus 1414 is not the exact value of .J2, but yet .J2 does have an exact value.
Using a calculator you may be able to get .J2 as 1'414213, but if you square 1'414213
you don't get exactly 2 and so 1414213 is not the exact value of .J2.
Numbers such as 1'4, 1'41, 1414 are rational numbers and are what we call rational
number approximations to .J2. It can be proved that there is no rational number that is
exactly equal to .J2.

Numbers such as .J2, -J), -15, ~ are said to be irrational numbers, i.e.
not rational numbers, and are called surds.
7r

2
is also an irrational number and 27 or 3142 is used only as a rational number approxi

mation for the value of 7r. Your calculator value of 7r is 31415927.


Since numbers such as .J2, .J3, -15, .J6 and 7r cannot be expressed as rational numbers,
their exact values can be written only as .J2, -J), -15, .J6, 7r.

1.16 Real numbers _ _ _ _ _ _ _ _ _ _ __


The entire collection of rational and irrational numbers is called the set of real numbers.
The following Venn diagram shows the various subsets of the set of real numbers.

26 NEW SENIOR MATHS: TWO UNIT COURSE

Real numbers R
Rational numbers Q

Irrational numbers I

The various subsets are represented by capital letters:

R
Q
N

Ireal numbersl
lrational numbersl
Inatural numbersl

I = Iset oj irrational numbersl

J = Iintegers I

R is the universal set and N, J, Q and I are subsets: N C J, J C Q, Q C R, I C R,


where C means 'is a subset of'.

1.17 Operations with surds _ _ _ _ _ _ _ __


2 x
.J36

..J4x~

i.e .

.../93
~

If
i.e.!f=

6
i.e.

..J4 =

-..)25 x 4
-J25 x 4

-J25
.J4 -- ~2 -

15
1'5

,j9

) 21

5 x 2

= -..)100
=

-J25 x

10

= 10

2.5

= "/625 = 25

. )25 _ ../25
I.e. 4 - -J4

These examples illustrate that:


If a and b are positive numbers,

(a) Simplification of surds


If a number has aperJect square as one of its factors, the square root of that number can
be simplified. The numbers
4, 9, 16, 25, 36, 49, 64, 81, 100, ...
are perfect squares because their positive square roots are
2, 3, 4, 5, 6, 7, 8, 9, 10, ...

I/BASIC ARITHMETIC 27

Numbers such as 12, 18, 28, 80 and many others contain perfect squares as factors.
Thus m, .JIB, m, V80 can be simplified. However, numbers such as 15,21,39 do not
contain perfect squares as factors and so ..JT5, -v'2i, ..J3 cannot be simplified.

EXAMPLE 33
(i) 02 = KX3 = .J4 x -13
2-13

(ii).JIB
.J9X2 = .J9 x -J2 = 3-J2

(iii) .J80 = -v'16 x 5 = Vf6 x -J5 = 4-J5


(iv) 3m

= 6-J7

3.J4X7 = 3.J4 x -J7

EXERCISES 1(n)
Express each of the following in simplest surd form.
1. .J8
6. .J4O
11.
16. -v'108

21.~

26. 5-v'128
31. 9-J68
36. 5-v'125

2. -J2O
7. -v45
12.
17. -v'ill
22. -v'200
27. 4-v'800
32. 3.J50
1
37. 2-v'320

3.
8.
13.
18.
23.
28.
33.

38.

-v'24
.J48

4.-J27
9. -J56
14. -J%
19. -v'147
24. 3.J54

.J84

-v'125
-v'288
2-v'150
4-v'90
1

29. 5.JIB
34. 4-v'250
4
39. 3-v'243

5075

(b) Addition and subtraction of surds


Using the Distributive Law in reverse

ab

ac

a(b

c)

we can add or subtract surds of the same kind, e.g.


2-13 + 5-13 = (2 + 5)-13
7-13

4M - M
(4
1)00
= 3M

3..J6 + 4..J6
-J5 = (3 + 4)..J6 - -J5 = 7..[6 - -J5

EXAMPLE 34
Simplify (i) .J8

.JIB + -J5O,

(ii) 5-13

+ -J2O

2m

..J4X2 = .J4 x -J2 = 2-J2

.J9X2 = .J9 x -J2 = 3-J2

-J5O -J25 x 2 = -J25 x -J2 = 5-J2

: . .J8 - .JIB + -J5O = 2-J2 - 3-J2 + 5-J2 =


5-13 + -J2O - 2m + -v45

+ -v45.

(i) .J8 =
.JIB =

(ii)

5-13 + 2-J5

4-13 + 3-J5

3)-J5

= (5 - 4)-13 + (2 +
= -J3 + 5-J5

4-J2

5. -J32

10. .J63

15. .J98

20. -JI62

25. 2..)44

30. 3-J52

35. 7-v'245
40. 3v1242

28 NEW SENIOR MATHS: TWO UNIT COURSE

EXERCISES 1(0)
Simplify:
1. .J3 + 2.J3

+ 4.;3
3. 3.J5 + 5.J5 - 2.J5
5.
+ V2 + 3.J5 - 6--/2
7.
-JT8 + .J3
9. -JT8 +
.J2

2.
4.
6.
8.
10.
12.
14.
16.
18.
20.
22.
24.
26.
28.
30.

m -

11.

13.
15.
17.
19.

21.
23.
25.
27.
29.

+ -J3 + ~
+ m - -J63
6.J5 + 4.J7 - 2.J5
-J2(j + .J5 + .Jf8
4.J7 - m + -J63
.J3 + 3m - .J48
.J6 + V24 + .J54

5-J45 2m
.JTI5 - 5.J2 + .J5O
~50
.J% - V24

2--17 + 4.J7
-J3 + 4.J3 - --/2
~
.J1
-J2(j + .J5
+ 2.J48 - 4.J3
2.J50
3M
.J5 +
v'45 + ~
5.J3 +
- v'45

5.J7
4.J1

3~

+ ..J6O - v'4O
3-JT8 + .J3
.JIso .J2OO
5--17 + 3-15 - 2m

2.J50

.J98 - 2-J2(j 7.J3


2.J1 + .JI2 + -J8
6.J1 +
2.J3 - 3~

m-

(cl Multiplication of surds


In this section we will make use of the properties

va x .Jb = .Jab
vaxva=a
EXAMPLE 35
.J7 x.J1 =.JI4
.J1 x -J6 =
.J4X3 = .J4 x.J3 2.J3
(iii) 2.J5 x 3-15 = 2 x 3 x .J5 x .J5 or 2.J5 x 3.J5 = 6-J25
=6 x 5
=6 x 5
(i)

(ii)

=
(iv) .J3

30

.JI2

.J3 x 2.J3
=2x.J3x.J3
2 x 3

= 6

(v) 4--/2 x V24


4.J1 x 2V6
=
= 16.J3

or

.J3 x

= 30
= ..J36
=6

or 4.J1 x

8m

V24

4.J48

4 x 4.J3

16.J3

EXERCISES 1(p)
Simplify, expressing the result in simplest surd form.

1. .J3 x .J5
5 ..f5 x
9. --/2 x

.J5
.JTI5

13. 2.J3 x 4v2


17. 4.J6 x 2.J3
21. 2-15 x 3--17

2.
6.
10.
14.
18.
22.

~ x

.J2

.J6 x .J3
.J6 x M
4.J5 x .J6

M x 2M
4.J3 x -JT8

3 . .J6 x .J1
7. .J5 x .J7
11. .J3 x
15. -J8 x 2.J1
19.
x 3.J1
23. 2~ x 02

5m

J'

4.
8.
12.

x .J7

.J6 x .Jff

x .J1
16. 2-15 x 5V2
20. 4.J3 x 3.J3
24. 4.J5 x -J2(j

I/BASIC ARITHMETIC 29

(d) Division of surds

When dealing with surds of the form

-Ja

i.e. with fractions that have a surd in the

denominator, it is usual to rationalize the denominator.


This means that we express the fraction with a rational number instead of a surd in the
denominator. If the denominator is of the form ..fi:J, we can rationalize the denominator
by multiplying numerator and denominator by ..fi:J since..fi:J x..fi:J
b.

EXAMPLE 36
Express each of the following in simplest surd form with rational denominator.
( 0)
I

.J7

.JTO
1I.J2

(0')

(000)

1II.J3

(0

.J)5
~
6

IV{j

.J7 x.J3 .J2T


1.J3 T3 x.J3 -3
o,).JTO .JTO x.J2 ~ 2-J5
(1I.J2
.J2 x .J2
2
..) ~ _ 1 x.J3 _.J3
(III
.J3 - .J3 x .J3
C)
-J5 -J5 x..J6 ~30
IV './ 6..J6
~ x ..J6
C).J7

'5

'V .J

rr-= =

Note that, in each answer, there is no surd in the denominator.

EXERCISES 1(q)
Express each of the following in simplest surd form with rational denominator.
1.

.J3

6vU

'.J3

5
11. -J5
16

.J2
'.J7

21.

.[80

7.J2

26. -J98

.[8

2.
7.
12

.J2
2

.J3
'-J5

17.Jf
22.
27.

3.J2
3-J5

3.

4.

3
.J2

8.

9.

13

14 .

7
.J7

'.J7

18.)[
-v'To
. 5.J2
28 2v1T8
23

'.[8

19.Jf
24.

M
3..J6

29

-J5O

'J5

5.

J5
I

10.

-J5

15

.J7

20.Jf
25

. 5.J3

30 2.J7
-J35

(e) Use of the Distributive Law


The Distributive Law
is true when a, band c are either all surds or a mixture of surds and rational numbers.

30 NEW SENIOR MATHS: TWO UNIT COURSE

EXAMPLE 37
(i) 2(.J3 + --/5)
2.J3 + 2--/5
(ii) .J3(~ + .J3) = .J2f + 3
(iii) ../6(.[2 + 2.J3) = .JT2 + 2v'18 = 2.J3

6.J2

EXERCISES 1(r)
Expand each of the following, expressing the result in simplest surd form.
2. --/5(--/5

5 ../6(.J3
2)
8. .J3(2)6 - 3.J3)

-18)

7. .J2( -J32
10. 3.J2(2)6

--/5)

3 . .[2(.[2 + -18)
6. 7(2--/5 - 1)
9. 2..[7 (.JT4 - 3)
12. -Ji(..JX
.JY)

+ .[2)

1. --/5(.[2 + .J3)
4 .J3(.J2 - )6)

-Ja(-Ja + .Jb)

11.

Similarly, we may use the extensions of the Distributive Law:


(a

b)( e + d)
(a + b)2

ae
a2

+ ad + be +
+ 2ab + b 2

bd

as well as the familiar difference of two squares:

EXAMPLE 38
(i) (.J3

(ii)

.J3( -J5 + ../6) + .J2(--/5 + )6)


= .JI5 + 3.[2 + .JIO + 2.J3

(.[2 + --/5)2 = (.[2)2 + 2.[2--/5 + (--/5)2


= 2 + 2.JW + 5

..[2)(--/5

../6)

7 + 2.JIO
i.e.
(iii) (.J5

i.e.

+ b)2

(a

2ab + b 2 where a
(--/5)2
(.J3)2

a2

.J3)(--/5 + .J3) =
= 5
(a - b)(a

= 2
b) = a 2 -

= .J2 and

= --/5

3
b 2 where a

= -J5 and

= .J3

EXERCISES 1(5)
Expand each of the following, expressing the result in simplest surd form.

1. (--/5 + .J3)(..[7
.[2)
3. (.J3 - 1)(.[2 + 3)
5. (2.J3 -

5)(2.J3

+ 3)

7. (..[7 + 1)(2..[7 - 5)
9. (2--/5
.[2)(2-J5 + 3)
11. (.J3 + 1)2
13. (2../6 + .J3)2
15. (2 + .J3)(2
.J3)
17. (.J7 - --/5)(.J7 + -J5)
19. (2../6 - .J3)(2../6 + .J3)
21. (.[7 - 2)(..[7 + 2)

2.
4.
6.
8.
10.
12.
14.
16.

(.J2

(-J5 +
(--/5 +
(.J3 (2.[2 (--/5 (.J3 +
(.J3 -

-J7)(.J3 + 2.[2)
2)(2--/5 + 3)
.J3)(../6 + 1)
.J2)(2.J3
..[2)
)6)(2.J3 - 1)
.J2)2

6)2

+ .[2)
1)(2V2 + 1)
20. (.JTI - .[7)(.JTI + .[7)
22. (3..[7 - 2)2
18. (2.J2 -

..[2)(.J3

I/BASIC ARITHMETIC 31

23~

(.J5 -

.J3)2
1)(.J3 + 2)
(-16 - .J5)(-16 + .J5)
(3.J5 - 2.J2)(3.J5 + 2.J2)
(2-16 - --./3)(-16 + 3.J3)
(5.J2 - 4)(5.J2 + 4)
(2.Ji5 + .J5)(
3.J5)
(2.J3
1)(2.J3 + 1)

25. (2.J3

27.
29.

31.
33.
35.
37.

m -

24. (v'TI + -J7)2


26. (v'TI - ~)(v'TI + ~)
28. (2.J2

.J3)2

30. (.J5 + 2.J2)( -16 - 1)


32. (4.J3 + 1)(2.J3 - 3)
34. (2-J7 + 3-16)2
36. (2.J2
38. (2.J3

3.J3)2

1)2

An important result of the use of the distributive law is that obtained from the product
(--./a + --./b)(-.)a - -.)b) resulting in a-b. This is the familiar difference of two squares, a

and b being squares of -.)a and -.)b respectively.


The two expressions -.)a
-.)b and -.)a + -.)b are called conjugate surds, each being the
conjugate of the other. The usefulness of conjugate surds lies in the fact that their product
does not contain any surd term. e.g.
(-.)5 + -.)3)(-.)5 - -.)3) = 5 - 3 = 2
(3-.)7 + 2-.)2)(3-.)7 - 2-.)2) = 63
8
55
This property enables us to rationalize the denominators of fractions with denominators
of the form .J(j + -Jb or .J(j - -Jb by multiplying numerator and denominator by the
conjugate surd.

EXAMPLE 39
(a) Express -=----= with a rational denominator

.J3 - .J2
1
.J3 -.J2 ( '
.J3 + .J2 X.J3
.J2 smce .J3
.J2

where .J3
.J2 is the conjugate surd of.J3 + .J2.

.J3-.J2

1
Thus - = - - - : : = = (.J3 + .J2)(.J3 - .J2)

.J3-.J2

3 - 2

=.J3-.J2

(b) Express -J7.J3 .J3 with a rational denominator

27-

.J3

.J3
2-J7 + .J3

2-J7 _ .J3 x 2-J7 +.J3 where 2-J7 + .J3 is the conjugate surd of
2-J2T + 3
2-J7 - .J3.
28 - 3

2-J2T + 3

25

EXAMPLE 40
, I
' WIt
'h ratIOna
'
Id enommator.
'
E xpress 2.J2.J2+ 1 + .J3 2+ 1 as a smg
e f
ractIOn
We may add the two fractions first and then rationalize the denominator of the result,
or we may rationalize the denominator of each fraction first, and then add the resulting
fractions. The latter procedure is preferable.

32 NEW SENIOR MATHS: TWO UNIT COURSE

+
2

+
.
.J2

.. 2.J2 + 1

.J2(2.J2 - 1)
4 - .J2
7
(2.J2 + 1)(2.J2
1)
_
2(-J3 - 1)
2(-J3 - 1)
2

1 - (-J3 + l)(-J3
1)
4-.J2
~-1
7
+
1

_ 4 - .J2 + 7(~
1)

= ~

7-J3 - .J2 - 3

EXERCISES 1(t)
Express the following fractions with a rational denominator (1. to 24.)
1. -=---=
1

4. 2-J5 - 3.J2
2-J5
7. 3-JIT - 2-J8
.J7 - 2-J5
10. 3-J5 - 2.J2
2-J3
13. 3-J3
2

-J2

16. --===---=

1
2. 2.../7 + .J6
3.J2
5 . .../5 - -J3
~ + 2
8

- 2

11 3.J2 + 2~
. 3.J2 - 2~
3.J6
14. 2.J2 + ~
3-J3
17. 2~ + .J2

19. -=-

20. 2.../5 -

22.

23.

----==-------:=

25. If x

3 - 2.J2 .
3 + 2.J2' fmd the value of x

26. If x = -J3
27. If x

-J2

+
.../5 + ~

+ 1, find the value of X2

3.J2

1, find the value of

X2 -

1
3 . .../5

+2

.J6
2.../5 - 3.J2

4.J2 + 3.../5

2.../5 - .J2

5-J3 + 3.../5

5.../5 - 3~
15 5-JTI + 3
3-JTI - 2
-J3
18 . .J24 _ ~
6.
9

12

21 .J6 + 2-J3
2.J6 - -J3
24 2.J2 - 5~
3.J6 - vTI

x'1

-\.

21x.

x
~.
X2 + 2x
28. I f x = 'V 5 - 2, fmd the value of - -
x+
29. Show that x = 2.J2 - 3 is one of the roots of the equation X2 + 6x + 1 =
30. Does x
.J2 - 2 make X2 + 4x + 2 = 0 a true statement?
31. Show that x = .../5 - 1 is one of the roots of the equation x 3
32. Show that x

= ~ 1_ 1 is one of the roots of the equation 2X2

o.

3x2 - 2x - 4

- 2x - 1

= O.

O.

Express each of the following (33. to 41.) as a single fraction with rational denominator:

-".1

I/BASIC ARITHMETIC 33

36.

~---=-=

37.

--==---

38.

-,:=----=

39.

~-

40.

41.

-==---=

x-

x+

where x = 2.[3

+ 1

. Ii

CHAPTER 2

Basic Algebra

2.1 Positive and negative numbers _ _ _ _ __

-4

-2

-3

-1

..

Number line

(a) Positive numbers


There are two uses for the sign' +'.
(1) It can indicate addition; for example, if we add 3 to 5 we get 8, i.e. 5 + 3 = 8.
(2) It can indicate direction. On the number line above, we consider numbers to the right
of 0 as positive; for example, +1, +2, +3 (or simply 1, 2, 3 without the positive sign)
indicate 1, 2, 3. units to the right of O.
These directed numbers are called positive numbers and are read as 'positive one,
positive two, positive three' without any reference at all to addition.

(b) Negative numbers


There are three uses for the sign' -'.
(1) It can indicate subtraction; for example, if we subtract 3 from 5 we get 2, i.e. 5 - 3 = 2.
(2) It can indicate direction. On the number line above, we consider numbers to the left of
o as negative, e.g. -1, -2, -3 indicate 1, 2, 3 units to the left of O.
These directed numbers are called negative numbers and are read as 'negative one,
negative two, negative three' without any reference at all to subtraction.
(3) It can also indicate 'the opposite of' 0
"t

-4

~: pPo~,es .~

-3

-2

..

Number line

For example, -3 is the opposite of +3, i.e. -3 = -(+3). We say that negative 3 is the
opposite of positive 3.
-(-3). We say that positive 3 is the opposite of
Also, +3 is the opposite of -3, i.e. +3
negative 3.

The opposite of negative is positive.

2/BASIC ALGEBRA 35

(c) Negative x
In the study of algebra, it is convenient to regard the - sign as meaning 'the opposite of'.

I -x means the opposite sign oj x.


If x = 3, then -x
-3
If x = -3, then -x = -(-3) = 3
Thus, if x is a positive number, -x is a negative number, and if x is a negative number, -x is

a positive number. Very confusing!


While it is true to say that a black cat is black, and a dead horse is dead, 'negative x' is
not necessarily negative.
For this reason, it is better to speak of -x as being 'the opposite sign of x' rather than
'negative x', and it should not be called 'minus x' unless the operation of subtraction is
involved, e.g. 3x minus x equals 2x, i.e. 3x - x = 2x.

EXAMPLE 1
If x
(i)
(ii)
(iii)
(iv)
(v)
(vi)
(vii)
(viii)
(ix)

5, y

-2,

x - y = 5
(-2) = 5 + 2 = 7
3(y + z)
3(-2 + 4) = 3 x 2
-(x + y)
-(5 + -2) =.-3
3x 2
3 x 52 = 3 x 25
75

3y2 = 3 X (-2)2 = 3 x 4
12
(3y)2 = (3 X -2)2
(-6)2 = 36
x(y - z) = 5(-2 - 4) = 5 x -6
-30
(y - z)2
(-2 - 4)2 = (-6)2
36
-x2y
-(5)2 x -2 = -25 x -2
50

EXERCISES 2(a)
If x

8, y

-4 and

-3, find the value of


2. xy + 7
5z
5. 4x
8. x + 3z
11. 2X2
14. -3z 2
17. 2y(x - z)

15. -x(y - z)
z
18. 2y

19. 2(y - z)

20. 5x 2

21.

22. 2x(x + y)
25. 2y + Z2
28. 2(yZ)2

23. x 2 - y2
26. 2x2y
29. 2X2 - 3y2

24. 2X2 - z
27. 2xy2

1. 3x - 4
4. 5(x

z)

7. 2x x 3y
10. (x

+ y)2

13. (2y)2
16. 2x

+ y

If a = 2, b

-3, e

3. 3y

x - z

30. yz

+ y2

-4 and d = 5, find the value of


33. a + b + e
36. 2d + 3b - a

31. a + 3

32. b

34. be
37. e(b

35. bed
38. (a - b)2

39. a

1
40. 2,ab 2

41 a + 2
b + 2

42. d + 2e

43. d(a - b)

44. b 2e2

45. 3a

d)

6. 2y + 3x 9. x + y2
12. 2y2

b2

4b - e

36 NEW SENIOR MATHS: TWO UNIT COURSE

2.2 Addition (and subtraction) of like terms _ _


You are familiar with the fact that you can add or subtract like terms, e.g.
a + a
4a 2b

-+

= 2a

3a 2b

3x

2x

Sx

7 a 2b
Sxy
2xy = 3xy
The reason for this lies in the use of the Distributive Law.
a + a
a(l + 1) = a x 2 = 2a
3x + 2x
= x(3 + 2) = x x S = Sx
4a 2b + 3a 2b = a 2b(4 + 3) = a 2b x 7
7 a 2b
Sxy - 2xy = xy(S - 2) = xy x 3 = 3xy
We can frequently simplify algebraic expressions by collecting like terms after removing
brackets, if they exist.

EXAMPLE 2

----------

(i)

3x
= 3x

+ 2y +

Sx

2y - 6y

Sx
8x - 4y

(iii)

2(3a - 4b) -

= 6~ - 8f)=-3a
= 6a
= 3a

3a
7b

8b

6y

3(a -

X2

(ii)

~ 2
+ 2x__x + 3x

= X2 + 3x2 +
= 4X2 + X

Sb)

(iv)

3x(x

2x - x

y) - 2x(x

3y)

=3x~xy

+ Tsb
+ ISb

3x2 - 2X2 X2 - 9xy

3xy - 6xy

EXERCISES 2(b)
Simplify the following expressions by collecting like terms.
1. 3x + S + 7x + 10
3. 4a + b
a
4b
5. 3xy + 2xy - yx
7. 3a 2b
3ab 2 + 2a 2b
9. 2x2y + 3x2y2 - x2y + 3x2y2
11. 3p2q - Spq
2p 2q
13. X2 - 3x + 2x + 4X2 .
15. 2X2 + Sy2 - 4X2

2. 7x - 3

3x -

4. 6ab + 3ab + Sa + 4a
6. mn

8mn

3nm

8. Sxy2 + 3xy2 - 2xy


10. 3abe

+ Sbea
+ 3m

- 2eba
6mn - m
14. a 2 + Sa 2 - 3a - Sa
16. 9x2 - 3xy + Syx - 6x 2

12. 12mn

Simplify the following expressions by removing the brackets and collecting like terms.
17. 'll

3(a

b)

19. 8m - S(2m - 3n)


21. S(2x + 3)
S(x + 7)
23. 7(2k + 1) - 3(k + 2)
25. Sx(x - 2y) + 3x(2x - y)
27. 2a + 3b - (a - b)
29. 3x(x - y) + 2x(x - y)
31. m(3m - Sn) + 3m(2m - n)
33. 2(X2
x - 6) - 3(X2 + 2x)
35. 3(X2 + Sx - 1) - (2X2 + X - 2)
37. a(a + 1) - 3(2a + 1)

IS.
20.
22.
24.
26.

2S.

30.
32.
34.
36.

3S.

4(2x - y)
6x
3(2x + Sy) + 4(x - y)
6(2a + 3b) + 3(a - b)
Sa(a + 2) - 3a(a + 1)
4a(2a + b) - a(a + 2b)
x + Sy - (3x + 2y)
Sx(2x + 1) - (x 2 + x)
IS(x - 2) + 4(3x - 3)
3x(x
2) - 4(x
1)
(Sx + 2y
3)
(x - 7y
3(m2 - m) - 2(m2 + 2m

+ 9)
+ S)

2/BASIC ALGEBRA 37

2.3 Substitution in formulae _ _ _ _ _ _ _ __


EXAMPLE 3
7fr2h find (a) V when r
3!, h = 5
(b) r when V = 275, h
14

If V

Use 7f ~ 3~

(a)

v=7fx

2 X 2 X5
2

~Il;r

x Z x :2 x 5 (using 7f ~ 3~)

192~

or V = 7f
(b)

22

275 =

:2 x 2' x

x r2 x W

245
T7f

(using 7f

(exact value)
~ 3~)

275
44
25

r2

r=f[
2!

or 275 = 7f

X r2

x 14

275

r - 147f

2 _

r --

)275

147f

(exact value)

EXERCISES 2(c)
(Use 7f
1. P

3~,,)

2(/

b); find P when /

20, b = 12.

2. E = IR; find E when I = 2'4, R


3. F

= ma; find F when m

40.

= 50, a = 0'2.

= ~c + 32; find (a) Fwhen C


5. A = 7fr2; find A when r 35.
4. F

(b) C when F

60

= 41.

7fr2h; find V when r = 42, h


10.
= me2 ; find (a) E when m = 10, e = 1'6 (b) e when E 13'5, m = 15.
v = u + at; find v when u = 20, a
l'S, t = 10.
s
ut + kat2; find (a) s when u 5, a = 6, t 2-4 (b) a when s = 50, t =

6. V
7. E

8.
9.

u = 10.
u 2 + 2as; find v when u = 12, a = 2, s = 20'25.

10. v2

= ~u + v)t; find s when


12. S = 27frh; find S when r =

11. s

2-6, v

2'5, h

3'5.

3'2, t

25.

2'5,

~-------------~-~--~---~------

38 NEW SENIOR MATHS: TWO UNIT COURSE

13. r

if

find (a) r when A

~m(V2

14. E

15. t
d

= 4, v = 4,
find (a) t when a = 3'8, n

= II.
= 2.
20~

= -0'2 (b) n when a = 56

5, t = 256.

16. F = m(v t

17. t

- u 2); find E when m

+ (n - l)d;

154 (b) A when r

= ar 5 ;

u); find F when m

find t when a

64, r

= 20,

= 4,

= 2,

= 5.

:2:

a(r3 - 1)
; find S when a
5, r
3.
r - 1
19. A = 7r(R2
r2); find A when R
5'6,r = 14.
20. V = 7r(R2
r 2)h; find V when R = 0'9, r
0'2, h

18. S

21. V

~7rr2h;

22. P =

25. A

3, h

V
5 ; find (a) P when R

~d7rr2h; find

W when d

= v v+u;
find (a) f when
U
P (1 +

I~O)n; find A

50, V = 20

3, r

= 20,

when P

1'5.

35.

2R -

23. W =
24. f

find V when r

25

1000, r

0'2, R

20.

~!

= 11' h
U'

(b) V when P

(b) v when f
=

10, n

20,

= 25.

2.

2.4 The Distributive Law _ _ _ _ _ _ _ _ __


The distributive law states that for any numbers a, band c:

We can extend the distributive law. If, in place of a, we have,


(x

+ y)( b +

c)

say~ (x

+ y) then:

= (x + y) b + (x + y) c
= b(x + y) + c(x + y)
bx + by + cx + cy

2.5 Quadratic trinomials _ _ _ _ _ _ _ _ __


A monomial is an expression that contains only one term, e.g. 5x; X2; 2ab; mn; s.
A binomial is an expression that contains two terms added or subtracted, e.g. x + y;
3a - 2b; X2 + 1; mn
p; 3x - 5.
A trinomial is an expression that contains three terms added or subtracted, e.g. X2 - 5x
+ y - 4; 3a - 2b + c; 4X2 - 2xy + y2; m + n - p.

A quadratic trinomial is a trinomial of the form


ax2 + bx + c (a :;::. 0, b :;::. 0, c :;::. 0)

t L the constant term

the x term (or linear term in x)


the X2 term (or quadratic term in x)

+ 6;

2/BASIC ALGEBRA 39

The word quadratic means 'squared'.


The following trinomials are quadratic trinomials:
(a) X2 - 5x + 6 is a quadratic trinomial in x.
(b) 3a 2 - 2a + 1 is a quadratic trinomial in a.
(c) X4 + 4X2 + 1 is a quadratic trinomial in x 2.

+ m )(x + n)
m)x + (x + m)n
mx + nx + mn
(m + n)x + mn

Consider the expansion of (x


(x

m)(x

+
+
+

n) = (x
= X2
= X2

(x

If n

m)(x

n) = X2

(m

m)2 = X2

2mx

m2

2ab
a2 - 2ab

+
+

b2
b2

= m, we get
(x

n)x

mn

This is frequently written as:


(a + b)2
(a - b)2

= a2 +

EXAMPLE 4
(a) (x + 2)(x + 3)
(b) (x - 5)(x 2)
(c) (x - 3)(x + 4)
(d)
(x - 3)2

= X2 + (2 + 3)x + 2 x 3 = X2 + 5x + 6
X2 + (-5 + -2)x + -5 x -2 = X2 - 7x + 10
= X2 + (-3 + 4)x + -3 x 4 = X2 + X - 12
X2
6x + 9

EXAMPLE 5
(a) (3x - 2)(2x

(3x - 2) 2x + (3x - 2) x -3
6x 2 - 4x - 9x + 6
6x2 - 13x + 6

3)

Observe the pattern:

6x 2 is the product of (3x

) and (2x

6 is the product of (
- 2) and (
- 3)

-13x is obtained from (3x - 2)(2x - 3) i.e. from -4x - 9x

(b) (2x - 5)(2x

(2x - 5)2x + (2x - 5)3


4X2
lOx + 6x - 15
4X2 - 4x - 15

3)

Observe the pattern:

4X2 is the product of (2x

) and (2x

-15 is the product of (


- 5) and (
+ 3)

-4x is obtained from (2x - 5)(2x + 3) i.e. from -lOx


~

(c)

(2x - 5)2

= (2X)2 + 2

x 2x x -5

+ (-5)2

+ 25
+ b 2 where a

= 4X2 - 20x

i.e. (a - b)2

= a2 -

2ab

= 2x

and b

6x.

40 NEW SENIOR MATHS: TWO'UNIT COURSE

EXERCISES 2(d)
Write down the expansion of:
1.
4.
7.
10.
13.
16.
19.

22.
25.

28.
31.
34.
37.

(x + 5)(x
(a - 3)(a
(c + 4)(c
(x + 4)(x

2. (x
5. (a
8. (x
11. (x

1)

+ 4)
5)

+ 8)

2)(x +
5)(a 6)(x +
4)(x -

3. (x - 2)(x - 3)
6. (p - 7)(P - 3)
9. (t + 5)(t - 1)
12. (x - 3)(x + 9)
15. (y + 7)2
18. (3m + 7)(2m - 1)
21. (5x
3)(7x + 2)
24. (5z - 14)(2z + 5)
27. (3x + 2)(3x + 2)
30. (x + I)(2x - 1)
33. (2a - 5)2
36. (2x
3)2
39. (2z + 6)2

4)
4)
1)
8)

14. (x - 1)2

5)2

(p

+
+

(2x + 3)(x + 5)
(3y - 2)(4y + 3)
(2m - 1l)(3m - 4)
(x -14)(6x
1)
(2p
9)(2p + 9)
(4p
5)2
(5y - 3)2
(5p - 1)2

17. (3x - 4)(x


2)
(6x - 7)(x - 4)
(y + 5)(3y
2)
(2p
I)(4p + 3)
(3x + 2)(2x + 3)
(3x + 4)2
(4y + 9)2
(3p + 7)2

20.
23.
26.
29.
32.
35.
38.

The distributive law can be further extended.

EXAMPLE 6
Expand
(a) (x
(a) (x

(b) (x -

2)(x2

2)(X2

I)(x

5x
5x

+ 6),

+ 6)

2)(x

(b) (x

I)(x

2)(x

3)

= x(x 2

5x + 6) + 2(x 2 - 5x + 6)
3
= x
5x2 + 6x + 2X2
lOx + 12
3
x + (- 5 + 2)X2 + (6
IO)x + 12
x 3 3x 2 - 4x + 12

+ 3)

(x = (x
= (x = (x
x(x 2
x3 +
x3 +

I)[(x + 2)(x + 3)]


I)[(x + 2)x + (x + 2)3]
1)[x2 + 2x + 3x + 6]
1)(x2 + 5x + 6)
+ 5x + 6) - 1(x 2 + 5x + 6)
5x 2 + 6x
X2 - 5x - 6
4X2 + x
6

By repeated use of the distributive law, we can expand the expression (a + b)3.
(a

b)3

(a + b)[(a + b)(a + b)]


(a + b)[a 2 + 2ab + b 2]
a(a 2 + 2ab + b 2) + b(a 2 + 2ab
= a 3 + 2a 2b + ab 2 + a 2b + 2ab 2
= a 3 + 3a 2b + 3ab 2 + b 3

+
+

b 2)
b3

In place of b write -b, and so,

(a -

b)3 = a 3

3a 2b

3ab 2

b3

It is useful to remember the expansions of (a + b)3 and (a - b)3:


(a + b)3 =a 3 + 3a 2b
(a - b)3
a 3 - 3a 2b

+
+

3ab 2 + b 3
3ab 2 - b 3

2/BASIC ALGEBRA 41

EXAMPLE 7
(2x

\,

3y)3

(2X)3 + 3(2x)2(3y) + 3(2x)(3y)2


8x 3 + 36x2y + 54xy2 + 27y3

(3y)3

EXERCISES 2(e)
Expand the following:
1. (x - 3)(2x 2

+ 3x +
+ 2)

3. x(x - 2)(x
5. 2(x - I )(x - 2)(x
7. (2a + b)3

+ 5)(x2 + 4b)3
13. (4m + n)3
15. (a 1l3 + b 1l3 )3
9. (X2
11. (3a

2x

2. (3x 2 - 5x + 2)(2x
4)
4. (x - I)(x - I)(x
2)
6. (2x - 1)3
8. (2x - 3y)3
10. (x - 2)(x + 2)(x + 2)
12. (2p - Sq)3
14. (x + h)3
16. (Sm - 2n)3
18. (a 2 + b 2)3

I)
3)
3)

17. (x 2 - y2)3

2.6 Factors _ _ _ _ _ _ _ _ _ _ _ _ _ __
(a) A common factor
If each term of an expression has a common factor, we 'take it out' by using the
Distributive Law in reverse:
ab

ac

= a(b + c)

EXAMPLE 8
Factorize (i) 5x

lOy, (ii) 6mn

21mp

15mq, (iii) 2a 2 - Sa 3

It is first necessary to find the highest common factor (H.C.F.) of the terms of the
expression. To do this we express each term in its prime factors and then pick out what is
common in the terms.
(ii)
(i)
Sx = S x x
6mn = 2 x 3 x m x n

21 mp = 3 x 7 x m x p

lOy = 2 x 5 x Y
H.C.F. = 5
15 mq = 3 x 5 x m x q
H.C.F. = 3m
:. Sx + lOy = S(x + 2y)
:. 6mn

(iii)

+ 21mp

- 15mq

3m(2n

2a 2 = 2 x a x a
5a 3 = S x a x a x a

H.C.F. = a 2

:. 2a 2 - 5a 3 = a 2(2 - Sa)

Note: Check the result in each case by expanding the answer.

EXERCISES 2(f)
Factorize
1. 2a + 2b
4. 2X2 - 6xy

2. 20x
5. m 2n

5y
mn 2

3. a

+ a2

6. r2 - 2rs

+ 7p

Sq)

,I

42 NEW SENIOR MATHS: TWO UNIT COURSE

7. 5a 3 + 10a 2b
10. px - py - pz
13. 8a 2b - 3a 2b 2
16. 9a 2b - 6ab 2
19. mp + m2p - mp2

-15x - 10x2
5a 2b 3 - 18a 3b 2
9m 3n 2 - 5m 2n 3
54x2y2 - 54x3y3
20. 5a 2b - 5a 2b 2 5a 2b 3

8.
11.
14.
17.

9. ab

ae

ad

12. 3a 2b + 6a 2b 2 + 9abe
15. 8p2r2q + 3p2rq
18. 4a 2 + 4b 2 - 4e 2

(b) Grouping in pairs


If we expand (x + y)(b + e) we get bx + by + ex + ey. Consider the situation in reverse:
r---l

bx

by

r---,

ex

ey = b(x + y) + e(x
(x + y)(b + e)

y)

We could also group the first and third terms and the second and fourth terms thus:

Ex +

by

+ e:x. +

ey
I

x( b + e) + y( b + e)
= (x + y)(b + e) as before

EXAMPLE 9
Factorize (i) x(x + 2) + y(x + 2), (ij) a(b - 3e) - x(b - 3e).
(i)
x(x + 2) + y(x + 2)
(x + 2)(x + y)
(ii) a(b - 3e) - x(b - 3e) = (b - 3e)(a
x)

EXAMPLE 10
Factorize (i) pq + pr + sq + sr, (ij) X2 - xy + 4x - 4y, (iii) m 2 - mn - 2m + 2n.
p(q + r) + s(q + r)

= (q + r)(p + s)

(ii)
X2 - xy + 4x - 4y = x(x
y) + 4(x - y)
(x
y)(x + 4)

(iii) m 2 mn
2m + 2n = m(m
n) -. 2(m - n)
= (m - n)(m - 2)
(i)

pq

pr

sq

sr

Check the result by expanding the answer in each case.


Also try these factors by grouping the first and third terms and the second and fourth
terms.
Note: -2m + 2n = -2(m - n). Note carefully the effect of taking out a negative
common factor.

EXERCISES 2(g)
Factorize each of the following and check your result by expanding your answer.
a(x + 2) + b(x + 2)
5a(p - 6) - 3(p - 6)
4x(x - y) + 5(x y)
p(a + b) + q(a + b) - r(a
x2(2x - 1) + 4(2x
1)
11. ax + ay + bx + by

1.
3.
5.
7.
9.

2. x(p

b)

4.
6.
8.
10.
12.

1)

4(P - 1)

e(a - 3b) + d(a


3b)
3a(2b
3e) - m(2b - 3e)
x(y + z)
y(y + z) - z(y
a(x + 2y) - b(x + 2y)
ax + 4a + bx + 4b

z)

2/BASIC ALGEBRA 43

13. X2 - xy + xz - yz
15. a 2 - ab - ae + be

+ 3a 2b + ab 2 + 3b 3
+ ax - 3b - 3x
X2 - 2xy - xz + 2yz
3a - 3b + 2a 2 - 2ab
ay - 2a + y2 - 2y
2axe + 2e - axd - d
p2q - pq2 + 5p - 5q
ax 2 - axy + bxy - by2
2xy - xz - 8y + 4z
ab - 3a -4b + 12
a 3 + 3a 2b + ab 2 + 3b 3
XW - yw + xy - y2

17. a 3
19. ab

'21.
23.
25.
27.

29.
31.
33.
35.
37.
39.

+ 2xz + y + z
lOy - 25y2 + 4x - 10xy
ae - 2be - 2ad + 4bd
3xy - 6y + 7x - 14
a 3 - a 2b - ab + b 2
2mn + 2mp + pn2 + p2n
x 3 + 3x2 + 4x + 12
4ax + 6bx - 6a - 9b
m2p + m 2 + np + n
2a 3 - a 2 + 2a - 1
x2y + X2 + y + 1
2x - 6y - xy + 3y2
3m 2 - 3mn - m + n
2x 3 - 2X2 + 2x - 2

14. 2xy

16.
18.
20.
22.
24.
26.

28.
30.
32.
34.
36.
38.
40.

(c) Difference of two squares

I a2 We can give

thi~

b 2 = (a - b)(a

b)

result the following geometrical representation.

~a

----.I

~(a-b)~

,,

a-----+-l

14

,l-t'

(a+b)--.t

:Q'
I

co

\-+-(a-b)-.j

EXAMPLE 11
Factorize (i) a 2 - 25, (ii) 16m 2 - 1, (iii) 9x2 - 49y2, (iv)
(i) a 2 - 25
(ii) 16m 2 - 1
(iii) 9x 2 - 49y2

(iv)

~~

~2

~~ _ ~2

= a 2 52
= (a - 5)( a + 5)
= (4m)2 -}2 = (4m - 1)(4m + 1)
= (3X)2 - (7y)2 = (3x - 7y)(3x + 7y)

(~y

_~y

(~

~) (~

~)

EXAMPLE 12

Factorize (i) (x
(i) (x
(ii) a 3

+
-

1)2 -

1)2 - (y -

(y - 1)2

a 2b - ab 2

b3

1)2, (ii) a 3

a 2b - ab 2

+ b3, (iii) 75mp2 1)] [(x + 1) + (y - 1)]

= [(x + 1) - (y = (x - y + 2)(x + y)

= a 2(a - b) - b 2(a - b)

= (a 2 - b 2)(a - b)

= (a - b)(a + b)(a - b)

48mq2.

;
;

44 NEW SENIOR MATHS: TWO UNIT COURSE

= 3m(25p'J- - 16q2)

(iii) 75mp2 - 4S mq2

3m[(5p)2 - (4q)2]

= 3m(5p - 4q)(5p

4q)

Note: In (iii), 3m is common. Take it out and we are left with the difference of tw,o
squares.

EXERCISES 2(h)
Factorize
1. p2

q2

2. m 2
5. 9a 2 - 25
8. p2q2 - r2

4. 64 - m
7. a 2b 2 - c 2
a2

10. 25 - 1
2

13. (x
X2
16. 4

3. X2

1
4
14. X2 - y2z 2
11. p2

1)2 - 9
1

- 9

19. 992 - 1
22. 12a 3 - 3ab 2
25. 24x2 - 54y2
28. a 2 - (a - b)2
31. x 3 - x2y - 9x

17. (a

12. 36c 2

2)2

20. 523 2 - 4772


23. 3x2y - 27 y
26. Sa 2 - 2b 2
29. a 3 ab 2
32. x 3 + 3x 2 4x - 12

9y

34. a x - x

35. 4Sa 2 - 75b 2

37. 3x2y2 - 12y2

38. (l

40. 5
43. (p
46. x 3

5x

h)2 -

a2

41. b 2

15. (2a)2

(3b)2

18. X2 - (y

Z)2

45a 2x - 20x
(x + y)2
4
2X2 - O-OS
p2q
p2 - 16q
X2
36. 25 - y2

24.
27.
30.
33.

42. m 2n
45. 2m 2 n 2

ISn 4

48. a 3 + 2a 2 - ab 2 - 2b 2

Two further identities are of use:


b 2)
b 2)

You can determine the truth of these by expanding the right-hand side_

EXAMPLE 13
Factorize the following:
(a) x 3
(c) (x

S
2)3

(b) 27y3 + 64x 3


(d) x2y3 - Z2y 3 - X2W 3

+ y3

(a)

x3

(b)

27y 3 + 64x 3

= x3

16

n3

(d) Sum and difference of two cubes

a 3 + b 3 = (a + b)(a 2 - ab +
(a - b)(a 2 + ab +
a3 b 3

39. Z3 -

44. 64 - (9X)2
47. Sa 2x2 - ISa 2y2

2)2 - (p - 2)2
3x2
9x + 27

49d2

21. a 3b - ab 3

b2
a2

16

6. X2
036
9. 9x2 - 4y2

23

+ 2x + 4)
(3y)3 + (4X)3
= (3y + 4x)(9y2 - 12xy

= (x - 2)(X2

16x2)

Z2W 3

2/BASIC ALGEBRA 45

(c)

(x

+ 2)3 + y3

(X
= (X

(d)

x2y3 - Z2y3 - X2W3 + Z2 W3

+ 2 + y)[(x + 2)2 (X + 2)y + y2]


+ 2 + Y)(X2 + 4x + 4 xy - 2y + y2)

y3(X2 - Z2) - W3(X2 - Z2)

= (x 2 - Z2)(y3 - W3)
(x - z)(x

+ z)(y - W)(y2 + yw + w2)

EXERCISES 20)
Factorize:
2. Z3 + 1
4. 54r 3 + 16
6. (x + 5)3 + (x - 2)3
8. b 6 a6

1. y3 - 125
3. 8p 3 + 27
5. 216 - a 3
7. (2x + 3)3 - (x - 4)3

9. 64a 3 + 8b 3

+ y6
14. a 3m 3 + a 3n 3 b 3n 3
Hi. 8x5 - 72x 3 + X2
9
18j a 3 + (a - b)3

11. p7X4 - p4X7

12. x 6

8 27
13. a3 - b 3
15. 4x 5 - 9x3 - 4X2
17. (x + h)3 - x 3

19. (a

8a 3

b)3 - (a - b)3

21. (3x - y)3 + 8y 3


23. (2x + 1)3 + (2x - 1)3
25. l000p3 - q6
27. (2a - b)3 + (a - 2b)3

29. a 5b 4

10. "j'rrR3 - "j'rrr3

20. 125

b3

22. 54a 3b 3

b 3m 3

16c 3

24.
1
X)3
26. 8 - (2
28. 125c 3 - 343d 3
30. 2(x - y)3 + 54
m 3n 3 -

a 2b

(e) Quadratic trinomials


From the previous section 24 we can see that

Let us consider example 4 in reverse.

EXAMPLE 14
Factorize (a) X2 + 5x + 6, (b) X2

7x

+ 10, (c) X2 + X - 12, (d) X2

6x

+ 9

+ 5x + 6 = (x + m )(x + n)
We look for two numbers m and n whose product is 6 and whose sum is 5.
6 and 1 are factors of 6 but their sum is 7.
2 and 3 are factors of 6 and their sum is 5.
:. X2 + 5x + 6 = (x + 2)(x + 3)
We may set out the working as follows:
X X + ff + 2

(a) X2

+)'+3

and accept only the combination that gives 5x when we cross multiply.

46 NEW SENIOR MATHS: TWO UNIT COURSE

+ 10 = (x + m)(x + n)
We need two numbers m and n whose product is 10 and whose sum is -7.
5 and 2 are factors of 10 but their sum is +7.
-5 and -2 are factors of 10 and their sum is -7.
:. X2 - 7x + 10
(x - 5)(x
2)

(b) X2 - 7x

+ x 12 = (x + m)(x + n)
We need two numbers m and n whose product is -12 and whose sum is 1.
The factors of -12 are 4 and -3, -4 and 3, 6 and -2, -6 and 2, 12 and -1, -12 and 1.
Of these factors, 4 and -3 have a sum of 1.
:. X2 + x - 12 = (x + 4)(x - 3)
x
+4 -;4' +jI -,if +JZ -J,Z
x
-3 +y :2' +% )f' ;Yf
The only combination that gives x when we cross multiply is the first one.

(c) X2

><

(d) X2

6x

9 = (x

m)(x

n)

We need two numbers m and n whose product is 9 and whose sum is -6.
3 and 3 are factors of 9 but their sum is 6.
-3 and -3 are factors of 9 and their sum is -6.
.'. X2 - 6x + 9 = (x - 3)(x - 3) = (x - 3)2
Note:

For multiplication of two numbers,


(;) like signs give positive,
(ii) unlike signs give negative.

EXAMPLE 15
Factorize (a) X2 + 7x + 12, (b) X2
5x - 24.
(a) The factors of 12 are 12 and 1, -12 and -1, 6 and 2, -6 and -2, 4 and 3, -4 and -3.
Of these, 4 and 3 give a sum of 7.
:. X2 + 7x + 12 = (x + 4)(x + 3)
(b) The factors of -24 are -24 and 1, 24 and -1, 12 and -2, -12 and 2, 8 and -3, -8 and 3,
6 and -4, -6 and 4. Of these, -8 and 3 give a sum of -5.
:. X2 - 5x - 24
(x
8)(x + 3)
x
-24 +24 +12 -12 +8 -8 +6 -6
x
+1 -1 -2 +2 -3 +3 -4 +4
The only combination to give -5x when we cross multiply is
8

><.

xX-+3

EXERCISES 2(j)
Factorize the following quadratic trinomials.
1. X2 + 4x + 3
4. X2 + 6x + 5
7. m 2 + 9m + 20
10. p2 + 19p + 18
13. X2 + 8x + 12
16. X2 - 8x + 12
19. m 2
21m + 20
22. p2 - 2p - 15

2. X2 + lOx + 21
5. a 2 + 7a + 6
8. p2 + 9p + 18
11. X2 + 7x + 12
14. X2 - 7x + 12
17. m 2 - 9 m + 20
20. X2 - 14x + 13
23. pi + 14p
15

3. X2 +
6. a 2 +
9. p2 +
12. X2 +
15. X2 18. m 2
21. p2 +
24. p2 -

llx
12a

+ 24
+ 32

lip + 18
13x + 12
13x + 12
12m + 20
2p
15
14p - 15

2/BASIC ALGEBRA 47

25. X2 28. a 2 31. X2 +


34. X2 37. X2 40. y2 -

26. X2 29. X2 32. X2 35. X2 -

2x - 35
4a - 12
6x - 72
X - 42

11 x - 42

38.

6y - 55

41. X2

X2

+
+

3x - 10
7x + 6
21x - 72
19x - 42
6x - 7
14x + 33

27. X2 + 17x + 72
30. X2 - X - 72
33. a2 + 13a + 30
36. X2 + 19x - 42
39. y2 + 6y - 55
42. X2 - 14x + 33

So far we have considered quadratic trinomials whose quadratic term has a coefficient
of 1. We consider now the situations where the coefficient of the quadratic term is not 1.

EXAMPLE 16
Factorize 6x2 - 13x

The factors of 6x 2 are either 6x and x or 3x and 2x. Since the constant term, 6, is positive
and the x term is negative, the factors of 6 will both be negative. Possible factors of 6 are
-6 and -lor -3 and -2. By 'trial and error' we combine the factors of 6x 2 and the factors of
6 until we obtain -13x as the x term:
(i)
(ii)
(iii)
(iv)
(v)

(vi)
(vii)
(viii)

Corresponding x term:

Possible factors:
- I)(x - 6)

(6x
(6x
(6x
(6x
(3x
(3x
(3x
(3x

6)(x - I)
3)(x - 2)
2)(x - 3)
1)(2x - 6)
6)(2x -I)
3)(2x - 2)
2)(2x - 3)

-x -6x -3x -2x -2x -12x -6x -4x -

36x =
6x =
12x =
18x =
18x=
3x =
6x =
9x =

-37x
-12x
-15x
-20x
-20x
-15x
-12x
-13x

Only the last possibility satisfies,

:. 6x2 - 13x + 6 = (3x - 2)(2x - 3)

Check that this is correct by expanding (3x - 2)(2x - 3).

Note: Since 6x 2 - 13x + 6 has no common factor, then neither of its binomial factors
can have a common factor. Thus in the above factor combinations (ii), (iii), (iv), (v), (vi)
and (vii) can be rejected because each contains a common factor of 6, 3 or 2.
Instead of writing down each of the eight possible combinations above, (i) to (iv) can be
written
6x
-I -6 -3 -2
x ><-6 -I -2 -3
and (v) to (viii) can be written
3x
-I -6 -3 -2
2x><-6 -I -2 -3
and we reject all of the combinations except the one that gives -13x when we cross
multiply. The only satisfactory combination is
3x
-2
2x'><-3

which gives (2x)(-2)

(3x)(-3) = -13x when we cross multiply.

EXAMPLE 17
Factorize 4X2 - 4x - 15.
The factors of 4X2 are either 4x and x or 2x and 2x. The factors of -15 are -15 and I, 15
and -I, -5 and 3, or 5 and -3.

48 NEW SENIORMATHS: TWO UNIT COURSE

Setting these down in cross multiplication style, we get


4x><-15 15 -5 5
or
4X>< 1 -1 3-3
x
1 -1 3 -3
x
-15 15 -5 5
and
or
2X><-15 15 -5 5
1 -1 3-3
2x
1 -1 3-3
2x
-15 15 -5 5
The only satisfactory combination that produces -4x is
2x
-5
2x><3
:. 4X2 - 4x - 15 = (2x
5)(2x + 3)
Check that the result is correct by expanding (2x - 5)(2x

2Xx

3).

EXAMPLE 18
Factorize 3x2 + 8x - 16.
The only factors of 3x 2 are 3x and x. Since the constant term is negative, the factors of -16
will be of opposite sign. Possible factors are -16 and 1, 16 and -1, 8 and -2, -8 and 2, 4 and
-4, -4 and 4.
3xX-16 1 16 -1 8 -2 -8 2 4-4
x
1 -16 -1 16 -2 8 2 -8 -4 4
The only possible combination is the last one.
3x 2 + 8x - 16 = (3x - 4)(x + 4)
Check by expanding (3x - 4)(x + 4).

EXERCISES 2(k)
Factorize
1. 2X2 + 3x + 1
4. 4a 2 + 13a + 3
7. 8x 2 - 14x + 3
10. 8x 2 + 14x + 5
13. 3x 2 - 17x + 10
16. 10x2 - llx - 8
19. 9x 2
12x + 4
22. 15m 2 + 17m
18
25. 12y2 + 14y
6
28. 6x 2
19x + 14
31. 6p2 + 25p + 21
34. 24x2 - 59x + 36
37.9x2 +9x
10.
40. 5x 2 - 2x - 3
43. 4X2 + 12x + 9

2.
5.
8.
11.
14.
17.
20.
23.
26.
29.
32.
35.
38.
41.
44.

3x 2 +
4X2 +
2X2 6x 2 +
6a 2 -

2X2 +
2X2 10a 2 6x 2 6x 2
10a 2 15x 2
2X2 3p2 9x 2 +

11 x - 4
5x + 1
9x - 5
17x + 12
13a - 63
3x
2
9x + 10
41a
18
25x + 14
20x + 14
11a
6
19x + 6
9x + 4
7p + 2
30x + 25

3.
6.
9.
12.
15.
18.
21.
24.
27.
30.
33.
36.
39.
42.
45.

2X2 + 7x + 6
3a 2 - 5a + 2
l3c 2 - 7 c - 6
3x 2 - l3x + 4
3x 2 - 11x - 4
4X2 - 12x + 9
6x 2 - 85x + 14
2y2 - 4y - 6
6x 2 29x + 28
8x 2 + 2x - 3
12y2 + 28y
5
3x 2 2x
10p2 + 11p + 3
8x 2 - 6x - 9
4x 2 28x + 49

To factorize the expressions in the next set of exercises, it will be necessary to use one or
more of the techniques we have studied:
(i) something common in each term,
(ii) grouping and using the Distributive Law in reverse,
(iii) difference of two squares,
(iv) sum and difference of two cubes,
(v) quadratic trinomials

2/BASIC ALGEBRA 49

EXERCISES 2(1)
Factorize completely
2. 2a 3 - 8a

4. x 2 8x
6. X2 - 8x - 9
8. 5x 3y - 20xy3
10. 10x2 + 9x - 1

1. x 2 3x
3. 3x2 + 9x
5. X2 - 9
7. 3x2y - -12y3
9. 1 - (b - c)2

11. (a + b)2 b 2

12. 6x 3 - 48
14. a(m + n) - b(m + n)
16. 3a 3 + 24a 2 + 21a
18. X2 - 3x - 10
20. ab 2 + abc + abd
22. X2 + x
24. x(y - z) + y(y - z)

13. a 2 - a - 42
15. 2x 3 + 14x2 - 16x
17. (x + 2y)2 - 4
19. a 3 - 1
21. X2 - 36y2
23. X2 + x - 12
25. 4X2 - 28x - 480
27. 6y3 + 3y2 - 3y
29. 15a 2 - 60

31. 5a 2x - 125x
33. 5t 3 + 5t 2 - 360t
35. 5 - 125a 2
37. m 2 mn + 6m - 6n
39. X2(X + 3) - 4(x + 3)
41. 4 - (x + 1)2
43. X2 - 2x + ax - 2a
45. 2axc - axd + 2c - d

a3

47. 8

26. bx2 - 14bxy + 49by2


28. 6y 3 + 26y2 + 8y
30. 9mn - 25m 3n 3
32. (x + y)2 - (x - y)2
34. X2 - 12xy + 20y2
36. ax + bx + ay + by
38. (x + 3y)(x - 3y) - 3z(x
40. mx2 xy + Iy - mix
42. 6x3 - 34x2 - 56x
44. X2 - lOx - 144
46. p3 + 125
48. (x

h)3

3y)

I, ,

Write an algebraic expression in factorized form for the shaded area in each of the figures
in questions 49. to 57.
49.

52.......1 - - - 4,

- - - - - I....
.

55.

c
a

.-~-.~~.

,-_._--

-~- .

~----------

----------------50 NEW SENIOR MATHS: TWO UNIT COURSE

2.7 Simplification of fractions _ _ _ _ _ _ __


The procedure in simplifying fractions is to
1. Factorize \humerator and denominator,
2. Cancel any common factors.

EXAMPLE 19

+ 6 (")
+ 2' II

' l'f (') 9x


SImp
1 y I 3x

15a 2 - 5ab () 9x2 y2


10ab
,III 6xy _ 2y2'

+ 6 = 3(lx---I-2) 3

3x + 2
(lx-rt)

"")
15a2
5ab
_ 5a(3a - b)
(
II
10ab
.1Qtd1

(i) 9x

3a - b

""') 9x2 - y2
( III
6xy - 2y2

Q.x---Y)(3x

y)

2y(lx----y)

3x

2Y

EXAMPLE 20
' l'f X2 - 5x
_
SImp 1 y
X2 - 5x

+6
X2

x
3x

X X2 - X -

X2

3x

_ (x.---2)~) x
x~
2 - (~)~)
(;v-2)(x + 1)
x
x + 1

EXERCISES 2(m)
Simplify
1. 8a - 4b
8x2 - 4xy
4.
8xy

3a - 5b
5ab
p2q _ pq2
~~--=---='-pq
X2 _ y2
(x + y)2
15a2 - 5ab
3ab _ b2
a2 + ab
ab + b2

7. 3a2
10.
13.
16.
19.

X2 - 6x + 8
22. x 2 - x- 2
25 X2 + 4x + 4
X2 - 3x - 10
3x2 x2y
28. ----=- x---=--
xy
3xy - y2

2.

15x

15

lOy

3. 14x - 7y

12ab - 6b 2
5.
9ab

2x - Y
8x + 2
6
4x + 1

2
m +m
8.

9. mn

12. 2rs

12r
r2 + rs

11. x ;x xy

X2 15. X2 +
X2 18.
X2

14 k + k
k + 1
4X2 - 4xy
17. x 2 - y 2
a2 - b 2
20. a2 + ab

9
3x
7x + 6
- 36

21. - - -

+ 3x + 2
X2 - 4
26 4x3y
16xy
X2 + 2x - 8
23

n 2

X2

24. --:------::

29.

m2 + m - 2
m

27. - - - -
12a

5
x -:-----=

2/BASIC ALGEBRA 51

2a 2
2a 2 - 2ab
3ab
ab _ b 2 X
12x2 - 4x . 10x2y
32.
_ x .. 5x2y2
2
2 _3_a_b_+_2_b_
34 (a + 2b )(a - b) x _a_
2
2
2
ab - b

a - 4b

30.

36. x

2x

38. x 3

40.

m2 - 9
35. m2 - m - 12

m2

3m

37 8x2 + 4x + 2

8x 3 - 1
(x + h)3 x 3

+ y3
2y

15x2 - 5xy . 3x - Y
10xy
..
2y
2
x 2x - 3
X2 - 25
33. x 2 _ 4x - 5 x (x - 3)(x + 5)

31.

X x

39.

-----"<---:----"--

----~---'~

2.8 Addition and subtraction of fractions _ __


EXAMPLE 21
'
(.)
3
. I
Express as a smg
e f
raction
I 5

2
- 7'

('0)
3x
II 5

- 2x 7+y

1,III
(...) 3

X
) 2x ..:.- y
( IV
3
- -6

Before adding or subtracting fractions, each fraction must have the same denominator.
This can be done by getting the lowest common multiple (L.C.M.) of the denominators.
The L.C.M. of 5 and 7 is 35.

2
3
(i)
7
5
21
10
35

21 - 10

21x -

The L.C.Mo of 5x and 7x is 35x.

2
3
(iii)
+

21

21x

11

21

2x -

=15

35

= 35x

3x

(ii)

(iv)

10

+ 5

lOx

+5

llx

2x - y

2(2x - y)

+ 10

= 4x -

2y - x

31

3x
_ 3(x - y)

EXERCISES 2(n)
Express each of the following as a single fraction.
1.

x - x

2.

3x

8" + 2"

3.

4a

a
6

52 NEW SENIOR MATHS: TWO UNIT COURSE

7. 3x

2 _ x

2a + b
9

10. a - 2b

14. ab 5
17. a2b -

yz

x - 3y

+l_x+
9 !
2
4

12. 1 _ ~

~ + ~

19 a + 1

2x - y

6.

3m - 2n + m + n
10

xy

2 _ a

11. 3(a: b) _ a

13. ~ + 1
16.

5 . a

4. 2 +

1
20 x+

a - 4
2a

3x

15. m _ n

18. ab + be

21. 1

+3

2 _

EXAMPLE 22
'
(e)
3
1
(e e)
. I
Express as a smg
e f
ractlon
I -2--4 + --2' II
x-

x-

x-y

x+

4 = (x
2)(x + 2), the lowest common multiple (LeCeMe) of X2
x - 2 is X2 - 4.

3
1
3
x + 2

=
+ -c------:c-:-:---_,:_:_
--+
X2
4
x
(x - 2)(x + 2)
(x - 2)(x + 2)
3 + x + 2

(i) Since X2 -

(x - 2)(x
(x
(ii) L.C.M. of x

1
x - y

x + 5

2)(x + 2)

+ y is (x - y)(x + y).
x+y
x-y
(x - y)(x + y)
(x - y)(x + y)
x + y
(x
y)
(x
y)(x + y)
x+y-x+y
(x - y)(x + y)
2y

(x - y)(x + y)

y and x

1
x

+ 2)

EXAMPLE 23
Express as a single fraction (i)

X2

2x - X2

4' (ii) X2

5x

+ 2x = x(x + 2)

and
X2 - 4
(x - 2)(x + 2),

2)e

the L.C.M. of X2 + 2x and X2 - 4 is x(x + 2)(x


3
2
3(x - 2)
x(x
.. x(x + 2)
(x - 2)(x + 2)
x(x + 2)(x
2)
3(x - 2) - 2x

x(x + 2)(x - 2)

X2 +

2x

(i) Since X2

2x

2)(x - 2)

4 and

2/BASIC ALGEBRA 53

(ii) Since x 2 - 5x +,6 = (x - 2)(x - 3)


and x 2 + 2x - 8 = (x - 2)(x + 4),
the L:C.M. of X2 - 5x + 6 and X2 + 2x - 8 is (x - 2)(x - 3,)(x

1
X2

1
2x - 8

1
(x

2)(x - 3)
x+4
(x -' 2)(x - 3)(x

4).

1
(x - 2)(x

4)

x-3
(x - 2)(x - 3)(x

+ 4)

+ 4)

4 - (x - 3)

(x
2)(x
3)(x + 4)

(x - 2)(x
3)(x + 4)

EXERCISES 2(0)
Write down the lowest common multiple (L.C.M.) for questions 1. to 16.
1. x
3 and x + 3
3. x andx
2
5. X2 - 4x and x - 4
7. X2 - 1 and x + 1
9. x + 2 and X2 + 4x + 4
11. X2 - 4 and X2 + 3x + 2
13. X2
4x + 4 and X2 - 2x
15. x - 5, x + 5 and X2 - 25

2. x - 5 and X2 - 25
4. 2x
4 and 3x - 6
6. X2 - 4x and X2 - 16
8. X2
9 and 2x - 6
10. x
y, x + y and X2 - y2
12. xy, x - y and x3y
xy3
14. x, x + 3 and X2 - 9
16. X2

y2, X2

xy, xy

y2

Express each of the following as a single fraction.


1

a- + a+
19. _x_ +
17. - - b

x-y
3a
b

21. a
2
- b2
x
23. X 2 - Y 2
1
25.

27.
29.

18.

+ a--b
X2

y
Y

+x-+-10

7a
31. 3a - 4 -

22.

1
x+

24. (X

3
-

26. _3_
28. 2a

+ x

2)2

+
6

1
x+

a
a2 - 9

6
30. 3x - 2
32. ---,:-=--- xy

X
X2 - 16

x+y

x-2
x-y

33. x

x+y

2
x+y

20. _x_

x-y

3
x-y

34.

x+

+ 1
x

---:---4---:

4
36'~4+ + a 2 +a +
x
5a + 6
x
I

38
3a + 1 _ 3a - 1
37. x + 1
3a - 1
3a + 1
x

35. a

39.

41. - -

3
2

CHAPTER 3

Equations and
Inequations
3.1 Linear equations in one variable _ _ _ _ __
An identity is a statement of equality that is true for all values of the variable for which
both sides are defined, e.g.
X2 - 4 = (x - 2)(x + 2)
5(x + 3)
5x + 15
are true statements for all values of x. Test that they are true for, say, x
7.
A conditional equation, or simply an equation, is a statement of equality that is true for
only certain values of the variable, e.g.
x + 5 8 is true only when x = 3.
A linear equation in one variable is an equation in which the pronumeral is of first
degree, e.g.
3x + 2 = 14,
and there is only one value of x, namely x = 4, for which this equation is true.
If the pronumeral occurs more than once in an equation, e.g.
5x - 3 = 2x + 9,
it is necessary to get the terms containing the pronumeral together on one side-either left
hand side or right-hand side, whichever may be more convenient.

EXAMPLE 1
Solve the equations (a) 5x - 3 = 2x
(a)

5x - 3 = 2x + 9
5x - 2x - 3 = 9
i.e. 3x - 3 = 9

3x = 12
x = 4
Test: Put x = 4 in (1).

L.H.S. = 17

9, (b) 5(x - 3) - 3(x - 2)

19.

.. . . . . . . . . . . . . . . . . . . . . . . . . . . . . . . . . . . . . . . . . . . . .. (1)
(Subtract 2x from both sides)
(Add 3 to both sides)

(Divide both sides by 3)

= R.H.S.

. ............................................ (1)

(b) 5(x - 3) - 3(x - 2) = 19


5x - 15 - 3x + 6 = 19 (Using Distributive Law)

2x - 9 = 19

3/EQUATIONS AND INEQUATIONS 55

2x = 28 (Add 9 to both sides)

x = 14 (Divide both sides by 2)

14 in (1).

Test: Put x =
L.H.S. = 5 x 11 - 3 x 12

55 - 36

19

R.H.S.

EXERCISES 3(a)
Solve the following equations and test your solutions.
2. 4(2x - 7) = 3x - 5
4. x - 7 = 3 - x

1. 5x
3x + 9
3. 3(x + 3)
4(9 - 3x)
5. 5(2a + 1) = 6(a - 5)
7. 4 - 3x ~ x + 9
9. 5(2x - 1) = 2(x - 5) - 3
11. 5 - 9y = 10 - lly
13. 5a
7 = 4(2a + 3)
15. m + 8 = 5(m - 1) - 2m
17. 18
3(a - 2)
2(a + 2)
19. 8(x + 2)
3(x + 5) = 2(x - 2)
21. 5(c - 7) = 3(3c + 8)
23. 7(m
2) = 5(2m - 1)

6. 3(5x - 1) - 2(4 - x) = 6
8. 3x - 2 = 5x
10. 4(3a + 2) - 6(3 - a) = 8
12. 4(3m
1) = 11 + 2m
14. 2c - (4 - c) = 5 - c
16. 4(x + 5) - (x
1) = 9
18. 3(8a - 2) - 3(2a + 4)
0
20. 3(2x + 5) = 4 - (3x - 2)
22. 6(2x - 3) = 8 - 2(3x + 1)
24. a + 6 = 3
a

If an equation contains fractions, it is advisable to multiply both sides of the equation by


the lowest common mUltiple of the denominators as the first step.

EXAMPLE 2
.

Solve the equatIons (a)

2x

+ 1

2x - 3

3' (b) x

(c) 3a - 2
1
2a

3a
2a

+
+

1
3

(a) Since the lowest common multiple of 4, 6 and 3 is 12, multiply both sides of the equation

by 12.
:. 3(2x

1) - 2(2x - ~)

6x

3 - 4x
2x

+6
+9

= 28

= 28 (Using Distributive Law)

= 28

2x = 19 (Subtract 9 from both sides)

x = 9~ (Divide both sides by 2)

(b) Since the lowest common multiple of x - 4 and x - 2 is (x - 4)(x - 2), multiply both
sides of the equation by (x
4)(x - 2).

3
2

x _ 4 x (x - 4)(x - 2) = x _
x (x - 4)(x - 2)
:. 3(x - 2)
2(x - 4)

3x - 6 = 2x - 8

3x - 2x
6
8

x =-2

(c) Since the lowest common multiple of 2a - 1 and 2a


both sides of the equation by (2a - 1)(2a + 3)
:. (3a - 2)(2a + 3) = (3a + 1)(2a - 1)
6a 2

5a - 6 = 6a 2
6a = 5

a=6

a-I

+ 3 is (2a

- 1)(2a

+ 3), multiply

56 NEW SENIOR MATHS: TWO UNIT COURSE

EXERCISES 3(b)
Solve the following equations and test your solutions.

1 2x+5_x+2

+
x+32x-l

3. -3-

5.

5x - 3

2. - 2 - =X+ 2

x + 1

2x - 1 _ 5 x
3

6.

+ 1=6-

2J

8.

7.
9. x

+4

11. x - 4

3 - 4x

=5

15. x

+ 2 = -

14.

5
23. y

x-7

~+

31. _1_ + _1_

x + 2

1 + _1_
-t
l+t

x + 1
6

= ---'--:---'

=2

3
p+

=2

x+

24.

25 x - 2 = ~
x+
5
3
2
27.
x
x+2
1
1
29. - 1 + ----x-

33 .

22

2
y-

x + 1

.3 +

2 x+4
x-5
x+
3x - 2

20 3x - 1
5x + 1 5x + 2

2m

=m +

18. x

y+2 y+4
19. 2x + 3 = 2x - 5
3x + 4
m

2)

5
16 a +

17. Y + 3

21. 2m

5(a

10. 3(x 2)
5

12.

x+2

13.

4. 2x - 1 = 3x + 1

26.

x
5

y+

28. ~ _
a

=
X2

30.

+ 3x + 2

2t

32

1
a(a

+ _1_
x - 3

_I_

. x
x + 3
34. _2_ _
3
x+5
x-

+ 2)
1

(x + 2)(x - 3)
1

X2 -

25

3.2 Inequations _ _ _ _ _ _ _ _ _ _ _ _ __
For an inequation the = sign is replaced by the inequality signs>, which means greater
than, or <, which means less than.
R stands for the set of real numbers and J stands for the set of integers (whole numbers).
x E R means that x can be any real number and can be represented by any point on the
number line. Thus x > 4, x E R means that x can be any number provided that it is greater
than 4; for example, x can be 52, 6'875,
8, 9~ and, of course, very many more.
i'

3/EQUATIONS AND INEQUATIONS 57

x E J means that x can only be a whole number. Thus x ~ -1, x E J means that x can
only be -1, 0, 1,2, 3, ... and on the number line can only be represented by these points.
It cannot be 5'2, ..j2, 6~, etc.
Example

Meaning

Sign

>

is greater than ...

X>4,XER

is greater than or
equal to ...

x~-l,EJ

4 5 6 1

..

-1

1 2 3

<

is less than ...

X< 2, XER

0----.--
x
-3-2-1 0 1 2

is less than or
equal to ...

x ~ 2, x ER

-3-2-1

<

Since
1. 10
2. 10
3. 10
4. 10
10
5. "2

<

1 <x < 5, XER


greater than ... but x is greater than
-0 I I I 0
less than ...
1 but less than 5
1 2 3 4 5
or x is between 1 and 5

10 > 6 it follows
+2> 6+2
- 2 > 6
2
x 2 > 6 x 2
x -2 < 6 x -2
6

> 2:

6. 10 <

_~

0 1 2

..

..x

that
i.e. 12 > 8 (Add 2 to both sides)
i.e. 8 > 4 (Subtract 2 from both sides)
i.e. 20 > 12 (Multiply both sides by 2)
i.e. -20 < -12 (Multiply both sides by -2)
i.e. 5 > 3 (Divide both sides by 2)
i.e. -5 < -3 (Divide both sides by -2)

You will notice in 4. and 6. that,

if both sides of an inequality are multiplied by or divided by a negative


number, the direction of the inequality is altered.
Solving inequations makes use of the following axioms of inequality for real numbers a,
band c.

If
a>
1. a + c >
2. a - c >
3.
ac >
4.
ac <
5.

6.

b, then
b + c
b - c
bc if c > 0
bc if c < 0
a
bif c> 0
->-1
c c
a < b if c < 0

Similar axioms also apply if a < b.


Note: If a > b, then a is to the right of b on the number line.
If a < b, then a is to the left of b on the number line.

I I

58 NEW SENIOR MATHS: TWO UNIT COURSE

EXAMPLE 3
~

Solve 2x + 3

9 for (a) x R, (b) x J and x


2x + 3
2x
x

(a)

o
x

(b)

~
~

8.

9
6 (Subtract 3 from both sides)
3 (Divide both sides by 2)

3 and x R

o
~

Since x J and x

8, x 13, 4, 5, 6, 7, 81

EXAMPLE 4

xfor(a)xR
:E;X~ 10andx
~x~ 10andx

3x> 19

Solve 3

3 - 3x > 19 - x
3 - 2x > 19
-2x> 16
x < 8

(Add x to both sides)


(Subtract 3 from both sides)
(Divide both sides by -2)

Answers to (a), (b) and (c) can be represented on the number line thus:
(a)

4'

-1

x < 8 and x

7'

R. Note that 8 is excluded.

2 3 4

(b)

-1

< 8 and x

R. Note that 2 is included but 8 is excluded.

x 12, 3, 4, 5, 6, 7)

EXAMPLE 5
Solve x

x + 1 for (a) x R, (b) x J.


x-I
3(x
1)
3x - 3

x + 1
5(x + 1)
5x + 5

-2x-3~5

-2x

-4

(Multiply both sides by 15)


(Using Distributive Law)
(Subtract 5x from both sides)
(Add 3 to both sides)
(Divide both sides by -2)

"3/EQUATIONS AND INEQUATIONS 59

-3

(a)

-4

-2

-1

-4 and x E R

...

-3 0

(b)

-4

-1

x E I -4, -3, -2, -1, 0, 1, ... 1

EXAMPLE 6
Solve -5 < 2x - 3

7 for x E R.
-5 < 2x - 3
:-2 <
2x
-1 <
x

~
~
~

10
5

(Add 3)

(Divide by 2)

__

------~O~--~;----~--~;----~--~-- .--~.~

-1

Note that -1 is excluded.

EXERCISES 3(c)
Solve each of the following for x E R unless otherwise stated and graph the solution set on
the number line.
1. 5x - 2 ~ 13
3. 2x + 5 < -5
5. 3x > 2x + 12
7. 7x < 3(2x + 1)

9.

x x
3" - 4 > 1

11. 3x - 1

2. 2x - 2 < 0
4. 6x + 2 ~ -10
6. 3(x + 1) ~ 9
8. 2(x - 6) ~ 8
3x
x
3
10. 2 - 3" < 2
12. 2x

< 3x + 10

13. x - I > -1

14. x + 3 < 7

15. x ..;.. 1 > 5 - x, x E {2, 3, 4, 51


17 3x 2x
-2
. 5
3 >

16. 4x < x + 15, x E {3, 4, 5, 61

19.

x - 5

> 5x
- -- -3

21. -5 < x + 4 < 1


2x - 1
23. -3 ~
< 3

18.

7x

~ 3

4x

20. 5x - 3 < x + 2
22. 22

5x - 3

32

24. x - 2 > -2 and x - 3

25. If 5 is subtracted from a certain positive integer, the result is greater than 5 but less
than 12. What values can the integer take?
26. If a certain number is divided by 2, the result is greater than 4 but less than 8. What
values can the number take?
27. The sum of two consecutive positive integers is no more than 35. What are their
possible values?
28. A committee is to have 3 more women than men and the number on the committee
must be at least 7 but no more than 15. What is the possible number of women on the
committee?

60 NEW SENIOR MATHS: TWO UNIT COURSE

29. The result obtained when a certain number added to 5 is divided by 5 is not more than
the result when the same number added to 13 is divided by 9. What is the largest value
the number can be?
30. The length of the base of an isosceles triangle is an integer and is 4 cm less than the
sum of the two equal sides. The perimeter is an integer less than 80 cm. What are the
possible lengths of the base?
31. The sum of three consecutive integers is greater than 7 and less than 25. Find all
possible values of the smallest of these integers.
32. The lengths of the sides of a triangle are 8 cm, 10 cm and x cm. What are the possible
values of x?

3.3 Square root and absolute value _ _ _ _ __


If a ~ 0, then -Jii is a non-negative number such that (-J{i)2 = a, and -J{i is called the
positive square root of a. (Non-negative square root is probably more precise since it
includes the case a = 0, but custom tends to retain the former term.)
The equation X2 = 9 has two solutions, namely the positive and negative square roots of
9. These are given by -J9 = 3 and --J9 = -3.
The important point to remember is that, when -J{i occurs in an expression, a positive
value (or zero) must be given to it.
What meaning can be given to
If x
+2, then .JX2
.J (+2)2 =.J4 +2.
If x
-2, then .JX2
.J (-2)2 = .J4 = +2.
If x
0, then R
=.JOi =.JO = O.
From this, we infer that

xijx>O
-x ij x < 0
=Oijx=O
Note that the right-hand side always consists of a non-negative number. Regard the
sign as meaning 'the opposite sign of'. That is, if x is a negative number, then -x, being
the opposite sign of x, is a positive number.
This definition frequently causes confusion because of the property it demonstrates
about -x, that is, negative x is not necessarily negative.
The absolute value (modulus) of a number x E R, written lxi, is given by the non
negative number that defines its magnitude.
0 = 1-01.
Thus 131 = 3 = 1-31 and 101
Hence

Ixl = x ijx > 0


= -x if x < 0
=Oijx
0
We observe that

Ixl

Since x is a real number, it can be represented by a point on the number line. It is useful,
then, to regard Ixl as the distance of a point x from the origin and since distance is
measured by a positive number, then Ixl is positive for all x
O.

'*

..

3/EQUATIONS AND INEQUATIONS 61

The points x = 3 and x = -3 are both 3 units from the origin and hence 131 = 1-31 = 3.
More generally, Ix - y I may be considered as the distance between two points x and y
on the number line and hence Ix
y I is a positive number if x > y or if x < y.

yl

Ix -

e.g. 15 - 31

x - y if x > y

y - x if x < y

o if x y
13 - 51
2

Two important theorems should be considered:


(a)
(b)

Ixllyl
Ixl + Iyl
Ixl + Iyl

Ixyl
Ix
and Ix

+ yl
+ yl

(the triangle inequality)


when and only when x and yare either
both zero or both have the same sign.

These theorems can be verified by giving x and y specific values. Test them for, say,
x = -7 and y = 3.

EXAMPLE 7
Demonstrate on a number line the values of x for which:
(a) Ixl > 1
(b) Ixl ~ 2

Ixl >
Ixl ~

1, then x > +1 or x < -1.


2, then x ~ +2 and x ~ -2 i.e. -2 ~ x ~ 2.
The required sets for (a) and (b) are shown in the following diagram:
(a) If
(b) If

--------~O~--~i-<O~-------

-1

-2

0 +1

EXAMPLE 8
Solve for x (a) 12x - 11

(a) 2x -

1 = 3 or 2x - 1 = -3
x
2 or
x-I

(b) 3x

(b) 13x - 21

1 or 3x - 2 = -1
1
1 or
x = 3

Find solution sets of the following inequations:


(c) 12x - 11 ~ 3
(d) 13x - 21 < 1
(c) If 12x -

11 ~ 3, then 2x

i.e.,
(d) If 13x - 21 <
i.e"

1 ~ 3 or 2x - 1 ~ -3
2x ~ 4 or 2x ~ -2

x ~ 2 or x ~ -1.

1, then 3x - 2 < 1 and 3x - 2 > -1


3x < 3 and 3x > 1
1
x < 1 and x > 3'

+2

62 NEW SENIOR MATHS: TWO UNIT COURSE

EXAMPLE 9
Give meanings to
(a) 12x - 31,

(b)

-J(y -

12x - 31

(a)

2)2,

(c) -J(2y - X)2

2x - 3 if 2x - 3

> 0 i.e. if x > ~

3 - 2x if 2x - 3 < 0 i.e. if x <

o if 2x
(b) Similarly

-J(Y -

2)2

=
-J(2y - X)2

"
I.e. f
1 x

= 2:3

= Iy =

(c)

- 3

21
y - 2 if y
2
y if y
o if y = 2

> 2
< 2

12y - xl

= 2y
= x

- x if y
2yify

o if y =

lx
< lx
>

lx

_lL

Before proceeding, it is important to revise the fundamental fact that an algebraic


expression is not defined if its denominator is zero. Thus, for example,
undefined when x
-=1= 0, 1.

= 0

x~:

is

or 1. We say that the given expression is defined for all

EXAMPLE 10
Simplify the following expressions:

(a)

(a)

I~I for x -=1= 0;

~
= ~x

(b)

= 1 if x

for x

-=1=

-2,2;

(c)

-J(x - 1)2

1 whenxlies between 0 and 1;


x

>0

-x = -1 if x < 0

x
(b)

Ix X2 -

21
4

(x - 2)
(x - 2)(x + 2)

(2 - x)

(x - 2)(x + 2)

, 1

if x

>

if x < 2, x

-=1=

-2

3/EQUATIONS AND INEQUATIONS 63

-1 since 0 < x < 1

Y(x - 1)2
1 - x
()
c
x-I =x-l

Failure to interpret ,.,;X2 correctly causes algebraic chaos akin to that created by division
by zero. Consider the following 'proof' that a hippopotamus weighs the same as an ant.
Denote the weights respectively by h and a, and the sum of the weights by 2w.

Then
so
and

+a

h
h - 2w

= 2w

= -a + 2w
= -a

......................... (1)
(2)

Multiply together the left- and right-hand-sides in (1) and (2)

(h
2w)h =
h2
2wh
h 2 - 2wh + w2 =
(h - W)2 =
h - w =

So

Le.,
Thus

Le.,
Hence
so

-a(-a + 2w)
a2 - 2aw
a2 - 2aw + w2 (Add w2.)
(a - W)2
a - w
(Take square roots.)
a

Where is the flaw in the reasoning?

EXERCISES 3(d)
Write expressions for the following:

1. "';81
3. 16x -< 41
5. Ix + yl
7. ~.x2 + x

2. -J(-2'5)2
4.

9 .JI6 + ~
11. -J9 - 6x + X2 when x

6.
8.

1-J3 - 21
Ixl + Iyl
Ix 51 + Ix

> 3

51

10. -J(2x + 3)2

12. Solve for x:

(a) Ix - 21
(c) 12x - 51
(e) 12 + xl

(b) 12x
(d) 15x

+ 11 = 2
+ 11 = 4

Represent the solution sets of the following inequations (13.-19.) on a number line:

13. Ix + 31 > 1
15. 12 + 4xl ~ 6
17. Ix + 31 > 1 and 12x +

11

19. IX2 -

~ 3 and Ix2

51 <

+ 11

51 <

14. 12x +
16. Ix2 18. 12x +

11

~ 4

51 <

3 or

12 + 4xl

~ 6

~ 0

Simplify the following expressions, stating the values of x for which your answers apply:
(20. to 25.)
..[Xl

20.

liT

21.

"';"';"(x-'-----:":4)-=-2

22.

1 - x

24. -Jx2 - lOx

-JXi

23.

25

= Ixl.lyl
~ Ixl + Iyl
for the following values of x and y:

(a)x = 5,y
2
(b)x
3,y = -2

26. Verify that (i)

(ii)

25.

11 - xl

Ix2 - 11

Ixyl
Ix + yl

(c)x

-6,y

64 NEW SENIOR MATHS: TWO UNIT COURSE

3.4 Quadratic equations _ _ _ _ _ _ _ _ _ __


An equation of the form
ax 2

bx

= 0, a

=1=

is called a quadratic equation in x. The elements of its solution set represent geometrically
the x values of the points of intersection of the parabolay = ax2 + bx + c and the X-axis.
From our experience of drawing parabolas, we have seen that a parabola can
(i) intersect the X-axis at two distinct points,
(ii) touch the X-axis at one point only,
~iii) not intersect the X-axis at all.
Solving of quadratic equations will involve the use of the Null Factor Law:

~XAMPLE

11
(b) If (x + 1)(x - 4) = 0,
then x + 1
0 or x
4 = 0
i.e.
x -lor x = 4

(a) If x(x - 2)

0,
0 or x
0 or x

then x
i.e. x

FXERCISES

2 = 0
2

3(e)

Solve the following equations (you should be able to do these mentally):


1. x(x

5)

+ 1)(x
:5. x(x + 6)
3. (x

2. (x - 2)(x - 3)
4. x(2x + 1)
0

- 2)

6. (x
7)(2x + 5) = 0
8. -5x(x + 1) = 0
10. 2x(2
3x) = 0
12. (2x - 9)x = 0
14. (x - 3a)(x + 2b)
0
0
16. (2x - 11)(2x + 11)
18. (2x + 3)2 = 0
20. (2x
1)(2x + 1) = 0

'7. 3x(2x - 9) = 0
9. (x
3)(x + 4) = 0
11. x(6
x) = 0
13. (x
a)(x - b) = 0
15. (x - 2)(x + 2)
0
17. (x - 1)2 0
19. 2x(5x - 8) = 0

Quadratic equations are divided into three classes.

Type 1:

Solving this type involves factorizing the difference of two squares.

EXAMPLE 12
Solve the equations
(i) X2 - 4 = 0
(iii) 9x 2 = 25

(ii) 2X2 - 18
(iv) (x - 2)2

=0
9

3/EQUATIONS AND INEQUATIONS 65

X2 - 4 = 0

(i)

i.e. (x - 2)(x

2)

0 (Difference of two squares)

or x + 2
0
x = 2 or x = -2

Graphically, the parabola y = x 2 - 4

cuts the X-axis at x = 2 and x = -2

x - 2

= 0

2X2 - 18 = 0

2(X2 - 9) = 0 (Common factor of 2)

2(x - 3)(x + 3)
0 (Difference of two squares)

(ii)

x - 3 = 0 or x + 3
x = 3 or x = -3
Graphically, the parabola y

9x 2

(iii)

i.e.

= 2X2

18 cuts the X-axis at x

= 3 and x = -3.

25

9x - 25
0

(3x - 5)(3x + 5) = 0 (Difference of two squares)

3x - 5 = 0 or 3x + 5
0

x = 1~ or x = -lj
Graphically, the parabola y = 9x2
(iv)

25 cuts the X-axis at x =

1~

and x

-1~.

(x - 2)2
9
(x - 2)2 - 9 = 0
(x - 2 - 3)(x
2 + 3) = 0 (Difference of two squares)
(x - 5)(x + 1)
0
x - 5 = 0 or x + 1 0
x = 5 or x = -1

EXAMPLE 13
Solve the equations (i) X2 - 6

0, (ii) 3x2 = 15.

0) If we consider 6 as being (-J6)2, thenx2 - 6 may be considered asthe difference of two


squares.
~-6=0

(ii)

3x 2 -

-J6 or x = --J6

3x2 = 15
15 = 0

3(X2
5)
3(x - .J5)(x + ...[5)

or

3x 2 = 15
X2 = 5

0 (Common factor 3)

= 0 (Difference of two squares)

x = ...[5 or x = -...[5

EXERCISES 3(f)
Solve the following equations.
1. X2 - 1
3. X2 - 49

5. X2

0
0

16

7. X2 - 64 = 0
9. X2 - 61 = 0

X2 = 6
x=-J6

X2 - (-J6)2
0
(x - -J6)(x + -J6) = 0 (Difference of two squares)

m~

i.e.

2.
4.
6.
8.
10.

X2
25
0
X2 = 0'25
X2 = 9
4X2 = 9
4X2 - 25 = 0

...[5

66 NEW SENIOR MATHS: TWO UNIT COURSE


11. X2 - 2l = 0
13. 5x2 - 5 = 0
15. 16 - X2 = 0
17. 9(x - 1)2 - 36 = 0
19. 5x 2 - 245 = 0
21. (2X)2 - 9 = 0
23. (x + 3)2 - 4 = 0
25. X2 - 5 = 0
27. X2 = 2
29. 5x 2 - 15 = 0
31. (x - 2)2 - 12 = 0
33. 10 - X2 = 0
35. 3x2
21

12. 4X2
9
0
14. 16x2 - 1
0
16. 25x2 = 49
18. (x - 2)2 = 16
20. 7x2
63
22. (5x - 1)2
16
24. (x - 1)2 - 4 = 0
26. 2X2 - 6
0
28. 3x2
33
0
30. (x + 1)2 = 8
32. X2 - 18
0
34. 2X2 - 24 = 0
36. 2X2
22

Type 2:
ax2 + bx

=0

This type contains x as a common factor. and so we use the Distributive Law in reverse.

EXAMPLE 14
Solve the equations (i) X2 - 3x
(i)

= 0, (ii) 4X2 =

8x.

X2 - 3x = 0
i.e. x(x - 3) = 0 (x is a common factor)

x = 0 or x
3

4X2

(ii)

8x

i.e. 4X2 - 8x = 0
4x(x - 2) = 0 (4x is a common factor)

x = 0 or x
2
'

EXERCISES 3(g)
Solve the following equations.
1. x 2 - 6x = 0
4. X2 + lOx = 0
7. 2X2 - 5x = 0
10. X2 = 2x
13. 6x 2 = 24x
16. 2X2 - 3x = 0
19. 4X2 + 8x = 0
22. 5x2 + x = 0

2. X2 - 5x = 0
5. X2 = 4x
8. X2 + 8x = 0
11. 3x2 14. 9x 2 =
17. 4X2 20. 2X2 23. 3x 2 =

21x = 0
27x
4x = 0
X = 0
-9x

3. X2 + 5x = 0
6. X2 - X = 0
9. X2 = 7x
12. 2X2 + 20x = 0
15. 5x2 + 20x
0
18. llx2 = 22x
21. 3x 2 - x = 0
24. 2X2 - 5x = 0

Type 3:

Solving this type involves factorizing a quadratic trinomial. Sometimes it may be


necessary to 'tidy up' the equation to get it in the form ax2 + bx + c = 0, with the right
hand side zero.

3/EQUATIONS AND INEQUATIONS 67

EXAMPLE 15
Solve the equations
(i) X2 - 5x + 6 = 0
(iii) x(x - 2)
3

(ii) 2X2 = x + 3
(iv) (3x + 4)(x - 3)

(iii)

(ii)

x(x - 2)
3
X2 - 2x = 3
X2 - 2x - 3 = 0
(x
3)(x + 1) = 0
x - 3
0 or x
x = 3 or x

(iv) (3x

1 = 0
-1

16

2X2 = x + 3

2X2 - X - 3 = 0

(2x - 3)(x + 1) = 0

2x - 3
0 or x + 1 = 0
x = I! or x = -1

X2 - 5x + 6 = 0

(x - 2)(x - 3)
0

x - 2 = 0 or x - 3 = 0
x
2 or x = 3

(i)

+ 4)(x - 3) = 16
3x2 - 5x - 12 = 16
(expand the left-hand side)
3x 2 - 5x
28 = 0
(3x + 7)(x - 4)
0
3x + 7 = O'or x - 4 = 0
x
-2tor x = 4

EXERCISES 3(h)
Solve the following quadratic equations.
1. X2
3x + 2 = 0
8 = 0
3. X2 - 2x
5. X2
6x + 9
0
7. X2 + 9x + 8 = 0
9. 5x 2 + 7x - 12 = 0
11. 5x2 - 11x + 2 = 0
13. 2X2 - x - 10
0
15. X2 + 5x + 4 = 0
17. 3x2 - 28x + 25 = 0
19. 5x 2 + 26x + 24
0
21. X2 + 7x - 30
0
23. 5x 2 = 8x - 3
25. x(x + 5) = 6
27. x(3x - 1) = 2
29. x(7 x - 30) + 8 = 0
31. (x
2)(2x + 5)
2x
33. X2 - 9 = 8x
35. (2x + 1)2 = 4
37. (x + 6)2 = X + 6
39. 7x 2 = 2(l7x
12)

2. X2 - 6x + 5 = 0
4. X2 - 4x + 3 = 0
6. X2 - 5x + 4 = 0
8. 9x2 + 4x
5 = 0
10. X2 + 4x - 12 = 0
12. 4X2 - 12x - 7 = 0
14. X2 + lOx + 25
0
16. 4X2 - 8x
21 = 0
18. X2 - 8x + 16 = 0
20. 3x 2 - 41x + 60 = 0
22. X2 - 7x + 10
0
24. x(2x - 11) = 6
26. x(3x + 19)
72
28. X2 + 15 = 8x
30. 5x2 = 3x + 2
32. 5x(x + 2)
3x - 2
34. 12 - 4x - X2 = 0
36. (x + 1)2 = 4x
38. 6x2 = 10 - 11x
40. X2 = 4(x + 24)

3.5 Completing the square _ _ _ _ _ _ _ __


X2

12x

+ 36

+ 6)2
(12,2
\2) = 62.

= (x

Observe that 36 =

i.e. the term not containing x is equal to the square of half the coefficient of x.

68 NEW SENIOR MATHS: TWO UNIT COURSE

lOx

X2 -

+ 25 =

Observe that 25

(x - 5)2

-lo'V
( 2) = (-5)2.

i.e. the term not containing x is equal to the square of half the coefficient of x.
Hence, if the first two terms of a quadratic expression in x are given, the expression can be
made into a square if we add the square of half the coefficient of x.

EXAMPLE 16
What must be added to (i)

X2

8x, (ii)

x is 8.

One-half of 8 is 4.

The square of 4 is 16.

Thus when 16 is added we get

3x to complete the square?

X2 -

(i) The coefficient of

(ii) The coefficient of

Thus when

8x

+ 16 =

(x

+ 4)2.

x is -3.

. -3

One-half of -3
The square of

X2

IS

2.

-3

2 is 4

~ is added we get X'

3x

~ = (X _ ~)'.

EXERCISES 3(i)
Write down the number to be added in order to complete the square in each of the
following.

1. X2
4. X2
7. X2
10. X2
13. X2
16. X2
19. X2

+
+
+
-

+
-

2. X2 - 6x
5. X2 + 2x
8. X2 -(8x
11. x 2 + ~X~\'>
14. X2 + x
17. X2 - 15x
20. X2 + cx

4x
lOx
20x
x
7x
9x
2bx

3. X2
6. X2
9. X2
12. X2
15. x 2
18. X2
21. X2

+
-

+
-

+
-

14x
12x
24x
3x
llx
2ax
ax

3.6 Solving quadratic equations by completing


the square _ _ _ _ _ _ _ _ _ _ _ _ _ __
EXAMPLE 17
Use the "completing the square" method to solve
(i) x 2

4x - 5 = 0, (ii)

(i)

X2
X2

i.e.

i.e.

X2 -

5x

6 = 0, (iii)

X2

= 8x.

4x - 5 = 0
(Put constant term on R.H.S.)
X2 + 4x = 5
+ 4x + 4 = 5 + 4 (Add 4 to complete the square)
(x + 2)2 = 9
(Taking square roots)
x+2 = --J9
= 3
x + 2 = 3 or x + 2 = -3

x = 1 or x = -5

3/EQUATIONS AND INEQUATIONS 69

or
i.e. (x

(ii)

(x + 2)2 =
(x + 2)2 - 32
=
2 - 3)(x + 2 + 3) =
(x - 1)(x + 5) =
x =

5x

X2 -

5x

0
1 or x

= 0

5x = -6

+ 25
4 =

G
- ~)'

i.e.

(Difference of two squares)

-5

+6

X2 -

X2 -

(Put constant term on R.H.S.)

+ 25
25 to compIete t h e square)
4 (Add 4

-6

x-~ = )"f
1

2"
i.e.

x -

2"

2" or x

5-1

2"

2"

x = 3 or x = 2

or

(iii)
X2

i.e.

G- ~)'
( x - ~)' - G)'
1)( 5. 1)
2" \x - 2' + 2"

= 0 (Difference of two squares)

(x - 3)(x - 2) = 0

x
3 or, 2

X2
8x
X2 - 8x = 0

8x + 16 = 16
(Add 16 to complete the square)

(x - 4)2 = 16
x - 4 = .JI6 (Taking square roots)

i.e.

x - 4 = 4 or x - 4 = -4

x = 8 or x = 0

EXERCISES 3(j)
Use the method of completing the square to solve the following quadratic equations.

1. X2 - 6x + 5 =
3. X2 + 4x - 5
5. X2 - lOx + 24
7. X2 - 26x + 25
9. X2 + X - 12 =
11. X2 + 7 x
30
13. X2 - 3x - 10
15. X2 + x = 72
17. X2 - lOx = 0

0
0

2.

X2 -

4.

X2

= 0
= 0

8.

X2

10.

X2

12.
14.

X2

16.

X2

18.

X2

0
0

6~ X2

X2

2x - 8

4x = 12
- 4x = 21
- 3x + 2
0
- 5x + 4 = 0
- 11x
12
= 7x - 10
lOx - 11 = 0
= 3x

Actually, all the equations in exbrcises 3U) could be solved by using factors. However, the
great advantage of the "completing the square" method is that it can be used to solve
quadratic equations that do not have rational factors.

70 NEW SENIOR MATHS: TWO UNIT COURSE

EXAMPLE 18
Use the "completing the square" method to solve (i) X2
+ 2 = O.

+ 2x - 5

0, (ii) X2

4x + 8,

(iii) X2 - 5x

+ 2x - 5 = 0
X2 + 2x = 5
X2 + 2x + 1
5 +
i.e. (x + 1)2 = 6

(i) X2

(Put constant term on R.H.S.)

(Add 1 to complete the square)

(Taking square roots)

x+l=-.[6
or x + 1
--.[6
x + 1 = -.[6
x = -1 + -.[6
or x
-1 - -.[6
~ -1 + 2'449
~ -1 - 2449
-3'449
= 1'449
In surd form, the exact solutions are
x
-1 -.[6
Using 2449 as an approximation to -.[6, the approximate solutions to four figures are
x = 1'449 or -3,449.
i.e.

(ii)

x2 =
4x
X2 - 4x + 4
i.e. (x - 2)2 =
x-2=
i.e. x - 2
x
i.e.

X2 -

4x + 8
8

8 + 4
12

(Add 4 to complete the square)

(Taking square roots)

orx - 2

2+M
orx 2 - M

~ 2 + 3464
~ 2 - 3464
5464
-1'464
The exact answers in simplest surd form are x
2 2-J3.
(iii) x 2

5x

-2

+ 25

-2

5x
X2 -

X2 _

i.e.

5x

(x - ~y
x

i.e.

+ 25
4

(Put constant term on R.H.S.)


25
(Add
to complete the square)

ffi

2
(Taking square roots)

ffi

x 2

orx

x=~+ffi
2

+ ffi

+ 4123

-ffi

5
2

5 ffi
2-2

5 - ffi

or x

2
~----

= 4'562

The exact answers in surd form are x

=
5

5 - 4123

0'439

ffi

3/EQUATIONS AND INEQUATIONS 71

EXERCISES 3(k)
Use the 'completing the square' method to solve the following quadratic equations,
giving your answers in surd form and also correct to three decimal places.
1. X2 - 2x ~ 4 = 0
2. x 2 + 4x - 4 = 0
4. X2 - 6x + 2
0

3. X2 - X - 5 = 0
2

S. X2 - 5x + 1 = 0
6. x + 2x - 2 = 0

7. X2 = 6x - 4
8. x 2
+ x - I = 0
10. X2 + 4x
1
9. X2 - 6x - 5 = 0
12. X2 + 3x - 6 = 0
11. X2 = 2x + 5
13. X2
x = 3
14. x 2 3x = 9
In each of the equations considered so far,' the coefficient of X2 has been 1. When the
coefficient of X2 is not 1, simply divide through by the coefficient of X2 and proceed as
before.

EXAMPLE 19
Use the 'completing the square' method to solve the equation 2X2 - 3x - 3
2X2 - 3x - 3 = 0
2X2 - 3x = 3
3x
3
X2 - T
:2

+ ~

3x

2 _

i.e.

16

(Put constant term on R.H.S.)


(Divide both sides by the coefficient of x 2)

= ~

+ ~ (Add {6 to complete the square)


16

(x - ~r = ~~
x

i.e.

= O.

-4

.J33

-4-

(Taking square roots)

.J33
4
3
.J33
x=4+ 4
3 + .J33
3

x- 4

orx -

4=

-'\/'33
-4

3
4

orx

.J33

3 - .J33

3 - 5'745

5745

= 2186

-0,686

EXERCISES 3(1)
Use the 'completing the square' method to solve the following quadratic equations.
1.
3.
S.
7.
9.
11.

13.
IS.

2X2 2X2 +
3x2 3x2 3x 2
2X2 2X2 +
4X2

X - 5 = 0
x
2
0
5x - 1 == 0
2x = 4
7x + 3
5x = 9
x = 4
9x - 4

2. 2X2 +
4. 2X2 +
6. 3x2 +
8. 2X2 10. 4X2 +
12. 3x2 14. 3x2 16. 6x2

6x
5
0
3x
1
= 0
'v

4i = 5

6x + 1
0
4x - 5 = 0
2x - 2 = 0
8x + 3
0
lOx +. 3 = 0

72 NEW SENIOR MATHS: TWO UNIT COURSE

3.7 Formula for solving quadratic equations _ _


The equation
ax2

+ bx + c = 0, a

'* 0

is called the general quadratic equation. By giving suitable values to a, band c the
solutions of this equation may be applied to any quadratic equation.
We proceed to solve it using the 'completing the square' method.
ax 2 + bx + c = 0
ax2 + bx = -c
(Put constant term on R.H.S.)
b
-c
X2 + -x =
(Divide both sides by the coefficient of x 2 )

+ ~x + (~)2 = ~2 _

X2

2a

4a

(Add

(:0)' to complete the square)

(x+ :a)' = b 4ac


2

i.e.

"jb 2

+ 2a

4ac

(Taking square roots)

This formula enables us to solve any quadratic equation and thus avoids the necessity of
hunting for factors of the quadratic expression or repeating the various steps in using the
'completing the square' method.

EXAMPLE 20
Use the formula to solve the following quadratic equations.
(i) X2 + 8x + 12 = 0
(iii) 2X2 - 4x + 1 = 0

0)

X2

8x + 12 = 0

-b "jb 2 - 4ac

(ii) X2 (iv) 4x 2

where a

3x

+ 5x

0
2

=0

1, b = 8, c = 12

-8
-8

"JT6

-8 + 4

= --2- or

-8 - 4

= -2
or -6
The solutions are rational numbers. Observe that b 2
4ac = 16 and
number. We could have solved this equation using factors:
(x + 2)(x + 6) = 0
x + 2 = 0 or x + 6 = 0
x = -2 or -6

"JT6 is

a rational

3/EQUATIONS AND INEQUATIONS 73

3x - 2 = 0
-b
---=---

(ii) X2 -

x =

3+ffi
3-ffi
.
2
or
(Exact solutIOns)

4123

3 - 4'123
or------

or -0'562
= 3'562
17 and ffi is a surd. It is not
The exact solutions are surds. Observe that b 2 - 4ac
possible to solve this equation by hunting for factors in the usual manner.
(iii)

+1 0
-b ..JlJ2 4ac

4x

2X2

wh~ea 2,b
-(-4) ..J(-4)2 - 4 x 2 x 1

..Jr:-16~----=8

4 -.[8
4+-.[8
4--.[8

4
or

2-12

or

2+-J2

or

4 - 2-J2

2--J2
2

(since -.[8

2-12)

(Exact solutions)

~ 1'707
or 0293
(-12 ~ 1414)
The exact solutions are surds. Observe that b 2 - 4ac
possible to solve this equation by trying for factors.

(iv)

4X2

+ 5x

8 and -.[8 is a surd. Hence it is not

- 2 = 0

-b

---4-a-c

..J~b-2

-5

+8 -J57

-5

7'550

= 0'319

or

-5 -

where a

.J57

= 4, b = 5, c = -2

(Exact solutions)

-5 - 7'550

or--~~--

or -1'569

EXERCISES 3(m)
Solve the following quadratic equations using the formula, giving solutions in simplest
surd form and correct to 3 decimal places where necessary:
1.

X2

6x

2.

X2

2x - 8 = 0
j

74 NEW SENIOR MATHS: TWO UNIT COURSE

3. X2 - 6x - 7 = 0
5. X2 + 2x - 1 = 0
7. X2 - 2x - 5 = 0
X2 - 2x - 9 = 0
X2 - 15x + 56 = 0
2X2 + 5x + 1 = 0
2X2 + 3x + 1 = 0
3x2 + 2x - 2 = 0
X2 + 6x + 1 = 0
21. 2X2 - X - 3 = 0
23. 4X2 - 9x + 4 = 0
25. 2X2 + X = 3
27. 2X2 + 6x + 1 = 0
29. x 2 = 2x + 2
31. 2X2 = 3x + 4
9.
11.
13.
15.
17.
19.

33. x 2 + 17 x = 60
35. 3x 2 - 15 = 0

4.
6.
8.
10.
12.
14.
16.
18.
20.
22.
24.
26.
28.
30.
32.
34.
36.

x 2 - 7x + 10 = 0
x 2 - 6x + 4 = 0
x 2 + 5x - 1 = 0
X2 + 4x + 2 = 0
X2 + 2x - 15 = 0
2X2 - 8x + 3 = 0
2X2 - 3x = 0
2X2 + 3x - 5 = 0
x 2 - 8x + 16 = 0
5x2 - 7x - 2 = 0
3x2 - 11 x - 4 = 0
x(x + 3) = 2
2X2 - 6x = 3
X2 = 6x + 2
2x2 + lOx + 5 = 0
3x2 + 9x + 5 = 0
x(x + 1) = 1

3.8 Problems involving quadratic equations _ _


EXAMPLE 21
One side of a rectangle is 2 cm longer than the other side and the area of the rectangle is

120 cm 2 What are the dimensions of the rectangle?

Let one side be xcm in length.

Then the other side is (x + 2) cm in length.

Since the area of the rectangle is 120 cm2 ,

x(x + 2) = 120 ................ (1)

x
X2 + 2x = 120
x 2 + 2x - 120 = 0
(x + 12)(x - 10) = 0
x + 12 = 0 or x - 10 = 0
x+2
x = -12 or x = 10
However, since x is a number that measures length, x cannot be negative. We can accept
only the solution x = 10. In this case one dimension is 10 cm and the other is 12 cm.
Equation (1) should be x(x + 2) = 120, x > O.
Note: The Null Factor Law, which we use to solve quadratic equations, states that if
ab = 0, then a = 0 or b = 0 or a = b = O.
You should observe that it is not necessary for both a and b to be zero. So long as one of
them is zero~ then ab = O. Thus, in practical problems, we sometimes have to reject one of
the solutions.

EXAMPLE 22
The height h m of a stone t seconds after being thrown vertically upwards is given by the
equation h = 30t - 5t 2 After what time is the stone at a height of 40m?
h = 30t - 5t 2
Thus
40 = 30t - 5t 2 when h = 40
5t 2 - 30t + 40 = 0

3/EQUATIONS AND INEQUATIONS 75

5( t 2 - 6t + 8) = 0
5(t - 2)(t - 4) = 0
t - 2 = 0 or t - 4
t = 2 or t = 4
Are both of these values of t acceptable? Why?

=0

EXERCISES 3(n)
1. In each of the following diagrams, the measurements are in cm. Find x in each case.
The area of the shaded regions is given in each case.
x +5
(a)
( )
(b)
c

(x -1)

(x

Area

x +1

+ 2)

r-------,
3

x+3

x+4
Area = 20 em1

40 em1

(e)

(d)

Area = 33 em1

x
4

x
-+--(X+3)~

Area = 44 em 1

Area = 22 em 1

2. In each of the following right-angled triangles, use the Theorem of Pythagoras to find
the value of x. Measurements are in cm.
(a)
x-l

(c)

x+3

(d)

x+3

3. The sum of a certain positive number and the square of that number is 12. What is the
number?
_ 4. The product of two numbers is 88. What are the numbers if one number is 3 more
than the other?

5. The product of two consecutive numbers is 72. What are the numbers?
6. The height h m of a stone t seconds after being thrown vertically upwards is given by
h = 40t - 5t 2 At what times is the stone at a height of (a) 60m, (b) 80m?

76 NEW SENIOR MATHS: TWO UNIT COURSE

7. The sum of the square of a positive number and four times the number is 60. What is
the number?

8. A rectangular swimming pool 12 m by 8 m is surrounded by a concrete path of


uniform width. If the area of the path is 224m 2 , find its width.
9. A carpet is placed in a room 6 m by 4 m, leaving a border of uniform width all round
it. Find the width of the border if the area of the carpet is 8 m 2
10. A picture on a wall measures 24 cm by 20 cm. It is surrounded by a frame of uniform
width whose area is 416 cm 2 What is the width of the frame?
11. A rectangle is 8 cm longer than its width. If the area of the rectangle is 48 cm2 , what
are its dimensions?

12. In a right-angled triangle, one of the sides adjacent to the right angle is 4 cm longer
than the other side. Find the length of each of the three sides if the area of the triangle
is 96 cm 2
13. The perimeter of a rectangle is 40 cm and its area is 84 cm 2
(i) If the width of the rectangle is xcm, express the length in terms of x.
(ii) Write down the area of the rectangle in terms of x.
(iii) Form a quadratic equation in x and solve it to find the length and width.

3.9 Linear equations with two variables ____


A linear equation in two variables x and y is an equation in which both of the pronumerals

are of first degree, e.g. x + y = 6, and there is an infinite number of values of x andy that

make this equation a true statement, e.g.

(0, 6), (1, 5), (2, 4), (4, 2), (-8, 14), ...

These points all lie on a straight line.


At the same time, let us consider the equation x - y = 2, the graph of which is also a
straight line that contains an infinite number of points, e.g.
(0, -2), (1, -1), (2, 0), (4, 2), (-10, -12), ...
You will observe that the ordered pair (4, 2)
satisfies both equations.
Hence x = 4, y = 2 is said to be the solution
of the simultaneous equations x + y = 6 and
x - y = 2 and graphically x = 4, y = 2 are the
L _
coordinates of the point of intersection of the
[/
two lines.

3/EQUATIONS AND INEQUATIONS 77

Two linear equations in two variables are said to be simultaneous if both


equations must be considered at the same time.

If an ordered pair (x, y) satisfies both equations, that ordered pair is a

solution of the simultaneous equations.

3.10 Algebraic solution of simultaneous


equations _______________
The most common algebraic methods of solving a simultaneous pair of linear equations in
two variables are (a) elimination, (b) substitution.
In the elimination method, which is more common for this type of simultaneous
equations, we eliminate one of the variables x and y by either addition or subtraction,
provided that the coefficients of one of the variables in each equation are equaL

If the coefficients are equal and opposite 'in sign, eliminate by addition.
If the coefficients are equal and have the same sign, eliminate by
subtraction.

EXAMPLE 23
Solve the simultaneous equations x

+y

= 6

2.

and x - y

x+y=6
x-y=2

......................... (1)

(2)

Elimination: Since the coefficients of yare equal and opposite in sign, we can eliminate y
by adding both sides of (1) and (2).
,'. 2x = 8
x

+y

Put x = 4 in (1)
:,4

Substitution:

y = 2
From (1), y
6

2
6+x=2
2x
8
x = 4
From (1), y
6

Substitute 6 - x for y in (2)


:. x - (6 - x)

= 6 -

We should check that the answers are correct by putting x = 4, y

2 into both equations.

EXAMPLE 24
Solve the simultaneous equations 3x

+ 2y

= 10 and 4x

3x + 2y
4x + 3y

+ 3y

= 13,

10

.......... , ............. (1)

= 13

(2)

i'

78 NEW SENIOR MATHS: TWO UNIT COURSE

Elimination: Make the coefficients of y both 6 by multiplying (1) by 3 and (2) by 2 .


.'. 9x
8x

Subtract (4) from (3)


= 4 in (1)

Put x

12

+ 6y = 30
+ 6y = 26
+

Substitution: From (1),

. . . . . . . . . . . . . . . . . . . . . . .. (3)

. . . . . . . . . . . . . . . . . . . . . . .. (4)

x = 4
2y = 10
2y = -2
y = -1
10 - 3x
y =
2

10 -2 3x f or y III
. (2)

SubstItute
:. 4x
8x

3(10 ;

3(10 - 3x) = 26
+ 30 - 9x = 26

8x

3x)

13

(Multiply by 2)

= -4
x=4
Put x = 4 in (1)
12 + 2y = 10
2y = -2
y = -1
Sometimes it is necessary to simplify and tidy up one or both of the equations before
solving.
-x

EXAMPLE 25
Solve for x and y:

:!+l=1

...................... (1)

7
4

...................... (2)

6
4
3x _ x - Y
4
2
Multiply (1) by 12. Why?
:. 2x

3y

= 12

...................... (3)

Multiply (2) by 4. Why?


:. 3x - 2(x - y) = 7
3x ~ 2x + 2y = 7
x + 2y = 7

...................... (4)

Make the coefficients of x in (3) and (4) both 2 by multiplying (4) by 2.


:. 2x + 3y = 12 ...................... (5)
2x + 4y = 14 ...................... (6)
Subtract (5) from (6)
:. y = 2
Put y
2 in (3)
2x + 6 = 12
2x = 6

= 3

Test your solutions by putting x = 3 and y = 2 into both (1) and (2).

EXERCISES 3(0)
Solve these simultaneous equations.
l.x+7y=5
x
7y
-9

2. x + 5y = 34
x - 5y = -6

3. 4x - 5y
4x - 2y

30

24

3/EQUATIONS AND INEQUATIONS 79

+ 5y =

16

5. 2m + 3n =
3m + 2n
8. 5x + 2y
9x - 7y =
11. 2x + 3y =

3y

-4

4x - 5y

4. 3x - y = 5
5x + 3y = -8

7. x

+ 5y

2x
, 10. 2x
lOx

13. Y
y

-13

=7

4x - 2
-3x + 5

14.

+ 8y

= 42

9x - (x - 2y)

78

16. 3(x - y)

x - 4
3(x

6. -2x

-3x

14
17

= 4(y

=9

_6a_-_b = 9

+ 7y
+ 5y

4
-5
9. 2a
3b = 5
2a - 5b = -1

-5

2) = 2y

17. 3a - 2b

19. 3(x - y) - 8(x + y) = 7


2(x + y) + 5(x - y) = -65
21. 5(2x - y) = 7x + 1
3(3x + y)
5(x - y + 12)

-4
-6
9

+ 2)
+ 20

12. 5m
2m

6n
9n

15. 2x

_l
4

x + 3y

12

= 20
5
-1

4
18. x - 3 = 2y

+ 1
3
3x - 1
2y + 1
-5- - -2- =

20. 2(3a - b)
= 3(a + b)
3(a - 4b) + 46
5a
22. 5(x + y) + 2(x - y) = 15
5x + 6(y - 2) = 0

3.11 Problem solving using simultaneous


equations _______________
EXAMPLE 26
Three books and 5 pencils cost $5'55. A book costs 10 cents more than 10 pencils. Find the
cost of a book and a pencil.
Let a book cost x cents and a pencil y cents.
3x + 5y = 555
(3 books and 5 pencils cost 555 cents) . . . . . . . ..
x - lOy = 10
(1 book costs 10 cents more than 10 pencils)
Multiply (1) by 2.
:. 6x + lOy = 1110
..........................................
x - lOy = 10
..........................................
Add (3) and (4).
:. 7x = 1120
x = 160
Put x = 160 in (2).
:. 160 - lOy = 10
lOy = 150
y
15
A book costs $160 and a pencil 15 cents. Check that this agrees with what is given.

(1)
(2)

(3)
(4)

EXAMPLE 27
In a pen consisting of rabbits and chickens I counted 25 heads and 80 legs. Assuming that
each rabbit has 4 legs and each chicken has 2 legs, how many rabbits and chickens are
there?
Let x equal the number of rabbits and y equal the number of chickens.
x + y = 25 (There are 25 heads) .............. (1)
4x + 2y = 80 (There are 80 legs) . . . . . . . . . . . . . . .. (2)

80 NEW SENIOR MA THS: TWO UNIT COURSE

Multiply (1) by 2.
:. 2x
4x

+
+

2y = 50
2y
80

.................................. (3)

(4)

Subtract (3) from (4).


:. 2x = 30
x
15

Put x = 15 in (1).
15

+y

25

10

There are 15 rabbits and 10 chickens.

mx + c is the equation of a straight line. If the line contains the points (1, 8) and
<-2, -1), form a pair of simultaneous equations in m and c and solve these equations.
y=mx+c
8 = m + c (y = 8 when x
1) ................... (1)
-1
-2m + c (y
-1 when x = -2) ................ (2)
Subtract (2) from (1).
:. 9 = 3m
m = 3
Put m = 3 in (1).
:. 8 = 3 + c
c
5
y = 3x + 5 is the equation of the line.
y

EXERCISES 3(p)
1. There are 450 students at a particular school. If there are 50 more boys than girls, how
many boys and girls are there?

2. A contractor has 8 trucks, some of which carry a load of 10 tonnes and others a load
of 5 tonnes. When the 8 trucks are filled, they contain a total load of 70 tonnes. How
many of each size of truck does the contractor own?
3. A father is 7 times as old as his daughter. In 5 years' time he will be 4 times as old as
his daughter will be then. Howald is each one now?
4. John's mother is 5 times as old as John is. Three years ago, she was 9 times his age.
What are their present ages?
5. Admission costs to a theatre are adults $5, children $2. If 1000 people paid to enter
the theatre and the total receipts were $3800, how many adults and children attended?
6. A box contains 50 coins made up of 10-cent coins and 20-cent coins. How many of
each are in the box if the value of the coins is $7?
7. The straight line ax + by = 12 contains the points (2, 2) and <-4, 5). Form two
equations in a and b and solve them.
8. A person spent 6 nights at a motel and 5 nights at a hotel for a total cost of $250. If he
had spent 5 nights at the motel and 6 nights at the hotel, the cost would have been
$245. What is the cost per night at the motel and at the hotel?

3/EQUATIONS AND INEQUATIONS 81

9. Let x be the numerator and y be the denominator of a fraction. The denominator is 5


more than the numerator. If 2 is subtracted from both the numerator and the
denominator, the denominator is then twice the numerator. What is the fraction?
10. The weekly salary of a television-set sales representative is made up of two parts, a
fixed amount $a plus $b for every set sold. In one particular week she sold 25 sets and
her salary was $495; in another week she sold 15 sets and her salary was $345. Find the
values of a and b.
11. Find two numbers such that if 18 is added to the first number it becomes twice the
second number and if 6 is added to the second number it becomes three times the first
number.

12. The weekly wages of 5 carpenters and 3 apprentices amount to $1880 and the wages of
3 carpenters and 5 apprentices amount to $1640. Find the weekly wages of a carpenter
and an apprentice.
13. Find the equation of the straight line that contains the points
(i) (4, 1) and (-1, -9)
(ii) (0, 4) and (1, 0)
(iii) (2, -l~) and C4, -6)
(iv) (2, 4) and (-6, 8)

3.12 Simultaneous equations-one linear, one


second degree
A second degree equation in x and y is one that contains X2 or y2 or xy. For example,
y = x 2 - 5x + 6
(parabola)
X2 + y2
4
(circle)
We have seen that the graph of a linear or first-degree equation in x and y is a straight
line. A straight line and a parabola or circle or hyperbola can intersect in up to two places
and the coordinates of the point or points of intersection can be found by solving the
simultaneous equations. We use the method of substitution.

EXAMPLE 29
Solve the simultaneous equations
y = X2

82 NEW SENIOR MATHS: TWO UNIT COURSE

We are asked to find the coordinates of the point or points of intersection of the parabola
Y
X2 and the straight line y = x + 2.
This can be done either graphically, as shown in the diagram, or algebraically, by using
the method of substitution.

In place of y in (1) put x


Then

y = X2
y = x + 2
2 from (2).

x
X2 -

X -

(x - 2)(x

(1)

........................... (2)

2 = X2
2 = 0
1) = 0
x = 2 or -1

Substitute these values of x into (1) or (2).


From (2), if x = 2, y = 4
if x = -1, y
1
Thus the straight line and parabola intersect at (2, 4) and (-1, 1).

EXAMPLE 30
Solve the simultaneous equations
x + y = 5

~~~~~~~~~~~~~~~~~

X2

y2

25 .

We are asked to find the coordinates of the point or points of intersection of the straight
line x + y = 5 and the circle X2 + y2 = 25.
This can be done either graphically, as shown in the diagram, or algebraically, using the
method of substitution.

x+y=5
+ y2 25

X2

From (l),y = 5 - x.
In place of y in (2) put 5 - x.
Then
X2 + (5 - X)2
X2 + 25 - lOx + X2
2X2 -

25

lOx

5x

x(x - 5)
x

X2

. .......................... (1)
(2)

25
0
(Divide by 2)
0
0
0 or 5

3/EQUATIONS AND INEQUATIONS 83

Substitute these values of x into (1).


From (1), when x = 0, Y = 5
when x = 5, Y = 0
Thus the straight line and circle intersect at (0, 5) and (5, 0).

EXERCISES 3(q)
Solve algebraically the
1. Y = 5x + 6
Y = X2
3. Y = x + 5
Y
X2 - 3x
5. Y = x - 3
XY = 10
7. x + Y = 5
X2 + y2
13
9. x - y
1
xy = 2
11. y = 2x - 6
X2 - xy + 2y2
13. y - 2x + 1 =
3y2 - Y
2X2 =
15. x - y + 3 = 0
xy = 10
17. x + y
2
X2 + y2 = 2
19. x + 2y = -8
xy = 8

following simultaneous equations.


2. Y = 3x - 2

y = X2

4. x + y = 15
y = X2 - 6x + 1
6. y - 2x = 1

X2 + y2 = 10

8. y = 2x - 2

y = X2 - X - 2

10. y = 2x - 5

y = X2 - 4x + 4

12. x + y = 5
16
3x2 + xy
y2 = 29
0
14. y - 4x
8 = 0
0
y = 4 - X2
16.
3x + y = 11
2X2 - xy - y = 10
18. x = 2y - 1
3x 2 = X + 2y2
20. y = x + 9
y = X2 - X - 6

EXAMPLE 31
Solve the simultaneous equations
3x + 2y = 4
x 2 + xy - y2 = 1
X2

3x + 2y
+ xy - y2

4
1

..... '.' .............................................. (1)

........................................................ (2)

From (1), 2y = 4 - 3x
4 - 3x
y=
4 - 3x
In place of y in (2) put -

Then

x 2 + x(4 - 3x) _ (4 - 3X)2 = 1

4X2 + 2x(4 - 3x) - (4 - 3X)2 =


4X2 + 8x - 6x2 - (16 - 24x + 9X2) =
4X2 + 8x - 6x 2 - 16 + 24x - 9x2 =
-l1x2 + 32x - 20 =
l1x2 - 32x + 20 =
(l1x - 10)(x - 2) =

4
4
4
0
0
0
10

(Multiply by 4)

(Divide by -1)

x = II or 2

I
<

84 NEW SENIOR MATHS: TWO UNIT COURSE

Substitute these values of x into (1).

10
30

When x
II'
II + 2y = 4

30

When x

22y = 44
22y = 14
7
Y = II

(MUltiply by 11)

6 + 2y = 4

2y = -2

y = -1

2,

The straight line and the curve intersect at

(!~,

7
11) and (2, -1).

Note: It is not essential to transpose (1) to express y in terms of x. We could express x in


terms of y.
From (1),
3x = 4 - 2y
x
4 - 2y
3
In place of x in (2) put 4_---=

+ y(4 - 2y) _ y2
9
3
(4 - 2y)2 + 3y(4 - 2y) - 9y2
9
16
16y + 4y2 + 12y
6y2 - 9y2 = 9
-lly2 --4y + 7 = 0
l1y2 + 4y
7 = 0
(11 y - 7)(y + 1) = 0
(4 - 2y)2

Then

y=

(Multiply by 9)

(Divide by -1)
7

Substitute these values of y into (1).

7
14

When y = II'
3x + II
4
33x + 14
33x

x
When y

-1.

=
=

44
30
10

(Multiply by 11)

II

3x - 2
3x

x = 2 as before.

EXERCISES 3(r)
Solve algebraically the following simultaneous :equations.
1.
2x - 5y = 3
2. Jy - 4x = 0
X2 - 2y2 - 3x = 2
X2 + y2 = 25
3.
x + 2y = 3
4.
3x - 2y
2
xy + 2x + y = 4
X2
xy + y2
21 .
5.
3x + 2y
1
6.
2x
3y + 1
xy + y2 - Y = 8
xy + x + y = 23

CHAPTER 4

Plane
Geometry
4.1 Points and lines _ _ _ _ _ _ _ _ _ _ _ __
We represent points by dots. Perhaps we could define a point as a dot, but how then could
we define a dot? Perhaps we could define a dot as a point!
The word point we leave undefined, but we know that it has position and is infinitely
small. Thus it is impossible to draw a point. It is possible only to draw a dot or a mark that
represents a point.
Similarly, a line cannot be defined. It is not satisfactory to say that a line is an infinite
set of points, but we can think of it as such. When we use the term line, we shall consider it
as a straight line. A line continues indefinitely in both directions and has no width.
If A and B are any two points, we represent point, line, ray and line segment as follows:

Symbol

Meaning

Illustration

.A

A BorAB

A line containing A and B

...

ABor AB

A ray with endpoint A

A.

A line segment with endpoints A and B

A.

Point

ABorAB

...

..

.B

Note: The notation AB may be used to describe a line, a ray, a line segment or the length
of an interval and the context will determine its meaning. For example, AB = CD is a
reference to lengths; X is a point on AB means that AB refers to a line segment.

86 NEW SENIOR MATHS: TWO UNIT COURSE

4.2 An angle _ _ _ _ _ _ _ _ _ _ _ _ _ __
An angle is the union of two rays that have
the same endpoint. The common endpoint A
is called the vertex.
The angle can be named as angle BA C or
angle CAB or angle A.
Angle BA C is written as L BA Cor / BA C
or BAC.

A"'--'----+-~.....

The unit of angle size is the degree and it represents l!oth of a half-turn. The

magnitude of an angle is the amount of turning, measured in degrees, between the arms of
the angle and is denoted by mag L BA C or simply L BA C.
There is a positive real number between 0 and 180 that gives the measure of every angle.
The number that gives the measure of L BAC is denoted by m L BAC. It refers to the
number of degrees and does not include the unit.
If m L BA C = 30, then mag L BA C = 30. We say that angle BA C is an angle of 30 .
However, we shall use the notation L BAC or BAC to describe an angle or its magnitude
and so we shall simply say that L BA C = 30.
8

..

. ..

A
c
Right angle 90

Straight angle 1 80

Acute angle

C
Reflex angle

Obtuse angle

Adjacent angles
In the diagram, LA OB and L BOC have a
common arm OB. These angles are said to be
adjacent.

Vertically opposite angles


If AB and CD intersect at 0, four angles are

formed. Any two of these that are not


adjacent are vertically opposite.
L AOD and
L AOC and
L AOC and
LAOC and

/D
A//O
B

L BOC are vertically opposite.


L BOD are vertically opposite.
C
L AOD are adjacent (and also supplementary).

L COB are adjacent (and also supplementary).

4/PLANE GEOMETRY 87

Supplementary angles
Two angles are supplementary if their sum is 180. For example, LAOe and L.-AOD are
supplementary.
Theorem:

If two straight lines intersect, the vertically opposite angles are equal.

o
A--------,,.,,:;:-------B

Given: AB and eD intersect at O.


To prove: LAOe = LBOD and LAOD = L eOB.
bO, LBOD = co.

Proof: Let LAOD = a O, LAOe


a + b = 180
(Supplementary)

a + c = 180
(Supplementary)

:. a + b = a + c
b
c
and so
LAOe = LBOD
Similarly
LAOD = L eOB
(The symbol :. means 'therefore'.)
Note: Throughout this chapter we will make certain statements without proving them.
Such statements are called axioms. Statements that are proved to be true are called
theorems.
An axiom is a statement that is accepted as true without proof
A theorem is a statement that is proved to be true.

EXERCISES 4(a)
Find the value of the pronumerals in each of the following (1. to 12.):
I

1.

4.

2.

5.

3.

88 NEW SENIOR MATHS: TWO UNIT COURSE

7.

8.
cab-70t (8 +2St
(28-14)0 (4b

10)0

10.

13. Two straight lines PQ and XY intersect at a point R. If the rays RM and RN bisect
L QR Y and L PR Y respectively, prove that L NRM is a right angle.
14. From a point 0, rays OP, OQ, OR, OS are drawn all in the same plane and points P
and R are on opposite sides of the line SOQ. If L POQ = L ROQ, prove that L POS
LROS.
15. How many pairs of adjacent angles are formed when three straight lines intersect at a
point? How many pairs of vertically opposite angles are formed?

4., Parallel lines

Two lines AB and CD are parallel if they are in the same plane and do not intersect. We
write AB II CD to indicate that the lines AB and CD are parallel.
The line PQ, which cuts across the parallel lines, is called a transversal.
L 1 and L 5, L 2 and L 6, L 3 and L 7, L 4 and L 8 are pairs of corresponding angles.
L 2 and L 8, L 3 and L 5 are pairs of alternate angles.
L 2 and L 5, L 3 and L 8 are pairs of co-interior angles.

4/PLANE GEOMETRY 89

Angle properties of parallel lines:

If
1.
2.
3.

two lines are parallel and they are intersected by a transversal, then
corresponding angles are equal,
alternate angles are equal,
co-interior angles are supplementary.

EXERCISES 4(b)
Find the value of the pronumerals in each of the following.

1.

2.

4.

5.

7.

8.

6.

10.

12.

90 NEW SENIOR MATHS: TWO UNIT COURSE

4.4 Angle properties of a triangle _ _ _ _ _ __

'--------~

4 _ _ _ _ _

a+ b+ c = 180

eO
_ __

e= a+b

1. The sum of the angles of a triangle is 180.


2. If one side of a triangle is produced, the exterior angle so formed is
equal to the sum of the two remote interior angles.

Theorem:
The sum of the angles of a triangle is 180.

......--------.:Io.C

A~:.:..----

Given: ABC is any triangle.


To prove: a + b + c
180.
Construction: Through B draw XY parallel to AC.
Proof'
x
a
(Alternate angles equal)
y = c
(Alternate angles equal)
x + b + y = 180 (Straight angle is 180)
.'. a + b + c = 180

Theorem:
A n exterior angle of a triangle is equal to the sum of the two remote
interior angles.

~~

____________

_____

~~e_o

Given: DABC with AC produced through D.


To prove: e = a + b.
Proof'
e + c = 180 (Straight angle 180)
a + b + c = 180 (Angle sum of Dis 180)
e=a+b

4/PLANE GEOMETRY 91

An isosceles triangle is one with two side


lengths equal. It can be proved that the base
angles are equal, i.e. L B
L C. The line
drawn from the vertex A to the midpoint X
of the base is an axis of symmetry. As a
consequence,
BX = XC and LAXB = LAXC
90

An equilateral triangle is a particular case


of the isosceles triangle in which the three
side lengths are equal and the magnitude of
each angle is 60.

"-- __1----- C
A

BU--------..C

'--'----~c

Obtuse-angled triangle
(one angle > 90)

Right-angled triangle
(one angle = 90)

If AB = AC and LBAC

Since AB
LABC
Let LABC
:. 40 + 2a

a
XO

40, find x.

AC, f:.:.ABC is
XO
isosceles
A ."-....;...;;..---+------
c
E
= LACB
aO
= 180 (angle sum of f:.:. = 180)
= 70
= L BA C + L ABC
(exterior angle = sum of interior
= 40 + 70
opposite angles)
110
110

EXAMPLE 2
If DA = DB
right angle.

A cute-angled triangle
(each angle < 90)
B

EXAMPLE 1

Solution:

B'-------C

= DC,

prove that L ABC is a

A ..e::-....:;..._ _ _ _--..;;._.... B

I .

92 NEW SENIOR MATHS: TWO UNIT COURSE

Proof:

L DAB = L DBA
L DCB = L DBC
In LABC,

(6.DAB is isosceles)
(6.DCB is isosceles)

2a + 2b = 180 (angle sum of 6. = 180)


:. a + b = 90
:. LABC = 90

EXERCISES 4(c)
Write down the value of the pronumerals in each of the following (1. to 12.):

4:

2.

3.

5.

6.

8.

9.

II

10.

4/PLANE GEOMETRY 93

13. Given that BA II DE, AB


LACE = 90.'

BC, CD = DE and B, C and D are collinear, prove that

BL..--I-------.fI-----'"

Note: The word "collinear" means "in the same straight line".
14. If one angle of a triangle is equal to the sum of the other two angles, prove that the
triangle is right-angled.
A
8
15. Given that AC = CB and DC
prove that AB II DE.

CE,

0'----------"'" E

16. ABC is a right-angled isosceles triangle with the right angle at C. D and E are points
L BCE. Prove that 6 CDE is isosceles.
on AB such that L ACD

17. In 6ABC, D is a point on BC such that BD = CD

AD. Prove that L BAC

= 90.

18. In each of the two situations, prove that 6ABX and 6CDX are equiangular, i.e. the
angles are equal, given that AB and CD are parallel.

X "'------'-----'
A
0

19. If AB /I CD and AE = AC,


prove that LACE = L ECD.
"0

20. If AB /I DC, AD = AC and AB = BC,


prove that
(a) LADC = LACD = LCAB = LBCA
(b) LDAC = LABC
(c) If L ADC = 40, find the size of L ABC

21. The three angles of a triangle are in the ratio 3:5:7. Find the magnitude of each angle.
22. ABC is a triangle in which AB = AC. AB is produced to D so that BD = BC. Prove
that LACB = 2 LDCB.

94 NEW SENIOR MA THS: TWO UNIT COURSE

4.5 Quadrilaterals _ _ _ _ _ _ _ _ _ _ _ __
A quadrilateral is a plane closed figure
bounded by four straight lines. If we join
one of the diagonals, say AC, the
quadrilateral is divided into two triangles
ABC andADC.
Sum of angles of DABC = 180 0
Sum of angles of DADC
180 0
... Sum of angles of quadrilateral ABCD =

.... ....

The sum of the four angles of any quadrilateral is 360 0 or 4 righ

Special quadrilaterals:
(a) Parallelogram

A parallelogram is a quadrilateral with


both pairs of opposite sides parallel.
(b) Rectangle

A rectangle is a parallelogram with one


angle a right-angle.

..
(c) Rhombus

A rhombus is a parallelogram with a pair


of adjacent sides equal.

(d) Square

A square is a rectangle with a pair of


adjacent sides equal.

A trapezium is a quadrilateral with


one pair of opposite sides parallel. If the
two non-parallel sides are equal, the
trapezium is isosceles and the base angles
will be equal.
(f) Kite

A kite is a quadrilateral with both pairs


of adjacent sides equal.

.... ....

.... ....

........
DL.----------::.I C

360 0

(e) Trapezium

.... ....

..

4/PLANE GEOMETRY 95

EXAMPLE 3
Prove that the opposite angles of a parallelogram are equal.

~I

To prove: LDAB = L DeB


Construction: Draw diagonal AC D C
Proof
LBAC = LACD (AB II DC)
LDAC = LBCA (DA II CB)
LBAC + LDAC = LACD + LBCA (addition)
Le.
L DAB
L DCB
Similarly, by drawing diagonal BD, we can show that LABC = LADC.

4.6 Angles of a polygon _ _ _ _ _ _ _ _ __


Apolygon is a plane figure bounded by any number of straight lines. Certain polygons are
given special names:
Triangle
3 sides
Quadrilateral
4 sides
Pentagon
5 sides
Hexagon
6 sides
Octagon
8 sides
10 sides
Decagon
A polygon which has all its sides equal and all its angles equal is called a regular
polygon. (e.g. equilateral triangle, square). A polygon in which each angle is less than
180 0 is called a convex polygon.
A polygon in which at least one angle is more than 180 0 is called a concave polygon.
convex
polygon

concave
polygon

Theorem:

The sum of the angles of a polygon of n sides is (2n - 4) right angles.

Construction: Join the vertices of the polygon to any point 0 inside it.
Proof: The polygon is divided into n triangles. The sum of the angles of each triangle is
2 right angles and so the sum of all the angles in the figure is 2n right angles.

96 NEW SENIOR MATHS: TWO UNIT COURSE

This, however, includes 4 right angles at O. Hence, by subtraction, the sum of


the angles of the polygon is (2n - 4) right angles.
If the polygon is regular each of its angles is 2n - 4 right angles.

Theorem:

If the sides of a convex polygon are produced in order, the sum of the
exterior angles so formed is 4 right angles.

Proof: At each vertex of the polygon, the sum of the interior and exterior angles is 2 right
angles. Since there are n vertices, the sum of all the interior and exterior angles is
2n right angles. The sum of all the interior angles is (2n - 4) right angles. Hence,
4) right angles
4 right angles.
the sum of all the exterior angles is 2n - (2n

Regular
Octagon

EXAMPLE 4
Find the size of an interior angle of a regular octagon.
Sum of the interior angles = (2n - 4) right angles
= 12 right angles when n = 8
1080

I
1080
E ac h mtenor ang e = -8

Alternatively
Sum of exterior angles

135

4 right angles
360

Each exterior angle

Each interior angle

45
180 - 45
= 135

4/PLANE GEOMETRY 97

EXERCISES 4(d)
1. ABCD is a trapezium in which AB is parallel to DC. The bisectors of angles A and D
intersect at E. Prove that L AED = 90.
2. ABCD is a rhombus in which L BAC
65
(a) Write down 3 other angles whose size

is 65.

(b) Write down 4 angles whose size is

25.

(c) Prove that the diagonals are at right

angles.

_----_8

DI:.-----~

3. Calculate the number of degrees in each interior angle of a regular (a) pentagon
(b) hexagon (c) decagon.
4. Calculate the number of degrees in each exterior angle of a regular (a) pentagon
(b) hexagon (c) decagon.
S. How many sides has a regular polygon, each of whose angles is (a) 90 (b) 108
(c) 165 (d) 160 (e) 168 (f) 170?
6. How many sides has a regular polygon, each of whose exterior angles is (a) 45 (b) 24
(c) 22!0 (d) 10.
7. Show that it is not possible for a regular polygon to have angles of 152.
8. Each interior angle of a regular polygon is 8 times an exterior angle. How many sides
has the polygon?
9. Three of the angles of a pentagon are 98, 112 and 114. If the other two are equal,
what is their size?

4.7 Congruent triangles _ _ _ _ _ _ _ _ _ __


Two plane figures are congruent if they are equal in all respects. If one figure is placed on
top of the other, with corresponding points touching, they fit perfectly.
C
F

AU8DDE

In the diagram above, LABC and LDEF are congruent. The three angles of one
triangle have the same si?:e as the three angles of the other and the sides opposite these
angles are equal in length, i.e.
LA = LD
BC
EF
LB
LE
AC=DF
LC = LF
AB
DE
LABC == LDEF, where == means "is congruent to".
In order to prove that two triangles are congruent, it can be sho~n that it is sufficient to
prove that:
.-------~--------------------------------------------~

(1) two sides and the included angle of one triangle are respectively equal

to two sides and the included angle of the other triangle (SAS);

98 NEW SENIOR MATHS: TWO UNIT COURSE

(2) the three sides oj one triangle are respectively equal to the three sides
oj the other triangle (SSS);

(3) two angles and a side oj one triangle are respectively equal to two
angles and the corresponding side oj the other triangle (ASA);

(4) the hypotenuse and one side oj a right-angled triangle are equal to
the hypotenuse and the corresponding side oj the other right-angled
triangle (RHS).

You will observe that two triangles are not necessarily congruent if
three angles of one triangle are equal to the three angles of the other triangle
(AAA);

(a) the

Such triangles are similar.

4/PLANE GEOMETRY 99

(b) two sides and an angle opposite one of these sides of one triangle are equal to two
sides and the corresponding angle of the other triangle (ASS).

(1)

(2)

(3)

(1) and (3) are congruent.


(1) and (2) are not congruent.

This situation is often referred to as the ambiguous case because sometimes it is

possible to draw two different triangles, (1) and (2), that are not congruent.

Congruence properties are useful in proving certain geometrical facts.

EXAMPLE 5
Prove that (a) the base angles of an isosceles triangle are equal;
(b) the line drawn from the vertex of an isosceles triangle to the mid-point of
the base is perpendicular to the base.
A
Given:
To prove:
Construction:
Proof:

An isosceles 6ABC with AB

= AC

= 90
Join vertex A to midpoint P of base
(a) In 6ABP and 6A CP,

AP = AP (common to both triangles)

BP = CP (construction)

AB
AC (given)

:. 6ABP == 6ACP (SSS)


B
(a) LABC

LACB

(b) LAPB

L..---fl---L---ff--~C

:. LABP = LACP
(b) It also follows from the congruent triangles that

LAPB

But LAPB + LAPC


:. LAPB

LAPC
=

180
90

EXAMPLE 6
Prove that the opposite sides and angles of a parallelogram are equal.
,.----.........----:::;;1IC

________- - - - - J

Given: ABCD is a parallelogram


To prove: AB = DC, BC = AD, L ABC
Construction: Draw the diagonal A C

L ADC, L DCB

L DAB

100 NEW SENIORMATHS: TWO UNIT COURSE

Proof' In 6ABC and 6ADC,


AC
AC (AC is common)
L BAC
L DCA (alternate angles equal)
L BCA
L DAC (alternate angles equal)
.'. 6ABC
6ADC (ASA)
AB = DC, BC = AD and LABC = LADC
Also, since LBAC
LDCA and LDAC
LACB,
then, by addition, L DAB = L BCD.

EXAMPLE 7
ABCD is a square, P, Q and R are points on AB, BC and CD respectively such that
AP = BQ
CR.
Prove that (a) PQ
QR
p
A
B
(b) LPQR
90
Proo!, In 6PBQ and 6 QCR
PB
QC
('.' AP
BQ and AB = BC)
Q
BQ
CR (given)
L PBQ
L QCR (right angles)

:, 6PBQ
QCR (SAS)

c
R
D
(a) :. PQ
QR
(b) L BPQ L RQC C: 6s are congruent)

But L BPQ + L BQP = 90 (sum of angles of a

:. LRQC + LBQP = 90

:. LPQR
90 (': L~QC + LBQP + LPQR = 180)

Note: '.' means 'because'

EXERCISES 4(e)

A~B

1. AB II DC and AO = OC. Prove that

BO = OD by proving that 6A OB and

6 COD are congruent.

D~C
A~

2. ABCD is a square and CX = CY. Prove

that (i) AX = A Y, (ii) L AXB

L A YD by proving that 6ABX and

6AD Yare congruent.

_ _ _.....,B

D .L-----ly-B-...... C

3. Two line, segments AD and BC bisect


each other at O. Prove that AB
CD

and AB II CD by showing that 6AOB

and COD are congruent.

4/PLANE GEOMETRY 101


A

4. Prove that, if the straight line drawn


from the vertex of a triangle to the mid
point of the base is perpendicular to the
base, the triangle is isosceles.

...---_ _ _ _...,.0

5. PQRS is a quadrilateral in which PQ =


SR and SP = RQ. Prove that
(i) DPQS == DRSQ,
(ii) L PQS = L QSR,
(iii) PQ II SR.
(iv) PQRS is a parallelogram.

6. Given that AB = AC and DB


show that L ABD = L ACD.

S 11:.------'

DC,

8 1-----'------.... C
E

7. If AC = AD and AB bisects L CAD,


prove that
(i) DABC == DABD,

(ii) BC = BD.

8. ABC is a triangle in which L B = L C. The bisector of angle A meets BC at D. Prove


that DABD and DACD are congruent. Hence prove that AD is perpendicular to BC.
A

9. DABC and DADE are isosceles. Show


that L BAD = LEAC.

DOE

10. DABC and DBCD are isosceles. Prove

~--+-----J

that
(i) LABD = LACD,
(ii) L BAD = L CAD.

102 NEW SENIOR MATHS: TWO UNIT COURSE

11. ABCD is a square. X and Yare points on BC and CD respectively such that

BX

CY. Prove that

(a) AX
BY,
(b) AX ..L BY.

12. ABCD is a parallelogram and BE

DF.
Show that AE = FC and AE II FC.

J21YE

13. P is a point inside a square ABCD such

A,..;::-_ _ _~B

that triangle PDC is equilateral. Prove


that
(a) 6APD == 6 BPC,
(b) 6APB is isosceles.

14. P and Q are the midpoints of the equal

sides AB and AC of the isosceles triangle


ABC. Prove that PC
QB by proving
th;:tt 6PBC == 6QBC.
8

IC-- _ _ _---""

15. ABC is an isosceles triangle with AB =

AC. 0 is the midpoint of BC and OP


and OQ are drawn perpendicular to AB
and A C respectively. Show that OP =
OQ.

16. Prove that the diagonals of a


parallelogram bisect each other i.e.
prove that in the parallelogram PQRS,
PO = OR and SO = OQ.

17. Prove that the diagonals of a rectangle


are equal.
Hint: Prove that 6PQR and 6PQS are
congruent.

L.--"':::'o"--~

Z>;<ZQ

4/PLANE GEOMETRY 103

18. Prove that the diagonals of a rhombus


bisect each other at right angles i.e.
prove that
(a) AO
OC and DO = OB.,
and (b) L AOB = 90.
Prove also that the diagonals of a
rhombus bisect the angle at the vertex
i.e. LDAO = LBAO, LADO
L CDO etc.
19.

F, G andH are the midpoints of


the sides AB, BC, CD and DA
respectively of the parallelogram
ABCD. Assuming that the opposite
sides and angles of a parallelogram are
equal, prove that
(i) D.AEH == D. CFG,
(ii) D.EBF == D.DHG~
(iii) EFGH is a parallelogram.

x------::::r

;JCyB

..---------".8

20. ABCD is a quadrilateral whose


diagonals A C and BD bisect each other
at right angles. Prove that ABCD is a
rhombus.

D ""-_ _ _---Jf

21. P and Q are two points inside a

parallelogram ABCD such that AP


QC and AP II QC. Prove that
(a) D.APC
D.AQC,
(b) PC II AQ,
(c) PC
AQ,
(d) AQCP is a parallelogram.

;:SS;;S;B

22. ABCD is a parallelogram and AP


QC. Prove that
(a) D.APD == D.BQC,
(b) PD = QB,
(c) PD II QB.

23. PQR is an isosceles triangle with PQ PR. SQR is also an isosceles D. with SQ = SR
and S is on the side of QR opposite to P. Prove that
(a) D.PQS == D.PRS,
(b) P, X and S are collinear, where X is the mid-point of QR.

104 NEW SENIOR MATHS: TWO UNIT COURSE

24. ABCD is a quadrilateral with AB = DC


and L BA C
L BDC. Prove that
(a) PB
PC,
(b) PA
PD,
(c) AC
BD,
(d) /:iABC == DDBC.

25. ABCD is a parallelogram whose


diagonals intersect at O. Through 0 a
straight line XY is drawn. Prove that
OX
OY.

l:'kl

Dye

26. ABCD is a parallelogram. P and Q are points on AB and CD respectively such that
AP = CQ. Prove that the perpendicular distances of P and Q from the diagonal BD
are equal.

27. Prove that if one pair of opposite sides of a quadrilateral are equal and parallel, then
the other pair of opposite sides are also equal and parallel.

28. From the mid-point 0 of AB, the line


CD is drawn at right angles. Prove that
(a)
OC == DBOC,
(b) DAOD == DBOD,
(c) DCAD == DCBD.
The quadrilateral A CBD sometimes is
called a 'kite'.

29. Which of the quadrilaterals (a) parallelogram (b) rectangle (c) rhombus (d) square
(e) trapezium possess the following properties:
(i) opposite sides are equal,
(ii) opposite angles are equal,
(iii) diagonals bisect each other,
A
(iv) diagonals are equal,
(v) diagonals are at right angles.
30. ABC is an acute-angled triangle. The
bisector of angle A meets the perpendi
cular bisector of BC at P.
(a) Prove that DPBM == DPCM and
hence that PB = PC.
(b) Draw perpendicular lines PX and
PY to AB and AC respectively.
Prove that DAPX
DAPY and
hence that AX
A Y and P X
PY.
(c) Prove that DPXB == DPYC and
hence that XB = YC.
(d) Hence show that AB
AC.

You have now proved that any


DABC is isosceles. Where is the
fallacy in the above proof?

4/PLANE GEOMETRY 105

4.8 Equal intercepts - - - - - - - - - - -


Theorem:

If the intercepts cut off on one transversal by three (or more) parallel
lines are equal, the intercepts cut off by them on any other transversal are
also equal.

Given: ABC and DEF are two transversals of three parallel lines AD, BE, CF and
AB = BC
To prove: DE = EF
Construction: Draw DG and EH parallel to A C
Proof' ADGB is a parallelogram
DG
AB (opposite sides of II gram are equal)
BEHC is a parallelogram
EH = BC (opposite sides of II gram are equal)
But AB = BC (given)
DG = EHandDG II EH
In D.DGE and D.EHF
DG
EH (proved)

L DEG
L EFH (BE II CF)

LGDE = LHEF(AD II BE)

.'. D.DGE == D.EHF

DE
EF

Theorem:

The straight line joining the mid-points of two sides of a triangle is


(a) parallel to the third side,
(b) one half of the length oj the third side.

~--------::'''c-----~ F
BL----------------~

106 NEW SENIOR MATHS: TWO UNIT COURSE

Given: D and E are the mid-points of AB and AC respectively


To prove: DE II BC and DE

~BC

Construction: Produce DE to F so that EF = DE; join FC


Proof' In 6ADE and 6FEC
AE = EC (given)
DE = EF (construction)
L AED
L FEC (vertically opposite)
6ADE == 6FEC (SAS)
CF = AD
BD
Also LADE = L CFE (congruent triangles)
CFII BA
.'. BCFD is a parallelogram (CF and BD are equal and II)
DEIIBC
Also
DF = BC (opposite sides of II gram are equal)
But
DE = EF (construction)
DE =

~DF

DE

!BC
2

EXERCISES 4(f) (For 3-Unit students)

1. D, E and F are the mid-points of the

sides AB, BC and CA respectively.


Name 3 parallelograms.
(b) Name 4 congruent triangles.
(a)

c
2. E, F, G and H are the midpoints of the

sides AB, BC, CD and DA respectively of


the quadrilateral A BCD. Prove that
(a) HE II DB,
(b) GF II DB,
(c) GF II HE,
(d) EFGH is a II gram.

E
A

3. D, E and Fare the mid-points of AB, BC


and CA respectively. Prove that
(a) FG = GD,
(b) AG
GE.
C

4. PQRS is a trapezium with PQ II SR. If A,

Band C are the mid-points of SP, PR and

QR respectively prove that

(a) AB II SR,
(b) BC II PQ,
(c) the points A, Band C are collinear.

4/PLANE GEOMETRY 107

5. PQRS is a trapezium with PQ II SR. If A


and Bare the mid-points of SP and RQ
respectively prove that
(a) AB is parallel to PQ and SR,
(b) AB =

~(PQ +

..

SR).

AI
/

~8

..

6. Prove that the three straight lines joining the mid-points of any two of the three sides
of an equilateral triangle, divides the triangle into four equilateral triangles.

7. D is the mid-point of the side BC of ,6,ABC. Perpendicular lines BX, DY and CZ are
drawn from B, D and C respectively to any straight line through A. Prove that
XY
YZ.

:~:

8. P, Q, Rand S are the mid-points of the


sides of a rectangleABCD. By joining the
diagonals A C and BD prove that PQRS is

a rhombus.

ORe

4.9 Similar triangles _ _ _ _ _ _ _ _ _ _ _ __


R

p .........- - - - - -_____-:-..:::....1

A~------------~

Two triangles are similar if


(a) the angles are equal,
or (b) the corresponding sides are proportional,
or (c) two pairs of corresponding sides are proportional and their
included angles are equal.
In the two triangles, the corresponding sides are:
AB andPQ
BC and QR

CA and RP and the equal ratios are:

AB
BC
-CA
PQ
QR
RP
The tests for similarity of the triangles ABC and PQR are:
(a) LA = L P; L B
L Q; L C = L R.
(b) AB = BC
CA
'PQ QR
RP

AB
AC

(c) PQ = PR and LA
LP.
We use the symbol
'is similar to'.

1\1

to denote 'is similar to'. Frequently the symbol - is used to denote

,6,ABC

1\1

,6,PQR

108 NEW SENIOR MATHS: TWO UNIT COURSE

EXAMPLE 8
If PQ II BC, prove that 6APQ III 6ABC.
Proof' In 6APQ and 6ABC,
L APQ = L ABC (corresponding angles)
L AQP = L ACB (corresponding angles)
L QAP L CAB (common angle)
A
3cm
P 2cm 8
:. The triangles are equiangular and so 6APQ III 6ABC.
Note: To prove that two triangles are.eqtiianglilar,it is sufficient to prove that .two
angles of one triangle are equal to two angles of the other. Why?
Since the two triangles overlap, it is convenient to draw them separately .

..c
Q

ALjp

_ _ _ _ _----L--I

3cm

The
AP _
AB The

corresponding sides are in the same ratio:

AQ _ PQ _ 3

AC - BC - .5

enlargement factor is l~.

The sides of 6APQ are

~ the corresponding sides of 6ABC.

The sides of 6ABC are

~ the corresponding sides of 6APQ.

If AQ

If BC

3'6cm, AC

3" x 3'6

3'2cm, PQ =

.53

x 32

6cm.

= 192cm.
A

EXAMPLE 9

Given:
To prove:
Proof'

L PAB
L BQC

6APB III 6BCQ

L PAB
L BQC (given)

L ABP = L CBQ (vertically opposite)

L APB = L BCQ (remaining angles are equal)


:. 6APB III 6BCQ

If AB = 42 cm and BQ

28 cm, the sides of 6BCQ are ~ of the corresponding sides of

6ABP.
BC
BP
BP
or BC

BQ
AB
AB
BQ

CQ _ 2
AP - 3"
AP
3

CQ
2

4/PLANE GEOMETRY 109

If PB = 3 cm, BC

= 3"2 x 3 cm = 2 cm

If CQ = 2'4cm, AP =

2:3

x 24cm

3'6cm

EXERCISES 4(g)
1. If the length of each side of a triangle is doubled, will the resulting triangle be similar
to the original one?

2. (a)

(b)

...c

8'------~

~----

If L APQ = L ABC, prove


DAPQ III DABC.

If LAXY = LACB, prove


DAXYIliDABC.
p

3. If L S
L Q,
(a) prove DPST III DRQT,

. PS
PT
(b) complete the ratios RQ =

ST

5
p

4. If ST II QR,
(a) prove DPST III DPQR,

. PS
?
(b) comp Iete t h e ratIos PQ = QR

= PR'

(c) If QR = 5 cm, ST = 2cm,

SP = 1 cm, find the length of PQ.

5. DABC and DADE are right-angled.


(a) Prove DABC III DAED,

. AB AC BC

(b) CompletetheratlOsT = T = T '

6. Find the value of the pronumerals in each of the following pairs of similar triangles:
(a)

(b)

3'5

(d)~
55

(c)

X~~'.5

110 NEW SENIOR MATHS: TWO UNIT COURSE

7. (a)

(b)

AotC--------'--._----3

In (a), EBFD is a parallelogram. Prove that DADE III D CDF. If AE = 3 cm and


EB

= 2 cm, what is the ratio ~~?

The instrument in (b) is called a pantograph; it is used for copying drawings, plans,
etc. on an enlarged scale from the original. The four rulers are loosely hinged at E, B,
F and D and can be moved at E and F so that EBFD is a parallelogram. Point C is
fixed. As point D moves around the original drawing, point A traces out an
enlargement. If E is at the midpoint of AB, what is the enlargement factor?
8. Find the value of the pronumerals in each of the following pairs of similar triangles:
(a)

(b)

(g)

(h)

4/PLANE GEOMETRY 111

(j)

(i)

9. A line drawn parallel to the side BC of a triangle ABC cuts AB at X and AC at Y.


Name the two similar triangles.
0) If AB
10 cm, BC
15 cm, AX = 6 cm, find XY.

OJ) If A Y
5 cm, YC = 25 cm, XY = 8 cm, find BC.

(iii) If AC = 8cm, YC = 3 cm and BC is 5 cm longer than XY, find BC.


10. Vicky is asked to measure the height of a tree. In order to do so she measures the
length of the shadow of the tree and also the length of her own shadow.

Shadow

Shadow

Vicky's height is 1'6 m and the length of her shadow is 2'4 m. What is the height of
the tree if its shadow is 36 m long?
T

11. On an overcast day, Chris was asked to


measure the height of a wall. Since there
was no shadow, he used a mirror M
placed on the ground.
The light from the top of the wall

travels along TM and ME to his eye.

Angles TMR and EMF are equal, and

so l:::. TMR and l:::.EMF are similar.

If EF = 1'6m, FM
0'8m and

MR
10m, what is the height of the

wall?

F Q8m M

R,

12. A stick 1 metre long placed vertically on the ground casts a shadow 60 cm long, and at
the same time a building casts a shadow 3 m long. What is the height of the building?
13. Jan is 12 m tall and casts a shadow 36 m long. Her friend Paul, who is standing next
to her, is 16 m tall. What is the length of his shadow?
L
:;:"',

// i

14. A boy, 1.'6 m tall, standing 3 m from a


street light casts a shadow 2 m long.
How high is the light above the ground?
15. A girl, 12 m tall, stands 45 m from a
street light and casts a shadow 15 m
long. How high is the light above the
ground?

I"

2m -,-

Shadow

3m

-----------~

--~- ~~~ ~ ~-

112 NEW SENIOR MATHS: TWO UNIT COURSE

16. To find the width of a creek, Vinh


walked from B to C to D to a point E
such that ECA was a straight line.
If BC = 20m, CD = 10m,DE = 5m,

what is the width AB of the creek?

17. A ladder 5 m long leans against a wall,


with the top of the ladder reaching 4 m
up the wall and its foot 3 m from the
wall. A man is at the point D 2 m up the
ladder.
(i) Prove that D ADE III D BDF.
(ii) Find the man's height DE above the
ground.
(iii) Find his distance DF from the wall.

4m

18. A road up a hillside has a constant slope


of 1 in 10, i.e. for every 10'units up the
hill there is a vertical rise of 1 unit. What
is the vertical rise if the distance up the

hillside is 80 m?

1 0 _ _-

19. The diagram (not to scale) shows how


GH measures the height of a tree TR
with the aid of a small mirror M placed
on the ground.
Graham, who is 18 m tall, places the
mirror 5 m from the tree and 30 cm from
himself in order to see the top of the
tree.
(a) Prove that D TRM III D GHM.
(b) Calculate the height of the tree.

---.~ 1

G
R

20. From the vertex A of a right-angled


D ABC, AD is drawn perpendicular to

BC.

(a) Prove that the 3 triangles D ABD,


D ADC ~ and D ABC are similar to
one another.
(b) If AB = 15 cm and AD = 12cm,
find the lengths of BD, DC and AC.

~-~--

4/PLANE GEOMETRY 113

21. A ladder AB, 10 m long, rests against a


wall with B 6m from the wall.

(a) How far up the wall does the ladder

reach?

(b) Dennis stands on the ladder at a


point 4 m up the ladder. Prove that

h,ADFIII h,DBE.

(c) Calculate the distances DE and DF.

22. The figure (not to scale) illustrates the


principle of a camera. :{...ight from the
object AB passes through a small hole P
to form an inverted image DC.
(a) Prove that h,APB III h,DPC.
(b) How far in front of P must an object

27 m high be placed so that an

image 25 cm high is formed 3 cm

behind P?

23. ABeD is a billiard table whose dimen


sions are 12 units by 6 units. A player
strikes a ball at E and it hits a ball at F.
(a) Name two similar triangles.
(b) If AE = 4 units and CF = 4 units,
what is the distance BX?

.,.,..'
"
............

,. '" '" '"

""

,,'

.......

.............

C 4

F
A

24. h,ABC is isosceles with side lengths


13cm, 13cm, 10cm. DEFG is, a
rectangle withEF
9 cm.
(i) Calculate the length of AX.
(ii) Prove that h,AXC III h,EFC.
(iii) Calculate (a) the length of FC,
(b) the width of the rectangle.

G X F

25. A man 2 m tall casts a shadow 3 m long


from a light 10m above the ground.
(i) What is the horizontal distance
from the man to the light?
(ii) What would be the length of the
shadow of a person 15 m tall stand
ing in the same position?
(iii) How far horizontally from the light
would a person 1'6 m tall have to
stand to cast a shadow 32 m long?

10m

3m

114 NEW SENIOR MATHS: TWO UNIT COURSE

26. A straight line through the vertex A of a parallelogram ABCD meets BC at E and DC
produced at F. Prove that
(a) DABE III DCEF (b) DABE III DADF

. AB
BE

Complete the ratIo FD


?

27. PQR is a triangle and a line drawn parallel to PQ meets QR at X and PR at Y.


Through P a line is drawn parallel to QR meeting XY produced at Z. Name three
similar triangles.
28. ABC is a triangle and BE and CF are drawn perpendicular to AC and AB
respectively. Prove that triangles AFC and AEB are similar.

=
DEDA. State two ratios each of which is equal to ~~.

29. The sides AB and DC of a quadrilateral are produced to meet at E. If L EBC

LADE, prove that DEBC III

30. D, E and F are the mid-points of BC, CA and AB respectively of DABC. Prove that
DABC III DDEF. What is the ratio of their corresponding sides?

4.10 Theorem of Pythagoras _ _ _ _ _ _ _ __


The square on the hypotenuse of a right-angled triangle is equal to the
sum of the squares on the other two sides.

Given:
To prove:
Construction:
Proof'
,

\1

DABC with LA = 90
BC2 = AB2 + AC2
Draw AD 1. BC
In DABD and DABC
LBDA
LBAC (right angles)8
LDBA
LABC
(common angle)
LBAD = LACB
(third angles equal)
DABDIII DABC

AB
BD

BC
AB

i.e.
. AB2
BC. BD
In DACD and DABC
LADC = LBAC (right angles)
LACD = LACB (common angle)
LDAC = LABC (third angles equal)
DACDIII DABC
AC
DC
BC
AC
i.e.
AC2 = BC.DC
Add (1) and (2)
:. AB2 + AC2 = BC.BD + BC.DC
= BC(BD + DC)
BC.BC
= BC2

(1)

(2)

4/PLANE GEOMETRY 115

Note: 6ABD and 6ADC are also 'similar, and hence ~~ =

~! i.e. DA 2

DB. DC (3)

In words, (3) can be expressed as:


The square on the perpendicular is equal to the product of the segments of the base.
In words (1) and (2) can be expressed as:
The square on either of the sides containing the right angle is equalto the product of the
hypotenuse and the segment of the hypotenuse adjacent to that side.
The converse of a statement is formed by interchanging the hypothesis and conclusion
or in a geometrical proof interchanging what is given and what has to be proved.
Converses are not always true e.g.
Statement: If two triangles are congruent, then they are similar (True).
Converse: If two triangles are similar, then they are congruent (False).
The converse of the Theorem of Pythagoras states:

If the square on a side of a triangle is equal to the sum of the squares on


the other two sides, the angle contained by these sides must be a right
angle.
A

c
Given:
To prove:
Construction:
Proof:

6ABC in which AC2= AB2 + BC2


L ABC = 90
Draw '6DEF in which EF = BC, DE = AB and L DEF
In 6DEF
FD2 = DE2 + EP (theorem of Pythagoras)
But A C2 = AB2 + BC2 (given)
FD2
AC2
(AB = DE and BC
EF)
FD = AC
In 6ABC and 6DEF

AB = DE
(construction)

BC
EF
(construction)

A C = FD
(proved)

:. 6ABC == 6DEF
(SSS)
. LABC = LDEF
90
(since L DEF = 90)

90

EXERCISES 4(h)
1. Calculate the length of the diagonal of a rectangle whose sides are: (a) 5 cm, 12 cm;
.
(b) 8 m, 10 m; (c) 15 cm, 36 cm.
2. Calculate the length of the diagonal of a square whose side is: (a) 5 m; (b) 36 cm. _,
3. Calculate the length of the side of a square whose diagonal is: (a) 12 cm; (b) 6.J2 cm.
4. Calculate the altitude of an equilateral triangle whose'side is 10 cm.

116 NEW SENIOR MATHS: TWO UNIT COURSE

5. Calculate the length of each side of a rhombus whose diagonals are: (a) 12 cm, 16 cm;
(b) 4 cm, 9'6 cm.
6. The lengths of the sides of a rhombus and the length of one diagonal are respectively
20 cm, 24 cm. Calculate the length of the other diagonal.
7. A motorist starts from a point A and travels 32 km north, 24 km east, 25 km south
and finally returns to A. What is the total distance travelled?
8. Two roads intersect at right angles. Two cars start at the same time from the inter
section. One travels at 60 km/h along one road and the other travels at 80 km/h along
the other road. How far apart are they after (a) 6 min, (b) 15 min, (c) 30 min,
(d) 36 min, (e) 45 min, (f) 60 min?
9. In figure (a), AB = 5 cm, BC = 6cm, DE

10cm, AD

13 cm. Find the length of

CD.
(a)

(b)

o
A

Bn--?'--_-uE

c
B

10~'

In figure (b), AB

24 cm, AC

CD

25 cm. Find the length of (i) BC, (ii) AD.

11. The two equal sides of an isosceles triangle are each 13 cm long and the base length is
10cm. Calculate the height of the triangle.
12. Calculate the length of the altitude of the isosceles triangles whose side lengths are
(a) 10cm, 10cm, 12cm
(b) 25cm,25cm, 14cm
(c) 52cm, 5'2cm, 4cm
(d) 3cm, 3cm, 36cm
24

13. A rectangular sheet of paper is 24 cm by

18cm. If it is folded flat along the line

CD, how many cm is B then closer to A ?

I'

1
18

1----------1

B

14. When a rectangular sheet of paper 16 cm

by 9 cm is cut in the manner shown, the

pieces can be rearranged to form a


square. What is the perip:1eter of this

square?'

4/PLANE GEOMETRY 117

15. Two vertical poles standing on horizontal ground at points 9 m apart are of lengths

6 m and 12 m. Find the length of the straight wire joining the tops of the poles.

16. ABC is a triangle in which AB is greater than AC. AD is drawn perpendicular to BC.

Prove that

+ DC2 = AC2 + DB2

AB2

17. ABCD is a quadrilateral whose dia


gonals intersect. at right angles. Prove
that
AB2 + CD2 = Be2 + DA2
B
18. In a trapezium ABCD, AB is.parallel to DC and the diagonal BD is perpendicular to

AB. Prove that

AB2

+ BC2

19. ABC is a triangle in which AB = 3 cm, BC


of the perpendicular from A to BC.

AD2

+ DC2

5 cm, LA = 90. Calculate the length

20. PQR is a triangle, right. angled at P. S is any point on PQ and T is any point on PR.

Prove that

QT2

+ RS2

QR2

+ ST2

21. In the parallelogram ABCD, the dia


gonal BD is perpendicular to AB. If the
diagonals intersect at E, prove that

AD2 = AE2

+ 3ED2

'

22. ABC is an equilateral triangle and X is a point on BC such that CX


that AX2 = 7BX2

2BX. Prove

4.11 Area _ _ _ _ _ _ _ _ _ _ _ _ _ _ __
Area of rectangle = lb
= length x width
b

h
I

Area of parallelogram = area of rectangle


lh
= base x 1- height

Area of triangle

21 area of rectangle

!bh

2 base x

1-

height

118 NEW SENIOR MATHS: TWO UNIT COURSE

Area of trapezium

= area

h,ADC

= !ah

+ !bh

+ area

h,ABC

= 2<a + b)h
1
..
.
= 2: sum of II sides x

Area of rhombus

= area

h,ADB

= 2:DB . AO

..L

height

area h,BCD

~---+------",,8

+ 2:DB . OC

= !DB(AO +

~C)

..

= 2:DB . AC
1

= 2: product of the diagonals


Units for area:
10000cm 2 = 1 m 2
10000 m 2 = 1 hectare (ha)
A hectare is an area equivalent to the area of a square of side length 100 m . .

EXERCISES 40)
1. Write down the area and perimeter of the following rectangles: (a) length 20 cm,
width 8 cm; (b) length 10 m, width 5 m; (c) length 122 m, width 150 cm; (d) length
18 cm, width 120 mm.

~i

2. Find, in hectares, the area of the following rectangles: (a) length 150 m, width 90 m;
(b) length 3 km, width 120m; (c) length 2 km, width 05 km; (d) length 180m, width
40m.
3. Find the perimeter of the following rectangles: (a) area 84 cm2, length 14 cm; (b) area
144 m2, width 240 cm; (c) area 5 ha, length 1 km; (d) area 72 ha, length 800 m.
4. Find the area of the following squares: (a) length 25 cm, (b) length 16 m,
(c) perimeter 24 cm, (d) perimeter 140 m.
5. Find the perimeter of the following squares: (a) length 31 cm, (b) area 49 m2,
(c) length 130 m, (d) area 256 cm2, (e) area 16 ha, (f) area 225 ha.
6. A room 10 m by 8 m has a rectangular carpet that leaves a border 05 m all round.
What is the area of the carpet?
7. A rectangular swimming pool 30 m by
20 m has a concrete path 3 m wide all
round. What is the area of the path?

30m

~~

20m
~

'' -

~.;

4/PLANE GEOMETRY 119

8. A rectangular field has dimensions 600 m by 250 m. How many tonnes of fertilizer are
needed if it is to be applied at the rate of 70 kg per hectare?
9. How many rectangularliles 10 cm by 5 cm are needed to cover a rectangular benchtop
3m by 2m?

10. The perimeter of a rectangle whose length is twice its width is 36 cm. What is its area?
11. The perimeters of a square and of a rectangle whose length is twice its width are each
24 cm. Show that their areas are in the.ratio 9:8.
12. To cover a rectangular benchtop, 400 tiles each 10 cm by 5 cm are used. If square tiles
of length 4 cm are used, how many would be needed?
13. The dimensions of a rectangle are both increased by 10 per cent. What is the
percentage increase' in the area?
14. The midpoints of the sides of a 2 m
square are joined to form a smaller
square. What is the ratio of their areas?

15. Find the area of the following triangles. (The use of the Theorem of Pythagoras will
be necessary in some cases.) All measurements are in cm.

(C)~

(b)

10

26

35
(f)

(d)

72

(e)28~

50

16

16. (i) Find the area of the coloured region in each of the following. (Measurements are
in cm.)
3
(a)

(c)

(b)
12

(d)

(f)

(e) 1.5

17

24

.J .

l20'NEW SENIOR MATHS: TWO UNIT COURSE

(ii) Find also the perimeter of the coloured regions (b) to (f).

17. A motor mower cuts strips 40cm wide. After 6 journeys with no overlaps round 4
sides of a rectangular lawn 32 m by 26 m, what percentage of the lawn has been
mowed?
18. If the length of each side of a square is increased by 50070, by what percentage is the
area increased?

CHAPTER 5

Probability

We often make uncertain statements like the following:


It will probably rain today.
I have a good chance of passing my exams.
There is a 50-50 chance that a head will come up when I toss a coin.
I have only a slight chance of being dux of the class.
In each case there is some doubt.:about the outcome, but the degree of doubt is not the
same in each case. Probability is a study of events that mayor may not happen, rather
than of events that will happen or that have already happened.

If we toss a coin, a head may turn up or it may not turn up. Since a head or a tail must
turn up, there are two possible outcomes, H or T, and each is equally likely. We say that

the probability of a head is


P (H) =

!. We write this as:

This can be expressed in a variety of ways:


There is a 50-50 chance that a head will turn up.
There is a 1 to 2 chance that a head will turn up.
The odds are 1 to 1 that a head will turn up.
It is an even-money bet that a head will turn up.
Before beginning a game of tennis, cricket, basketball, etc., the opposing sides toss a
coin. Why?

5.1 Experiment with a coin _ _ _ _ _ _ _ _ __


Toss a coin 10 times, 50 times, 100 times, 200 times, 500 times and count the number of
heads. Complete the following table:

122 NEW SENIOR MA THS: TWO UNIT COURSE

Number of tosses

10

50

100

200

500

Number of heads
If you toss the coin 10 times, how many heads would you expect? If you toss the coin 50
times, how many heads would you expect? Would you be surprised if you didn't get
exactly the number you expected?
An American soldier, while a prisoner of war in World War II, performed the
experiment of tossing a coin 1000 times. He actually performed the experiment 10 times
and obtained the following numbers of heads: 502, 511, 497, 529, 504,476,507,528,504,
529.

r;-;l r;:;l
~ ~

[;]

Faces

Die

A die (plural: 'dice') is a small cube with its 6 faces numbered with dots from 1 to 6 as
shown in the diagram. If the die is rolled on a table, one of the 6 numbers will appear
uppermost; for example, in the diagram, 5 appears uppermost.
Each number is equally likely to turn up. Why?
What is the probability that a 5 will appear uppermost? There are 6 possible outcomes
and one of these 6, namely the 5, is favourable to what we want. Thus the probability of
.

. 1

a 5 appeanng uppermost
P (5) =

IS {

This can be expressed in a variety of ways. There is a 1 to 6 chance that a 5 will turn up.
The odds are 5 to 1 against a 5 turning up. This means that, for every 1 favourable
/' outcome, there are 5 unfavourable outComes.
We may define probability as being the ratio of the number of favourable ol,ltcomes to
the number of possible outcomes, assuming that the outcomes are equally likely. The
probability P (A) of a particular result A is given by:
(A)

number offavourable outct;Jmes


number of possible outcomes

5.2 Experiment with a die _ _ _ _ _ _ _ _ __


Roll a die 6 times, 60 times, 120 times, 300 times, 600 times and count the number of times
a 5 (or any other number) appears uppermost. Complete the following table:
Number of rolls of die

60

120

240

300

600

Number of times 5 appears


If you roll the die 6 times, how many fives would you expect? If you roll the die 60
times, how many fives would you expect? Would you be surprised if you didn't get exactly
the number you expected?

5/PROBABILITY 123

From the definition of probability as the number of favourable outcomes + number of


possible outcomes, we can see that, if the number of favourable outcomes is equal to the'
number of possible outcomes, the probability is 1. In this case the event is certain to
happen.
It is certain that the sun will rise in the east tomorrow. The probability of a number less
than 7 when a die is rolled

1 (a certainty). Can you think of other situations where

the probability is I?
If there are no favourable outcomes, then the probability is O. Many athletes have run
in a 100-metre race but no athlete has run the distance in 5 seconds. Thus the probability
that an athlete will run 100m in 5 s is O. Probability that a 7 will appear when a die is
rolled =

O. Probabilities then will lie between 0 and 1.

o <;

P (A) <; 1

To win Lotto, you must usually pick the 6 winning numbers and you have the 40
numbers 1 to 40 to pick from. The number of different ways you can pick 6 numbers when
you have 40 to pick from is 3 838380 and only one of these is favourable.
Probability of picking the 6 winning numbers

= 3 83~ 380 = 0'0000003. Rather small!

The following table shows the number of live births and the number of males and
females born in Australia over the three-year period 1975 to 1977.

Year

Live births

Males

Females

Proportion
of males

1975
1976
1977

233012
227810
226291

119850
116838
116551

113162
110972
109740

0'514
0513
0'515

With rare exceptions, statistics over the years indicate a slight excess of male over
female births.
number of males born
P (a newborn child is a boy)
total number of births
~ 051
Life assurance companies prepare 'life tables', Which show the number of Australian
people per 100000 alive at any given age. Such a table shows, for example, that after 60
years 74251 are expected to be alive.
P (an Australian will live until age 60) =

17062~~ ~

074

The probabilities of the events discussed are shown on the scale on page 124.

EXAMPLE 1

A die is rolled on the floor. What is the probability that the nu.mber appearin~ uppermost
is (0 odd, (ii) divisible by 3, (iii) odd or divisible by 3, (iv) odd and divisibl4 by 3?
As we have seen, there are 6 possible outcomes as a result of rolling a die and they are
equally likely.
(i) There are 3 odd numbers, namely 1, 3, 5, and hence 3 favourable outcomes.

3
1

:.P(odd)
6
2

124 NEW SENIOR MATHS: TWO UNIT COURSE

(ii) There are 2 numbers divisible by 3, namely 3, 6, and hence 2 favourable outcomes.

=~ =~

:. P (divisible by 3)

(iii) 1, 3, 5, 6 are favourable; hence there are 4 favourable outcomes.

:. P (odd or divisible by 3)

=~ =~

(iv) 3 is the only number that is odd and also divisible by 3; hence there is only 1

favourable outcome.

:. P (odd and divisible by 3) =

Certain

. . - - The Sun will rise in the east tomorrow

og

oa

An Australian will live until age 60

07
06
A newborn child will be a boy

05

A head will turn up when a coin is tossed

04

03

02

A 5 will turn up when a die is rolled


01
You will pick the 6 winning numbers in Tattslotto
Impossible 0 ....-~--. An athlete will run a 100 m race in 5 s

5.3 Complementary events _ _ _ _ _ _ _ _ __


The complement of event A is the event 'A does not occur' and is denoted by A. If an
experiment has n possible outcomes, m of which are associated with event A and so (n - m)
are associated with event A, then
P(A)

m
n

P(A) = n - m

i.e.

I P(A)

m
n

+ P(A)

=II

1 - P(A)

"

5/PROBABILITY 125

EXAMPLE 2
A die is rolled on the floor. What is the probability that the number appearing uppermost
is not 4?
1
P(a 4 appears uppermost)
6
1
5
:. P(a 4 does not appear uppermost) = 1

An ordinary pack (deck) of playing cards contains 52 cards made up of 13 diamonds, 13


hearts, 13 clubs and 13 spades. If the pack is well shuffled and a card is drawn at random,
each of the 52 cards is equally likely to be drawn and hence has a probability of
being drawn.

,,

+
A

5~

of

~+ 4
tit

+ +.5

EXAMPLE 3
From a pack of 52 playing cards one card is drawn at random. What is the probability that
it is (i) a diamond, (ii) not a spade, (iii) an ace, (iv) a diamond or an ace, (v) a diamond and
an ace, (vi) a heart and the ace of clubs?
There are 52 equally likely outcomes.
(i) There are 13 diamonds and so
P (diamond)

= ~~

(ii) There are 13 spades and hence 39 cards that are not spades.

:. P (not a spade)

~~

(iii) There are 4 aces and so


4
1
P (ace)
52
13
(iv) There are 13 diamonds and 4 aces but there is one diamond that is also an ace. Hence
there are 16 cards that are favourable to what we want.

16
4

:. P (diamond or an ace) 52
T3
(v) The ace of diamonds is the only card that is a diamond and also an ace .

.'. P (diamond and an ace)

(vi) There is no card that is a heart and also the ace of clubs.
:. P (heart and the ace of clubs) = 0

EXERCISES 5(a)
1. Select 20 lines in a page of a novel. Count the number of times each letter of the
alphabet appears. Would you expect each of the 26 letters of the alphabet to be used

126 NEW SENIOR MATHS: TWO UNIT COURSE

approximately the same number of times? If a letter in these 20 lines is selected at


random, would you say that the probability that it is 'e' would be the same as the
probability that it is 'z'?
2. Select any page in a telephone directory and count the number of times 0, 1, 2,
3, ... 9 appears as the last digit in a telephone number. Prepare a table showing your
results. Do you think these results were to be expected?
3. What is wrong with the following arguments?
(i) The probability that a child born in Australia is born in New South Wales is }

because Australia has 7 states.


(ii) Since there are fewer road accidents in Hobart than in Sydney, it is safer to drive

in Hobart.
4. Would the following have probabilities close to 0, 0'5 or I?
(a) A person will die because of accidental drowning.
(b) A student in your class has fair hair.
(c) There will be an aircraft crash in Australia today.
(d) The cost of living will increase this year.
(e) A household has a television set.
(f) It will rain in Sydney tomorrow.
5. Perform this experiment. From a bag containing 7 black marbles and 3 white
marbles, withdraw a marble, note its colour and then replace it in the bag. Shuffle the
marbles and then withdraw a marble again. Repeat 10, 50, 100, 150, 200 times.
Complete the following table:
Number of withdrawals

10

50

100

150

200

Number of black marbles


How many black marbles do you expect to get in each case?
6. Perform this experiment. From a pack of 52 playing cards, withdraw a card and note
whether it is a diamond, heart, spade-or club. Replace the card in the pack, shuffle
well and then withdraw another card. Repeat 20, 40, 60, 80, 100 times. Complete the
following table:
I Number of withdrawals
I

20

40

60

80

-100

Number of hearts

How many hearts do you expect to get in each case?


7. A box of matches has 50 matches of which 40 are 'live' and the remainder are 'dead'.
If a match is selected at random, what is the probability that it is (i) live,
(ii) dead?
8. A carton contains a dozen eggs of which three are brown and the rest are white. An
egg is chosen at random from the carton. What is the probability that it is (i) brown,
(ii) white?
9. A bag contains 2 white marbles, 2 black marbles and 1 red marble. A marble is
selected from the bag. What is the probability that it is (i) red, (ii) black, (iii) not
white, (iv) red or black, (v) not black nor white, (vi) red or white?

5/PROBABILITY 127

10. A set of 20 cards is numbered 1, 2, 3, ... 20. A card is drawn at random from the set.
What is the probability that the number on it is divisible by (i) 3, (ii) 5, (iii) 3 or 5 or
both, (iv) 3 and 5?
11. A die with its faces numbered 1 to 6 is rolled on the floor. What is the probability that
the number appearing uppermost is (i) greater than 2, (ii) greater than 3 but less than
5, (iii) odd, (iv) odd and divisible by 3?
12. A set of 10 cards is numbered 3, 4, 5, ... 12. A card is drawn at random. What is the
probability that the number on it is (i) even, (ii) odd, (iii) odd or even, (iv) greater than
7, (v) divisible by 3, (vi) even and divisible by 3, (vii) divisible by 5, (viii) odd or
divisible by 5, but not both?
13. A letter is chosen at random from the letters of the word 'Sunday'. What is the
probability that it is (i) a vowel, (ii) a consonant, (iii) 'd' or 'a'?
14. A card is drawn at random from a pack of 52 playing cards. What is the probability
that it is (i) a heart, (ii) a king, (iii) a heart or a king, (iv) a heart and a king, (v) the
queen of diamonds?
15. A digit is chosen at random from the digits 7, 8, 9, ... 15. What is the probability
that the digit is (i) odd, (ii) odd and divisible by 5, (iii) odd but not divisible by 5, (iv) a
multiple of 2, (v) a multiple of 3, (vi) a multiple of both 2 and 3, (vii) a multiple of 2 or
3, (viii) a multiple of 2 or 3, but not both?
16. In a group of 25 students, 18 study French, 12 study German and 5 study neither
French nor German. If a student is chosen at random, what is the probability that he
or she studies (i) French only, (ii) German only, (iii) French or German or both,
(iv) both French and German?
17. A bag contains 5 blue marbles, 3 red marbles and 2 green marbles. One marble is
drawn at random from the bag. What is the probability that it is 0) blue, (ii) red,
(iii) not green, (iv) blue or green, (v) not red nor blue?
18. A box has marbles of the same size but different colours-red, white and blue. If a
marble is drawn out at random, the probability that it is red is the same as the
probability that it is white and twice the probability that it is blue.
(i) What is the smallest number of marbles the box could have?
(ii) If a marble is chosen at random, what is the probability that it is (a) red,
(b) white, (c) red or white, (d) not blue?
19. A bag contains 10 balls of which 6 are red, 3 are yellow and 1 is white. A ball is drawn
out at random. What is the probability that it is (i) red, (ii) not yellow, (iii) white or
yellow, (iv) neither red nor white, (v) not red?
20. A bag contains 12 discs of which 7 are black, 3 are white and 2 are red. A black disc is
withdrawn. What is the probability that the next disc withdrawn is (i) black, (ii) not
white; (iii) neither black nor white?
21. A survey of a certain district showed that 6010 of families have 1 child, 38010 have 2
children, 42010 have 3 children, and 10010 have more than 3 children. A family is
selected at random. What is the probability that it will have (i) some children, (ii) no
children, (iii) at least 2 children, (iv) not more than 2 children?

-----

-- . - .~. - -

..

- .~ -

128 NEW SENIOR MATHS: TWO UNIT COURSE

22. The diagram shows a circular disc


divided into 4 sections A, B, C and D. A
pointer, pivoted at the centre 0, is free
to rotate. If the pointer is rotated, find
the probability that the end of the
pointer stops inside (i) A, (ii) B, (iii) A or
B, (iv) not C, (v) B, C or D.

23. A horse is grazing inside an enclosed


rectangular paddock 50 m by 20 m and is
free to move anywhere inside the
paddock. What is the probability that at
any given time the horse is (i) more than
5 m from the fence, (ii) less than 5 m
from the fence, (iii) not more than 5 m
from a corner?

50m
C
D ~-------------,
20m

24. A French roulette wheel has 37 com


partments around its rim. One of these
is numbered 0 and is coloured green.
The others are numbered 1 to 36 and
half of these are coloured red and the
other half are coloured black.
The wheel is spun in one direction and

a small ball is rolled in the opposite

direction. The chances of the ball falling

into any of the 37 compartments are

equally likely.

Find the . probability that the ball will land on (i) black, (ii) an odd number,
(iii) a multiple of 3, (iv) any number from 1 to 12, (v) 15.
The payouts for a win are:
even money for red or black, odd or even numbers; 35 to 1 for any number.
If a gambler invests $1 in each of the situations (i) to (v) and wins, how much does
he or she receive back in each case?
If the gambler invests $1 on black for 37 consecutive spins, how much would you
expect he or she would win or lose?

5.4 Finite sample space _ _ _ _ _ _ _ _ _ __


A set of favourable outcomes may be considered as an event. The tossing of even numbers
with a die (3 favourable outcomes), the drawing of a heart from a pack of cards (13
favourable outcomes) and the tossing of a head with a coin (1 favourable outcome) may
be considered as events.

5/PROBABILITY 129

The tossing of a die, the drawing of cards from a pack, etc., are called experiments or
trials.
In modern pFobability theory, all possible outcomes of an experiment are considered as
points in a space, called a sample 'space or probability space, <5. If <5 contains a finite
number of points, n, and if the outcomes of an experiment are equally likely, then we can
assign to each point, called a sample point, 'a probability of!, The sum of the pron
babilities of all the sample points is, therefor.e" J.,
A sample space 'of the experiment of tossing a die consists of 6 sample points. The set
<5
11, 2, 3, 4, 5, 61 gives the possible outcomes of the experiment. Three points
correspond to event A, the tossing of an even number, the other three to event B, the
ttlSsingofan odd number. 'If'mare'interested only in tbe tossing ofev,en numbers (e) and
odd numbers (0), the sample space may be considered as consisting of only two sample
points, In this case, the set <5 = Ie, 01 is a sample space, one point of which corresponds to
event A, the other to event B.
An event, then, is a set or collection of sample points in
Le" it is a subset of <5,
,A sample -space -of she ,experiment ,of .chowjng a lettec of the alphabet may be
considered as consisting of 26 sample points. The set

<5 = {a"b, c, .. " x, y, zl; n(<5) = 26


gives the possible outcomes of the experiment, each outcome being equally likely,
[Five lpoints correspond to event A" the choosing of a vowel:
A

la,

e, i, 0, uJ; n(A) = 5

On the assumption that all sample points are equally likely, the probability measure of
event A is simply a measure of proportion. If we denote the probability of event A by
P(A), then

P(A)

number of sample points in A


n(A)
number of sample points in <5 = n(<5)
number of outcomes corresponding to A
total number of possible outcomes
5

26
We may, of course, consider the sample space as consisting of 2 sample points, vowels
I v, c1. However, this is not advisable, because each point
is not equally weighted.

(v) and consonants (c), i.e. <5 =

5.5 Mutually exclusive events _ _ _ _ _ _ _ __


If two or more events cannot occur simultaneously, they are said to be mutually exclusive
or disjoint. In the language of sample space, the events have no points in common.
If a coin is tossed, either a head or a tail may turn up, but both events cannot occur in
the one throw. If a card is drawn at random from a pack, it could be a heart or the ace of

spades or some other card, but the drawing of a heart excludes the possibility of drawing
the ace of spades. These two events are mutually exclusive. The events, drawing a heart or
an ace, however, are not mutually exclusive. Why?

130 NEW SENIOR MATHS: TWO UNIT COURSE

EXAMPLE 4
If one card is drawn at random from a pack of 52 playing cards, what is the probability
that it is a heart or the ace of spades?

<5

CD.

Mutually exclusive events

Event A is the drawing of a heart from a pack of 52 playing cards and is, therefore, a: set
of 13 points in a sample space, <5, of 52 points.
Event B is the drawing of the ace of spades and is a set containing 1 point. That is,
n(<5)

52, n(A)

13, n(B) = 1

A U B is the set of all points belonging to A or B or both and is called the union of the
two sets. Thus A U B is a set of 14 points. Tnat is,
n(A U B)

P(A U B)
P(A)
P(B)

14
14

52
13
52
1
52

A and B have no points in common and are thus mutually exclusive, in which case:

P(A U B) = P(A)

P(B)

Before adding probabilities, one must be sure that the events are mutually exclusive.
Sometimes some of the outcomes of event A ~re included in the outcomes of event B.

EXAMPLE 5
From a set of 15 cards whose faces are numbered 1 to 15, one card is drawn at random.
What is the probability that it is a multiple of 3 or 5 or both?

Union and intersection

5/PROBABILITY 131

The sample space, 6, is a set of 15 points.


A = 13, 6, 9, 12, 151 and so n(A) = 5
B = 15, 10, 151 and so nCB) = 3

There is one point in the intersection of A and B; i.e., there is one point that belongs to
both A and B, namely the number 15.
The intersection or product of A and B is the set of all points belonging to both A and B
and is denoted by A n B or AB and so n(A n B)
1.
n(A U B)
7, because the number 15 is a multiple of both 3 and 5 and must be
included only once, regarding it as a multiple of either 3 or 5.
Thus

Thus

peA U B)

peA)

= 15

PCB)

= 15

P(AB)

peA U B)

=1=

peA U B)

1
P(A)

P(A)

P(B)

+ PCB)

but

- P(AB)

In this case, A and B are not mutually exclusive. A and B are mutually exclusive if the
O.
set of points in the intersection of A and B. is a null set, in which case P(AB)

EXERCISES 5(b)
1. A card is drawn at random from a pack of 52 playing cards. A is the event 'drawing a
heart', B the event 'drawing the ace of clubs', C is the event 'drawing an ace'. Find

the probability of these events:


(a) A
(d) A U B

(b) B
(e) A U C

(c) C
(f) B U C

Are A and B mutually exclusive?

Are A and C mutually exclusive?

Are Band C mutually exclusive?

2. From a set of 17 cards numbered 1,2, 3, ...., 17, one card is drawn at random. A is
the event 'a multiple of 3', B is the event 'a multiple of 8', C is the event 'a multiple of
5'. Find the probability of these events:
(e) jj
(a) A
(b) B
(c) C
(d) A
(j) jj U C
(f) A U B
(g) A UC
(h) A U B
(i) A U C
Which of the events A, Band C are mutually exclusive?
3. A die is rolled on the floor. A is the event 'an even number', B is the event 'an odd
number', C is the event 'a number less than 4'. Find the probability of these events:
(a) A U B
(b) A U B
(c) A U C
(d) jj U C

4. An urn contains 8 red marbles, 7 white marbles and 5 black marbles. One marble is
drawn at random from the urn. What is the probability of
(a) red or black,
(b) not white,
(c) neither black nor white?

---,

132 NEW SENIOR MATHS: TWO UNIT COURSE

5. A coin is tossed three times. A denotes the event 'at least two heads', B the event
'head, tail, head', C the event 'not more than one head'. Find the probability of these
events:
(a) A
(b) B
(c) C
(d) A U B
(e) A U C
(f) B, U C
(g) A U B U C

Which of the events A, Band C are mutually exclusive?

6. An integer is chosen at random from the first 50 positive integers. A is the event
'divisible by 2', B the event 'divisible by 3', C the event 'divisible by 5'. Find the
probability of these events:
(a)" AB
(e) A U B

>,

.'(>b) AC
(f) A U C

(c). .. BC
(g) B U C

(d) ABC
(h) A U B U C

7. A die is tossed twice. A is the event 'the sum of the two numbers uppermost is 4 or
more' , B the event 'the'sum of the-two numbers ,uppermost isJess than 6', C the event
'the two numbers uppermost are the same'. Find the probability of these events:
(a) AB
(e) A U B

(b) AC
(f) A U C

(c) BC
(g) B 'U C

(d) ABC
(h) A U B U C

8. A die is tossed twice. A is the event 'the sum of the two numbers uppermost is greater
than 5', B the event 'the sum of the two numbers uppermost is less than 8'. Find the
probability of these events:
(a) A
(b) B
(c) AB

(I) A U B

Hence show that P(A U B) = P(A)


Are A and B mutually exclusive?

9. If P(A)

P(B) - P(A

B).

P(B) = 1, does it follow that either event A or event B must occur?

15

10. P(A U B) = 16; P(A

B) =

1
3
8;
P(A) = 8.

Find P(B).

11. In a group of 50 students, 30 study Mathematics, 25 study Physics and 20 study both
Mathematics and Physics. One student is selected at random from the group. What is
the probability that the student studies
(a) Mathematics but not Physics,
(b) Physics but not Mathematics,
(c) neither Physics nor Mathematics?

Use a Venn diagram.

12. From a group of 100 students, 50 study History, 30 study English Literature and 20
study both. If a student is selected at random from the group, what is the probability
that the student studies
(a) at least one of these subjects,
(b) History but not English Literature,
(c) History, given that the student also studies English Literature?

5/PROBABILITY 133

5.6 Successive Outcomes ___________


EXAMPLE 6
A coin is tossed twice. What is the total number of possible outcomes?
In the first toss, there are two possible outcomes, H or T. Each of these can be associated
with H or T in the secon~ toss as shown in the following tree diagram.
First toss

Second toss

Possible outcomes
HH
HT
TH
TT

There are 4 possible outcomes (2 x 2), namely HH, HT, TH, TT, where HH means
head in the first toss followed by head in the second toss and so on. Note that HT and TH
are two different outcomes-HT is head first toss, tail second toss; TH is tail first toss,
head second toss.
Are each of these four equally likely? If so,
1 2 1
P(2 heads)
4' P(l head) = 4' P(O heads) = 4
The 4 possible outcomes can also be illustrated
well by a lattice diagram.

EXAMPLE 7

First toss

A card is drawn from a pack of 52 playing cards and also a coin is tossed. If we are
interested only in whether the card drawn is a heart (h), diamond (d), spade (s) or club (c),
how many possible outcomes are there?
In drawing the card from the pack, there are 4 possible outcomes h, d, s or c. Each of
these can be associated with H or T when the coin is tossed as shown in the following tree
diagram.
Cards

Coin

Possible outcomes

h~H

hH

hT

d~H

dH

dT

s~H

sH

sT

c~H

cH

cT

,~~---."

--~

~~---------------~---.--~.~-~.

-.

134 NEW SENIOR MA THS: TWO UNIT COURSE

There are 8 possible outcomes (4 x 2), where


hH means heart and head, hT heart and tail, etc.
Are these 8 outcomes equally likely? If so, each
has

probability~. Thus the probability of drawing

a spade and tossing heads is

~.

Similarly, the 8 possible outcomes can be illus


trated by a lattice diagram.

Cards

EXAMPLE 8
On school sports day Joanna runs in the 100-metre, 200-metre and 400-metre races. In
terms of win (W) and loss (L), there are two possible outcomes for each race. How many
possibilities are there for the three races?
200 metre

100 metre

400 metre

Possibilities
WWW
WWL
WLW
WLL
LWW
LWL
LLW
LLL

There are 8 possible outcomes (2 x 2 x 2), where WWW denotes a win in each of the
three races, WWL denotes a win in 100m, 200 m and a loss in 400 m, etc. Are the 8
outcomes equally likely? The answer is "no" unless she has a 50-50 chance of winning
each race. If this is so, then
P(she wins the 3 races)

I
8

P(she wins any 2 races) =

P(she wins any I race) =

P(she wins no races) =

This example cannot be illustrated on a two-dimensional lattice diagram. Why?

Multiplication principle
Each of the three exampies 6, 7 and 8 is an illustration of the multiplication principle,
which states:

If one operation can be performed in m different ways and if, when it has
been performed in anyone of these ways, a second operation can be
performed in n different ways, the number of ways of performing the
two operations is m x n.
.

5/PROBABILITY 135

EXERCISES 5(c)
1. A coin and a die are tossed at the same time. Draw ,a tree diagram and a lattice
diagram and list all the possible outcomes. Are they equally likely outcomes? If so,
what is the probability of (i) a head and an even number, (ii) a tail and a number
greater than 4?
2. Two cubes each have two faces painted red, another two white and another two blue.
They are rolled on the floor. Draw a tree diagram and a lattice diagram showing nine
possible outcomes of the colours appearing uppermost. Are they equally likely
outcomes? What is the probability that (i) both colours are the same, (ii) one is red
and the other white?
3. A committee consisting of one boy and one girl is to be selected from 3 girls Anne,
Karen and Helen and 4 boys Joe, Harry, Tim and Leon. How many pairs can be
formed? Illustrate with a tree diagram and a lattice diagram.
4. A carvery offers roast lamb, roast beef, roast pork and roast chicken as the main
course and apple pie, cheese cake and orange cake as sweets. Draw a tree diagram and
a lattice diagram to show the number of possible selections consisting of one item
from the main courses and one from the sweets.
If each is equally likely to be selected, what is the probability of selecting (i) roast
pork and either cheese cake or orange cake, (ii) roast chicken but not apple pie?
5. Mal has 3 shirts, 2 ties and 2 pullovers. Illustrate with a tree diagram the different
shirt-tie-pullover combinations.
6. A die is rolled twice. Explain why there are 36 possible pairs of numbers. Are they all
equally likely? What is the probability that
(i) both numbers are the same,
(ii) the sum of the two numbers is 5,
(iii) the sum of the two numbers is 2, 3 or 12?

Illustrate with a lattice diagram. Why is it preferable to a tree diagram in this case?

7. A coin is tossed three times. Illustrate on a tree diagram the eight possible outcomes.

Are the outcomes equally likely? Find (i) P(O heads); (ii) P(1 head), (iii) P(2 heads),

(iv) P(3 heads).


8. A box has 4 balls marked 1, 2, 3, 4. A ball is
elected at random from the box and the
pointer is rotated on the circular disc.
Illustrate on a tree diagram the 12 possible
outcomes. What is the probability of (i) Rand
an odd number, (ii) Wand a number greater
than 2 or R and an even number?

9. Three students Arnold, Brenda and Christina work independently on a mathematics


problem and each has a 50-50 chance of success (8). Draw a tree diagram illustrating
the possible outcomes. What is the probability that (i) all three solve it, (ii) only
Brenda and Christina solve it, (iii) none of them solves it, (iv) at least one of them
solves it?
10. One cube has 4 red faces and 2 white faces; another has 3 red and 3 white; another has
2 red and 4 white. The three cubes are rolled. Draw a tree diagram indicating the
colour of the faces uppermost. Are the outcomes equally likely?

136 NEW SENIOR MATHS: TWO UNIT COURSE

11. A 20-cent coin, a 10-cent coin and a 5-cent coin are tossed. Draw a tree diagram
showing the possible outcomes of heads and tails. What is the probability of (i) 3
heads, (ii) a head with the 5-cent coin only, (iii) at least 2 tails?
12. The diagram shows a circular disc divided into
four equal sections 1, 2, 3, 4. A pointer,
pivoted at the centre, is free to rotate. The
pointer is rotated twice. Draw a tree diagram
to illustrate the 16 possible outcomes. What is
the probability that (i) the pointer lands on the
same number twice, (ii) the sum of the two
numbers is at least 5?
13. Use the lattice diagram of question 6. to find the following probabilities resulting from
rolling a die twice:
(i) the two numbers add to more than 5,
(ii) an even number with the first and an odd number with the second,
(iii) the highest total possible,
(iv) both numbers the same,
(v) the numbers sum to 10 or less,
(vi) a multiple of 3 with the first and a multiple of 2 with the second,
(vii) both numbers different and they sum to 8.
14. A card is drawn at random from a pack of 52 playing cards. It is replaced, the pack is
shuffled and a card is drawn again. What is the probability that (i) both cards are
spades, (ii) neither card is a heart, (iii) both cards are the same suit? Draw a tree
diagram and a lattice.

5.7 Independent events _ _ _ _ _ _ _ _ _ __


Events A and B are independent if

I P(AB)

= P(A) .P(B)

This definition of independent events can be illustrated by the following examples.

EXAMPLE 9
If two dice are tossed, what is the probability of an even number with the first die and a 3

or a 5 with the second die?


The sample space consists of 6 X 6, or 36, points as listed in the following table. For
convenience, the numbers on the first die are printed in bold type.
(1, 1)

(2, 1)

(3, 1)

(4, 1)

(5, 1)

(6, 1)

(1, 2)

(2, 2)

(3, 2)

(4, 2)

(5, 2)

(6,2)

(3, 3) 1(4, 3) I (5, 3) 1(6, 3) I

(1, 3)
(1, 4)

(2, 4)

(3,4)

(1, 5) I (2, 5) I (3, 5)


(1, 6)

(2,6)

(3, 6)

(4,4) . (5, 4)

(6, 4)

(5, 5) 1(6, 5)
(5, 6)

(6,6)

5/PROBABILITY 137

18 points correspond to event A, an even number with the first die and any number with
the second die.
12 points correspond to event B, a 3 or a 5 with the second die and any number with the
first die.
Thus

P(A) = 18 =

and

P(B) = 12 =

36

36

The intersection of A and B is the set of 6 points, indicated in the table thus:
A

Hence

B
6

P(AB) = 36 =
P(A)
P(B)
P(A) .P(B)

18
36
12
36

'----'

(2, 3), (2, 5), (4, 3), (4, 5), (6, 3), (6, 5) I
1

6
1
2

= P(AB)

This result conforms to our definition of independent events and to the everyday
meaning we attach to the word 'independent'. It is quite apparent that whichever number
happens to turn up when the first die is tossed will have absolutely no influence on the
number likely to turn up when the second die is tossed.
This example could also be solved by a careful observation of the 36 sample points.
Observe that

~ of the outcomes contain an even number with the first die and} of this

~ i.e. ~ contain an even number with the first die and a

3 or a 5 with the second die.

This two stage process of tossing two dice can be represented by a tree diagram with two
sets of branches. For convenience we will call a 3 or a 5 with the second die a success Sand
not a 3 or a 5 a failure F.
1st die

2nd die

E<:S

Possible
Outcomes

<'
; o~:

Probability

EF

OS

X ~

ES

OF

= ij

x
6

The tree diagram shows the four possible out


comes but they are not all equally likely. An
even number E with the first die and a 3 or a 5
with the second die is given by ES with prob
'l' 1
1
1
abllty
2: x 3
= 6'
The situation can also be represented by the
lattice shown,

1H\

2
1

1 st die

138 NEW SENIOR MATHS: TWO UNIT COURSE

EXAMPLE 10
A fair coin is tossed 3 times. What is the probability of
(a) 2 heads and 1 tail
(b) at least 1 head
The experiment of tossing a coin 3 times can be represented by a tree diagram with 3 sets
of branches or with a 3-dimensionallattice.
First
toss

Second
toss

Third
toss

Possible
outcomes

H<H~:
, <;::H

n(S)

I(H H H), (H H
(TTnl

HHT

:2

H TH

:2

:2

HTT

..1

THH

T<H~T
HenceS

:2

n,

,
,

TTH

T T T

x 1.2

x :2

x :2

x :21

,
,
1

,
8

X !2

T H T

T~H
T

HHH

Probability

4x

(H T H), (H Tn, (T H H), (T H

X 1.2

n, (T T H),

8 and each of the 8 outcomes is equally likely.

(a) From an observation of the 8 sample points, we can pick out the points corresponding
to the event 2 heads and 1 tail.

A = I(HHn,(HTH),(THH)1

:. P(2 heads and 1 tail) =

k+ k+ k

3
8
(b) Each of the 8 outcomes except (T Tn has at least 1 head.
:. P(at least 1 head) = 1 - P(O heads)

= 1 -

!8

7
8

EXAMPLE 11
Two friends, David and Colin, frequently play golf and tennis with each other. In the long
run, it has been found that David wins 3 rounds of golf out of every 5, and 1 game of
tennis out of every 4. If they play one round of golf and one game of tennis, find the
probability that David
(a) wins both,
(b) loses both,
(c) wins the round of golf only,
(d) wins either the golf or the tennis but not both.
------------"---~

5/PROBABILITY 139

We assume that no game ends in a draw,


Let A be the event 'David wins the golf', and so A is the event 'David loses the golf',
3
2
P(A) = 5 and P(A) = 5
Let B be the event 'David wins the tennis' , and so lJ is the event 'David loses the tennis'
1
3
P(B) = 4 and P(B) = 4
On the assumption that A and B are independent events, then
(a) P(AB)

= Pr(A) ,Pr(B)
3

(b)

P(AB)

P(A) ,P(B)

(c)

P(AB)

5'4

5'4

3
20

3
10

P(A) ,P(B)

3 3

5'4
9
20

(d) The statement 'David wins either the golf or the tennis but not both' implies that
(i) David wins the golf and loses the tennis, or
(ii) David loses the golf and wins the tennis,
(i) and (ii) are mutually exclusive,
(i)

P(AB)

= P(A) ,P(B)
3

P(AB)

(ii)

5'4
2
20

5'4
9
20
' d pro b a b'l'
H ence t h e reqUIre
Iity

9 :+- 20
2 = 20'
11
= 20

In this example, there are four possible outcomes (not all equally likely). If we denote a
win by David by Wand a loss by L, then the possible outcomes are WW, WL, LW and
LL, Their respective probabilities can be illustrated by the following probability tree

diagram,

<
1.

<~
5

Possible
Outcomes

Tennis

Golf

4"

WW

WL

L<~

Probability
1

4"

LW

LL

EXAMPLE 12
A die and a coin are tossed in that order, and then a letter is taken from the set la, b, cl,
Find the probability of these events:
(a) A: a 3 with the die
(b) B: a head with the coin
(c) C: b from the set {a, b, c)
(d) ABC

140 NEW SENIOR MATHS: TWO UNIT COURSE

A sample space consists of 6 x 2 x 3, or 36, equally weighted points (multiplication

principle). List the 36 ordered triples.


Of these 36 points, 6 correspond to event A, 18 to event Band 12 to event C. Hence
(a)

P(A)

6
1
36 = 6

(b)

P(B) =

(c)

P(C) = 12 =

(d)

ABC

.. P(ABC) =

36

2
3

1(3, H, b)1
1
36
Pr(A). Pr(B) .Pr( C)

A, Band C are independent events.

Represent the situation by a tree diagram with 3 sets of branches.

EXERCISES 5(d)
1. I toss a coin and throw a die. What is the probability of

(a) a head with the coin,


(b) a number greater than 4 with the die,
(c) a head and a number greater than 4?
2. From a class of 25 boys and 15 girls, of whom 8 boys and 7 girls wear glasses, I select
one student. What is the probability that the student
(a) is a boy,
(b) wears glasses,
(c) is a boy and wears glasses?
3. A coin is tossed three times. Find the probability of these events:
(a) A: at least two heads,
(b) B: head, tail, head,
(c) C: not more than one head.
4. A red die and a blue die are thrown on a table, and the numbers appearing uppermost
are noted.
Find the probabilities of the events:
(i) the same number with each die,
(ii) the sum of the numbers exceeds nine,
(iii) a 3 with the red and a 4 with the blue,
(iv) an odd number with the red and an even number with the blue,
(v) the sum of the numbers is less than two.
S. A coin and a die are thrown together. What is the probability of a head and a number
greater than 4 or a tail and a number not exceeding 3?
6. A die is thrown and a number from the set (1, 2, 3, 4, 51 is selected.

Find the probabilities of

(i) A, the event in which an even number is selected from the given set,
(ii) B, the event in which the die shows an odd score,

5/PROBABILITY 141

(iii) C, the event in which the total score exceeds 9,

(iv) D, the event in which the total score is 10.


7. A coin is tossed three times. A is the event 'at least two tails'; B is the event 'three
heads or three tails'; C is the event 'at least one tail'. Which of the following are
independent?
(a) A and B
(b) A and C
(c) Band C
8. Consider two spin dials. On one of them the letters A, B, C, D and E are printed, and,
on the other, the digits 1, 2, 3, 4 and 5. When the dials are spun, it is equally likely
that they will stop on any letter or number.
What are the probabilities of the events
(i) B and an even number,

(ij) C or D and an odd number,

(iii) E and an even number, or C and a number greater than 3,


(iv) a consonant and an odd number, or a consonant and a number greater than 2?
9. A coin and a die are thrown together. What is the probability of a head and a number
greater than 3 or a tail and a number not exceeding 4?
10. In a large school, 25070 of the students ride bicycles to school, and 40070 of the students
have fair hair. One student is selected at random.
What is the probability that the student
(i) has fair hair and rides a bicycle to school,
(ii) does not have fair hair and does not ride a bicycle to school,
(iii) has fair hair but does not ride a bicycle to school,
(iv) rides a bicycle to school but does not have fair hair?
3
3
11. A die is loaded so that the probability of a 6 is 10' the probability of a 5 is 10' and

each of the other numbers is equally likely. If the die is tossed twice, find the
probability of
(a) two sixes,
(b) no sixes,
(c) at least one six,
(d) the sum of the numbers being 6.
12. One card is drawn at random from a pack of 52 playing cards. It is replaced, and the
pack is shuffled. A second card is then drawn. What is the probability that
(a) .both cards are diamonds,
(b) neither card is a diamond,
(c) only one of the cards is a diamond,
(d) the first card only is a diamond,
(e) the second card only is a diamond,
(f) at least one of the cards is a diamond?
13. For a certain species of bird, there is a chance of 4 in 5 that a fledgling will survive the
first month after birth. From a brood of 3 chicks, what is the probability that
(a) all will survive,
(b) none will survive,
(c) at least one will survive?

142 NEW SENIOR MATHS: TWO UNIT COURSE

14. An athlete competes in 100m, 200m and 400m races, and he estimates his chances of
winning as
(a)
(b)
(c)
(d)
(e)

he
he
he
he
he

~, ~ and ~ respectively.

Calculate the probability of these events:

wins all three races,


loses all three races,
wins the 100 m race and loses the other two,
wins the 400 m race and loses the other two,
wins the 100m race and the 200m race, and loses the 400m race.

15. Cube A has 4 red faces and 2 white faces; cube B has 3 red and 3 white faces; cube C
has 2 red and 4 white faces. The three cubes are tossed. What is the probability of
(a) 3 red faces uppermost,
(b) 3 white faces uppermost,
(c) red with A and B and white with C,
(d) red with A and white with Band C,
(e) at least one red face?
16. A coin is tossed 4 times. What is the probability of
(a) 4 heads,
(b) 4 tails,
(c) head, tail, head, tail, in that order,
(d) heads in the first 3 tosses, but not in the fourth,
(e) heads in anyone of the four tosses?
17. A die is tossed three times. What is the probability of
(a) 3 sixes,
(b) 0 sixes,
(c) 3 odd numbers,
(d) 3 even numbers,
(e) a six in the first two tosses only,
(f) a six, not a six, a six, in that order?
18. Janet and Brian play 3 tennis matches and Janet's chance of winning <;tny one match
is

j. What is the probability that Janet

(a)
(b)
(c)
(d)

wins all 3 matches,


loses all 3 matches,
wins the first and the third but loses the second,
loses the first and wins the other two?

19. An urn contains 5 white, 3 black and 2 blue balls. A ball is withdrawn, replaced and a
second is drawn. This is repeated 3 times. What is the probability of the events:
(a) 3 white balls,
(b) a black ball in the first two drawings, but not in the third,
(c) white, black, blue, in that order,
(d) white, black, white, in that order,
(e) not more than 2 white balls,
(f) a white or a black ball in each drawing?
20. A Gallup Poll establishes that, on the average, 2 out of every 3 people interviewed are
in favour of a certain proposal. What is the probability that, out of a group of 3
people interviewed:
(a) all will be in favour,
(b) none will be in favour,

5/PROBABILITY 143

(c) the first, and third persons interviewed will be in favour, but not the other one?
21. A student estimates that his chances of passing English, Mathematics and Physics are
respectively
(a)
(b)
(c)
(d)

he
he
he
he

~, ~ and ~.

Estimate the probability of these events:

passes English only,


passes English and Mathematics only,
passes the three subjects,
fails in each subject.

22. The probability of a cure with drug A is 08 and with drug B is 0'6. One randomly
selected patient is treated with drug A and another with drug B. What is the
probability that
(a) neither patient is cured,
(b) both are cured.
23. One urn contains 2 red cubes and 4 blue cubes and a second urn contains 4 red cubes
and 3 blue cubes. One cube is selected at random from each of the two urns. What is
the probability that one of the cubes is red?
24. The probability that a man will be alive in 20 years is
wife will be alive is

~ and the probability that his

j. What is the probability that in 20 years time (i) both will be

alive, (ii) only one of them will be alive, (iii) at least one will be alive?
25. A man finds that he is late for work on 10 per cent of occasions if he is on time the
previous day and on 20 per cent of occasions if he is late the previous day. Given that
he was on time on Monday, what is the probability that he is on time on Wednesday?
Illustrate with a probability tree diagram.
26. In a certain factory, machines A, Band C manufacture 25, 35 and 40 per cent
respectively of the articles. Of their output 5, 4 and 2 per cent respectively are
defective. An article is selected at random. What is the probability that it was
manufactured by:
(a) A and is defective,
(b) B and is not defective?
27. Given three urns A, Band C. A contains 3 black and 2 white cubes, B contains 3 black
and 1 white cubes, C contains 3 black and 3 white cubes.
(i) An urn is chosen at random and from it a cube is chosen at random. Illustrate this
two-stage process by a probability tree diagram.
(ii) What is the probability that the cube is black?
28. A student finds that on the average he 'misses' the school bus once every eight weeks
(5 days in a school week). Find the probability of
(a) not missing the bus on anyone morning,
(b) catching the bus on two successive mornings,
(c) catching the bus each morning for a week (5 days),
(d) missing the bus on at least one morning.
29. To open a locked safe requires a correct three-digit combination. Calculate the
probability of:
(a) succeeding at the first attempt,
(b) failing at the first attempt.

144 NEW SENIOR MATHS: TWO UNIT COURSE

30. The first race at Randwick has 13 runners and the second race has 16 runners.

Assuming that all horses have an equal chance of winning, calculate the probability
of:
(a) a double (i.e. a winner in each race),
(b) a quinella (Le. first and second in either order) in the first race,
(c) a quinella in the second race,
(d) a quinella in both races.
31. To gain a driving licence in N.S.W. both a written test and a practical driving test
must be passed. Statistics show that 70070 pass the written test on the first attempt and
90% of those who use a second test pass that test. Also 60070 pass their first practical
test and 80070 pass their second test. The written and practical tests are independent.
Calculate the probability of:
(a) passing the written test on the second attempt,
(b) passing the written test after no more than two attempts,
(c) requiring a third written test,
(d) passing the practical test on the second attempt,
(e) receiving a licence after requiring one practical test and two written tests.

5.8 Dependent events _ _ _ _ _ _ _ _ _ _ __


EXAMPLE 13
Two dice are tossed. A is the event 'a 5 with the first die' and B is the event 'sum of the
numbers on the two dice exceeds 10'.
(a) Find P(AB).
(b) Are A and B independent?
(c) Find the probability that the sum of the numbers exceeds 10, given a 5 with the first
die.
(a) A

1(5, 1), (5, 2), (5, 3), (5, 4), (5, 5), (5, 6) I

Hence

P(A)

= 36 = 6"

1(5, 6), (6, 5), (6, 6)1

Hence

P(B) = 36
A

nB =

12

1(5, 6)1 and so

1
P(AB) = 36

=1=

L:h

"-

E)

5
4

2nd

die 3

2 3

P(A) .P(B)

1st die

(b) Since P(A) =1= P(A) .P(B) , events A and B are not independent. Hence they are
dependent. The situation can be represented by (i) a lattice as shown (ii) listing the
6 x 6 possible outcomes. It cannot in this case be represented by a tree. Why?
(c) We are asked to find the probability of event B given that event A has happened. This
is written as P(BIA).

Since A has happened, the only possible outcomes in this case are

(5, 1), (5, 2), (5, 3), (5, 4), (5, 5), (5, 6).

Of these 6 outcomes, only one is favourable, namely (5, 6).

5/PROBABILITY 145

Hence P(BIA)
But
and
i.e.

1
P(AB) = 36
P(A)

6
36

P(BIA) = P(AB)
P(A)

Transposing, we get
P(AB)

P(A) .P(BIA)

5.9 Sampling without replacement from a small


population _ _ _ _ _ _ _ _ _ _ _ _ _ __
EXAMPLE 14
From a set of 5 cards numbered 1, 2, 3, 4, 5 two cards are selected at random without
replacement. What is the probability that both are odd numbered cards?
Let A be the event 'odd number first draw' and B be the event 'odd number second
draw'. We are asked to find P(AB).
The probability that first card drawn is odd =
i.e.

P(A)

~,

3
5

The probability that the second card drawn is odd could be either ~ or

~ depending on

whether the first card is odd or even.


If the first card is odd, the probability that the second card is odd

i.e.

P(BIA)
P(AB)

= ~.

=~
P(A) ,P(BIA)

3 2

5'4
6

A sample space of the experiment may be considered as consisting of 20 sample points,


each equally likely.
(1,
(1,
(1,
(1,

2)
3)
4)
5)

(2,
(2,
(2,
(2,

1)
3)
4)
5)

(3, 1)

(4, 1)

(3, 2)
(3, 4)
(3, 5)

(4, 2)
(4, 3)
(4, 5)

(5, 1)
(5, 2)
(5, 3)
(5,4)

Note that the ordered pairs (1, 1), (2, 2), (3, 3), (4, 4), (5, 5) are missing. Why?
,

146 NEW SENIOR MATHS: TWO UNIT COURSE

Of these 20 equally likely outcomes, 6 are favourable, namely


(1, 3),

(1, 5), (3, 1), (3, 5), (5, 1), (5, 3).
6
P(AB) = 20

Hence

We can represent the situation by a tree diagram or a lattice.


1st draw

2nd draw

Possible
outcomes

Probability

00

ixt=i%

OE

20

EO

x {

5.

~ 4

~ 2

20

1st draw

tXt = k

EE

indicates missing points

indicates favourable outcomes

EXAMPLE 15

A carton contains 10 transistors, 2 of which are defective. A sample of 3 transistors is


drawn at random from the carton. Find the probability that not more than one transistor
is defective if the sampling is done (a) without replacement; (b) with replacement.
(a) Let D denote 'defective' and N 'non-defective'.
1st draw

2nd draw

3rd draw

Outcomes

<:D~N

10

Probability

DDN

10

x-.l9 x 1

=90

DND

+Ox{-x+

16
720

DNN

10

N~D~:
~ N<D

NDD

iO

"9

X 81

16
720

NDN

iO

"9

X 87

720

NND

iO

NNN

1.

X 86

D~ <:D
9

112
= 720

10

112

112

120

.,

336
72fj

Note that there is a branch missing. It is not possible to have an outcome DDD. Why?
The statement 'not more than 1 defective' means 0 defectives or 1 defective.
Required probability

P(DNN U NDN U NND U NNN)

112
720
672
720

112

112

336

+ 720 + 720 + 720

(b) Since each transistor withdrawn is replaced in the carton before the next withdrawal is
2
made, the probability of a defective being withdrawn remains the same 10 for each
withdrawal. Note carefully the difference between the two tree diagrams.

5/PROBABILITY 147

1st Draw

<;:0<::
<:0
,
0<:0
<;: <0
10

10

10

10

!O

!O

!O

!O

!O

ONN

2
x
,iJ

!O

ro

NOD

8
ro x

NON

10

!O

NNO

8
ro x

!O

NNN

PO

!O

DON

!O

ONO

!O

,aoo

32

,aoo
32

,aoo

!O

!O

!O

!O

:2

'28

,aoo
32

'0
8

,aoo
128

,aoo

,aoo

,aoo

128

!O

512

P(DNN U NDN U NND U NNN)

128
128
1000 + 1000
0896
Alternatively:
Required probability

10

000

!o

Required probability

!o

Probability

Outcomes

3rdOraw

2nd Draw

128
1000

512
1000

1 - P(2 defectives + 3 defectives)


1 - P(NDD U DND U DDN U DDD)
32
1 - ( 1000
0896

32
1000

32
1000

8 )
1000

5.10 Sampling without replacement from a large


population _ _ _ _ _ _ _ _ _ _ _ _ __
In example 15, we considered sampling from a small popUlation of 10 transistors. If the
sampling is done without replacement, the probability that the first transistor withdrawn
is defective is

1~'

The probability that the second transistor withdrawn is defective is

~,

given that the first is defective and so on. The probability of a defective has altered
considerably.
If, however, we take a sample without replacement from a large population of mass
produced articles, say 1000 or more, of which 20 per cent are defective, then the
probability that the first article selected is defective is

12~~.

The probability that the

second article selected is defective, given that the first is defective is then

!~~ which

is

approximately 02. Hence the probability of any article being defective remains practically
unaltered by the sampling.

EXAMP.LE 16
A shoe-manufacturer makes ladies' and men's shoes. Sixty per cent of the output are
ladies' shoes. If a sample of 3 pairs is chosen at random from the factory, find the
probability that the sample contains
(a) exactly 2 pairs of ladies' shoes,

148 NEW SENIOR MATHS: TWO UNIT COURSE

(b) at least 1 pair of men's shoes,


(c) the first 2 pairs chosen are ladies' shoes, and the third pair is men's.
Let p
06 and q = 0'4 be respectively the probabilities of choosing a pair of ladies'
shoes and a pair of men's shoes.
1st choice

2nd choice

<
p

M
(a)

<
q

3rdchoice

Outcomes

L-<L

Probability

LLL

p3

LLM

p2q

~
q
M

LML

p2q

LM'M

pq2

~L

MLL

p2q

MLM

pq2

MML

pq2

MMM

q3

----c;---- M

M~.L

----q---- M

Required probability = P(LLM U LML U MLL)

3p2q

= 3(0'6)2(0'4)

= 0432
(b)

Required probability =

- P(O men's shoes)


1 - P(LLL)

1 p3
1 - (0'6)3
0784
(c)

Required probability = P(LLM)


= p2q
=
=

(0'6)2(0,4)
0144

EXERCISES 5(e)
1. A box contains 5 black cubes and 3 red cubes. Two cubes are drawn at random from
the box. Find the probability that:
(a) both cubes are black,
(b) both cubes are the same colour,
(c) both cubes are different colours.
2. A box contains 6 green balls and 4 white balls. A batch of 2 balls is drawn at random
from the box. Find the probability that the 2 balls are the same colour.
3. An angler has caught 15 fish of which 3 are undersized. A random sample of 3 fish is
drawn without replacement by an inspector. The angler is fined if one or more of the
fish in the sample is undersized. What is the probability that he is fined?
4. A carton contains a dozen eggs, 3 of which have a double yolk. If 3 eggs are required
to make a cake, find the probability that the 3 eggs used have double yolks.
5. From a group of 9 people made up of 3 males and 6 females, a random sample of 2 is .
selected. What is the probability that:
(a) the first person selected is a female,
(b) both people selected are females,
(c) one is a female and the other a male?

5/PROBABILITY 149

6. From a pack of 52 playing cards, two cards are selected without replacement. What is
the probability that:
(a) both cards are diamonds,
(b) both cards are the same suit,
(c) one card is a spade and the other a club,
(d) both cards are different suits?
7. An urn contains 3 white balls, 4 red balls and 5 black balls. Two balls are drawn at
random without replacement. What is the probability that they are:
(a) both white
(b) both red
(c) both black?
8. From 7 teachers and 5 pupils, a random selection of 2 is made. What is the probability
that:
(a) they are both teachers,
(b) they are both pupils,
(c) one is a teacher and the other a pupil?

How can the answer to (c) be deduced from (a) and (b)?

9. From the letters of the word 'promise', three letters are chosen at random. What is
the probability that the three letters are (a) vowels, (b) consonants, (c) vowel,
consonant, vowel in that order?
10. A punter correctly picked the first and second in a race of 10 horses. What is the
probability of this, if each horse is equally likely to win?
11. From a group of 5 boys and 6 girls, two are selected at random for a class committee.
What is the probability that a boy and a girl are selected?
12. A carton contains 10 electric light globes, 3 of which are defective. Two are drawn at
random. What is the probability that:
(a) the first globe drawn is defective,
(b) both are defective,
(c) neither is defective,
(d) exactly one is defective?
13. In a raffle, 20 tickets are sold and there are 2 prizes. If I buy 5 tickets what is the
probability that I win at least one of the prizes?
14. Group A contains 10 females and 5 males. Group B contains 4 females and 6 males.
Two people are selected at random. What is the probability that:
(a) they are both females if they are selected from Group A,
(b) they are both males if they are selected from Group B,
(c) one is a male, the other a female if one is selected from both groups?
15. A box of 10 chocolates contains 4 hard-centred and 6 soft-centred chocolates. If two
are selected at random, what is the probability that:
(a) they both have hard centres,
(b) they both have soft centres,
(c) one has a soft centre and the other a hard centre?
16. In a lottery game, three numbers are selected from 1, 2, 3, 4, ... 40. Find the
probability that the three selected numbers are all even.
17. A sample of three items is selected at random without replacement from a batch of

i.

150 NEW SENIOR MATHS: TWO UNIT COURSE

ten items, four of which are defective. Find the probability that there is at most one
defective item in the sample.
18. If every possible order of finishing is equally likely in a race with nine contestants,
find the probability of picking the first three placegetters in their correct order.
19. Three cards are drawn at random from a pack of 52 playing cards. What is the
probability that they are all from the same suit?

20. A bag contains 6 red balls and 4 white balls. A random sample of 3 is withdrawn.
What is the probability that they are the same colour if the sampling is done
(a) without replacement (b) with replacement?
21. Three cards are drawn at random from a pack of 52 playing cards. What is the
probability that 3 aces are drawn if the drawing is done (a) without replacement
(b) with replacement?
I.

22. A bag contains a large number of one-cent coins and two-cent coins in the ratio of 2
to 3. Find the probability that if 3 coins are selected from the bag:
(a) 2 of them are one-cent coins,
(b) at least 2 of them are one-cent coins,
(c) not more than 2 of them are one-cent coins.
23. A manufacturer finds that 10 per cent of the articles made in his factory are defective.
If 3 articles are taken at random, what is the probability that:
(a) all are defective,
(b) none is defective,
(c) there are more defectives than non-defectives?
24. A hand of three cards dealt from a pack of 52 playing cards contains the ace of clubs.
What is the probability of this?
25. In a raffle, 30 tickets are sold and there are two prizes. What is the probability that a
person buying 5 tickets wins (a) neither prize (b) both prizes (c) at least one prize?
26. From a set of 10 cards numbered 1 to 10, two cards are drawn without replacement.
What is the probability that:
(a) both numbers are even,
(b) one is even and the other is odd,
(c) the sum of the two numbers is 12?
27. Three cards are selected without replacement from a pack of 52 playing cards. What
is the probability of:
(a) exactly 3 hearts,
(b) exactly 3 aces,
(c) at least 1 heart?
28. A manufacturer of metal pistons finds that on the average 20 per cent of his pistons
are rejected because they are either oversize or undersize. What is the probability that
a batch of 3 pist.ons will contain:
(a) no more than 2 rejects,
(b) at least 2 rejects?
29. It is known that in the long run 7 out of 10 students from the year 12 class of a certain
school will enter the University. Find the probability that out of a group of 3 students
chosen at random from this class:
(a) all will enter the University,
(b) some will enter the University.

5/PROBABILITY 151

30. In the long run, a marksman finds that the ratio of successes to failures in hitting the
target is 9 to 1. If 3 rounds are fired, what is the probability of:
(a) 3 successes,
(b) at least 2 successes,
(c) not more than 1 success?
31. In an assortment of chocolates the ratio of hard centres to soft centres is 3 to 5. Find
the probability that, if 3 chocolates are chosen at random:
(a) exactly 2 will have hard centres,
(b) at least 1 will have a soft centre.
32. In an opinion poll, the ratio of those in favour to those against a particular proposal
was 7 to 3. What is the probability that if 3 randomly chosen people are interviewed:
(a) all will be in favour,
(b) the majority will be in favour,
(c) not more than 2 will be against the proposal?
33. On average, it rains on 2 days out of every 7 in a particular area in summer. Find the
probability that in a given weekend it will rain (a) on both days (b) at least one day?
34. An urn contains a number of cubes, 60 per cent of which are white and the remainder
black. Two cubes are randomly selected without replacement. What is the probability
that:
(a) they are the.,same colour,
(b) they are different colours?
35. An
are
(a)
(b)

urn contains 10 cubes, 6 of which are white and the remainder black. Two cubes
randomly selected without replacement. What is the probability that:
they are the same colour,
they are different colours?

36. In a group of 10 people,S are males and 5 are females. If two people are selected at
random, what is the probability that:
(a) they are both males,
(b) at least one is a female?
37. In a large gathering of people, 50 per cent are males. If two are selected at random
what is the probability that:
(a) they are both males,
(b) at least one is a female?
38. Urn X contains 1 white cube and 2 black cubes. Urn Y contains 2 white cubes and 1
black cube. An urn is selected at random and from it two cubes are selected without
replacement. Draw a tree diagram to represent this three-stage process and find the
probability that both cubes drawn are:
(a) the same colour,
(b) different colours.
39. A and B playa set of tennis. When a player wins two games, the set is won. If A has
probability 0-6 of winning anyone game, what is the probability that A wins the set.

Construct a tree diagram.


40. Of 2 coins A and B, A is a fair coin, B is loaded with a probability of 0'6 for a head. A
coin is chosen at random and tossed twice. What is the probability of 2 heads?

CHAPTER 6

Relations and
Functions
6.1 Relations _ _ _ _ _ _ _ _ _ _ _ _ _ __
You are familiar with non-mathematical phrases such as these:
is the brother of
plays football with
is taught by
travels to school by
is married to
was born in
and you are familiar with mathematical phrases such as these:
is equal to
is greater than
is the square of
is parallel to
is a multiple of
is equal to or less than
To become a sentence, each of these phrases would need to be completed by a first and
a last word. The sentence would then tell us that a certain relation exists between the two
things named. The two things named are members of two different sets, say set X and set
Y. One or more elements x E X are related to one or more elements y E Y in some way or
other, thus forming a set of ordered pairs (x, y).

EXAMPLE 1
Consider two sets of numbers X and Y such that X
{2, 3, 4, 6\ and Y
{l, 2, 4, 51.
If x is an element of X, and y is an element of Y, then a typical ordered pair is (x, y).
The set of all possible pairs is called the Cartesian product of X and Y. In symbols,
X x Y

I(x, y): x EX, Y E Yl.

Thus X x Y contains 16 pairs:


X x Y = 1(2,
(3,
(4,
(6,

1),
1),
1),
1),

(2,
(3,
(4,
(6,

2),
2),
2),
2),

(2,
(3,
(4,
(6,

4),
4),
4),
4),

(2,
(3,
(4,
(6,

5),
5),
5),
5)1

Any subset of this Cartesian product is a relation, and, in most cases, the relation is
defined by some rule; i.e., y is related to x in some way.
Suppose that the rule is that y is greater than x. In symbols, it is expressed as
y > x for x E X, Y E Y

6/RELATIONS AND FUNCTIONS 153

This relation contains five ordered pairs, 1(2, 4), (2, 5), (3, 4), (3, 5), (4, 5)1, and can be
represented graphically (fig. 6-1).

Fig. 6-1

This type of relation is sometimes called a many-many correspondence.

The relation can also be represented by plotting points on a plane (fig. 6-2).

6
5
4
3
2
1

23456

Fig. 6-2
A relation., is a set of ordered pairs, and is usually defined by some property or rule.
The domain of the relation is the set of all first elements of the ordered pairs.
The range of the relation is the set of all second elements of the ordered pairs.
Thus, in the preceding example, the domain is the set 12, 3, 41 and the range is the set
14,51.

x is called the independent variable since it may be chosen freely from the elements of
the domain and y is called the dependent variable since its value depends on the value
chosen for x.

EXAMPLE 2
Let S be the set of ordered pairs defined by y > x, x E R where R is the set of all real
numbers.
Although the rule defined by y > x is the same as in example 1, the domain of the
relation is the set of all real numbers, R. Also, the range is the set of all real numbers. The
graph of the relation is shown in fig. 6-3 as the half-plane above the line with equation
y = x. S contains an infinite number of ordered pairs, many of which have the same first
element, e.g. (1, 2), (1, 5), (1, 1r), (1, -J2), (..[3,3), (..[3, 2'5), ...

i (

154 NEW SENIOR MATHS: TWO UNIT COURSE

x
y>x, xeR

Fig. 6-3

EXAMPLE 3
Graph the relationx2 + y2 = 9
The graph of the relation (fig. 6-4) is the circumference of a circle whose centre is the
origin and whose radius is 3 units. The domain of this relation is not explicitly stated; in
which case it is assumed to be all real numbers for which the given relation can apply. The
domain is -3 ~ x ~ 3 and the range is similarly -3 ~ y ~ 3
y
3

-3

x2 +

y2

= 9, - 3 ~ x ~ 3

-3

Fig. 6-4
There are two ordered pairs in the relation that have the same first element (with two
exceptions), e.g. (0,3), (0, -3), (1, .J8), (1, -.J8), (-J2, -J7), (-J2, --J7), (-1, .J8), (-1, -.J8), ...
For all values of x greater than 3 or less than -3, the relation is not defined. Why?

6.2 Functions _ _ _ _ _ _ _ _ _ _ _ _ _ __
A function may be defined as a set of ordered pairs with the special property that no two
pairs have the same first element. It is, then, a special case of a relation.
In each of the following figures, a vertical line has been drawn to intersect the graphs.
In fig. 6-5 and fig. 6-7, this line intersects the graph in one point only. Thus, in
fig. 6-5, the only ordered pair with first member 2 is (2, 3), and, in fig. 6-7, the only
ordered pair with first member 2 is (2, 4). Hence these figures are the graphs of functions.
In fig. 6-6 and fig. 6-8, the vertical line intersects the graphs at more than one point.

'-'"" - --

--

- -_

.. _-

- --

-'-

.-

' - --"

'

- "~

--_. ' , ...- i _

6/RELATIONS AND FUNCTIONS 155

Function: y

=x +

1,

X E

Relation: x 2

Fig. 6-5

+ y2 = 4, -2

Fig. 6-6

12
1

I
I

I
Function: y

X2,

Relation: y2

Fig. 6-7

= x,

X ~

Fig. 6-8

Hence the relations represented by these graphs are not functions.


However, by transposing the equation
X2

y2 =

thereby obtaining
y=
andy =
each of these passes the test for defining a function.
y
...)9 - X2 defines the function whose graph is the top semicircle in fig. 6-6 and
y = -...)9 - X2 defines the function whose graph is the lower semicircle, both with the
same domain, -3 E;'; x E;'; 3.
y2 = X
(fig. 6-8)

Similarly,
is equivalent to
y = -JX

and each of these equations can be used to define a function.

y
-JX defines the function whose graph is the top branch of the parabola in fig. 6-8
and y = --JX defines the function whose graph is the lower branch each with the same
domain x ~ o.

156 NEW SENIOR MATHS: TWO UNIT COURSE

A function, f, is fully defined when the domain and rule are stated or when the
complete set of ordered pairs is given.
We use the notation f(x) or y to mean the value of f for any value of the independent
variable x in the domain. The expression 'y is a function of x' is frequently used to
indicate a rule of the form y = f(x) e.g. y = X2, -2 ~ x ~ 2 or f(x) = x 2 , -2 ~ x ~ 2 are
used to mean the same thing. The rule tells us what to do, namely square any numbers
provided they are between - 2 and 2.
e.g.

f(l)
f( - 2)

= 12

= 1

= (- 2)2

f(3) is meaningless because the rule applies only for -2 ~ x ~ 2. Hence the rule, without
a statement of the domain, is not in itself the function. It is common, however, to state the
rule for f(x) without reference to its domain. If this is so, the domain is implied as being
the largest possible domain.

EXAMPLE 4
State the largest possible domain for the functions defined by the rules:

WA~=~

~A~=~

~A~

(d) f(x)

(e) f(x) = -2--1

x
x
y

(a) Since any real number can be squared,


the largest possible domain is all real
values of x. Furthermore since any
number squared is non-negative, the
range is all positive numbers including
zero.
.'. f(x) =

X2,

x ER

(b) Since the fraction 1 can be defined for all values of x except those which make the

denominator zero, the largest possible domain is all x except x = O. What is the range?
.

.. f(x) = -, x*-O

(c) Since the positive square root of a negative number does not exist in the field of real
numbers, x ~ O.
~, x ~ O. What is the range?

:. f(x)

(d) 4 -

X2 ~

0 and so -2

~ 2.

-2

:. f(x)

(e) X2

2. What is the range?

1 *- 0 and so x *- -1, 1.
:. f(x)

= X2 ~

l' x *- -1, 1

Sometimes a function is defined by a certain rule for part of the domain and a different
rule for another part of the domain.

6/RELATIONS AND FUNCTIONS 157

EXAMPLE 5
The functionf(x)

={X22, x ~

1
- x,x >
has a domain all real x, but has two different
rules: one for x ~ 1, the other for x > 1.
e.g.
f( -2) = (_2)2
4
f(3)
= 2 - 3 = -1

6.3 Open and closed intervals _ _ _ _ _ _ __


Sometimes we are interested in function values for a particular subset of R, consisting of
all values of x between two real numbers a and b, b > a. This subset is called an interval,
and a and b are the end points of the interval.
If the end points are included in the interval, then the interval is a closed interval,
denoted by [a, b].
If the end points are excluded, the interval is an open interval, denoted by (a, b).
If one end point only is included, the interval is half-open, denoted by (a, b] if open on
the left, or by (a, b) if open on the right.

...x

Closed interval [a, b] = {x:a ~ x ~ b}.

...

----------~O
~

O~------------~
b
X

Open interval (a, b) = {x:a <x <b}.

..

--------~o~-------------------~----------~~
a
b
X
Half-open on left (a, b]

{x:a

<x

~ b}.

O~----------~~

..

Half-open on right [a, b) = {x:a ~ x

< b}.

6.4 Restriction of a function _ _ _ _ _ _ _ __


Consider the functions:
y

(a) f(x)

= X2

for all x.

(b) g(x) = x 2 , -1

(c) h(x)

= x2, x

158 NEW SENIOR MATHS: TWO UNIT COURSE

The different letters f, g and h, used to name the three functions, emphasize the fact
that we have three different functions since they are defined on different domains, even
though the rule is the same in each case. g and h are called restrictions of f since their
domains are subsets of the domain of f. Note also that the range has altered.

6.5 Odd and even functions _ _ _ _ _ _ _ __


An odd function is characterized by the property that
f( -x)
f(x)
f( -x)

e.g. if
then

- f(x)
x3
(-X)3

-8

-x3

-f(x)
fIx)

= x3

Since f(x) and f( - x) are opposite in sign, the portion of the graph of f for x
mirror image in the origin of the portion for which x ~ O.

0 is a

a and

An even function is characterized by the property that


f( -x)
f(x)
f( -x)

e.g. if

f(x)

x 2 , then

(_X)2

X2

= f(x) and sofis even.

The graphs of even functions are symmetrical about the Y-axis.


The statement f( -a) = f(a) implies that the function is defined for both x

x = - a.
~ 0 is not an even function sincef( -a) is not defined.
The recognition of oddness or evenness property of functions is useful in curve
sketching. Only details of one half of the graph need to be calculated. The other half may
be drawn using the symmetry properties. Most functions are neither even nor odd.

f(x)

x 2, x

6.6 Special functions _ _ _ _ _ _ _ _ _ _ __


Throughout this course, you will meet many important functions such as these:
(a) Quadratic functions, chapter 12
(b) Polynomial and rational functions, chapters 14 and 15
(c) Exponential (index) and logarithmic functions, chapters 13 and 18
(d) Circular functions, chapter 17
(e) Sequence functions, chapter 11
There is another important function, the absolute value function. Since the notion of
absolute value was examined in chapter 1, we will now consider the function briefly.

6.7 Absolute value function - - - - - - - -


The function f(x)

= Ixl

is called the absolute value or numerical value function.

6/RELATIONS AND FUNCTIONS 159

EXAMPLE 6
Sketch the graphs of
(a) y = #
(b) y = 12x (c) y
x + Ixl
(d) y
Ix2 - 41

11

(a) y

= #- =

Ixl

= x if x >
= -x if x

o if x =

(b) y

12x -

11

< 0
0

1
2x - 1 for x > 2"

1 - 2x for x <

= 0 for x

(c) y

= x + Ixl

2x if x > 0

= 0 if x

2"

:E;;

y=o

(d) y

= Ix2 - 41
= X2 - 4 if X2 - 4

0 i.e. x :E;; -2 or x
4 - X2 if X2 - 4 < 0 Le. -2 < x < 2
~

EXERCISES 6
1. Which o f the following relations are functions? State their largest possible domain
and range.
(a) 1(1, 1), (2, 2), (3, 5), (3, 7)1
(b) 1(-3, 1), (3, 1), (8, 7), (9, -2), (11, 6)1
(c) (x, 5) for all x
(d) (2, y), y a natural number
(e) X2 + y2 = 4
(f) y
-../4 - X2
(g) y
X
(h) y ~ x

+
+

2
2

160 NEW SENIOR MATHS: TWO UNIT COURSE

2. Sketch the graph of each of the following functions. State their largest possible
domain and range.
(a) f(x) = 9 - x
(b) g(x) = 9 - X2
(c) h(x) = -....)4 - X2
(d) f(t) = -Jt

3. Consider the function defined by f(x) = 3x - 6 for all x.


(a) Find the value of f(1), f( - 2), f(a).
(b) For what value of a is f(a) = a?
(c) For what values of x is f(x) > x?
(d) Sketch the graph of f.
4. Consider the linear function g(x), defined for x ~ o.
Given that g(3) = 2 and g(4) = 4, write down the rule that defines the function and
state the range.

S. Consider the functionf(x) = X2 - l,x any real number.


(a) Find f(3), f( - 3).
(b) Find f(a), f(b), f(a + b).
(c) Is f(a) + f(b) = f(a + b) for all a and b?
(d) State the range of f.
6. For the function g(x) = -./X, x ~ o.
(a) find g(X2), g(x+ 2), g(x + 2)2,
(b) write down the range of g.

7. Sketch the graphs of the following absolute value functions defined for all x and state
the range in each case.

21

(a) f(x) = Ix (b) g(x) = Ixl - 2


(c) h(x) = Ix + 11
(d) f(x) = ....)(x + 2)2
(e) y = 13x (f) y =
2x I
(g) y = 4 - 12x I

14 -

61

8. Sketch the graph of the relation Ixl + Iy I = 1.

Is the relation a function? State the greatest possible domain and the range.

9. Find f(x) for all x in each of the following, given that


(a) f(a + 2) = a + 5,
(b) f(x - 1) = X2 - 1,
(c) f(2a) = 4a 2 - 4a + 1,
(d) f(t) = at2 + b, f(2) = 5, f(3) = 20.

10. [x] is defined to mean the greatest integer not exceeding x,


e.g. [1] = 1, [26] = 2, [-16] = -2.

For each of the following functions find f(2) , f( - 2), f(I4), f( -14).

(i) f(x) = [x]


(ii) f(x) = [x + 1]
(iii) f(x) = Ixl + [x]
(iv) f(x) = [2x]

6/RELATIONS AND FUNCTIONS 161

11. Sketch graphs of the following functions:

= {x +

(i) f(x)

1 ~f x
2 If x

~ 1
< 1

(ii) f(x) = { x

2x if x ~ 0

(iii) f(x) {
-2x if x

"*

-2

if x = -2
4
x,x> 1
{I, -1 ~ x ~ 1
-x, x < -1

(iv) f(x)

<0

if x

12. State the largest possible domain and range for:


(i)f(x) ~
(ii) f(x) '/3~
(iii) f(x) ..Jx2 - 9
(iv) f(x) =
+ ~ (omit range)
(v) f(x) =

x
lXl

13. State whether the following functions, defined on their largest possible domain, are
odd, or even, or neither:
(a) f(x)
(d) f(x)

x
x3

(g)

(b) f(x)
(e) f(x)

(h) f(x)

4 -

(c) f(x)
(f) f(x)

x
X

14. A function is defined by the rulef(x)

1
X2

Ixl
(x - 2)2

(i) f(x)

+ 1, x

= { -1, -1 <
1

=
=

x,

< 1

x'~

-2

Find, if they exist,


(a) f(l)

(b) f(

(c) f(O)

1)

15. A function is defined by the rule f(x)

(d) f(2)

+ f(

2)

{ x'- x < 0
~' x ~ 0

Find
(a) f(O)

(c) f( -2)

(b) f(2)

16. A function is defined by the rule f(x)

(d) f(a 2 )

..Jx - 1, x ~ 1
{ x-I, x < 1

Find
(a) f(1)

(b) f(

(c) f(10)

1)

+ f( -10) (d) f(5)

17. Consider the linear functionf(x)


ax + b where a and b are constant.
(a) Find the values of a and b given that f(1) = -1 and f(2) =
(b) Findf(3) andf(10)
(c) Find the value of x for which f(x)

x.

18. Find the largest possible range for the following functions:
(a) f(x)
(c) f(x)

(x - 3)2
..J16 - X2

19. Sketch the graph of f(x)


f(x) = 3.

(b) f(x)
(d) f(x)

12x -

x + Ixl
16 - x 2

51 and indicate' on it the values of x for which

CHAPTER 7

Trigonometric
Ratios
7.1 Unit circle _ _ _ _ _ _ _ _ _ _ _ _ _ __
y

(-1,0)

{1.0}X

Consider a circle of unit radius, whose centre is at the origin. The equation of this circle is

Take any point P whose coordinates are (x, y) on the circumference of this unit circle.
Consider the point P as starting from A and rotating in an anti-clockwise direction, taking
up various positions on the circumference as shown in the diagrams above. In each
position L AOP = O.
We define cosine and sine as
cos 0
sin 0

x
y

x-coordinate of P
y-coordinate of P

7/TRIGONOMETRIC RATIOS 163

The definitions of sine and cosine apply to angle () of any magnitude.

Since -1 ~ x ~ 1, cos () lies between -1 and 1

and -1 ~ y ~ 1, sin () lies between -1 and 1.

If the point Pis:


(i) at A, ()
0, the coordinates of A are (1, 0) and so cos 0
1, sin 0 = 0
(ii) at B, () = 90 0 , the coordinates of Bare (0, 1) and so cos 90 0 = 0, sin 90 =
(iii) at C, () = 180 0 , the coordinates of Care (-1, 0) and so cos 180 0 = -1, sin 180 0 = 0
(iv) at D, () = 270 0 , the coordinates of Dare (0, -1) and so cos 270 0 = 0, sin 270 0
= -1
(v) at:4again,()= 360,thecoordinatesofAare(I,0)andsocos360 = 1, sin 360 0 =0.

Note that cos 360 0 = cos 0 0 = 1 and sin 360 0 = sin 0 0 = O.


If we consider the point P as rotating more than once round the circle, then angle A OP
can be regarded as 360 0 + (), 720 0 + () etc and so cos (360 + ()
cos (), sin (360 0 + ()
= sin () etc.
There are four other trigonometric ratios:
tangent, cotangent, secant and cosecant (their abbreviations are tan, cot, sec and cosec
respectively) that can be defined in terms of cos and sin.

tan () = l = sin () cos ()


x
cos
x
cos () .
= ---:--()' Sin ()
cot () =
sec () =
cosec () =

Sin

1
x

= --()'
COS

COS ()

*0
*0
*0

-J-(), sin () * 0

Sin

Thus there is a restriction on the values of (). Tan and sec are not defined when cos () 0

Le. when () = 90 0 , 270 0 , 450 0 , cot and cosec are not defined when sin () = 0 i.e. when ()

= 0 0 , 180 0 , 360 0
Observe that cot, sec and cosec are the reciprocals of tan, cos and sin respectively.
The tangent may be considered as a ratio without reference to sin and cos and may be
given a distinct geometrical representation.
y

At the point A (1, 0) where the unit circle cuts the X-axis, draw a tangent line A T. If angle
(), we define tan () as the y-coordinate of T. (Actually, this is where tan gets its
name).
If P is at B, () = 90 0 , OB is parallel to AT and so tan 90 0 is not defined.

AOP

r -

164 NEW SENIOR MATHS: TWO UNIT COURSE

< e < 180, tan e < O. What are the values of tan 0, tan 180, tan 3601
Is tan e defined when e = 27{)01
Draw the above diagrams for ebetween 180 and 270 and also between 270 and 360
and note the positions of T.
If 90

7.2 Symmetry properties of trigonometric


ratios __________________
The coordinate axes divide the circle into four quarters, called quadrants.
y
First quadrant 0 < e < 90
Consider the point P(a, b) where Plies
between A and B. Both the x and y
coordinates of P are positive numbers and so
all the ratios are positive.

Second quadrant 90 < e < 180


Consider the point P(a, b) in the first
quadrant such that L A OP = e and a point
Q in the second quadrant such that LA OQ
180 - e. By symmetry, the coordinates
of Q are (-a, b). Hence

cos (180 - e) = -a
sin (180 - e) = b
b

tan (180 - 0.) = -a

-cos e
sin e

-tan

From this we can see that, for every angle in the 2nd quadrant, there is a corresponding
angle in the 1st quadrant whose sine, cosine and tangent ratios are numerically the same.
This angle is found by subtracting the 2nd quadrant angle from 180. Since the
x-coordinate is negative in the 2nd quadrant, the values of cos and tan are now negative;
sin remains positive.
e.g. cos 140
sin 140
tan 140

cos (180
40) = -cos 40
sin (180 - 40)
sin 40
tan (180 - 40)
-tan 40

>:::
>:::
>:::

-0,7660
0'6428
-0'8391

Third quadrant 180 < e < 270


Consider the point P(a, b) in the first
quadrant such that LA OP = e and a point
Q in the third quadrant such that LA OQ =
180 + e. By symmetry, the coordinates of
Q are (-a, -b). Hence
cos (180 + e) = -a
sin (180 + e) = -b
-b
tan (180 + e) = -a

x
3rd Quadrant

-cos e
-sin e
tan

7/TRIGONOMETRIC RATIOS 165

For every angle in the 3rd quadrant, there is a corresponding angle in the first quadrant
whose sine, cosine and tangent ratios are numerically the same. You can find this angle by
subtracting 180 from the 3rd quadrant angle. Since in the 3rd quadrant x < 0 and y < 0,
only tan is positive; sin and cos are both negative.
e.g. cos 220 = cos (180 + 40) = -cos 40
sin 220 = sin (180 + 40) = -sin 40
tan 220 = tan (180 + 40) = tan 40

-0,7660
-0'6428
0'8391
y

Fourth quadrant 270 < 0 < 360


Consider the point P(a, b) in the first
quadrant such that LA OP = 0 and a point
Q in the fourth quadrant such that LA OQ
360
O. By symmetry, the coordinates
of Q are (a, -b). Hence
cos (360 - 0)
sin (360 - 0)

a
=

-b

cos 0
-sin 0

4th Quadrant

-b
tan (360 - 0) =
= -tan 0
a
For every angle in the 4th quadrant there is a corresponding angle in the first quadrant
whose sine, cosine and tangent ratios are numerically the same. You can find this angle by
subtracting the 4th quadrant angle from 360. In the 4th quadrant, only cos is positive.
e.g. cos 320 cos (360 - 40) = cos 40
sin 320 = sin (360 - 40) = sin 40
tan 320 = tan (360
40) = -tan 40

~
~

0'7660
- 06428
-0,8391

7.3 Sign of the trigonometric ratios _ _ _ _ __


The sign of cos, sin and tan for the four quadrants can be summarized as follows:

First quadrant:
Second quadrant:
Third quadrant:
Fourth quadrant:

All are positive (A)

Sin only is positive (S)

Tan only is positive (n

Cos only is positive (C)

Try to think of some way to remember these e.g. All Stations to Central (ASTC).

7.4 Complementary angles: () and 90 - () _ __


y

166 NEW SENIOR MATHS: TWO UNIT COURSE

Consider the point P(a, b) on the unit circle such that L AOP = (J and a point Q such that
L AOQ = 90 - (J. From congruent triangles, the coordinates of Q are (b, a) because ON
PM = band QN = OM = a

sin (90 -

(J)

tan (90

(J)

sec (90 -

(J)

a
a

b
1
b=

b = sin
b

cos

(J

cos (90

(J) =

cot

(J

cot (90

(J)

cosec (90

(J) = -

cosec

(J

= -a =

(J

tan

(J

1
= sec
a

(J

These relationships are said to be complementary. This accounts for the prefix' co' which
appears in cosine, cosec and cot.
sine and cosine, tan and cotan, sec and cosec are complementary pairs.
e.g. sin 50 = cos 40
tan 75 = cot 15
sec 80 = cosec 10

cos 60 = sin 30
cot 54 = tan 36
cosec 32 = sec 58

7.5 Negative angles: () < 0


So far consideration has been given to
(J > 0. We have seen that if we start from
the point A and rotate in an anticlockwise
direction to P, then (J > O. If, however, we
rotate in a clockwise direction to Q so that
L AOQ = L AOP, then (J < O.
Hence, by symmetry,

cos ( - (J) = a
sin (-(J) = -b
tan

e.g.

cos
tan
sin
cot

(-160)
( 245)
( - 200)
( 110)

cos ( - 40)
tan (-25)
sin (-70)
= cos 160
= -tan 245
- sin 200
= -cot 110

(-(J) =

= cos 40

cos (J
-sin

(J

-b = -tan
-

(J

;:::: 0'7660

= -tan 25;:::: -0,4663


= - sin 70 ;:::: - 09397

cos 20
tan 65
= sin 20
= cot 70

;:::: -0,9397
;:::: -2'1445
;:::: O' 3420
= tan 20 ;:::: 03640

EXERCISES 7(a)
1. In which
(a) sin (J
(d) sin (J
(f) cos (J

of the four quadrants is:


(b) tan
(J < 0
< 0 and tan (J > 0

> 0
< 0 and tan

(J

< 0

(c) cos (J < 0


. (e) sin (J > 0 and cos (J < 0
(g) cos (J > 0 and tan (J > O?

7/TRIGONOMETRIC RATIOS 167

2. State the quadrant in which each of the following angles lies:


(a) 72 0
(b) 114 0
(c) 95 0
(d) 200 0
0
0
0
(f) 183
(g) 83
(h) 216
(i) 300 0

(e) 321 0
(j) 155 0

3. Express each of the following as a trigonometric ratio of angle A:


(a) sin (180 0 - A)
(b) cos (90 0 - A)
(c) tan (360 0 - A)
0
0
(d) cos (180 + A)
(e) sin (360 - A)
(f) cot (90 0 "1 A)
4. If 0 is an angle in the 2nd quadrant, state whether the following are positive or
negative.
(a) cos (180 0 - 0)
(b) tan (180 0
0)
(c) sin (~Qo - 0)
0
(d) sin (360 - 0)
(e) cos (180 0 + 0)
(f) tan (90
0)
0

5. Use tables or a calculator


following values of 0:
(a) (i) 125 0
(ii)
(b) (i) 205 0
(ii)
(c) (i) 282 0
(ii)
(d) (i) -25 0
(ii)

to evaluate sin 0, cos 0, tan 0 and cot 0 for each of the


152 0
217
301 0
-122

(iii)
(iii)
(iii)
(iii)

6. If sin a = 02 write down the value of


(a) sin (180 0 - a)
(b) sin (360 0
0
(d) cos (90 - a)
(e) sin (180 0
7. If tan 0 = t, express in terms of t
(a) cot 0
(b) cot (90
(d) tan (360 0 - 0)
(e) cot (180 0

a)
a)

0)

117 0
251 0
342
-215
0

(c) sin (-a)


. (f) cosec a

(c) tan (180 0

(f) tan (180 0

0)
0)

8. If cos A c, express in terms of c


(a) sec A
(b) cos (-A)
(d) cos (360
A)
(e) sec (-A)

(c) cos (180


(f) cos (180 0

A)

+ A)

9. Use tables or a calculator to evaluate


(a) tan 125 0
(b) sin 212 0
(d) sin (-160 0 )
(e) cot 42 0

(c) cos (-140 0 )


(f) cos 260 0

0)

10. If 90 0 < 0 < 180 0 , show with the aid of a diagram of a unit circle that
(i) cos (180 0 + 0) = -cos 0
(ii) sin (360 0 - 0)
- sin 0
11. Use the fact that sin (360 0
write down the values of
(a) sin 400 0
(d) sin 610 0

0)

= sin 0 etc and, with the aid of tables or a calculator,

(b) cos 425 0


(e) cos 580 0

(c) tan 520 0


(f) cot 490
0

7.6 Graphs of the trigonometric functions


In fig. 7-1, the graph of sin 0 is represented by the continuous curve, and the graph of cos 0
by the broken curve. The graphs are drawn in the domain -360 0 to 360 0 The graphs,
however, for domain R are unlimited curves, and their form from -360 0 to 0 0 is the same
as from 0 0 to 360 0 and from 360 0 to 720 0 and so on. In other words, as 0 increases, the
values of sin 0 and cos 0 repeat themselves after an interval or period of 360 sin and cos
are therefore called periodic junctions, the period being 360 0 We saw earlier in this
0

168 NEW SENIOR MATHS: TWO UNIT COURSE

chapter that the point P(x, y) corresponds to angle 0, 360 0


(360 + 0) = sin 0 and cos (360 + 0) = cos O.
0

0, 720 0

0, and hence sin

Fig. 7-1
The maximum and minimum values of sin 0 and cos 0 are 1 and -1 respectively; in
other words their amplitude is 1. If the cos curve is translated 90 to the right, parallel
with the O-axis, it coincides with the sin curve. This follows from the fact that cos 0 =
sin (90 + 0).
y

Fig. 7-2
Fig. 7-3
Fig. 7-2 shows the graph of tan 0 for 0 ~ 0 ~ 360. As 0 increases, the values of tan 0
repeat themselves after an interval or period of 180. The tan function is discontinuous at
o = 90, 270, ...
Fig. 7-3 shows the graph of cot 0 in the domain 0 to 360. The values of cot 0 also
repeat themselves after a period of 180. The function is discontinuous at 0 0, 180,
360, ...
What transformation is necessary for the two curves to coincide?

y
1

!v=secoll

I y=coseco
I
:
I
I

Fig. 7-4

Fig. 7-5

7/TRIGONONIETRIC RATIOS 169

Figs. 7-4 and 7-5 show the graphs of cosec () and sec () respectively. They also are
periodic, with period 360. What restrictions are placed on their domain? What is the
range in each case? What transformation is necessary for the two curves to coincide?
The graphs of the six trigonometric functions have been drawn in pairs of
complementary functions-sine and cosine, tangent and cotangent, secant and cosecant.
As an exercise, draw thegraphs in pairs of reciprocal functions-sin and ~osec, tan and
cot, cos and sec, remembering that the reciprocal of 1 is 1 and the reciprocal of 0 is not
defined. These two facts should enable you to deduce the graphs of cosec, cot and sec
from sin, tan and cos respectively.

7.7 Some exact values _ _ _ _ _ _ _ _ _ _ __


It is necessary to use tables or a calculator to find the trigonometric ratios of most angles

and these ratios have only an approximate rational number value. For angles in the first
quadrant of 0 , 90, 30, 60, 45 , the ratios can be calculated exactly without the aid of
tables or a calculator.
0 and 90 can best be associated with the unit circle.
30 and 60 can best be associated with an equilateral triangle of side lengths 2 units.
45
can best be associated with a right-angled isosceles triangle of equal side
lengths 1 unit
(0,1) 8

() =

0 when P is at A

() = 90 when P is at B

sin

cos

tan

cosec

sec
cot

~
2
1
2

-J2

2
~

-J2

2
~

-J2

1
~

1
2
~
2
1
~

1
1

-J2

These exact values can best be remembered if we get a mental picture of the three
diagrams above.
These exact values can be extended to certain angles in the other quadrants.

170 NEW SENIOR MATHS:.TWO UNIT COURSE

EXAMPLE 1
Without the aid of tables or a calculator:
(i)
(ii)

(iii)

(iv)

sin 150

cos 225 = cos (180 + 45)

sin 30

1
2

= -cos 45

tan 240 = tan (180 + 60) = tan 60


-.fj
sin 270 = - 1 when the point P is at (0, -1) on the unit circle
- sin 90
= - 1
or sin 270 = sin (180 + 90)

(v) cos ( 300)


(vi)
(vii)

sin (180 - 30)

cos 300

cos 60

sec 120
sec (180 - 60)
- sec 60
cosec 330 = cosec (360 - 30) = - cosec 30

1
2

-2
-2

EXAMPLE 2
Find all values of () between 0 and 360 for which
(a) cos ()

(d) sec () =

(b) sin () =
2

(c) tan ()

(e) sin () = -1

(a) Since cos () is positive, () lies in the 1st and 4th quadrants
1
cos () = 2

() =

60,360

60

= 60, 300
(b) Since sin () is negative, () lies in the 3rd and 4th quadrants
. ()

sm

= - ..J2

() = 180 + 45,360 - 45

= 225, 315
(c) Since tan () is positive, () lies in the 1st and 3rd quadrants
tan ()
1
() 45, 180 + 45
45, 225
(d) Since sec () is negative, () lies in the 2nd and 3rd quadrants
2
sec ()
. cos ()
()

-2
180 - 30, 180 + 30
150, 210

(e) Since sin () is negative, () lies in the 3rd and 4th quadrants
-1
sin ()
()
180 + 90,360 - 90

7/TRIGONOMETRIC RATIOS 171

EXERCISES 7(b)

Without the aid of tables or a calculator, write down the exact values of: (1. to 4.)

1. (a) sin 90 0
(e) sec 135 0

(b) cos 120 0


(f) cot 150 0

(c) tan 150 0


(g) cosec 90 0

(d) cos 180 0


(h) sin 120 0

2. (a) sin 180 0


(e) cos 240 0

(b) cos 210 0


(f) sec 225 0

(c) tan 225 0


(g) tan 180 0

(d) cot 240 0


(h) sin 210 0

3. (a) sin 270 0


(e) cot 315 0

(b) tan 300 0


(f) cos 330 0

(c) cosec 330 0


(g) sin 300 0

(d) tan 315 0


(h) cosec 300 0

4. (a) sin 360 0


(e) sec 390 0

(b) cos 390 0


(f) cos 450 0

(c) tan 405 0


(g) sin 420 0

(d) cot 420 0


(h) tan 495 0

5. Complete the following table and draw the graph of y


to 360 0
()

00

90 0

30 0

150 0

180 0

sin () for values of () from 0 0

210 0

270 0

330 0

360 0

sin ()
6. Complete the following table and draw the graph of y
to 360 0
()

00

60 0

90 0

120 0

180 0

= cos () for values of () from 0


240 0

270 0

300 0

360 0

cos ()
Find all the values of () between 0 and 360 0 for which: (7. to 18.)
7. sin () = -

-J3

2
9. cosec () = 2
11. cot () = -J3
13. sin () = cos ()
15. 2 cos 0 + 1
0
17. sin () + -J3 cos 0

8. tan ()

10.
12.
14.
16.
18.

-1

cos () = -1
sec () = -.J2
sin () = 0
2 sin () = -J3
cosec () = sec ()

7.8 The fundamental identity _ _ _ _ _ _ __

172 NEW SENIOR MATHS: TWO UNIT COURSE

Since the equation of the unit circle is

X2 + y2

1,

and, by definition,

x = cos

it follows that

cos2 (j + sin 2 (j = 1
1 - sin 2(j = cos2 (j
and 1 - cosl (j = sin 2 (j

(j,

sin

(j,

(1)

Divide (1) by cos2 (j:

(2)

Divide (1) by sin2 (j:

(3)

EXAMPLE 3
If sin

(j =

~ and 90 <
0

(j

cos2

Since

< 180 0 , find the value of cos


(j

+ sin2

cos2 (j

then

(j =

9
+ 25

and tan

(j

< 180 0

(j.

1,

=
16
25
4

cos2 (j
cbs (j

tan

(j

(j =

since 90 0 <

sin (j
cos
3
4

Alternatively, consider a 3, 4, 5 triangle in


the 2nd quadrant of a circle of radius 5 units.
Hence
. (j
.
)
sm
=:53 (gIven
cos

(j

tan

(j

4
5
3
4

EXAMPLE 4
Simplify

2
(i) 11 - cos2 (j
- sin

(iii) ~a2
(i)

(ii) 1
(iii)

cos2 (j

-Jli2+7

+ X2 where x
sin2
cos 2

+ tan2 (90

(ii) 1

(j

= ~a2

(j
(j =

(j) =

tan 2 (90 0

a tan

(j

and 0 0

(j)

<

(j

< 90

2 (j

an

1 + coP

+ a2 tan2 (j =

(j

cosec 2 (j

~a2(1

tan2 (j)

.Jill. sec 2 (j

a sec

(j

7/TRIGONOMETRIC RATIOS 173

EXAMPLE 5
Prove the following identities
cos (J
.
1
sin2 (J
2
(I) . 2 (J
(ii) 1
..
sm
+ cos2 (J cos (J
- sm
(iii) tan A sin A + cos A = sec A

cos (J
1 - sin

(ii)

(J -

tan

sec

(J

- - = cos 2 (J

(J =

=
(iii) tan A sin A

(J

cos2 (J
1

1 - sin2 (J
sin2 (J + cos2 (J

(i)

tan

(J -

+ cos A

cos (J
sin (J
- sin (J
cos
cos2 (J - sin (J (1 - sin
(1 - sin (J) cos (J
cos2 (J - sin (J + sin2 (J
(1 - sin (J)cos (J
1
sin (J
(1 - sin (J)cos (J
1
cos
sec (J

sin

(J =1=

(J)

sin A
.
+ cos A
cos A
sin2 A + cos A
cos
1
sin2 A + cos2 A
cos

= - - . sm A

cos A
sec A

EXERCISES 7(c)

1. If

tan

(J =

;4

and 180 0 <

(a) sin (J
2. If tan (J =

(J

< 270

find the value of

find the value of (a) cot

(b) cos (J.

-~ and 90 <
0

(J

< 180

(J

(b) sin (J (c) cos (J.

~ and u is not in the first quadrant, find, in simplest surd form, the value of

3. If cos u

cos u - 2 cot u
tan u
u

4. Simplify:
(a)

sin2

(J

+ cos2 (J

2 cot ex

(c)
1 + cot2 ex
sin A
cos A
(e)
+
cos
cot2 (J
(g) cosec2 (J

(b)

2
sin (J
1 - sin2

(d) (sec2 (J
(f) sin3 (J

(b) 1

(J

1) tan (90 0

sin

(J

cos2 (J

sin2 (180 0 +

(J)

(J)

174 NEW SENIOR MATHS: TWO UNIT COURSE

5. Simplify:
(a)

X2
--;=::;:====

when x

17

6. If sec ex

(b) -Ja 2

asec 0

X2

when x

acos 0

and 90 < ex < 180, find the value of

(a) cos ex,


(b) sin ex,
(c) tan ex.

7. If 180 < 0 < 270 and sin 0 = p, express cos 0 in terms of p.


8. Find the value of sec 0 if tan 0 = 06 and 0 is not in the first quadrant.
9. If sin 0

x, express

10. If asin2 0 + bcos 2 0


11. If x

1 - cos2 0 .
20m terms of x.
sec

= c, express sin 0 and cos 0 in terms of a, band c.

asin 0 and y

bcos 0, find an expression that does not involve O.

12. If tan2 0 + 2 sec2 0 = 5, find the value of sin2 O.


13. Simplify:
(a) (1 + tan 2 u)(1 - sin2 u)
(c)

sin 0

+ 1 + cos 0
sin 0

(d)

+ -,-----:----

1 - sin 0

+ sin 0
x --;--------;:

(e) sin 0 cos (90 - 0) + cos 0 sin (90 - 0)


(g) 2 cos2 30 -

~in

(b)

(h) 1

(f) 1

tan
sin 0 cos (90 - 0)

Prove the following identities:


14. (1
tan x)2 + (1 + tan x)2 = 2 sec2 x

1 + cos t
15. (cot t + cosec t)2 = 1
- cos t
16. sin2 ex cos 2 {3 - cos 2 ex sin2 {3 = sin2 ex - sin2 {3
17. sec 0 + tan 0 =

sin 0
0
cos

18. sin2 0 tan 0 + cos2 0 cot 0 + 2 sin 0 cos 0


19. sin 0(1 + tan 0) + cos 0(1 + cot 0)

20. tan 0(1

22.

23.

1 + cot 0

cosec

sin 0 + cos 0

cot2 0) + cot 0(1 - tan2 0) = 0

cot 0 cos 0
21. cot

tan 0 + cot 0

cos 0
1 + sin 0
0
sec 0
-ta-n-O-=--+-c-o-t--':-0 = cos

(cos teat t - sin t tan t) sin teas t

cos t - sin t

1 + sin teas t

REVISION EXERCISES A

1. (a) Expand (2x - 5)2


(b) Write 5600000 in standard form
(c) Factorize (i) 81 '- 4a2 (ii) 10xy - 5y

=4

2. Solve for x: (a) X2


3. Solve for x: (a)

:5

= 4x

(b) X2

(c) X2

= 4x

- 4

(b) 3x - 1 = ~

= 20

20

4. Simplify 02 x 001 x 0'0003, expressing the result in standard form.


5. If a = - 3 and b = 4, evaluate -Ja 2
3x 3
a

..

6. SImplIfy (a) -42 x

b2

ay - a . 3y - 3
2.... 4 ay 2
xy

(b) 5x _ 2x :
7. Simplify (a) 25 112

+ ~.

(b) 3 0

(c) -J36

+ 64

(d) 3112

x 12112

8. Factorize (a) 5x2y - 10xy2 - 5xy


(b) 64 - 9a2

(c) a 2 18a + 56

9. The hypotenuse of a right-angled triangle is (x + 1) cm in length and the other two


sides are x cm and (x - 7) cm. Form an equation in x and solve it to find the length of
each side.
10. Represent the inequations on the number line

(b) -8 ::e;; 3x - 2 < 16

(a) 3x - 2 > 2

(c)

Ix -

11 > 1

11. (a) If V = 'lral (d - a), find the value of d if V = 1760, 'Ir = 3~, I = 35, a = 2
(b) If V = 'Ir(R + r)(R - r)h find the value of V to 3 significant figures, using a
calculator, if R = 7 '6, r = 4'1, h :;: 129

12. The perimeter of a rectangle is 18cm and its area is 20cm2.


(a) If the length is xcm, express the width in terms of x.
(b) Write down the area in terms of x.
(c) Form a quadratic equation in x and solve it to find the length and width.
.

12m 2 - 4m
10m 2n
+ -5
22
m - m
mn

13. (a) SImplIfy 3 2


3x
(b) E xpress (')
I .

+4

5
X

(")
II

+3

X2

1 + 1 as smg
. I
'
+ 2x
e f
ractIOns.

14. Solve the simultaneous equations

+ 1)

+ 1) + 10

107 and y = 5 X 10-3 find in standard form (a)

3(x - 3) = 2(2y
2(3x - 1) = 5(2y

15. If x = 3 X

x2y3

(b) (xy-l)2

176 NEW SENIOR MATHS: TWO UNIT COURSE

16. ABCD is a rectangular block 10cmby


6 cm leaning against a vertical wall.
Point D is 5 cm above the horizontal
ground.
(a) Prove that l:,EAD and l:,FAB are
similar.
(b) Find, in simplest surd form, the

vertical height. of Band C above the

ground.

17. Solve the quadratic equation 2X2


(i) in simplest surd form
(ii) correct to 3 decimal places.

x + 5 giving your solutions

18. Express in simplest surd form with rational denominator:


(a) -Jf2

(b) -J5

+ 4.J3 +

- 3..J54

+ .J2O + .J8O

19. Solve for x: (a) (x 2 - 3X)2

16

(b) (X2 - 3x - 10)(x2

sin2 0
20. Prove that (a) (l _ sin 0)(1 + sin 0)
(b) (sin x
21. Solve the equation

x
x+

cos X)2

3x - 4)

tan 2 0

(sin x

cos X)2

3 giving your solutions in simplest surd form.

7
22. Express (a) the fraction 30 as a recurring decimal
(b) the recurring decimal 042 as a fraction.

23. ABCD is a parallelogram. AE and CF


are drawn perpendicular to the diagonal
DB. Prove that
(a) l:,AEB and l:,DFC are congruent
(b) AE = FC.

24. The edges of three cubes have lengths acm, bcm and ccm respectively. Give
expressions for:
(a) the average volume of the three cubes,
(b) the edge of a cube equal in volume to the sum of the three volumes,
(c) the volume of a cube whose edge is the average of the three edges.

25. The combination lock on a safe has three concentric circular discs, each with the
digits 0 to 9 on its circumference. Only one combination of digits will open the safe.
What is the probability of opening the safe with one single trial?
26. One student has a pencil whose cross-section is square and whose faces are coloured
black, white, green and red. Another student has a pencil whose cross-section is
hexagonal (six sided) and whose faces are coloured black, white, green, red, yellow
and orange. Both pencils are rolled on the floor and the colours appearing uppermost
are noted.

REVISION EXERCISES A 177

Find the probability of the events:


(i) both colours uppermost are the same,
(ii) both colours uppermost are different,
(iii) black is uppermost on at least one of the pencils,

(iv) neither black nor white appears uppermost.


(b) sin (270 0 - 0)
(d) tan (0 - 180 0 )
(f) sin (0 + 180 0 )

27. Simplify: (a) cos (90 0 - 0)


(c) cos (90 0 + 0)
(e) tan (180 0 - 0)

28. Write down the exact value of:


(b) sin 225 0
(a) tan 315 0
0
(d) tan 360
(e) sec 60 0
29. (a) Express

21- \

(c) cos 180 0


(f) cos 210 0

with a rational denominator.

5/4
3/4
(b) Simplify 3 x 15
x

-=--c-,.----=__

30. Factorize: (a) 8 - (x


31. Expand: (a) (a l13
32. Evaluate (a) (l!)3

(b) 3x

h)3

b 1I3)(a 2/3

7x 2

6x 3

+ a ll3 b ll3 + b2/3) (b) (a 1l2

(b) (0'2)2

(c)

-J7f

b- I12 )2

(d) "",0'09

33. Find the exact value of:


(a) a:~ where

a W', W (j)',

(b) X4

2X2

b =

1 where x

3.J2.

34. Factorize:
(a) 16x2 - 1
(c) a 2 - b 2 + 2a - 2b

(b) 3x 2
(d) 8a 3

4x - 7

27

35. Find the value of x in each of the following diagrams:


12

10

36. Calculate the length of each side of a rhombus whose diagonals are 48 cm and 64 cm
long.
37. (i) 4 boys' names and 5 girls' names are in a hat. Find the probability that if two
names are drawn out without replacement:
(a) two girls' names are drawn out,
(b) at least one boy's name is drawn out.
(ii) A jar contains red buttons and white buttons in the ratio of 3 to 2. Find the

probability that if 3 buttons are chosen at random from the jar:

(a) exactly two are red,


(b) not more than one is white.

:I

'

178 NEW SENIOR MATHS: TWO UNIT COURSE

38. State the largest possible domain for functions with the following rules:

rx=t

(a) f(x)
(c) f(x)

(d) f(x)

39. (i) Simplify:


(a) cos 0 sin (90 0 - 8) + sin 0 cos (90 0
(b) cos 3 0 + cos 0 sin2 0
(c) 1
cos2 (180 0 - 0)

~ and

(ij) If tan 0
(a) sin 8

Ix -

11

0)

< 8 < 270 write down the exact value of

180 0

(b) cos O.

40. If sin ex = 0-6 and 0 0


(a) sin (180 0
ex)
(d) cosec ex
41. If tan 8

(b) f(x)

<

ex

<

90 0 , write down the exact values of:


(b) cos (90 0 - ex)
(c) cos (180 0
0
(e) tan (180
ex)
(f) sin (360 0

= t, express in terms of t

(a) tan (90 0

8)

(b) tan (180 0

8) (c) cot (180 0

42. Find all values of 8 between 0 0 and 360 for which:


1

0) (d) tan (360 0

+
-

ex)
ex)

8)

(a) sin 8

(d) sec 8

(b) cos 8 = 0

(c) tan 8

(e) cot 8 = ..J3

(f) cosec 8

43. Express as a single fraction


(a)

4
5
+ -1 x 4
595
9

(b) - x -

~"---;---"--;-

44. (i) Write down the expansion of (2x - Y)(X2 - xy + 'y 2)


(ii) Factorize: (a) 8y2
6y - 9
(b) X4 - 8x
45. Simplify: (a)

(b) a

(c)
I E xpress
46 ( .)

_ 4x - 6y
-;- 2X2 - 2xy

2X2 xy

m2

.J5
+

-J2.m simp
. Iest surd f orm Wit. h ratIona
.
1 d enommator_
' .

(ii) Show by substitution that

x3

+ 3x2 + x

3m + 2

O.

x = .J5 - 1 is a root of the equation

-1

= -J2

CHAPTER 8

Applied
Trigonometry
8.1 Definitions _ _ _ _ _ _ _ _ _ _ _ _ _ __
From your study of the trigonometry of a right-angled triangle in previous years, you have
seen how a consideration of the unit circle and similar triangles enables us to express sine,
cosine and tangent of an angle in terms of a ratio of the opposite side, the adjacent side
and the hypotenuse of a right-angled triangle.

Ba>
"0

"w

a>

+-'

"w0

0.
0.

Adjacent side

Fig. 8-1

sine A
cosine A
tan A

opposite side
hypotenuse
adjacent side
hypotenuse
opposite side
adjacent side

a
c
b

c
a
b

These trigonometric ratios provide the relationship between the angles and sides of a
right-angled triangle, while the Theorem of Pythagoras,

I c2

= a

+ b2 , I

provides the relationship between the lengths of the sides without reference to the size of
the angles.

180 NEW SENIOR MATHS: TWO UNIT COURSE

8.2 Some exact values _ _ _ _ _ _ _ _ _ _ __


The exact values of sin, cos and tan of 30,60 and 45 can best be remembered if we get
a mental picture of the following two diagrams, as seen in Chapter 7.7.
A

D..------..

.......... B

A -~---

sin

1
2

.J3

cos

.J3
-

1
2

.J2

tan

.J3
-

.J3

.J2

-2

-2

EXAMPLE 1
BC
= 12
Fic = 12 sin

. 30 0
(a) SIll

12

=
=

AB
12

12 cos 30

12 x

cos 30
AB

c
30

A ""--~~----.... 8

.J3
2

=6.J3

QR
8

QR = 8 tan 60

(b) tan 60 0

= 8.J3 .

PR2 = QR2 + PQ2

= (8.J3)2 + 82

= 192 + 64

= 256

PR = -J256

= 16

EXERCISES 8(a)
Give exact answers to each of the following, expressing lengths in simplest surd form
where necessary. (Do not use tables or a calculator).

= 90, A =
C = 90, A =

1. In DABC, B

30, AC

2. In D ABC,

45, BC

=
=

20 cm. Calculate the lengths of (i) BC, (ii) AB.


10 cm. Calculate the lengths of (i) AC, (ii) AB.

3. A vertical pole of height 15 m stands on level ground and a straight wire 30 m long
joins the top of the pole to a point on the ground. Find (i) the distance of this point
from the foot of the pole, (ii) the angle the wire makes with the ground.

'----- ---- ..

8/APPLIED TRIGONOMETRY 181

4. A ladder 10m long standing on level ground leans against a vertical wall and makes
an angle of 60 with the ground. Calculate (i) how high up the wall the ladder reaches,
(ii) the distance of the foot of the ladder from the wall.
12cm, AC 8cm, A = 60. CFis drawn perpendicular to AB to
5. In 6ABC, AB
meet AB at F. Calculate the length of (i) AF, (ii) FC, (iii) BC.
6. In the diagram, calculate the lengths of (i) AC, (ii) DC, (iii) AB, (iv) BC.

7. In the diagram, A C
(iv) AD.

20cm

12 cm. Calculate the lengths of (i) AB, (ii) BC, (iii) DC,

'I

8. Calculate the perimeter of the trapezium ABCD given that AD


A = 30, B = 45.
D Scm

16 cm, DC

8 cm,

~:~

9. A stepladder stands on horizontal ground with its feet 2 m apart. If the angle formed
by the legs is 60, how high above the ground is the top of the ladder?
10. The magnitude of the angle formed by the diagonal of a rectangle and one of its
longer sides is 30 . Find the dimensions of the rectangle if the length of the diagonal is
60cm.

8.3 Direction and bearing _ _ _ _ _ _ _ _ __


Navigators and surveyors measure direction by reference to the points of the compass:
north, south, east, west. Directions are indicated in terms of the number of degrees east or
west of north or south, or are measured clockwise from north and given in standard three
figure notation.

-------------~.~~---

182 NEW SENIOR MATHS: TWO UNIT COURSE

W~---------""E

s
N 30 E or 030 T means that the direction is 30 east of north. It could also be
expressed as 30 E of N or E 60 N or 60 N of E. However, the directions are usually
indicated in terms of the number of degrees east or west of north or south, as stated
above.
N

w-fw-kE~EW+E

or 250 0 T

NW
or315T

S
N300E
or 1300 T

or0300T

A bearing is a direction angle and indicates the direction of one point relative to another
point. In the diagram, the bearing of B from A is N 70 E or 070 T. The bearing of A
from B is S70 W or 250 0 T.
N

All bearings are in a horizontal plane.

EXAMPLE 2

Two yachts sail in a straight line from a buoy B. One sails 12 km in the direction 038 T
and the other sails 16 km in the direction 128 T.
(i) How far apart are they then?
(ii) What is the bearing of the first yacht as seen from the second yacht?
90. Why?
Applying the Theorem of Pythagoras to l::,ABC,
AC2 == 122 + 162
144 + 256
400

AC == 'J'400
20

The two yachts are 20km apart.

(I) L ABC

81 APPLIED TRIGONOMETRY 183

A
(ij) It is necessary to find the size of L ACN.

12
In 6ABC, tan LACB = 16

075

LACB = 3652'
LBCW = 38 Why?
LACN = 90 - (3652'

+ 38)
158'
The bearing of A from Cis N 158' W or 34452' T
What is the bearing of C from A?

EXERCISES 8(b)
1. The bearing of B from A is 120 T, the bearing of C from A is NE and the bearing of
C from B is N. What is the bearing of (i) A from C, (ii) A from B, (iii) B from C?

2. On level ground, A is 50 m due east of O. The bearing of B from 0 is 030 T and the
distance of B from 0 is also 50 m. Find the distance and bearing of B from A.
3. A is 5 km N 37 Wand B is 12 km S 53 W from a lighthouse. Find the distance and
bearing of B from A.

4. Karen and David set out from home at the same time. Karen cycles due north at
15 kmlh and David due east at 20 km/h. Find
(i) how far apart they are after I hour,

(ij) after how many minutes they are 10 km apart,

(iii) the bearing of Karen from David at any time.


S. Two towns A and Bare 15 km apart, with B due west of A. Town C is due south of B
and 12 km away. Calculate the distance and bearing of A from C.
6. A and B are two lighthouses, A being 20 km due north of B. The bearing of a ship is
145 T from A and 055 T from B. Calculate the distance of each lighthouse from the

ship.
7. Nick cycles 15 km due north, then 12 km due east and finally 20 km due south. What
are his distance and bearing from his original position?

8. Two ports A and B are such that B is due west of A. A is due north of a ship S. The
ship is on a course N 32 Wand reaches B after travelling for 3 hours at 25 km/h.
Calculate the distance between the two ports and the time it would have taken the ship
to reach A from S.
9. A hiker walks 15 km from camp in the direction S 3652' Wand then walks 7 km due
west. What are the distance and bearing of his position then from the camp?
10. A ship sails for 20 km on a cours~i S20 Wand then 25 km on a course S25 W.
Calculate (i) how far south the ship now is from its original position, (ij) how far west
the ship now is from its original position, (iii) the bearing of the ship now from its
original position.

184 NEW SENIOR MATHS: TWO UNIT COURSE

8.4 Angles of elevation and depression _ _ _ __

(a)

(b)

Wall

Fig. 8-2
If, in (a), we look up at A from the point P, the angle ojelevation of A from P is the angle
between the line of vision PA and the horizontal line P X.

L APX is an angle oj elevation.

The point P is the eye of the observer and A could be, say, a point on top of a wall.
If, in (b), we look down at A from the point P, the angle ojdepression of A from P is the
angle between the line of vision PA and the horizontal line PX.

: LAPX is an angle oj depression.

The point P is the eye of the observer, say, on top of a cliff and the observer is looking
down on a boat A in the water below.
If we look from A up to P, then L PAB is an angle of elevation.
LAPX = LBAP
i.e. angle of depression from P = angle of elevation from A.

Angles oj elevation and depression are in a vertical plane.

EXAMPLE 3
From a point A, level with the foot of a vertical pole and 25 m from it, the angle of
elevation of the top of the pole is 40. Calculate (i) the height of the pole, (ii) the angle of
elevation from A of a point halfway up the pole.
p

A.-::...-l......:..=...---.....

In the diagram, PQ is the vertical pole and L PA Q is the angle of elevation of P from A.

8/APPLIED TRIGONOMETRY 185


PQ

(i) In DPAQ, tan 40

25

PQ = 25 x tan 40

= 25 x 08391

= 2098

The height of the pole is 2098 m.


(ii) RQ

! of 2098

1049
1049 = 04196

In DRAQ, tan L RAQ

L RAQ
2246' (using tables or a calculator)

The angle of elevation of R from A is 2246' .

EXAMPLE 4
An observer in a lighthouse 100 m above sea-level is watching a ship sailing towards the
lighthouse. The angle of depression of the ship from the observer is 15. (i) How far is the
ship from the lighthouse? (ii) Some time later, the angle of depression is measured as 25 .
How far has the ship travelled in this time?

x---

A~-----~=------------------------------------------------~

In the diagram, PQ is the lighthouse, A is the ship and L APX is the angle of depression.
However, in DPAQ, L PAQ = 15 and L APQ = 75.
(i) In DPAQ, tan 15

= ~O~

or

JOO

AQ

10~

tan 75

AQ = 100 x tan 75

= tan 150

100

02679

= 3732
The ship is 3732m from the lighthouse.

100 x 3'732
3732

150

x~p

100

15

t------

25

---------1

3732
(ii) In the diagram, the ship has moved from A to Band L BPX is the angle of depression

when the ship is at B.


In DBPQ, tan 25 =

~~

BQ _
100
- tan 25

100

04663
214'5

or

BQ
100

tan 65
BQ

100 x tan 65
100 x 2145

214'5

186 NEW SENIOR MATHS: TWO UNIT COURSE

AB = AQ - BQ

= 3732 - 2145

= 1587

The ship has travelled 1587 m.

EXERCISES 8(c)
1. A person 2 m tall is standing on the ground and looking up at the top of a building. If

the person is 18 m from the building and the angle of elevation of the top of the
building is 30, calculate the height of the building.
2. An aircraft flying in a horizontal straight line at an altitude of 2000 m passes directly
over an observer on the ground. One minute later, the observer finds that the angle of
elevation of the plane is 13 24' . Calculate (i) the distance flown by the aircraft in that
time, (ii) the speed of the aircraft in km/h.

3. The diagram represents a vertical


flagpole AB on top of a building. From
pL-____________ __
a point P on the ground, the angle of
o
elevation of A is 3652' . PD = 55 m,
CB = 5m, AB = 12m.
Calculate (i) the height of A above the ground, (ii) the distance from A to P, (iii) the
angle of elevation of A from C.
~

4. From an aircraft 1000 m above the ground, the angles of depression of the tops of two
houses (the same height) in line with the aeroplane are 40 and 60 respectively. How
far apart are the two houses? (Ignore the height of the houses.)
5. AB and CD are two vertical buildings with their bases A and C on level ground. The
height of AB is 50 -m. The angle of elevation of B as seen from Cis 20 and that of D
as seen from A is 35.
Calculate (i) the horizontal distance between the buildings, (ii) the height of CD,
(iii) the angle of elevation of D as seen from B.

6. From the top, T, of a cliff, an observer sees two ships P and Q in line with him and at
horizontal distances of 50 m and 70 m. The angle of depression of P from Tis 25.
Calculate (i) the vertical height of the cliff, (ii) the angle of elevation of T from Q.
7. From a point 5 m above the ground, the angle of depression of the bottom of a wall is
21 and the angle of elevation of the top of the wall is 32. Find (i) the horizontal
distance from the point of observation to the wall, (ii) the height of the wall.
8. From a point A on the ground, the angle of elevation of the top of a tower is 38 and
the angle of elevation of the top of a vertical flagpole on top of the tower is 41 0. A is
80 m from the foot of the tower. The ground between A and the tower is horizontal.
Calculate the length of the flagpole.

'--- ---.- ------

--

- - - -- - -

190 NEW SENIOR MATHS: TWO UNIT COURSE

Note: Since sin B is positive, B could be in first or second quadrant or perhaps both.
You will observe that the calculator provides only the first quadrant angle 31 41' . The
second quadrant angle 148 19' is obtained by subtracting 31 41' from 180.
Can B have only one of these values, or can it have both?
Since a > b, it follows that A > B.
Thus B < 52 and cannot assume the value 14819'. Also, if B
A + B > 180, which is impossible in a triangle.
Hence

B=3141'

C = 180
(A + B)

= 180 - (52 + 31 41')

= 9619'

sin C
C

Therefore, B

sin A

12

sin 52

12 sin 96 19 '

=
sin 52

15'14

3141', C

9619',

= 15'14

EXAMPLE 7
Solve the triangle ABC, given A

= 40, BC = 20 cm and AC

30.cm.

By the sine rule,


b

sin A
20
sin 40
sin B

sin B

30

sin B

30 sin 40

B = 7437' or 10523'

(using calculator as in Example 6)

Now test to see whether both these values of B will satisfy the specifications given.
180 - (40 + 7437')
When B = 7437', C
= 6523'
10523' , C = 180 - (40 + 10523')
When B
= 3437'
Thus the ambiguous case applies in this example.

8/APPLIED TRIGONOMETRY 187

9. A city building is 45 m high. From the top of this building, the angle of depression of
an object 0 on the wall of a building opposite is 50. The width of the street is 20 m.
Find (i) the height of 0 above street level, (ii) the angle of elevation of 0 from the foot
of the first building.

10. Two buildings of unequal height stand at a distance apart on horizontal ground. The
taller building is 60 m high and from its top an observer finds that lines of sight to the
bottom and the top of the shorter building are at angles of depression of 25 and 10
respectively.
Calculate (i) the horizontal distance apart of the buildings, (ii) the height of the
shorter building.
11. From the top of a lighthouse 75 m above sea level, the angles of depression of two
buoys due north of the lighthouse are 60 and 30 respectively. Find, in simplest surd
form, (i) the distance of each buoy from the lighthouse, (ii) the distance between the
two buoys.
12. From a point P on the horizontal ground, the angle of elevation of the top of a
building 40 m high is 30 and from a point Q on the same horizontal level as P and in
line with the foot of the building, the angle of elevation is 60 . Calculate, in simplest
surd form, the distance PQ.

8.5 The sine rule _ _ _ _ _ _ _ _ _ _ _ _ __


In previous years, your study of trigonometry has been confined to the solution of right
angled scalene triangles. Now we consider the solution of scalene triangles that have either
three acute angles or one obtuse and two acute angles. For this purpose, we establish the
sine rule and the cosine rule.

[~=~=sfnc

where a, band c are the length measures of the sides of a triangle ABC opposite the angles
whose magnitudes are A, Band C respectively.

c
I
I
I

P:

b____"-_ _---"'"

-4-------c----......
Fig. 8-3

DAB

...-..---c--....

Fig. 8-4

Let p be the measure of the length of the perpendicular from C to AB. The foot of this
perpendicular lies on AB if triangle ABC is acute angled (fig. 8-3) and on BA produced if
triangle ABC is obtuse angled (fig. 8-4).
In the right-angled triangles A CD and BCD,
p
b sin A = a sin B
and
p = b sin (180 - A) = a sin B
But sin (180 - A) = sin A.

188 NEW SENIOR MATHS: TWO UNIT COURSE

Hence, in both cases,

b sin A

i.e.,

sin A

a sin B;

sin B

Similarly, by drawing a perpendicular from A to BC it can be seen that . b


sm B
abc
Hence, sin A = sin B = sin C'

The sine rule can be used to solve a triangle when given:

.c '
sm C

(a) two angles and the length oj one side,


(b) two sides and the magnitude oj an angle that is not included by these
two sides, i.e., an angle opposite one oj the two sides.

EXAMPLE 5
(This example illustrates the use of the sine rule in case (a).)
In DABC, given A
45, B = 30 and BC
5 cm, calculate C, band c.

c
::,..

"<:$

A~8
Since A

C = 180, it follows that


C = 180 - (45 + 30)

105
a
b

sin A
sin B

5
b

sin 45 = sin 30

b = 5 sin 30

sin 45

5 x 0-5 x ..J2

= 3536 (using a calculator)

To evaluate

5 sin 30
.
. 450 , usmg a calculator, proceed as follows:
sm

Operation sequence

[9

Display

5 ~ 30 Isinl

a
sin A

35355 ..
c

sin C

5
sin 45
sin 105

5 sin 105

c =
sin 45

6-830 (using a calculator)

8/APPLIED TRIGONOMETRY 189

The ambiguous case


Sometimes, when two sides of a triangle and an angle opposite one of these sides are
given, it is possible to construct two triangles.
Given: 6ABC in which A
30 0, BC
4 cm and AC = 6 cm. Construct the triangle
ABC.
The diagram (fig. 8-5) shows that there are two possible triangles ABC with the
specifications given. Note the following facts concerning this diagram:
(i) The given angle, A, is acute.

Fig. 8-5
(ii) a is less than b, but greater than CD, i.e. greater than bsin A, which, in this case, has

magnitude 3 cm.
Only when conditions (i) and (ii) are present is it possible to draw two triangles. This is
called the ambiguous case. Examples 6 and 7 will illustrate the use of the sine rule when
two side lengths and an angle opposite one of these sides are given.

EXAMPLE 6
12cm and AC = 8cm.

Solve the triangle ABC, given A

By the sine rule,

sin A
12
sin 52 0
sinB

8
8 sin 52
=--

= 0'5253

31 041' or 148 019 '

To evaluate B, using a calculator, proceed as follows:

Operation sequence

521sinl 8El12

B = 31.6912 0

31 041'

(multiply .6912 0 by 60 to convert to minutes)

Display
3169116 ...

190 NEW SENIOR ,MATHS: TWO UNIT COURSE

Note: Since sin B is positive, B could be in first or second quadrant or perhaps both.
You will observe that the calculator provides only the first quadrant angle 31 41' . The
second quadrant angle 148 19' is obtained by subtracting 31 41' from 180.
I'

Can B have only one of these values, or can it have both?


Since a > b, it follows that A > B.
Thus B < 52 and cannot assume the value 14819'. Also, if B
A + B > 180, which is impossible in a triangle.
Hence

B
31 41'
C = 180 - (A + B)

= 180 - (52 + 31 41')

= 9619'

c
a

sin C
sin A

c
12

sin 96 19' - sin 52

12 sin 96 19 /

c
sin 52

1514

Therefore, B = 3141', C

9619', c

= 1514

EXAMPLE 7
Solve the triangle ABC, given A

40, BC = 20 cm and A C

30,cm.

By the sine rule,


a

sin A
20
sin 40

sin B

30

sin B

30 sin 40

sin B = ------

B = 7437' or 10523'

(using calculator as in Example 6)

Now test to see whether both these values of B will satisfy the specifications given.
When B = 7437', C = 180 - (40 + 7437')
= 6523'
When B = 10523', C = 180 - (40 + 10523')
= 3437'
Thus the ambiguous case applies in this example.

8/APPLIED TRIGONOMETRY 191

Case 1: When B = 7437', C = 6523'


c _
a
sin C - sin A
20 sin 65 23 '
c=
28'29
Case 2: When B = 10523', C = 3437'
c

c=

a
sinA
20 sin 3437'
1768

The two possible solutions are shown in figure 8-6.

= 2829
Fig. 8-6

EXERCISES 8(d)
1. In the triangle ABC, a

8, A

30 and B

75. Calculate C, band c.

2. In the triangle ABC, b = 10, B = 45 and C = 120. Calculate A, a and c.


3. InthetrianglePQR,P = 65, Q = 70 andPR = 25cm. Calculate the length ofPQ.
4. In the triangle ABC, a
calculator.

5, A = 60 and B = 45. Find b without using tables or a

5. In the triangle ABC, sin B

~,

6. In a triangle ABC, B = 2A, b

6 and b

5 and a

9. Find sin A.

3. Show that 5 sin A

7. Find the length of the longest side of a triangle ABC in which A


and AC = 8cm.
8. In the triangle ABC, angle A
length of AB.

42, angle B

28 and BC

3 sin 2A.

36 52' , B

30

6 cm. Calculate the

9. In the triangle ABC, a = 16, b = 12 and sin A = 0'4. Calculate


(a) sin B
(b) the length measure of the altitude from C to AB.
10. A wooden stake, S, is 13 m from a point, A, on a straight fence. SA makes an angle of
20 with the fence. If a goat is tethered to S by a 10m rope, where, on the fence, is the
nearest point to A at which it can graze?

192 NEW SENIOR MATHS: TWO UNIT COURSE

11. In a triangle ABC, AC


of angles A and C.

30 cm, AB = 44 cm and B

37. Find two possible values

12. 0, A and B (in that order) are three points in a straight line. The bearings of A and B

from 0 are 020 T. From a point P, 4 km from 0 in a direction NW, the bearings of A
and Bare 112 T and 64 T respectively. Calculate the distance from A to B.
13. In the triangle PQR, PQ
20cm, QR = 22cm and R = 15. Find two possible
values for angle RPQ and PR.
14. In the triangle ABC, A
3652' , B = 30 and the perpendicular distance from C to
AB is 3 units. Calculate lhe perimeter of the triangle ABC.
15. In DABC, the lengths of BC and AC are in the ratio 2: 1 and sin L ABC
Calculate (a) sin L BA C (b) two possible sizes for L BA C.
16. In DABC, angle B
25, angle C = 55, BC
5 m. Calculate
(a) the length 'of AC,
(b) the length of AX, where X is the foot of the perpendicular line from A to BC.
17. In DPQR, PR = 3 cm, L RPQ
40, L PQR = 60. Calculate
(a) the perimeter of DPQR,
(b) the length of the perpendicular line from R onto PQ.
18. In DXYZ, XY

5 m, L ZXY

68, L XYZ

82, calculate the length of XZ.

19. An aircraft flies from point A to point B 400 km on a cour.se 040 T. It then flies on a
course 160 T from B to a point C 500 km from A. Calculate
(a) the distance BC,
(b) the bearing of C from A.
20. The bearing of a ship from a lighthouse A is N75 E and its bearing from a second
lighthouse B 44 km south of A is N 40 E. Find the distance of the ship from B.
21. Three points A, Band C lie on a horizontal plane. B is 2000 m due south of A. Cis
145 T from A and 052 T from B. Calculate the distance of C from both A and B.
22. Two points A and B on the same bank of a river are 50 m apart. C is a point on the
70. Calculate the width of the river.
other bank. Angle BA C = 80, angle ABC
A

23. In DABC, AD bisects L BAC. Use the


sine rule to prove that
AB
AC

BD

D _ _ _ _C
8.6 The cosine rule _ _ _ _ _8_ _ _
_

a2 = b 2

+ c2

2bccos A

::}

cos A

c2

+ a2

.
2ca cos B

::}

cos B =

+b

2 -

2abcos C

::}

cos C

b2

c2

a2
2

ca

b2

8/APPLIED TRIGONOMETRY 193

In words, the cosine rule may be expressed as follows:


The square on one side ofa triangle is equal to the sum of the squares on
the other two sides, minus twice the product of these two sides and the
cosine of the included angle.

c
I
I

pi
I
I
AL-~------:=,-----IB
1
. .....
11---- C -~-t
....._tl

Fig. 8-7

__ ,---:L.----::o.

b_~

Fig. 8-8

To prove: a 2 = b 2 + c 2 2bccos A.
From C draw a perpendicular, p, to meet AB at D (fig. 8-7) and BA produced at D
(fig. 8-8). Let AD
x units.
.
Apply the Theorem of Pythagoras to DBCD.
a 2 = p2 + (c - X)2
a 2 = p2 + (c + X)2
2
p2 + c
2cx + X2
= p2 + c 2 + 2cx + X2
2
But p2 + X2
b
But p2 + X2
b2
and
x
bcos A
and
x = bcos(180 - A)
a 2 = b 2 + c 2 2bc cos A
= -bcos A

a2
b 2 + c 2
2bccos A.
The cosine rule is really an extension of the Theorem of Pythagoras to apply to acute

angled or obtuse-angled triangles.

If A = 90,
cos A
cos 90 = 0

a 2 b 2 + c2
If A < 90,
a2 < b 2 + c2
and if A > 90,
a 2 > b 2 + c2
Use the cosine rule to assist in solving a triangle when you are given

I
I

(a) three side lengths,


(b) two side lengths and the included angle.

EXAMPLE 8
Find the magnitude of the largest angle of a triangle, the lengths of whose sides are 3 em,
5cm and 7cm.
The largest angle is the angle opposite the longest side, i.e. angle A in fig. 8-9.
Applying the cosine rule, we get
b 2 + c 2 - a2
cos A
2bc
32 + 52
72

194 NEW SENIOR MATHS: TWO UNIT COURSE

+ 25 - 49

Hence

30
15
-30
-0'5
180 - 60
120

Note: Since cos A is negative, A is an obtuse angle.

Fig. 8-9

EXAMPLE 9
In triangle ABC, a

10, c

5 and B

3652 ' . Calculate

(a) b,
(b) magnitude of C.

_9~

AN

8=10

Since two sides and the included angle are given, the cosine rule is applicable.
b 2 = c 2 + a2 - 2 cacos B
2 x 5 x 10 cos 3652'
= 52 + 102
= 25 + 100 - 100 x 08
45
b
v'45 since b > 0
= 6'708

(a)

(b) To find the magnitude of C, we can either use the cosine rule again, or use the sine
rule. It is probably quicker to use the sine rule. Using the sine rule, we get

c
b
sin C = sin B
5
6'708
sin 3652'
sin C
5 sin 3652'
sin C
6'708
C = 2634' (using a calculator)

8/APPLIED TRIGONOMETRY 195

EXAMPLE 10
Two men set out from point P at the same
time. One travels at 20 km h- 1 along a
straight road in the direction 032 0 T. The
other travels at 25 km h-1 along another
straight road in the direction 132 0 T. Find
their distance apart after 3 h.
After 3 h, one man is at A, 60 km from P,
and the other is at B, 75 km from P, as
shown in fig. 8-9.
132 0 32 0
L APB
100 0
Hence, in the triangle APB, two sides and
the included angle are known.

Fig. 8-10

Applying the cosine rule, we get


AB2 = 6Q2 + 75 2 - 2 x 60 x 75 cos 1000
3600 + 5625 + 120 x 75 cos 80 0
3600 + 5625 + 1563
10788
AB = '-"10788
103'9
Distance apart is 104 km (approx).

8.7 Area of a triangle _ _ _ _ _ _ _ _ _ _ __


base x altitude
Area of h,ABC = - - - - -

~cp.
In fig. 8-7
and, in fig. 8-8

p
p

=
=

=
area of h,ABC =
=

(Fig. 8-7 and fig. 8-8)

b sin A,
b sin (180 0 - A)
b sin A, and so

~bC sin A

.21 product of two sides and the sin of the included angle.

EXERCISES 8(e)
1. Calculate the cosine of the smallest angle of the triangle whose sides measure 9, 11
and 13cm.

3652' , b = 7 and c = 10. Calculate a, B and the area of


2. In the triangle ABC, A
the triangle, given that cos 3652' = 08.
3. Two adjacent sides of a parallelogram have lengths of 12 cm and 15 cm and the
included angle is 50. Calculate the length of the diagonals.
4. In triangle ABC, b = 4, c
5 and the magnitude of angle BAC is 538' . Calculate
the perimeter of the triangle and its area measure, given that cos 53 8 I ~ 06.

196 NEW SENIOR MATHS: TWO UNIT COURSE

5. In triangle ABC, B = 12652', a = 12 and c = 15. Find band A.


6. The lengths of the sides of a triangle are in the ratio 5: 6 : 9. Find the magnitude of the
largest angle.
7. In triangle ABC, BC 8 cm, A C
calculate
(a) the magnitude of angle ABC,
(b) the length of AD,
(c) the magnitude of angle DAC.

7 cm. If D is the midpoint of BC,

9 cm and AB

8. In triangle ABC, BC
11 cm, AC
5 cm and AB = 8cm. Calculate
(a) the magnitude of angle ABC,
(b) the length of the perpendicular from A to BC,
(c) the area of the triangle ABC.
9. Two adjacent sides of a parallelogram have lengths of 8 cm and 10 cm. If the length of
the longer diagonal is 14 cm, calculate
(a) the magnitude of the angles of the parallelogram,
(b) the length of the other diagonal.
10. Find the area measure of the triangle ABC given that a
11. In triangle PQR, q

12, r

5 and angle QPR

= 6,

= 7 and c = 11.

= 108. Calculate

(a) p,

(b) the magnitude of angle PQR.


12. In triangle ABC, BC = 11 cm, AC = 5 cm and AB
that BP = 4 cm. Calculate the length of AP.

= 8 cm. P is a point on BC such

13. Two cars, A and B, depart from the same position. A travels along a straight road due
east at 30kmh- l B departs 15min after A, and travels along another straight road in
a north-easterly direction at 40 km h -I. How far apart are they 15 min after B departs?
14. A, Band C are three towns such that B is 20 km from A in a direction 330 T and C is
30 km from A in a direction 2038' T. Find the distance from B to C.

15. P and Q are two towns 50 km apart with Q due east of P. A third town, R, to the
north of the line joining P and Q, is 70 km from P and 30 km from Q. Find the
bearing of R from (a) Q, (b) P.
16. A lighthouse is 10 km north-west of a ship travelling due west at 16 km h- l . How far is
the ship from the lighthouse 45 min later? What is the bearing of the lighthouse from
the ship then?
17. ABC is a triangle in which AC = 7 cm. A circle, centre B and radius BC, cuts AB
internally at D. AD
5 cm, DC = 4cm. Calculate the length of BC and the area of
6ABC.
18. P, A, Band C are four points in a plane such that the angles BPA and CPA are
obtuse and on opposite sides of PA. PA = 8cm, BP = 10cm, PC = 12cm,
AB = 14cm andAC = 18 cm. Calculate the length of BC and the area of the triangle
. ABC.
19. P, A, B, C are four points, in order, on a straight road that runs up a hillside and
makes a constant angle of 10 with the horizontal. A flagpole whose height is h m
stands at P. From A and B, the top of the flagpole has elevations of 30 and 5

8/APPLIED TRIGONOMETRY 197

respectively above the horiiontal. If AB is 100 m long, what is the height of the
flagpole?
If BC is also 100 m long, what is the elevation of the top of the flagpole from C?
20. From a point P, a man observes that the angle of elevation of the top of a cliff A is
40. After walking 100m towards A along a straight road inclined upwards at an
angle of 15 to the horizontal, the angle of elevation of A is observed to be 50. Find
the vertical height of A above P.
21. A ship sailing in a direction 065 T observes two lighthouses in a line due north. After
travelling 4 km one of the lighthouses bears 285 T and the other 315 T. Calculate the
distance between the lighthouses.

22. The equal sides AB and A C of an isosceles triangle ABC are each 5 cm and
BC = 4cm. D is a point on AC such that DC = 1 cm. Calculate
(a) the size of angle A,
(b) the length of BD,
(c) the area of 6ABC.

= 8 cm, BC = 5 cm, L DAB = 60. Calculate


(a) the length of the diagonals A C and BD
(b) the area of the parallelogram.

23. ABCD is a parallelogram AB

24. Two sides of a triangle have lengths 32 cm and 4'8 cm and the included angle is 65.
Calculate the perimeter of the triangle to 1 decimal place.
25. Two sides of a triangular field are 60 metres and 50 metres and the included angle is
140. Calculate
(a) the length of the third side,
(b) the area of the field.
26. The sides of a triangular field have lengths 80 m, 90 m, 100 m. Calculate the area of .,
the field.
27. Two cars leave a point A at the same time. One car averages 80 kmlh along a straight
road in direction 025 T. The other car averages 90 km/h along a straight road in
direction 135 T. How far apart are they after 3 hours?

CHAPTER 9

Coordinate
Geometry
Straight Line
Mathematics leapt ahead in the seventeenth century with two great advances-the
discovery of analytical geometry and the discovery of the calculus. Rene Descartes
(1596-1650) was one of the mathematicians responsible for the creation of analytical
geometry. He did this by setting up a coordinate system and applying algebra to geometry.
The name of Descartes is now commonly used as an adjective in such phrases as Cartesian
coordinates, Cartesian plane, and Cartesian axes.

9.1 Gradient of a straight line _ _ _ _ _ _ _ __

----,
X2

X1

:Yi

Y1

18(X21 Y2)

Fig. 9-1

Fig. 9-2

Consider two fixed points A(Xl' Yl) and B (x2, Y2) with B to the right of A.
The gradient (or slope) m of AB is defined by

9/COORDINATE GEOMETRY- STRAIGHT LINE 199

If x 2 = Xl' the lineAB is parallel to the Y-axis. The gradient is not defined in this case.

If Y2 = YI' the line AB is parallel to the X-axis. The gradient in this case is zero.

In fig. 9-1, the line rises from left to right and makes an acute angle f) with the X-axis. If

X and yare measured in the same units, (x 2 - Xl) and (Y2 - Y 1) are both positive numbers,
and so the gradient is positive.
In fig. 9-2, the line falls from left to right and makes an obtuse angle f) with the X-axis.
(X2 - Xl) is a positive number, but (Y2 - Yl) is negative, and so the gradient is negative.
Since in both cases tan f)
m, the gradient (or slope) may be defined by

Note: Since

'---=-----'--'

X2 -

Xl

'--"----"-=--

it makes no difference which point we label (Xl' YI)

EXAMPLE 1
Find the gradient of the line joining the given points and the angle the line makes with the
positive direction of the X-axis:
(b) (-3, 5) and (2, -1)
(a) (2, 3) and (4, 7)
(a)

m=

(b)

-1 - 5

i.e., tan

f)
f)

=2
= 6326'

i.e., tan

f)

f)

2 + 3

-1,2

12

180
50 12'

12948'

9.2 Parallel lines _ _ _ _ _ _ _ _ _ _ _ _ __


Parallel lines make angl~s of equal
magnitude with the positive direction of
the X-axis, and hence have the same gradient
(fig. 9-3). If a line undergoes a translation
parallel to the X-axis or Y-axis, its gradient
remains unaltered.

Fig. 9-3

9.3 Perpendicular lines _ _ _ _ _ _ _ _ _ __


Fig. 9-4 shows two lines OP and OQ, perpendicular to one another, with gradients m] and
m 2 respectively. Triangles ORP and OSQ are congruent. Why?

200 NEW SENIOR MATHS: TWO UNIT COURSE

By definition,
tan LPOR
tan LROQ

m lm2

b
a

ml

= m2
a

1 provided

i.e.,
that m l , m 2

'* O.

ml.

Fig. 9-4

Hence, if two straight lines are perpendicular, the product of their gradients is -1.
If the gradient of a line L is, say,

l,

then the gradient of a line perpendicular to L is -

~.

If a line L is rotated in the plane through 90, the gradient of the image line under this
transformation is the negative reciprocal of the gradient of L.

EXAMPLE 2
The coordinates of the vertices of a triangle ABC are (-2, 1), (3, 2) and (4, -3) respectively.
Show that AB is perpendicular to BC.
2 - 1
1
Gradient of AB = +
5
Gradient of BC

- 5

Since the product of the gradients is -1, AB is perpendicular to BC.

EXERCISES 9(a)
1. Find the gradient of the line containing these points:
(a) (2, 4), (0, 6)
(b) <-3, -1), (-5, 6)
(c) <-2, 2), (-6, 2)
(d) (2, -3), (-3, 2)
(e) (a, b), (b, a)
(g) (-1, -5), (-4, -3)

(f) (3b, 2c), (2b, -3c)

(h) (0, 5), (-8, -7)

2. Find the magnitude of the angle that the line joining the given points makes with the
X-axis:
(a) (-4, -2), (4, 6)
(b) (0, 5), (-2, 4)
(d) (4, 5), (-2, -4)
(c) (-5, 6), (3, 3)
(e) (2a, b), (2b, a)

(f) (b, c), (c, b)

3. Show that the line joining the points (-2, 3) and (4, -2) is parallel to the line joining the
points (-1, 7) and (-7, 12).
4. In each of the following, show that ABCD is a parallelogram:
(a) A(O, 0), B(3, 0), C(5, 5), D(2, 5)
(b) A(-3, -1), B(4, 1), C(8, 5), D(I, 3)
(c) A(-I, 4), B(4, 6), C(2, 7), D(-3, 5)
(d) A(-2, -3), B(6, 2), C(8, 7), D(O, 2)

9/COORDINATE GEOMETRY - STRAIGHT LINE 201

5. In each of the following, show that PQR8 is a trapezium:


(a) P(-I, 2), Q(3, 4), R(S, -1), 8(-4, -7)
(b) P(-2, 3), Q(3, 7), R(9, -5), 8(2, -5)
6. Show that the points (-2, 0), (2, 12) and (-5, -9) are collinear.
7. Show that the points (2a, b), (a, 2b) and (-a, 4b) are collinear.
8. In each of the following, show that ABC is a right-angled triangle:
(a) A(2, -3), B(5, 2), C(-3, 0)
(b) A(-I, 2), B(3, 4), C(7, -4)

9. In each of the following, show that PQR8 is a rectangle:


(a) P(1, 4), Q(5, 2), R(2, -4), 8(-2, -2)
(b) P(4, -7), Q(S, -4), R(2, 4), 8(-2, I)
(c) P(I, 3), Q(5, 2), R(6, 6), 8(2, 7)
10. A(-S, 6), B(2, 4), C(5, -7) and D(-5, -3) are the vertices of a quadrilateral. Prove that

the diagonals of the quadrilateral are perpendicular.

9.4 Equation of a straight line _ _ _ _ _ _ __


(a) Equation of a line passing through
a fixed point (x11 Y1) and having a gradient m
One, and only one, straight line can be drawn when we know the coordinates of one point
on the line and the gradient of the line.
y

Iy
I

y,

A(X1, Y1) ____ .J

x -

Xl

Fig. 9-5

Let A(x1, Yl) be a fixed point and P(x, y) be any other point on the line AP whose
gradient is m.
, Thus
That is,
This is the equation of the line AP.

EXAMPLE 3
Find the equation of the straight line that contains the point (2, 3) and has a gradient Of~.
Y - Yl

That is,

Y - 3 =

4y

m(x -

~(X

XI)

- 2)

4y - 12
3x - 6
3x - 6 = 0 is the equation of the line.

202 NEW SENIOR MATHS: TWO UNIT COURSE

(b) Equation of a line passing through


two fixed points (x11 Y1) and (x21 Y2)
One, and only one, straight line can be drawn when we know the coordinates of two fixed
points.

Fig. 9-6
Let A(x l , YI) and B(x2, Y2) be two fixed points and P(x, y) be any other point on the line
APB (fig. 9-6).

But
That is,

EXAMPLE 4
Find the equation of the straight line passing through the points C3, 4) and (2, -6).

m=

-6

-2

YI
m(x - XI)
Y - 4 = - 2(x + 3)
2x + 2
0 is the equation of the line.

That is,
Y

(c) Tangent form of straight line


Let the fixed pointA(O, c), where XI = 0 and
c, be the point where the line AP cuts
the Y-axis, and let P(x, y) be any other point
on the line (fig. 9-7).

YI

Thus

x-

y
<

m.

lye

_ _ _ _ ...J

That is,

x
!

y=mx+c!

is the equation of the line; c is the Y-intercept.

Fig. 9-7

9/COORDINATE GEOMETRY

STRAIGHT LINE 203

This is a convenient form of the equation and is useful for determining the gradient of
the line when its equation is known.

EXAMPLE 5
For the line whose equation is 2y + 3x - 5 = 0, find
(a) the gradient,
(b) the angle at which the line crosses the X-axis,
(c) the Y-intercept.
2y + 3x - 5 = 0
2y -3x + 5
3
5
y = - -x + - and so
2
2
3

(a)

m=

tan () = -1'5
() 180 - 5619'
12341'

(b)

(c) Smce
y

3 + 2
5.IS 0 f t h e f orm

= -2x
y = mx + c, t h e Y'
-mtercept

.
IS

5
2'

(d) Equation of a line parallel to the X-axis


If the straight line is parallel to the X-axis,
m = O.
We have seen that the equation of any line
cutting the Y-axis at (0, c) and having
gradient m is y = mx + c. Thus

y=c

(OtC)

is the equation if m is zero.

x
Fig. 9-8

(e) Equation of a line parallel to the Y-axis


When the straight line is parallel to the
Y-axis, the gradient is not defined. Hence we
cannot deduce its equation from any of the
preceding forms.
It is apparent from fig. 9-9 that, for all
points on the line, x = c irrespective of the
value of y.
Thus

(C,O)

x=c
is its equation.

Fig. 9-9

204 NEW SENIOR MATHS: TWO UNIT COURSE

(f) General equation of a straight line


Every line has a first degree equation in x and y of the form

ax

by

c = 0

where a, band c are constants. Either a or b, but not both, may be zero, and c may be
zero.
This equation may be written
-J- 0
Y = - ax
b _ ~
b if b ..,..,

mx

which is of the form y

-~ and V-intercept

c; thus it is the equation of a straight line of gradient

1. If a = 0, the line is parallel to the X-axis.


2. If b = 0, the line is parallel to the Y-axis.
3. If c = 0, the line passes through the origin.

Parallel and perpendicular lines


The lines with equations
ax + by + c
alx + b1y + C1

and

ax _ ~ and y
b
b

when expressed in the form y =

........................ (1)

0,

(2)

a IX

-Ii;"

~: respectively, can be

seen to have
(a) the same gradient, and hence are parallel when ~ = ~I,
I

(b) the product of their gradients equal to -1, and hence are perpendicular when
a . Ii:
a1 =
b

-1'
, I.e., when aa l

EXAMPLE 6
Find the equation of the line passing through the point (2, -3) that is (a) parallel to,
(b) perpendicular to, the line with equation 3x + 4y - 5
O.
(a)

3x

4y - 5 = 0
4y = -3x
3
y = --x

+5
+ 45
4

Hence the line passing through the point (2, -3) has gradient
y - Yl
y
4y

3 =

m(x

-~(x
4

12 = - 3x

Xl)

- 2)

Thus 3x + 4y + 6 = 0 is the equation of the line. Compare the equation of this line

5
O. What do you observe?

with the line 3x + 4y

Alternative method (a):

Since the line passing through the point (2, -3) is parallel to 3x
equation is of the form 3x + 4y + k = O.

4y - 5 = 0, its

91COORDINATE GEOMETRY - STRAIGHT LINE 205


Put x = 2 and Y = -3, and so k = 6. Hence 3x
line.
4
(b) The gradient of the perpendicular is 3.
Y - YI

+ 4y + 6 = 0 is the equation of the

m(x - Xl)

+ 3 =j(X - 2)
3y + 9 = 4x - 8
Y

Thus 4x - 3y - 17 = 0 is the equation of the perpendicular. Compare the equation


of this line with the equation 3x + 4y - 5 = O. What do you observe?

Alternative method (b):


Since the line passing through the point (2, -3) is perpendicular to the line with
equation 3x + 4y - 5 = 0, its equation is of the form 4x - 3y + k = O.
Putx = 2 andy
-3, and so k = -17. Thus 4x 3y - 17 = 0 is the equation of
the perpendicular.

9.5 Intersection of two lines _ _ _ _ _ _ _ __


The point of intersection of two (or more) lines may be found algebraically.by finding the
solution set of the system composed of the equations of the lines.

EXAMPLE 7
Find the equation of the line that passes through the point A(4, -2), and the point of
intersection, B, of the lines with equations 4x + 2y + 2
0 and 3x + 5y - 9
o.
The coordinates of Bare (-2, 3); these are found by solving the simultaneous equations.
We now know the coordinates of two points on AB, and so we can find its equation.
5
3 + 2
m = -2 - 4 = -()
y - YI = m(x - Xl)
5
Y + 2 = -()(x
4)
5x

6y + 12 =
5x + 20
6y - 8
0 is the equation of AB.

Alternative method:
Any line passing through the intersection of 4x + 2y + 2
the equation
4x
Put X

4, y

+ 2y + 2 + k(3x + 5y - 9)

= 0 and 3x +

= 0 Why?

-2
:. 16 - 4

+ 2 + k(12 - 10 - 9)

k = 2

The equation of the line AB is


4x + 2y + 2
i.e.
Two straight lines will
(i) intersect at a point
or Oi) be parallel
or (iii) coincide.

+ 2(3x + 5y - 9) 0
lOx + 12y - 16 = 0
5x

+ 6y

- 8

0 as before

5y - 9 = 0 has

206 NEW SENIOR MATHS: TWO UNIT COURSE

If we solve the simultaneous equations we will get


(i) a unique solution if the lines intersect
(ii) no solution if the lines are parallel
(iii) an infinite number of solutions if the lines coincide.

This is what we would expect. We can determine which of these three situations exists
without actually solving the equations. For the pair of equations

a,x + b1y + c, = 0
a2 x + b2 y + C2 = 0, we have
(i) intersecting lines if

a
2

(ii) parallel lines if

bb)

a2

(iii) coincident lines if

* ~:; their gradients are not equal.


2

a2

* 1; their gradients are equal but the lines are different.


C2

bb,
2

= L; the equations are equivalent.


C2

EXAMPLE 8
Consider the simultaneous equations
(a) 3x + 2y
5
6x + 4y = 16

(b) 3x

6x

(a) The two lines have the same gradient


3

+ 2y = 5

+ 4y = 10

6" = 4

* 16

The lines are parallel. There is no solution to the equations.


3x + 2y
5
. . . . . . . . . . . . . . . . . . . . . . . . .. (i)
Algebraically:
6x + 4 y
16
. . . . . . . . . . . . . . . . . . . . . . . .. (ii)
Multiply (i) by 2
6x + 4y
10 ......................... (iii)
Subtract (iii) from (ii)
0
6
This is a false statement. There are no values of x and y for which this statement is
true.
(b) The two lines have the same gradient

3
2
5
6
4
10
The lines coincide. There is no unique solution. Actually, on division by 2, 6x + 4y =
lOis equivalent to 3x + 2y = 5.
Name some values of x and y which satisfy.
3x + 2y
5
. . . . . . . . . . . . . . . . . . . . . . . . .. (i)
Algebraically:
6x + 4y
10 .. .. . .. .. . .. . .. .. . . .. . ... (ii)
Multiply (i) by 2
6x + 4y
10 ......................... (iii)
Subtract (iii) from (ii)
0
0
This is a true statement, independent of x and y. We can proceed no further.

9/COORDINATE GEOMETRY - STRAIGHT LINE 207

EXERCISES 9(b)
Find the equations of the following straight lines (1. to 12):
1.

Gradient~,

2. Gradient

passing through (-6, 5)

-~, passing through (4, -3)

3. Passing through (3, 3) and (-4, -5)


4. Passing through (2, -8) and (7, 2)
S. Passing through (6, 6), and making an angle of 45 with the X-axis
6. Passing through (-2, 3), and making an angle of 538' with the X-axis
7. Passing through (-5, -2), and making an angle of 135 with the X-axis
8. Passing through C7, 4), and making an angle of 1438' with the X-axis
9. Parallel to the X-axis and passing through the point (5, 2)
10. Parallel to the Y-axis and passing through the point (-2, -4)
11. X-intercept 2, Y-intercept -5
12. X-intercept -3, Y-intercept -2
Express the equations of the following lines in the form y
gradient of each (13. to 18.):

mx + c, and hence state the

13. 2x + 3y = 4

14. 3x - 2y = 7
15. 8x

+ 2y

= 3

16. 2y = 6 - 3x
17. 4x

5y = 3

18. 5y - 2x

19. For the line 2x + 3y - 12 = 0, find


(a) its intersection with the axes,
(b) its gradient.
20. (a) Show that (2, 3) is on the line 2x + 3y - 13 = O.
(b) If (-1, 2) is on the line ax - 4y + 11
0, find a.
21. Find the equation of the line containing the point (2, -3) that is
(a) parallel to,
(b) perpendicular to, the line 3x + 2y - 6
O.
22. Find the equation of the line passing through the origin that is
(a) parallel to,
(b) perpendicular to, the line with equation 4x - 5y + 3 = O.
23. The line 5x + 2y - 10 = 0 is translated
(a) 2 units to the left, parallel to the X-axis,
(b) 2 units upwards, parallel to the Y-axis.

Find the equation of the image line under each transformation.

208 NEW SENIOR MATHS: TWO UNIT COURSE

24. The line 4x - 3y = 12 is translated


(a) 3 units to the right, parallel to the X-axis,
(b) 3 units downwards, parallel to the Y-axis.

Find the equation of the image line under each transformation.

25. The line y


2x
4 is rotated through an angle of 90 in the plane about its point of
intersection with
(a) the X-axis, (b) the Y-axis.

Find the equation of the image line under each transformation.

26. The line with equation y = 3x - 6 is rotated in the X - Y plane through a right angle
about its point of intersection with the Y-axis. Find the equation of the image line
under the transformation.
27. Show that the point (4, -1) belongs to the line with equation 2x + 3y = 5. Find the
equation of the image line under a rotation of 90 about the point (4, -1).
28. The coordinates of two points A and Bare (0, -2) and (3, 0) respectively. The
x-coordinate of a point C on AB is 6. Find
(a) the equation of AB,

(b-) the magnitude of the angle that AB makes with the X-axis,

(c) the y-coordinate of C,


(d) the equation of the line that contains the point C and is perpendicular to AB.
29. Show that the line with equation 2x - y
(-1, 5) and (1, 9).

5 is parallel to the line joining the points

30. ABCD is a parallelogram. The coordinates of A, Band Care (-1,4), (4, 6) and (2,7)
respectively. Find
(a) the equation of CD,
(b) the equation of AD,
(c) the coordinates of D.

31. ABCD is a rectangle. The coordinates of A and Bare (1, 4) and (5, 2) respectively.
The x-coordinate of D is -2. Find:
(a) the equation of AB,
(b) the equation of AD,
(c) the y-coordinate of D,
(d) the equations of BC and DC,
(e) the coordinates of C.
32. A (0, 0), B(2, 1) and C(1, 5) are the vertices of a triangle ABC. The triangle is rotated
anti-clockwise about the point A, through a right angle in the X - Y plane. Find the
equation of the image lines of
(a) the line AC,
(b) the line AB,
(c) the line BC.
33. Find the equation of the line that contains the point of intersection of 2x + 5y
o and 3x - 4y + 6 = 0 and is parallel to the line with equation 4x - y

19
8.

34. The coordinates .of the vertices A, Band C of a triangle are (-1, 3), (2, 5) and (1, -1)
respectively. Find:
(a) the equation of the perpendicular line from A to BC,
(b) the equation of the line through B, parallel to this perpendicular line.

9/COORDINATE GEOMETRY

STRAIGHT LINE 209

35. Consider the equations:


x+y+l
0
y
2
x
3y = 2x + 1
2x - 3y + 6 = 0
(a) Show that these equations represent the sides of a parallelogram.
(b) Find the coordinates of the vertices of the parallelogram.
(c) Find the equations of the diagonals of the parallelogram.
36. Find the equation of the straight line that contains the point of intersection of the
lines with equations 3x + 2y = 12 and 5x - y
7, and which
(a) passes through the point (-4, -5),
(b) is parallel to the line 2x
y + 4 = 0,
(c) is perpendicular to the line y = 5.
37. ABCD is a parallelogram. The coordinates of A, Band Care (-1,4), (4, 6) and (2, 7)
respectively. Find the coordinates of D.

38. ABCD is a quadrilateral. The coordinates of A, Band Care (-8, 6), (2, 4) and (5, -7)
respectively. If the diagonals are perpendicular, and DC is parallel to the X-axis, find:
(a) the coordinates of D,
(b) the coordinates of the point of intersection of the diagonals.
39. The coordinates of the vertices of a triangle ABC are (2, 5), (6, 1) and (4, -3)
respectively. Find:
(a) the equation of the line through C perpendicular to AB,
(b) the equation of the line through A perpendicular to BC,
(c) the coordinates of the point of intersection of these two perpendicular lines,
(d) the magnitude of angle ABC.
40. The equations of the sides of a triangle ABC are as follows:
AB: 5x + y
10
BC: 3x - 2y = 6
CA: x - 5y = - 24
(a) Show that angle BA C is a right angle.

(b) Find the coordinates of the foot of the perpendicular from A to BC.
41. Without actually solving the simultaneous equations state whether the following pairs
of lines (i) intersect, (ii) are parallel or (iii) coincide.
(a) 2x
3y = 8
(b)
x + 3y + 7
0

4x - 6y = 16
2x + 7y + 16 = 0

(c) 6x - 5y
24 = 0
(d) x + y
7

9x
4y - 22 = 0
x + y = 8

9.6 Linear inequations* ____________


Figure 9-10 shows the set of points on the Cartesian plane that lie on or above the line with
equation x + y = 1. This set of points is represented by the inequation x + y ~ 1.
Figure 9-11 shows the set of points on the Cartesian plane that lie on or below the line
with equation x + y
1. This set of points is represented by the inequation x + y ~ 1.
Figure 9-12 shows the set of points on the Cartesian plane that lie above the line with
*Frequently the word "inequality" is used in place of "inequation" .

: I

- -- .. ----.-- -..----'---.->..--..-. -... ..

...

~~.---- -.- ~ ~~~'. ~~-----------~~~~-~~

210 NEW SENIOR MATHS: TWO UNIT COURSE

x+y

i!

Fig. 9-11

Fig. 9-10

x+y>l

x+y< l

Fig. 9-12

Fig . 9-13

equation x + y = 1. This set of points is represented by the inequation x + y > 1. Since


the line x + y = 1 is not included in the set of points, it is drawn with a broken line.
Figure 9-13 shows the set of points on the Cartesian plane that lie below the line with
equation x + y = 1. This set of points is represented by the inequation x + y < 1. Again,
since the line x + y = 1 is not included in the set of points, it is drawn with a broken line.
Each inequation divides the plane into two regions. Each region is a half-plane.
y

y ~

...

.~

-1

y~

Fig. 9-14

-1 <

y~2

Fig. 9-15

9/COORDINATE GEOMETRY -

STRAIGHT LINE 211

Figure 9-14 shows the set of points on the Cartesian plane that lie on or below the line
with equation y = 2. This set of points is represented by the inequation y ~ 2.
Figure 9-15 shows the set of points on the Cartesian plane that lie on or below the line
with equation y = 2 but also above the line with equation y = -1. This set of points is
represented by the inequation -1 < Y ~ 2. Note that the line y = 2 is a continuous line
whereas the line y = -1 is a broken line. Why?
y
l

-X

,~

0 1

-1

Xi

1,

.x > 1

x:::. 1 or

Fig. 9-16

x~

-1

Fig. 9-17

Figure 9-16 shows the set of points to the right of the line with equation x = 1. It does
not include the line x = 1, which is therefore a broken line. This set is represented by the
inequation x > 1.
Figure 9-17 shows two sets of points, one set being to the right of the line with equation
x = 1 and the other set being on and to the left of the line with equation x = -1. This
situation is represented by two inequations: x > 1 or x ~ -1.

EXERCISES 9(c)
Sketch the graph of the following inequations and shade in the required region.
1.
3.
5.
7.
9.
11.
13.

> 2x

2x

3y

3x - 4y ~ 6
x - y < -2
y < x + 1

2. x

4. y

3
2x < 1
6. 2y - 5x < 10

8. -2 < y

10. y ~ x + 1
12. x ~ Ii
14. -3 < x + y

< 2

9.7 Simultaneous linear inequations ______


We have seen that linear equations in x and y can be represented by straight lines and that
linear inequations can be represented by regions of the Cartesian plane.
We have also seen that it is possible to have two equations true at the same time and that
graphically this is represented by the point of intersection of two lines.
Is it possible to have two inequations true at the same time? If so., how can the solution
set be represented?

The solution set oj two simultaneous linear inequations is represented by


the region oj the Cartesian plane that is common to the two regions.

212 NEW SENIOR MATHS: TWO UNIT COURSE

EXAMPLE 9
Find the solution set of the simultaneous inequations
y~x

x+y<2

x+y=2
A: y

8: x + y< 2

A n 8: y

x nx + y

<2

Fig. 9-18
Fig. 9-19
Fig. 9-20
The shaded region A in figure 9-18 is the set of points that lie on and above the line with
equation y = x and is thus y ~ x.
The shaded region B in figure 9-19 is the set of points that lie below the line with
equation x + y = 2 and is thus x + y < 2. Note that this set does not contain the set of
points on the line x + y = 2, which is therefore a broken line.
The region that is common to both A and B is represented in figure 9-20 as the shaded
region A n B and is the set of points on and above the line y = x and also below the
line x + .y = 2. It is an unbounded region i.e. it is not completely enclosed.
It is usual to indicate the three regions A, B and A n Bon the one graph, as shown in
figure 9-21.
A is the shaded region marked thus: IIII
B is the shaded region marked thus:
A

B is the shaded region maked thus: _

Two intersecting straight lines divide the plane into four regions, each of which is
defined by a pair of linear inequations.
y

y=x

x
x+y=2

Fig. 9-21
In the table below, we have taken some points at random on the plane and tested
whether they belong to A, B, A n B. Note that those. points which belong to A n B
belong also to both A and B.

9/COORDINATE GEOMETRY -

Point

AnB

(0,0)
(-1, 3)
(0, 1)
(2, 1)
(1, -4)
(-3, -2)

Yes
Yes
Yes
No
No
Yes

Yes
No
Yes
No
Yes
Yes

Yes
No
Yes
No
No
Yes

STRAIGHT LINE 213

y=x+1

EXAMPLE 10

.. X

"'~I---'-'---+-- ' -

-1

- .....

,
.

x~2n

y:>-l

y~

x+ln

x~

Fig. 9-22

Fig. 9-23

Figure 9-22 shows the set of points on and to the left of the line with equation x = 2 and
also above the line with equation y = -1. This set is expressed as x ~ 2 n y > -1
Figure 9-23 shows the set of points on and below the line with equation y = x + 1 and
also on and to the left of the line with equation x = 1. This set is expressed as
y~x+lnx~1

EXAMPLE 11
Show, by shading on a sketch, the region defined by the three inequations

x - y

x + 3y

-1
-1

19

5x

3y

x-v = -1

x
Fig. 9-24
x+3y=-1

Fig 9-24 shows the required region on or below the line x - y = -1, on or above the line
3y = -1 and on or below the line 5x + 3y = 19. The region is a bounded region.
A(2, 3) is the point of intersection of x - y = -1 and 5x + 3y = 19 found by solving
the simultaneous equations.
B(-I, 0) is the point of intersection of x + y = -1 and x + 3y = -l.
C(5, -2) is the point of intersection of x + 3y = -1 and 5x + 3y = 19.

214 NEW SENIOR MATHS: TWO UNIT COURSE

EXERCISES 9(d)

Describe the sets that are shaded in each of the diagrams in questions 1. to 6.

1.

....

3.

6.

=2

x =-1
-_-1~+--~~ X

4.

5.

x =-1
--~~---1~X

-1

Graph the solution set of each of the simultaneous inequations in questions 7. to 12. and
state whether the given points are in the region.
7. x

+y

8.

y~x

(0, 0), (2, 3), (-1, -2)

+Y >

+ 2y

10.

3y ~ 2x + 6
x+y>2
(2, 0), (3, 3), (4, -1)

+y

12.

y > 3x + 3
x+y<3
(0, 3), (2, 7), (-1, 4)

~ 8
y < 7
(0,4), (-1, 1), (9, 2)

9. x

2y > x + 2
-1
(0, 0), (0, 1), (2, 5)

~ 4
x ~ -2
(0, 0), (-3, 1), (1, 0)

11. 4x

In each of questions 13. to 18., write down the equation of the intersecting lines and
describe the four regions labelled P, Q, Rand S.
13.

14.
y

I:

. l

15.
y

9/COORDINATE GEOMETRY -

16.

STRAIGHT LINE 215

17.

--~--,'~---1""'X

Describe the sets that are shaded in each of the diagrams in questions 19. to 21. and find,
by solving simultaneous equations, the coordinates of the points A, Band C in each case.
19.

21.

20.
y

y
Y=

x
2

+1

I~~_-r- Y =6 +1
~--"""
x+2Y=6

Shade in the region and find the coordinates of the points of intersection of the boundary
lines for each of questions 22. to 27.

22.

x + 2

23.

y - x
y

2x + Y ~ 4
x+y~2

4x

24. Y - 3x < 4

25.

y+2x<7
y

x -

x+y~2

0, y

3
2x - 6

+ Y > -8

y
3x < 3
3x + 4y < 3

x - 2y > 4

y~x+2

26.

>

27. x

x +y

-2

x-y~2

-2

9.8 Distance between two points _ _ _ _ _ __


Consider two points A (Xl' Yl) and B(X2' yJ. Construct a right-angled triangle ABC, as
shown in fig. 9-25, with AC and CB parallel to the X-axis and Y-axis respectively. The
coordinates of Care (X2' Yl)' We use the notation d(A, B), or simply AB, to denote the
distance from A to B. This is a non-negative number, and AB
BA.

216 NEW SENIOR MATHS: TWO UNIT COURSE

x
x, I

C(X2, y,)

Fig. 9-25

AC = IX2 - xII or IXI - x21. For example,


-3 and x 2 = 5, then IX2
xII
15 + 31
BC = IY2 - YII or IYI - Y21. For example,
if Yl =
2 andY2
3, then IY2 - YII
13 + 21

if XI

S.

5.

Applying the Theorem of Pythagoras to triangle ACB, we get


AB2

= AC2 + BC2
= (x2 xlF +

(Y2 - YIF

Hence

EXAMPLE 12
Find AB where A
Let

(-2, 3) and B

A
(Xl' YI)
(-2, 3), and
B
(x2, Y2) = (6, -1).
IX2 - Xj I = 16 + 21
S
IY2
Yll
1'-1 - 31 = 4
AB ..J(x2 - Xj)2 + (Y2 - YI)2
= ..J64 + 16

(6, -1).

y
A(-2.3)

(-2,-1)

..J8O

4.J5

EXERCISES 9(e)
1. Find the distance between the following points:
(b) (5, 1) and (5, -4)
(a) (4, 6) and (-5, 6)
(d) (7, -2) and (-6, 5)
(c) (-5, -3) and (3, -1)
(f) (a, b) and (b, a)
(e) (a, 2a) and (-2a, 6a)
2. Find the perimeter of the triangle whose vertices are the points (5, 1), (S, 5) and
(-3, 7).
3. Prove that the points (0, -2), (6, 6), (S, 4) are the vertices of an isosceles triangle.
4. Show that the points (2, -3), (5, 2) and (-3, 0) are the vertices of a right-angled
triangle.

9/COORDINATE GEOMETRY

STRAIGHT LINE 217

5. Show that the points A(-I, 4), B(4, 6), C(2, 7) and D(-3, 5) are the vertices of a
parallelogram A BCD, by finding the length of the sides. Find the length of each
diagonal.
6. Show that the points P(-2, 4), Q(5, 5) and R(-2, -2) are equidistant from the point
(2, 1).
7. Find the length of each side of a triangle whose vertices have coordinates (-2, -3),
(-5, 1) and (6, 3). Hence show that the triangle is right-angled.
8. The vertices of a quadrilateral are the points A(2, -3), B(5, 1), C(1, 4) and D(-2, 0).
Show that the quadrilateral is a square.
9. Show that the points (-4, -1), (2, 3) and (6, -3) are the vertices of a right-angled
isosceles triangle.
10. Show that the points (1, 1), (4, 5), (0, 8) and (-3, 4) are the vertices of a square.
11. The coordinates of A, Band Care (2, 19), (-3, 7) and (10, 7) respectively.
(a) Show that DABC is isosceles
(b) Find the length of the perpendicular from A to BC.

12. Show that the points (2, -1), (4, 3) and (5, 2) are equidistant from the point (3, 1).
13. Show that the points (-2, -2), (4, 6) and (-4, 2) are equidistant from the point (1, 2).
14. The coordinates of A, Band Care (0, 4), (5, 1) and (1, -3) respectively. Find:
(a) the length of AB
(b) the perimeter of the triangle ABC.

9.9 Perpendicular distance of a point


from a line _ _ _ _ _ _ _ _ _ _ _ _ _ __
y

ax +by+c

Fig. 9-26
The perpendicular distance of the fixed point P(x1 , Yl) from the line with equation
ax + by + c = is given by the formula

There are a number of ways to prove this formula. One method is as follows:
(i) Write down the equation of PM
(ii) Solve the simultaneous equations to find the coordinates of M
(iii) Use the distance formula to find the length of PM.

218 NEW SENIOR MATHS: TWO UNIT COURSE

EXAMPLE 13
Find the distance from the point (4, 5) to the line with equation x
Applying the formula, we get
PM
11 x 4 + 2 x 5 - 41
-v'P + 22
_ 1101
-..[5

.... x

----+---......;;:.,~~

x+2y-4=O

Alternatively, follow the steps given:


(i) Since PM is perpendicular to the given line its equation is of the form 2x
Putting x
4, Y = 5 gives k = -3.
3.
The equation of PM is 2x - Y

-../(x2 - X 1)2

+ (Y2 -

+k

O.

= 3 and x + 2y = 4 yields x = 2, Y = 1.

(ii) Solving the simultaneous equations 2x - Y


The coordinates of Mare (2, 1).
=

O.

10
..[5
2..[5
The distance is 2..[5 units.

(iii) PM

2y - 4

Yl)2

-../(2 - 4)2 + (l - 5)2

=m
=2..[5

Proof of the formula


The equation of the line through P, perpendicular to the given line is
This meets

Thus

= bX l - aYI
- e at M(xo, Yo), where
= b 2xl - abYI - ae
.................... (i)
-abxl + a 2Yl
be .................. (ii)
_ (a 2 + b 2)Xl - (b 2xl - abYl
tie)
a2 + b 2

bx - ay
ax + by
(a 2 + b 2)xo
(a 2 + b 2)yo
Xl -

Xo

a(ax1

a2

and

(a 2

Yl - Yo

bYI

+ e)

b2

b 2)Yl - (-abx l

a + b
2

b(ax l

bYI

e)

a2 + b 2
(ax l

Hence

bYI

a2 + b 2

and taking the positive square root gives the distance as

+ bYI + el
-../a 2 + b 2

lax)

EXAMPLE 14
Find the distance between the parallel lines
2x - Y + 2 = 0
and 2x - Y
5 = 0

e)2

a 2Yl - be)

9/COORDINATE GEOMETRY

STRAIGHT LINE 219


2x-y+2=o

Take any point on either line e.g. (0, 2) on


2x - y + 2 = O.
We then find the distance PM from (0, 2)
to the line 2x - y - 5 = 0
PM = 12 x 0 + -1 x 2 - 51
-..)22 + p

1-71

2x-y-5=

-J5

x
Can you think of another way of doing it?

EXERCISES 9(f)
Find the distance between the point and the line in each of the following (1. to 8.)
1.
3.
5.
7.

(1,4); 3x - 4y
12 = 0
(5, 6); y + 2 = 0
(1, -12); 5x
12y + 20
0
<-2, -3); 2y
x - 6 = 0

(2, 3); 7x - 24y + 8 = 0


(-5, -2); x - 3y
6 = 0
(0, 0); x + y
4 =0
(-2, -2); 4x
3y = 0

2.
4.
6.
8.

Find the distance between the parallel lines (9. to 11.)


9. 3x

4y - 4 = 0; 3x - 4y

+ 8 = 0; 7x
+ 1 = 0; x + y

10. 7x - 24y
11. x

+y

24y
- 5

16

=0

+ 58

12. A(-2, -3), B(O, 3) and C(2, 4) are the vertices of a triangle ABC. Find the altitude
from A.
13. P(4, 2), Q(-8, -7) and R(O, 4) are the vertices of a triangle PQR. Find the altitude

from R.
14. Find the area of the triangle whose vertices are (-1, -1), (1, 2) and (2, 1).
15. Find the area of the triangle whose vertices are (1, 3), (3, 4) and (0, 2).
16. Prove that the lines x - 2y
equidistant from the origin.

= 0,

2y - 5

= 0 and

2x

= 0 are

9.10 Midpoint of an interval _ _ _ _ _ _ _ __


B(X2, Y2)

I...Q

:~
-- -;,. -_-; __ .rl D
I

I;:;:

I I
I...Q

(x" y,)A

________ d
a - x,
C

Fig. 9-27

220 NEW SENIOR MATHS: TWO UNIT COURSE

Let P(a, b) be the midpoint of the interval joining A(x1, Yl) and B(x2, Y2)'
The triangles APC and PBD are congruent
AC
PD
and
PC = BD
i.e. a XI
x2 - a
i.e. b - Yl
Y2 - b
2a = XI

+ x2

2b = Yl

Yl

Y2

+ Y2
2

Thus the coordinates of the midpoint of an interval are

XI

+ x2
2

YI

'

+ Y2)
2

EXAMPLE 15
Find the coordinates of the midpoint of the interval joining the points (-4, 2) and (5, -7).

2; 7)

. (12' -21)
I.e.

EXAMPLE 16
Find the equation of the perpendicular bisector of the line joining the points A(6, 4) and
B(-2, -2).

Mid-point of AB

6 - 2 4
2 '
(2, 1)
4 + 2

=
Gradient of AB

A(6,4)

3
4

.'. Gradient of
Equation of

.l

.l

3
bisector: Y - Y 1 = m(x
y - 1

3y
3
4x + 3y

-~x

(-2-;2 )8
XI)

1:
3

- 2)

3
-4x + 8
11

EXERCISES 9(g)
1. Find the coordinates of the mid-point of the interval joining the points
(i) (5, 1) and (-2, -3)
.
(ii) (8, 8) and (-4, 6)
(iii) (-2, -7) and <-6, -9)
(iv) (-3, 5) and (5, -3)
(v) (a, b) and (c, d)
(vi) (a + b, a) and (a - b, b).
\1

2. The coordinates of the mid-point of the join of A(3, -2) and Bare (-4,6). Find the
coordinates of B.
3. The vertices of a triangle are A(2, 5), B(7, -3) and C(-2, 1). Find the coordinates of D
and E, the mid-points of AC and AB respectively. Show that DE is parallel to CB.

9/COORDINA TE GEOMETRY -

STRAIGHT LINE 221

4. A(4, 3), B(6, -2), C(I, -5) and D(-3, -2) are the vertices of a quadrilateral. Find the
coordinates of E, F, G and H, the mid-points of AB, BC, CD and DA respectively,

and prove that these points are the vertices of a parallelogram.


5. Find the equation of the perpendicular bisector of the interval joining the points
(-5, -6) and (3, -2).
6. Find the equation of the line parallel to the line 3x + 4y - 6 0 and passing through
the mid-point of the interval joining the points (-5, -6) and (2, 8).
7. The points A (5, 3), B(3, -6), C(-3, -2) and D(-l, 7) are the vertices of a parallelogram.
Find the coordinates of the mid-point of each of its diagonals. What do you
conclude?
8. Find the equation of the line perpendicular to the line 2x
5y + 3 = 0 and passing
through the mid-point of the interval joining the points C8, 6) and (4, -2).
9. (a)
(b)
(c)
(d)

Show that the triangle whose vertices areA(3, 5), B(1, 1) and C(-l, 3) is isosceles.
Find the coordinates of the mid-point D of BC.
Write down the gradients of AD and BC.
What property of isosceles triangles is illustrated in (c)?

10. A(-3, -1), B(4, 1), C(8, 5) and D(I, 3) are the vertices of a parallelogram.
(a) Find the coordinates of the mid-points of the diagonals AC and BD.

(b) What property of parallelograms is illustrated in'(a)?


11. (a) Show that the four points (0, 0), (2, 1), (3, -1), (1, -2) are the vertices of a square.
(b) Find the coordinates of the mid-point of each diagonal. What do you conclude?
(c) Write down the gradient of each diagonal. What do you conclude?
12. A(2, 1), B(1, 4), C(-4, -1) are three points.
(a) Show that angle BAC is a right angle.
(b) Find the coordinates of the mid-point D of BC.
(c) Find the distance from D to each vertex of the triangle ABC.
13. AC1, 4), B(4, 6), C(2, 7) and D(-3, 5) are the vertices of a parallelogram.
(a) Find the coordinates of the mid-points P, Q, Rand S of AB, BC, CD and DA

respectively.
(b) Prove that PQRS is also a parallelogram.
14. A(-4, 0), B(4, 0) and C(2, 4) are the vertices of a triangle.
(a) Find the equation of the perpendicular bisector of AB.
(b) Find the equation of the perpendicular bisector of BC.

(c) Find the coordinates of the point S where the perpendicular bisectors meet.
(d) Prove that S is equidistant from each of the vertices.

CHAPTER 10

Locus and

Subsets of

the Plane

In this chapter, we consider functions and relations defined by geometrical conditions,


with associated graphical representation.
Hence this chapter may be seen as a broadening, both of coordinate geometry, and of
functions and relations.

10.1 Locus _ _ _ _ _ _ _ _ _ _ _ _ _ __
Definition: We may define locus as a set of points in a plane that satisfies some geometric
condition or some algebraic equation. Frequently, locus is defined as 'the path traced out'
by a particle moving in a plane (or space), and a Cartesian equation gives us the nature of
the curve along which the particle travels. We have already seen that:
(a) the equation ax + by + c = 0 defines a straight line.
(b) the equation y = ax2 + bx + c defines a parabola.
In this chapter, we shall consider certain geometric conditions that define a straight line, a
circle, a parabola, etc. For example, from your earlier study of Euclidean geometry, you
have seen that the locus of all points P in a plane such that P is equidistant from two fixed
points is a straight line. We shall see that the locus of a point P in a plane such that its
distance from one fixed point is twice its distance from another fixed point is a circle. We
shall redefine the parabola in a later chapter as the locus of all points P in a plane such
that the distance from P to a fixed point is equal to its distance from a fixed straight line.

10.2 Straight line _ _ _ _ _ _ _ _ _ _ _ __


EXAMPLE 1
Find the locus of a point P(x, y) such that its distance from A (-1, 3) is equal to its distance
from B(5, 1).

i'

10/LOCUS AND SUBSETS OF THE PLANE 223

A (-1.3 )11<.:: -

,PC, x,

y)

-- - ' ,

--~B(5,1)

o
Fig. 10-1

We use the notation PA to denote the distance from P to A


Applying the formula for the distance between two points, we get
PA = ..J(x + 1)2 + (y - 3)2, and
PB = ..J(x - 5)2 + (y - 1)2.
PB, it follows that
Since PA
..J(x + 1)2 + (y - 3)2
..J(x - 5)2 + (y - 1)2.
Squaring both sides of this equation yields
(x + 1)2 + (y - 3)2 = (x - 5)2 + (y - 1)2
X2 + 2x + 1 + y2 - 6y + 9
X2 - lOx + 25 + y2
2y + 1
i. e. ,
12x - 4y - 16 = 0

3x - y - 4 = 0

Since this equation is of the form ax + by + c = 0, the locus is a straight line.


Check that this straight line is the perpendicular bisector of the line segment AB.
Referring to fig. 10-1, we see that, for any position of the point P, the triangle PAB is
isosceles.

EXAMPLE 2
What is the locus of a point P(x, y) that is always three units from the Y-axis?
P(x, y) is any point on the locus, and PM denotes the distance of P from the Y-axis
(fig. 10-2).
y

PM
3

i.e., Ixl = 3
Hence the locus is a pair of lines
M - - - - - - P(x.y)
with equation x = 3.

Fig. 10-2

-3

EXERCISES 10(a)
1. Find the locus of a point P(x, y) which moves in a plane such that PA

PB where
(b) A = (2, 4), B = (2, -6)
(d) A = (1, 7), B = (4, -2)

Show in each case that the locus is a straight line perpendicular to AB and passing

through the midpoint of AB.

(a) A = (0, 3), B = (0, 7)


(c) A = (-2, 3), B = (4, 5)

224 NEW SENIOR MATHS: TWO UNIT COURSE

Find the equation of the locus of a point P(x, y) that is subject to the following
conditions: (2. to 11.)
2. A point is three units above the X-axis.
3. A point is four units to the left of the Y-axis.
4. A point is equidistant from the line y = -5 and the X-axis.
5. A point is

.J2 units

x.

from the line with equation y

6. The distance of a point from the Y-axis is three times its distance from the X-axis.
7. The distance of a point from the line x

4 is equal to its distance from the line y

1.

8. Twice the distance of a point from the X-axis is three times its distance from the
Y-axis.
9. The distance of a point from the line y
line x
2.

-5 is three-quarters of its distance from the

10. The distance of a point from the line x


y
-l.

= -3 is two-fifths of its distance from the line

11. A point is equidistant from (3, 5) and (-1, -1).


12. Find the locus of a point P which moves so that
(a) PA2 - PB2 := 5,
(b) PA := PB, where P = (x, y), A
(2, 3) and B

(-2, -1).

13. A and B are the fixed points (a, 0) and (-a, 0), Find the locus of P(x, y) such that the
gradient of AP is twice the gradient of BP.
I .

10.3 The circle _ _ _ _ _ _ _ _ _ _ _ _ __


A circle may be defined as the set of all points, P, in a plane at a given
distance from a fixed point in the plane. The fixed point is the centre of
the circle and the given distance is the radius.
y

Fig. 10-3

(x - h)2

+ (y - k)2

r2

Consider a circle of radius r units, having its centre at C(h, k). If P is a point in the
plane with coordinates (x, y), then 'P is on the circumference of this circle if the distance

10/LOCUS AND SUBSETS OF THE PLANE 225

from C to Pis r units. If the measure of the distance from C to P is denoted by CP, then P
is on the circle if CP = r.
Applying the Theorem of Pythagoras to the triangle CBP in fig. 10-3, we have
BC2 + Bp2
k)2

i.e., (x h)2 + (y
Thus the circle is defined by the relation

The domain is h - r
The range is
k - r

~ x ~
~

h
k

Cp2
r2

+ r
+ r

Note: The equation (x - h)2 + (y - k)2


However, if we transpose the equation so that
(y

k)2
y - k

r 2 does not define a function. Why?

= r 2 (x
= ~r2 -

Y = k

- h)2
(x - h)2
~r 2 - (x - h)2

defines a function whose graph is the top semicircle


y
k - ..,;r-c
and
r2=--_-(-,-x-_-h-: -c)- =-2
defines a function whose graph is the lower semicircle both with the same domain
h - r ~ x ~ h + r.
If the centre of the circle is at the origin, then h

0 and k

0, and the equation of

such a circle is then

EXAMPLE 3
Find the equation of the circle with centre (-3, 4) and radius 6 units.
(x - h)2 + (y - k)2 = r 2
(x + 3)2 + (y - 4)2 = 36

i.e.,
X2

6x

X2

+ 9 + y2
y2 + 6x

- 8y + 16
8y - 11

36
0

EXAMPLE 4
Find the coordinates of the centre, and the length of the radius, of the circle whose
O.
equation is X2 + y2 - 4x + lOy + 14
X2 + y2 - 4x + lOy + 14
0
(x 2 - 4x + 4) + (y2 + lOy + 25) + 14
4 + 25
(x
2)2 + (y + 5)2 = 15
Since this is of the form (x h)2 + (y - k)2 r2, the coordinates of the centre are (2, -5),
and the length of the radius is vT5 units.

EXERCISES 10(b)
1. Find the equation of each of th~ following circles:
(a) centre (-1, -4), radius 3 uIllts
(b) centre (3, -3), radius

(c) centre

(-2, ~). radius ~ units

(e) centre (4, 0), radius 3 units

(d) centre

.J5 units

(0, -~). radius 4 units

226 NEW SENIOR MATHS: TWO UNIT COURSE

2. Find the equation of the following circles:


(a) centre (3, 2) and passing through the point (5, -5)
(b) centre (-1, 4) and passing through the origin
(c) centre (0, 0) and passing through the point (-3, 4)
3. Write down the coordinates of the centre, and the length of the radius, of the
following circles:
(a) X2 + y2 - 5x + 3y - 1
0
(b) X2 + y2 + 4x + 2y
5 = 0
(c) (x - 3)2 + y2
3
(d) (x + a)2 + (y -:- b)2 = 8
(e) 2X2 + 2y2
8x + 5y + 3 = 0
(f) 3x 2 + 3y2 + 9x
4y - 24
0
4. Using the fact that the coordinates of the midpoint of the interval joining the points

2.Y2) are (XI; x 2. Y,

(x,. Y,) and (x

Y2), find the equation of the circle that has,

as endpoints of a diameter, the points


(a) (3, 4) and (9, -6),

(b) (0, 0) and (5, -3),


(c) (5, 8) and (-2, 3).

S. Show that the point (4, -3) does not lie on the circle X2 + y2 - 5x + 3y + 2
Does it lie inside or outside the circle?
6. Does the origin lie inside or outside the circle X2

+ y2 - 4x - y +

7. Show that the equation X2 + y2 - 6x + 2y + 10

= O.

1 = O?

0 represents a point-circle.

8. For the circle with equation X2 + y2 + 6x - 8y = 0,


(a) find the coordinates of the centre and the length of the radius,
(b) show that the origin is on the circle,
(c) find the equation of the diameter drawn through the origin.
9. Find the equation of the circle that touches the X-axis at (4, 0) and the Y-axis
at (0, 4).

10. Show that the point (2, 1) lies on the circle with equationx 2 + y2 + 6x 2y - 15 = 0,
and find the coordinates of the other end of the diameter through (2, 1).
11. (a) Find the equation of a circle whose centre is the point (-1,2) and whose radius has
a length of 5 units.
(b) What is the length of the intercept cut off by this circle on the X-axis?
(c) Find the length of the tangent from the point (6, 4) to this circle.
12. Find the equation of the circle whose centre is the point (1, 3) and which touches the
X-axis.
13. The equation of a circle is X2 + y2 + 4x - 2y - 20 = O. Find
(a) the length of the tangent to this circle from the point (5, 2),
(b) the length of the intercept on the Y-axis.
14. A diameter intersects the circle at the points (6, -4) and <-2, 6).
(a) Find the centre and the radius of the circle.
(b) What is the length of the tangent from the point (-5, 5)?
15. The coordinates of two points, A and B, are (-1, 3) and (5, 7). Find
(a) the coordinates of the midpoint of AB,
(b) the equation of the circle of which AB is a diameter,
(c) the coordinates of the points of intersection of the circle and the Y-axis.

10/LOCUS AND SUBSETS OF THE PLANE 227

16. (a) Find the coordinates of the centre and the length of the radius of the circle
X2 + y2 - 4x - 8y - 5 = o.
(b) The point (3, 2) is the midpoint of a chord of this circle. Find the distance of the
chord from the centre and the length of the chord.

EXAMPLE 5
Find the locus of a point P which moves so that PA
and B = (4, 1). Show that the locus is a circle.

= 2PB where P

= (x, y), A

= (-2,4)

y
A (-2,4)-_ ...

-- -- -- -

P( x. y}
'\
'\
'\

\8(4.1}

(6,0)

Fig. 10-4

PA = 2PB
i.e.,
Square both sides of this equation:
(x + 2)2 + (y - 4)2 = 4[(x - 4)2 + (y - 1)2]
X2 + 4x + 4 + y2
8y + 16 = 4[X2 - 8x + 16 + y2
2y + 1]
= 4X2 - 32x + 64 + 4y2 - 8y + 4
i.e.,
3x2 + 3y2
36x + 48 = 0
X2 + y2 - 12x + 16 = 0 (Divide by 3)
(x2 - 12x + 36) + y2 + 16 = 36
(x - 6)2 + y2 = 20
Since this equation is of the form (x - h)2 + (y - k)2 = r2, the locus is a circle whose
centre is (6, 0) and the radius is 2-J5 units.

EXAMPLE 6
A <-1, 3) and B(3, 1) are two points on the plane. Find the locus of P(x, y) such that PA is
perpendicular to PB.

Since PA is perpendicular to PB, the product of their gradients is -1.


Gradient of PA

Gradient of PB =

x+

3, x*-- 1.

- 1

*- 3.

228 NEW SENIOR MATHS: TWO UNIT COURSE

x
Fig. 10-5
Hence
i.e.,

y - 3
y
1
x+lxx-3
-1
(y - 3)(y - 1)
-(x + 1)(x - 3)
y2
4y + 3 = - X2 + 2x + 3
X2 + y2
2x
4y
0
(x 2 - 2x + 1) + (y2 - 4y + 4) = 5
(x
1)2 + (y - 2)2 = 5

h)2 + (y
k)2
r2, it represents a circle, centre
Since this equation is of the form (x
(l, 2) and radius -J5 units. This result confirms your previous knowledge of the theorem:
an angle in a semicircle is ~ right angle.

10.4 The equation

X2

y2 _ 0 _ _ _ _ _ _ __
y

X2 - y2 = 0
i.e. (x - y)(x + y)
0
x -y
0
or
x +y = 0
y

or
y
-x
Hence the locus is the pair of intersecting straight lines
whose equations are y = x and y
-x.

Fig. 10-6

EXERCISES 10(c)
1. Find the equation of the locus of a point P(x, y) that is 4 units from the point (2, -1).

2. What can be said about the centres of all circles that pass through the points (2, 0) and
(6, 4)? What is the locus of the centres?
3. Find the locus of a point P(x, y) that moves in a plane so that its distance from
A (3, -1) is twice its distance from B(-I, -1).
4. Find the locus of a point P(x, y) that moves in a plane so that its distance from
Q(5, -2) is twice its distance from R(-2, 3).
5. Find the locus of a point P(x, y) which moves in a plane such that PA 2 + PB2
44
where A = (-3, 2) and B
(3, -2). Show that the locus is a circle and find its centre
and radius.

- ., I

10/LOCUS AND SUBSETS OF THE PLANE 229

6. A ladder that is 6 m long rests with one end on the horizontal ground, and the other
end against a vertical wall. Considering the ground and the wall as the X- and Y-axes
respectively, find the locus of the midpoint of the ladder.
7. A point moves in a plane so that the sum of the squares of its distances from the
points (1, 2) and (5, -3) is 45. Find the equation of the locus.
8. A point P(x, y) moves so that the sum of the squares of its distances from the points
A(4, 0) and B(-4, 0) is 82 units.
(a) Find the equation of the locus of P.
(b) At what points does the locus cut the coordinate axes?
9. Find the equation of the locus of a point that moves so that its distance from the point
C4, 2) is always twice its distance from the point (5, -1). What point on the line joining
the points (-4, 2) and (5, -1) is on the locus?
10. A = (4, 2) and B = (-2, -8). Find the locus of a point P(x, y) that moves so that the
angle APB is a right angle.
11. A = (2, 1) and B = (-2, 1). Find the equation of the locus of P(x, y) if angle APB is a

right angle.
12. A point P(x, y) moves so that its distance from (3, 4) is proportional to its distance
from (-1, 2). Find the equation of the locus of P if the origin is a point on the locus.

13. A is a point where the circle with equation X2 + y2 = 16 cuts the X-axis. Find the
locus of the midpoints of all chords of this circle that contain the point A.
14. Find the equation of the locus of the midpoints of all chords of length 4 units of the
circle with equation X2 + y2 - 4x + 2y = 4.

10.5 Sets of points inside and


outside a circle _____________
A circle divides the plane into three sets of points, namely, the sets of points on the circle,
inside the circle and outside the circle. We have already considered the set of points on the
circle, centre (h, k) and radius measure r as the graph of
(x - h)2 ,+ (y - k)2 = r2,
and the point P(x, y) lies on this circle if CP = r.

IIC"'Y)
Y - k.

lI1.k) Ix - hi'

x
Fig. 10-7
Graph of
(x - h)2 + (y - k)2 < r 2

x
Fig. 10-8

Graph of

(x - h)2 + (y - k)2 > r 2

_...,..,.'1 "

"

---"---- "----".--.----'------'-~"

230 NEW SENIOR MATHS: TWO UNIT COURSE

If the point P(x, y) lies inside the circle (fig. 10-7), then

CP < r

i.e. CP2 < r 2


But
Cp2 = CB2 + BP2
= (x - h)2 + (y - k)2
Thus the graph of (x - h)2 + (y - k)2 < r 2 is the set of points inside the circle (fig. 10-7).
Similarly, the graph of (x - h)2 + (y - k)2 > r 2 is the set of points outside the circle
(fig. 10-8).

Note that in Fig. 10-7 and Fig. 10-8 the circles are drawn with a broken line. Why?

10.6 Graphs of solution -sets of


simultaneous inequations _________
The solution set of a system of inequations is the intersection of the solution sets of the
individual inequations of the system.

EXAMPLE 7
Sketch the region of the cartesian plane which includes all points on or inside the circle
centre (0, 0) and radius 3 units and all points
(a) to the left of the line x = 2,
(b) op. or above the line x + y = 3.
y

Y
3

3
/

-3

\
\

\.
" ......

Fig. 10-9

Fig. 10-10

(a) The region is represented by the inequations X2 + y2 ~ 9, x < 2. (Fig. 10-9)


Note that the line x = 2 is drawn with a broken line, the circle with a continuous
line.
(b) The region is represented by the inequations X2

+ y2

9, x

+y

3. (Fig. 10-10).

EXAMPLE 8
Describe the solution set and draw the graph of the simultaneous inequations y
y ~ 2x + 3.

X2 and

The graph of the inequation y ~ X2 is shown in fig. 10-11 as the set of all points in the
shaded portion of the plane, above and including the points on the parabola with equation
y = x 2
The graph of the inequation y ~ 2x + 3 is shown in fig. 10-12 as the set of all points in

\ "'-----

10/LOCUS AND SUBSETS OF THE PLANE 231

x
Fig. 10-11: y

~ X2

Fig. 10-12: y

.1

+3

2x

the shaded portion of the plane, below and including the points on the line with equation
+ 3.

y = 2x

(3.9)

(-l,n

x
Fig. 10-13
The parabola and straight line intersect at (-1, 1) and (3, 9). Check this by solving the
system of equations y == X2 and y = 2x + 3.
The intersection of the two shaded portions is the graph of the solution set, S.
In set language, it is expressed thus:
S = I(x, y):y ~ x 2 1

I(x, y):y ~ 2x

+ 31

Thus the graph of S is the shaded portion of fig. 10-13.

EXAMPLE 9
Describe the region of the x-y plane whose
points satisfy the inequationsy < 2 + x - x 2 ,
Y + 2x ~ 2.
Weare concerned with points on the plane
that are below the parabola with equation
y = 2 + x - X2, and that are also on or below
the line with equation y + 2x = 2. The
parabola and straight line intersect at (0, 2)
and (3, -4). Verify this.
Thus the graph of the solution set is the
shaded portion of fig. 10-14.

\
\
\ (3.-4)
-'I---~ I

Fig. 10-14

232 NEW SENIOR MATHS: TWO UNIT COURSE

Note that the graph of the parabola is drawn with a broken curve to indicate that points on
the parabola are not elements of the solution set; the graph of the line is unbroken to indicate
that points on the line in the interval x = to x = 3 are elements of the solution set.

EXERCISES 10(d)
Sketch the graph of the relations: (1. to 10.)
1. X2

+ y2

16

2. X2 + y2 < 4
3. (x - 1)2 + y2 > 9
4. (x - 3)2 + (y + 4)2
5~ (x + 3)2 + y2 < 1

6. y ~ X2 + 1
7. Y > 9 - X2
8. y ~ Ixl
9. y < 2x + 4
10. y ~ 12x + 41

25

Sketch the region of the plane defined by the inequations: (11. to 20.)
11. X2 + y2 ~ l,x ~ O,y ~ 0.
12. (x - 1)2 + (y - 1)2 < 1, x > 0, y
13. y < 4 x2,y~ 0.
14. y < lxi, y > 0, 1 < x < 2.

> 0.

15. X2 + y2 ~ 4, y > x + 2.
16. x > 0, y ~ X2 - 4, x + Y < 2.
17. Y > Ix
21, y > 3.
18. y ~ 1 - x2,y ~ 0.
19. X2 + y2 ~ 1, y ~ 2x, x ~ 0.
20. (x-l)2+y2~1,x~0,y~1.
Sketch the region of the cartesian plane whose boundary consists of: (21. to 30.)
21.
22.
23.
24.
25.
26.
27.
28.
29.
30.

X2, the ordinates at x = 1 and at x


2 and the X-axis.
the curve y
the circle centre (0, 2) and radius 2 units and the lines y
1 and y = 3.
the circle (x - 3)2 + (y - 4)2 25 and the y-axis.
an arc of the parabola y = 4 - X2 between x = 2 and x
-2 and the X-axis.
the graph of y
Ix - 11, the X-axis and the Y-axis.
the circle x 2 + y2 36, and the lines y
6 and x = 6.
the circle centre (-2, 0), radius 2 units and the lines x
1 and x - I .
the parabolay X2
2 and the liney x.
the curve y = .Jx, the y-axis and the line y = 2.
the semicircle y = .JI=X2 and the X-axis.

CHAPTER 11

Sequences

and Series

11.1 Sequences _ _ _ _ _ _ _ _ _ _ _ _ __
The word 'sequence' is used frequently in everyday language. We speak of a sequence of
events; your history teacher may ask you to write down certain events in their proper
sequence. We can think of the sequence of the topics treated in a text book; in a
mathematical proof it is important to put down the steps of the proof in their proper
sequence.
In all contexts in which the word 'sequence' is used, we consider a set of objects, ideas,
steps or events in some definite order, and we associate each element of the ordered set
with an element of the set of natural numbers 11, 2, 3, ... I.
Consider the set
{95, 92, 89, 84, 81, 791,
which represents the marks obtained by the best six candidates in an examination. Each of
these numbers can be associated with an element of the set of natural numbers
11,2, 3,4, 5,61,
which represent 1st, 2nd, 3rd, ... 6th places in the examination. From these two sets, we
can then form a set of ordered pairs:
1(1, 95), (2, 92), (3, 89), (4, 84), (5, 81), (6, 79)1
There is a one-to-one correspondence between the elements or terms of the sequence
and the first n natural numbers. Thus we associate a sequence with a sequence junction
whose domain is either
1. N, the set of natural numbers, in which case the sequence has an infinite number of
terms, or
2. {I, 2, 3, ... , n 1 where n EN, in which case the sequence has a finite number of terms.
The sequence function may be written thus:
(1, t 1), (2, t2), (3, t3), , (n, tn), . ..
where t 1, t2 , t3, , tn, . .. are the terms of the sequence, tn being the nth term.
In order to know the values of t 1 , t2 , t3 , we must have some rule that will tell us what
1, 2, 3, ....
the nth term is for each n

234 NEW SENIOR MATHS: TWO UNIT COURSE

EXAMPLE 1
Consider the sequence function defined by tn
When n

tn =
tl =
t2 =
t3=

= 1,
= 2,

2n - 1. Find t l , t2, t3' ...


2n - 1

2 - 1
4 - 1
6 - 1
The sequence is 1, 3, 5, ... , i.e. the set of odd natural
n
n = 3,

= 1,
= 3,
= 5,

numbers.

EXAMPLE 2
Define the sequence function for the following sequences:
(a) 2, 4, 6, 8,. . .
(b) 1, 4, 9, 16, ...
(c) 1, 2, 4, 8, ...

These sequences afford an opportunity of studying patterns of numbers.

tl = 2 = 2 x 1
t2 = 4 = 2 x 2
t3 = 6 =,2 x 3
Thus tn = 2n

(a)

tl =
t2 =
t3 =
Thus tn =

(b)

(c)

tl
t2
t3
Thus tn

1 = }2

4 = 22
9 = 32

n2

= 1 = 2
= 2 = 21
= 4 = 22
= 2n - 1

EXERCISES 11 (a)
2. Write down the first four terms of each of the following sequences whose nth term is
defined by
2
1
(b) tn = n +
(a) tn = 2
n

(c) tn

= an +

tn =
3 n

(f)

tn = n(n + 1)

(_l)n

(e) t =
n
n

(d)

3. Write down the first eight terms of the sequence defined by


tl = 1, t2 = 1, tn = tn- 2 + tn-I' n > 2.

This sequence is called the Fibonacci Sequence.

4. Study the pattern of each of the following sequences, and write down the next two
terms and the nth term.
(a) 3, 5, 7, 9, ...
1 1 1

(c) 1, 4' 9' 16'~"

(b) 0, 3, 8, 15, ...

(d) 1,1,1,3,5,9, ...

II/SEQUENCES AND SERIES 235


5. A sequence is such that t1 = 1, t2 = 2, t3 = 3 and each term thereafter is equal to the
sum of the three terms immediately preceding it. Write down ts, tm t2k + l'
In the remainder of this chapter, attention will be given to two particular sequences that
are of importance in mathematics, namely
1. arithmetic sequence or progression,
2. geometric sequence or progression.

11.2 Arithmetic sequence (or progression) _ __


An arithmetic sequence or progression is a sequence of terms in which
each term after the first is formed by adding a constant number, called
the common difference, to the preceding term.
The sequence

a, a + d, a + 2d, . .. , a + (n - l)d, ...


is called an arithmetic sequence, where a and d are real numbers and n is a natural
number.
a is the first term and d the common difference.
An arithmetic sequence may also be defined by the rule

The graph of this function is a set of points, which, if joined, would form a straight line.
It is a linear function (fig. 11-1). Are we justified in joining these points?

a+3d

a+2d

a+d

tn

(n

1 )d.

Fig. 11-1
Some examples of arithmetic sequences are as follows:
1.3,5,7,9,...
a
3,d=2
2. 6, 2, -2, -6, -10,. . .
a = 6, d = -4
3. 2, 3!, 4!, 5i,. . .
a = 2, d = I!
4. 1, 1 + .j2, 1 + 2.j2,. . .
a = 1, d = .j2
5.7r+3,27r+5,37r+7, ... a
7r+3,d= 7r+2

EXAMPLE 3
Write down the arithmetic sequence whose fifth and ninth terms are -17 and -29
respectively.

236 NEW SENIOR 'MATHS: TWO UNIT COURSE

t5 = a
a

t9

+ 4d = -17
+ 8d = -29

...................... (1)

(2)

Subtract (1) from (2).

From (1),

4d
d
12

12

-3

-5

-17

a
The sequence is -5, -8, -ll, ...

11.3 Arithmetic mean _ _ _ _ _ _ _ _ _ _ __


If a, band c are three consecutive terms of an arithmetic sequence, b is the arithmetic

mean of a and c.
Since d
b - a and d = c - b
:. b - a
i.e.
2b

=a +c

The arithmetic mean of two numbers a and c is half their sum.

EXAMPLE 4
Insert five terms in arithmetic sequence between 25 and 7.
t)
a = 25
t7 = a

25

+ 6d
+ 6d

= 7
d = -3

The five terms are 22, 19, 16, 13, 10.

11.4 Arithmetic series _ _ _ _ _ _ _ _ _ _ __


A series is the sum of the terms of a sequence. Thus

t)

t2

+ ... +

t3

tn

is an infinite series of the sequence whose terms are t), t 2, t 3, . .. , t n, . ..


However, in most situations, we are concerned with the sum of only the first n terms of
a sequence. Such a sum is called a partial sum or finite series. Thus the series
a

(a

d)

(a

2d)

+ ... +

[a

(n -

l)d]

is the sum of the first n terms of the arithmetic sequence


a,a

d,a

2d, ... ,a

(n -l)d

It is useful to be able to express this series as a formula, rather than add the successive

terms of the sequence.


Denote the sum of the first n terms by Sn and the nth term by I.
Then
Sn = a

(a

+ d) + (a + 2d) + ... +

(I - 2d)

+ (I

d)

Write this series in reverse:


Sn

(l - d)

(I

2d)

+ ...

+(a

2d)

(a

+ d) +

ll/SEQUENCES AND SERIES 237

Adding, we get
2Sn
Sn

= (a + I) + (a + I) + (a + I) + ... + (a + I) + (a + I) +
n(a + I)
+ I)

(a

I)

i(a'

2n [a +
n

= 2[2a

1) d]

(n -

(n -

since 1

(n -

1) d

l)d]

Thus the finite arithmetic series is defined by the quadratic function

S, "" q[2a

(n - l)dl. n EN.

Note the connection between Sn and tn.

tn = Sn - Sn-l' n

>

Is this obvious?

EXAMPLE 5
Find the sum of the first twenty terms of the sequence 3, 5, 7, ...
a

3, d
n

Sn = 2[2a

2, n = 20

20

and so

S20

= 2[6 +

(n -

l)d]

19 x 2]

= 440

EXAMPLE 6
How many terms of the

seri~s

+ 7 + 10 + ... must be taken to give a sum of 531?


a= 4, d = 3, Sn = 531
n

Sn = 2[2a

n
531 = 2[8

(n -

+ (n
1062 = n(3n + 5)

Thus

+ 5 n - 1062
+ 59) (n - 18)

3n 2
(3n

l)d]
1)3]

0
= 0

n = 18

since n is a positive integer.,

EXAMPLE 7
Find the sum of the first twenty terms of an arithmetic sequence given that the tenth term

is 39 and the sum of the first ten terms is 165.

Since the tenth term is 39,

then
a + 9d = 39
Since the sum of the first ten terms is 165,

then

1~[2a + 9d] = 165


2a + 9d = 33

. . . . . . . . . . . . . . . . . . . . . . . . . . . . . . . .. (1)

. . . . . . . . . . . . . . . . . . . . . . . . . . . . . . . .. (2)

238 NEW SENIOR MATHS: TWO UNIT COURSE

Subtract (1) from (2)


a

From (1)

-6

9d
9d
d

= -6
= 39

45

= 5
n

Sn = 2[2a

(n

l)d]

20

+ 19 x 5]
= 10 ( - 12 + 95)
= 830

S20 = T[ -12

EXAMPLE 8
2n 2 + n.

Find the arithmetic sequence whose series is defined by Sn

tn
Sn
Sn-I

Sn - Sn-II
2n 2 + n
and so
= 2(n - 1)2 + n
= 2n 2 - 3n + 1
Thus
~
4n - 1
tl
3, t2 = 7, t3 = 11
Hence the sequence is 3, 7, 11, ... , 4n - 1, ...

Alternative method:
Sn
S,

= 2n 2 + n
= 3 = tl

10:. t2 ==
S3 = 21 .'. t3 =
a = 3, d = 4
tn
a + (n -,
= 3 + (n -,
S2

Thus
and

10 - 3
21 - 10

7
11

l)d
1)4

4n - 1
Hence the sequence is 3, 7, 11, ... , 4n - 1, ...

EXERCISES 11 (b)
1. Which of the following are arithmetic sequences?
(a) 5,2, -1, -4,...
(b) 7, 17,27, ...

1 1 1 1
5

(c) 2' 3' 4> 5" . .


(d) 8' 1, 1~, Ii, ..
2. Find the eighth term and the fourteenth term of the arithmetic sequence 8, 14, 20, ...
3. For the arithmetic progression 17'2, 16'6, 16, ... , find ts and til'
4. The first and second terms of an arithmetic sequence are p and q respectively. Find
the tenth term.
S. Find the (n

2) th term of the sequence 14, 11, 8, ....

6. Find the arithmetic progression, where ts

17 and t12

= 52.

II/SEQUENCES AND SERIES 239

7. Find t6 of an arithmetic sequence given that t3 = 5'6 and t12

-7.

8. Find the 7th term of-the arithmetic progression whose 5th term is m and whose 11 th
term is n.
9. Find the value of p so that p
arithmetic progression.

+ 5, 4p + 3, 8p - 2 will form successive terms of an

10. How many terms are there in the sequence 9, 12, 15, ... , (6p

+ 15)?

11. Insert 5 terms in arithmetic sequence between -8 and 22.


12. In an arithmetic sequence, the first term is 6 and the fifth term is twice the fourth
term. Find the common difference.
13. If 36, 31, 26 are the first 3 terms of an arithmetic progression, and if the nth term is
-4, find the value of n.

14. Find the sum of the first 16 terms of the arithmetic sequence 3, 4!,

5i, ....

15. Find the sum of the first 12 terms of the arithmetic sequence in which the first term is
8 and the twelfth term is 41.

16. Find the sum of all integers between 20 and 50 that are divisible by 3.
17. A body, falling freely from a height, travels 49m (metres) in the first second, 147 m
in the second second and 24'5 m in the third second. How far has it fallen (a) after 6 s
(seconds), (b) during the 6th second?
18. Find the sum of the first 50 terms of an arithmetic progression, given that the 15th
term is 34 and the sum of the first 8 terms is 20.
19. Find the sum of the first 20 terms of an arithmetic progression whose 8th term is 6 and
12th term is 9.
20. Show that the sum of the first n odd natural numbers is a perfect square.
21. The first term of an arithmetic sequence is 7, the common difference is 2 and the sum
of the first n terms is 247. Find the value of n.

22. t3 = -2, t9 = 28. How many terms of this arithmetic sequence, beginning with the first
term, are required to give a sum of 1092?
23. The sum of three numbers in an arithmetic progression is 15 and their product is 105.
Find the numbers.
24. Find the sum of all the natural numbers between 0 and 101 that are (a) divisible by 2,
(b) divisible by 5, (c) divisible by 2 and 5, (d) divisible by 2 or 5 but not both.
25. The sum of the first six terms of an arithmetic sequence is -12 and the sum of the first
fourteen terms is 196. Find (a) the sum of n terms, (b) the smallest value of n if the
sum is to exceed 250.
26. The sum of the magnitudes of the angles of a pentagon (5-sided polygon) is 540 0 The
magnitudes of the angles form the terms of an arithmetic progression. If the largest
angle has a magnitude of 136 0 , find the magnitude of each of the other four angles.
27. The sum of the first four terms of an arithmetic sequence is 34 and the sum of the next
four terms is 14(). Find the sum of the 9th and 10th terms.

240 NEW SENIOR MATHS: TWO UNIT COURSE

28. The rungs of a ladder decrease uniformly in length from 40 cm (centimetres) at the
bottom rung to 30 cm at the top rung. How many rungs are there if their total length
is 525 m?
29. How many terms of the series 7

11

+ ...

must be added to give 352?

30. The first three terms of an arithmetic sequence are 45,41,37. If the nth term is 1, find
the value of n and the sum of these n terms.
31. The track of a gramophone record is in the shape of a spiral curve and may be
considered as a number of concentric circles of inner and outer radius 525 cm and
10'5 cm respectively. The record rotates at 33i rev/min and takes 18 minutes to play.
Find the length of the track. (Take

7r

32. For the series function defined by Sn


sequence is arithmetic.

= 2;.)

3n 2

11 n, find tn and hence show that the

33. A series is such that the sum of its first n terms is n(3n + 2) for all n EN. Prove that
the sequence is arithmetic and find the eighth term.
34. The sum of the first nine terms of an arithmetic sequence is 81. If the sum of the first
and third terms is zero, find the first term and the common difference.
35. Find the sum of all integers between 200 and 400 that are divisible by 6.
36. The first and second terms of an arithmetic sequence are a and b respectively. If the
nth term is c, express n in terms of a, band c, and hence find the sum of these n terms.
37. Find the first three terms of an arithmetic sequence in which the fifth term is three
times the second term, and the sum of the first six terms is 36.
38. Logs of wood are stacked in a pile so that there are 15 logs on the top row, 16 on the
next, 17 on the next, and so on. If there are 246 logs altogether,
(a) how many rows are there;
(b) how many logs are on the bottom row?
39. The lengths of the rungs of a ladder increase uniformly from 40 cm in the top rung to
75 cm in the bottom rung. If 13'8 m of wood are used to make the rungs, how many
rungs are there?
40. The lengths of the sides of a right-angled triangle form the terms of an arithmetic
sequence. If the hypotenuse is 15 cm in length, what is the length of the other two
sides?
41. How many terms of the series 6

10

42. The sum of the first n terms of the


possible values of n.

+ ... must be taken to give a sum of 880?


series 30 + 26 + 22 + ... is 120. Find two
14

43. Find the sum of (a) the first n odd positive integers (b) the first n even positive integers
(c) the first n positive integers and find this value of n if the sum is 210.
44. For a potato race, a straight line is marked on the ground from a point A, and points
B, C, D, ... are marked on the line so thatAB = BC CD
... = 2 metres, and a.
potato is placed at each of the points B, C, D, . ... A runner has to start from A and
bring each potato by a separate journey back to a basket at A. Find the number of
potatoes so that the total distance run during the race will be 480 metres?

ll/SEQUENCES AND SERIES 241

45. Cans of fruit in a supermarket display are stacked so that there are 3 cans in the top
ro~, 5 in the next row, 7 in the next and so on. If there are 10 rows in the display, find
(a) the number of cans in the bottom row,
(b) the total number of cans in the display.

46. The first term of an arithmetic sequence is 5. The ratio of the sum of the first four
terms to the sum of the first ten terms is 8 : 35. Find the common difference.
47. The angles of a hexagon form the terms of an arithmetic sequence. If the smallest
angle is 95, find the size of each of the other angles.

11.5 Sigma notation I: _ _ _ _ _ _ _ _ _ __


The symbol E is the Greek letter sigma, corresponding to the English letter S and is used in
mathematics to denote 'the sum or.
5

1;xn denotes the sum of a numbe,r of terms, each of the form xn and we give n the
n=l

values 1, 2, 3, 4, 5.
5

Thus

1; xn

+ X2 + x 3 + X4 + x 5

n=l

1; (2k +

1) denotes the sum of a number of terms each of the form 2k

+ 1 for

k=O

k = 0, 1, 2, 3, ... n.
n

Thus

1; (2k +

+ 3 + 5 + ... +

1) = 1

(2n

+ 1)

k=O
10

1;rxr -

2x

3x2

+ ... +

10x9

r=1

In reverse,
n

12

22

32 +

... + n 2

1;k2
k=l
10

1 . 2 + 2. 3

+ 3.4 + ... + 10. 11

1;n(n

1)

n=1
8

2x

2X2

+ ... +

1; 2xk

2x 8

k=O

The arithmetic series we have just considered

(a

+ d) +

(a

d)

(a

2d)

+ ... +

(a

(n -

1) d)

is the sum of n terms each of which has the form of the nth term a + (n - l)d for values
of n = 0, 1, 2, 3, ... n and can be expressed very neatly using the sigma notation.
n

Thus a

(a

+ 2d) + ... +

(a

(n - 1) d) =

1; a + (k
k=1

- 1) d

242 NEW SENIOR MATHS: TWO UNIT COURSE

EXAMPLE 9
8

Evaluate 'E(1

4k)

k=1
8

'E(1

= 5 + 9 + 13 + ... + 33

4k)

k=l

The R.H.S. is an arithmetic series with a = 5, d = 4, n


n
Sn = 2(a + l)

= 2(5 + 33)

S8

152
8

:. 'E(1

4k) = 152

k=l

EXERCISES 11 (c)
1. Write out expansions of the series defined by:
p

(ii) 'E3r

(iii) 'E2k
k=1

r=O
8

(iV) 'Ek(k

1)

'-'X
I
r

n+1

1)2

(viii) 'E(3k - 2)

(ix) 'E Xk

k=1

k=O

(vi) '\"""r

r=1

k=1

(vii) 'E(2k

k.

(v) 'Erxr

k=1

2. Use sigma notation to express each of the following. (There can be more than one
form.)
(i) 1 + x + X2 + ... + x lO
(ii) 12 + 22 + 32 + ... + 92
(iii) 1.3 + 2.4 + 3.5 + ... + 10. 12
(iv) 1 + 6 + 11 + ... + (5p - 4)

1
1
1
1

(v) 2.3 + 3.4 + 4.5 + ... + pcp + 1)


(vi) 2X2 + 3x 3 + 4X4 + ... + 12x l2
(vii) a + ar + ar 2 + ... + ar n - 1

3. Show that each of the following represents an arithmetic series and hence evaluate:

1)

n
k=1

(ii) 'E(4k
k=1

k=1

(iv) 'E(6k

10

(i) 'E(3k - 7)

2)

(v) 'E(2k - 1)
k=1

(iii) 'E(4k - 1)
k=l
10

(vi) 'E(3k

2)

k=l

11.6 Geometric sequence (or progression) - -


A geometric sequence or progression is a sequence ofterms in which each
term after the first is formed by multiplying the preceding term by a
constant number, called the common ratio.

-,
II/SEQUENCES AND SERIES 243

The sequence
a, ar, ar 2 , ar 3 , , arn - I, ..
is called a geometric sequence, where a and r are real numbers and n is a natural number.
a is the first term and r the common ratio.

A geometric sequence may also be defined by the rule

Examples of geometric sequences are:


1. 2, 6, 18, 54, ...
2. 3, -15, 75, -375, ...
3. 18, 9, 4~,
4. 3,
5. x,

21, ...

3-J'i, 6, 6-J'i, ...


X2,

x3, . ..

a = 2, r = 3

a = 3, r = -5
1

a = 18, r = 2"

a = 3, r = -J'i
a = x, r = x

EXAMPLE 10
Write down the geometric sequence whose third and sixth terms are 12 and 96
respectively.
t3
ar 2 = 12 ......................... " (1)
(2)
t6 = ar s = 96
Divide (2) by (1).
ar S
96
ar2
12

r3
r
From (1)

= 8
= 2

a x 4
a

12

=3

The sequence is 3, 6, 12, ...

11.7 Geometric mean _ _ _ _ _ _ _ _ _ _ __


If a, band c are three consecutive terms of a geometric sequence, b is the geometric mean

of a and c.
By definition

The square of the middle term equals the product of the other two terms.

EXAMPLE 11
Insert three terms between 2 and 162 so that the five numbers form a geometric sequence.
Show that there are two possibilities.
The first term is 2 and the fifth term is 162.
i.e. a
2 and ar4 = 162.
:. r 4 = 81
r = 3
The terms are 6, 18, 54.

244 NEW SENIOR MATHS: TWO UNIT COURSE

11.8 Finite geometric series _ _ _ _ _ _ _ _ __


As in the case of the arithmetic sequence, a finite geometric series is the sum of the terms
of a finite geometric sequence.
Thus the series a + ar + ar 2 + ... + arn - 1 is the sum of the first n terms of the
geometric sequence a, ar, ar2 , , ar n- 1
Similarly, it is useful to be able to express this series in a convenient form, rather than
add the successive terms of the sequence.
Denote the sum of the first n terms by Sn- Then
n

+ arn- 1 = 'Eark- 1

= a + ar + ar 2 +

Sn

............ (1)

k=l

Multiply both sides of (1) by r.

+ arn- 1 + arn . ................. (2)


ar + ar 2 +
Sn - rSn = a - arn
i.e. Sil - r) = a(l - rn)
/

rSn
Subtract (2) from (1):

I Sn = a(:

=;nl j

Use when r < 1.

Or, subtracting (1) from (2), we get

Sn

a(rn - I)
r- 1

Use when r > 1.

Thus the finite geometric series is defined by the


1)
S = a(I - rn) or a(f'/
n
-----
- r
r

EXAMPLE 12
Find the sum of the first eight terms of the geometric sequence 3, 6, 12, ....

Sn = a(rn -11) where a


r

3(2n 2 = 3(2n :. S8 = 3(28 -

3, r

= 2, n

1)

1
1)
1)

765

EXAMPLE 13
For the series 4

+ ... , find S6 andS 1o '

S = a(I - rn) where a


n
1 - r
_ 4[1 - (!)n]
1 - ~

=8[I-orJ

4, r =

2'

II/SEQUENCES AND. SERIES 245

Fig. 11-2 is the graph of Sn = a(:


S, = 8 [1 -

GrJ

1.I.e., It
..IS t h e grap h 0 f
2;

rn) where a = 4 and r

- r

wher"sn EN.

8
6

----------;--.--.-.

Fig. 11-2

11.9 Infinite geometric series _ _ _ _ _ _ _ __


In example 13, we observe that, for a = 4, r =
Sl = 4
82 = 6

S5
S6

S3 = 7

S7

S4 =
It appears that, as n increases, Sn

7!

S10

1
2'

7l
=

7~
7I~

= 7:U

7 + a fraction less than 1, and fig. 11-2 above


shows that as n increases indefinitely, i.e. n - 00 (n approaches infinity); Sn - 8.
We define Sco as n_oo
lim Sn. That is,
Sco
8
Consider a piece of string 8 cm long; it is cut so that the first piece cut off is 4 cm long.
The remainder is then cut in half, so that the next piece is 2 cm long. The remainder is then
cut in half, so that the next piece is 1 cm long, and so on. It is conceivable that there will
always be a piece left over, however small, to be cut in half, and, as the number of pieces
being measured becomes increasingly large, their total length will get closer to 8. That is,
.

Soo = 4

+ 2 + 1 + 2 + 4 + ...

We have seen that


Sn

a(1 - rn)
1 -r

a
..

If

Irl <

arn

=I-r-

1, i.e. if r lies between -1 and 1, rn - 0 as n -

-r
00.

246 NEW SENIOR MA THS: TWO UNIT COURSE

(~)n

n
1
4
10
-00

As n -

00,

(!)

o.

Thus lim rn = 0, and so lim


n-oo
n-oo

arn
-r

IS~
Soo does not exist if

1
2
1
16
1
1024
-0

Irl

o.

~ if ITI < 1 I

1. Why?

EXAMPLE 14
I .

A rubber ball is dropped from a height of 20 m. Each time it strikes the ground, it

~ of the distance of the previous fall. Find the total distance it travels.
3
4'
n - 00.
For downward motion, a = 20, r

rebounds

00

a
-r

20
1 -

=
For

~pward

motion, a = 15, r

80

4' n Soo

00.

15

1 _ ~
4

= 60

Total distance = 80 m

60 m

140 m

11.10 Recurring decimals _ _ _ _ _ _ _ _ __


The infinite geometric series formula can be used to express a recurring decimal as a
rational number.

EXAMPLE 15
Find the fractional equivalent of (a)

) o 23 =

0232323 ...

+ 00023 +

= 023

023, (b) o 57.

II/SEQUENCES AND SERIES 247

where a

0'23, r = 001

023
1 - 001

23

99

(b) 0'57 = 0'5777 ...


0'5 + 007

+ 0007 +

O' 5

+ a where a
r

0.5

+ 007

= 2

0'07, r = 01

1 - 01

90

52
90
26
45

EXERCISES 11 (d)
1. Which of the following are geometric sequences?
(a) 3, 6, 12, ...

1
1

(b) 8, -2,

2' -8""

(c) 2, 5, 11, 23, ...


1 1
(d) 9' 3,1,3, ...

111 1

(e)

2' 4' 6' 8""

2. Find (a) the 6th term and (b) the sum of the first 6 terms of the sequence 4, 6, 9, ...
3. Find the first term and common ratio of a geometric sequence, given that t3
t5 = 156i.

= 25 and

4. Find the first term of the geometric progression whose 6th and 7th terms are 3; and
64
. 1y.
27 respectIve

5. Find the first three terms of a geometric sequence in which


(a) the sixth term is 160 and the seventh term is 320,
(b) the fifth term is 4 and the eighth term is
6. If 2p + 1, 5p and 12p
value of p.

-k.

4 are successive terms of a geometric sequence, find the


I

7. Find (a) the 10th term a:pd (b) the sum of the first 10 terms of the sequence 8, -4, 2, ...

8. Find the sum of the giveh,number of terms in each of the following:


(a) 1 + 2 + 4 + ... (8 terms)
(b) 125

+ 25 + 5 + ... (7 terms)

248 NEW SENIOR MATHS: TWO UNIT COURSE

(6 terms)

(c)

(d) 1 + 11 + 121 + ...


(e) 100

(lOterms)

+ 100(0'9) + 100(0-9)2 + _..

(8 terms)

9. 2k + 2, 5k + 1 and 10k + 2 are tl)ree successive terms of a geometric sequence. Find


the value of k.
10. For the sequence 2, 2.[3, 6, ... find (a) the 10th term, (b) the sum of the first 10

terms.

11. Evaluate the series 16

8 + 4 ~ 2 + ... + 16'

12. Find the sum of 9 terms of the series 3 + 34/ 3 + 35/3 +

13. The

fi~st 4 terms of a sequence are x, iX2, ~X3,

287x4. Find, from first principles, the

sum of n terms, and evaluate the sum when n

6 and x

3.

14. Find x and y, given that 1, x and yare in arithmetic sequence, and 1, y and x are in
geometric sequence.
15. The first two terms of a geometric sequence are
+ .J5 and ..J7
simplest surd form with rational denominator, the common ratio.

.J5.

~jnd, in

16. The population of a certain town is 10000; if its population will decrease each year by
10 per cent of its population in the preceding year, find its population in 5 years' time.
17. The value of a car when new is $3000. If it depreciates at the rate of 15 per cent of its
value at the beginning of each year, find its value after 6 years.
18. A pump removes one-half of the water remaining in a tank in 10 minutes. What
fraction of the original amount of water will be left in the tank after one hour?
19. A pump removes one-quarter of the water from a tank every 15 minutes. If the tank
initially holds 256000 litres, how much water will remain in the tank after one hour?
20. How many terms of the series 6 + 3 +

~ + ...

must be taken to give a sum of

11 : ~? What is the sum of an infinite number of terms?


21. Evaluate the following:
(a) 8
4 + 2 - ...
(c) 25 -

(b) 4 + 3 + 2~ +
(d) (.[3

10 + 4 +

22. If 1 + 2x + 4X2 + ...

~,

+ 1) + 1 + ~(.[3 - 1) +

find the value of x.

23. Find the first three terms of a geometric sequence given that the sum of the first four
terms is 21j, and the sum to infinity is 27.
24. The three numbers a, band c, whose sum is 15, are successive terms of a geometric
sequence, and b, a and c are successive terms of an arithmetic sequence. Find the
values of a, band c.
25. Insert 4 terms in geometric sequence between

~ and

128

II/SEQUENCES AND SERIES 249

26. How many terms of the sequence 6, -12, 24 ... are required to give a sum of 1026?
27. The first term of a geometric sequence is 8 and the sum to infinity is 32. Find the
common ratio.
28. Find a number which when added to each of -1, 1 and 5 gives three terms in geometric
sequence.
29. 2m - 8,2m

+ 4 and 5m - 2 are successive terms of a geometric sequence. Find the

values of m.
30. The sum of the first 8 terms of a geometric series is 17 times the sum of its first 4
terms. Find the common ratio.
31. The sum of the first 4 terms of a geometric series is 30, and the sum of the infinite
series is 32. Find the first three terms.
32. Find the sum of the series 12 + 8 +
33. Find the sum of the series 1 +

5t +

....
1

1
a+
+ (a + 1)2 + .... For what range of values of

a has this infinite series a sum?

34. For the geometric sequence a, ar, ar2, ... show that the sequence log a,
log (ar2) , . " is an arithmetic sequence.

lo~

(ar),

35. Find the sum of 10 terms of the series logio 3 + 10giO 6 + logio 12 + ... , given that
logio 3 = 04771 and logio 2 = 0'3010.
36. Find, without using tables or a calculator, the sum of 7 terms of the series 10giO 27
logio 9 + 10glO 3 + ....
37. For the geometric sequence
sum of the infinite series.

.J5 +

..j3,

.J5

..j3, . .. , find the common ratio and the

38. Find the fractional equivalent of (a) 2-38, (b) 4'62 and (c) 041717 ....
39. 0323232 ...

~, wherep and q are integers with no common factor. Find the value

ofp and q.

40. Show that 1'2888 ... is a rational number by expressing it in the form m where m and
n
n' are integers with no common factor.

42. Find a number which, when added to each of 2, 6, 13 gives three numbers in geometric
sequence.

11.11 Compound interest


A practical example of a geometric sequence is the growth of money invested at
compound interest. If $100 is invested at compound interest of 8 per cent per annum at the
beginning of a certain year, its magnitude is:

-....

---..

~-----

250 NEW SENIOR MATHS: TWO UNIT COURSE

x 108 .at the end of the first year,


x (1'08)2 at the end.of the second year,
x (1'08)3 at the end of the third year,
x (1'08)n at the end of n years.
In general, if $P is invested at compound interest of r per cent per annum, it grows to
r
PR at the end of the first year, where R
+ 100'
$100
$100
$100
$100

PR 2 at the end of the second year,


PR 3 at the end of the third year,
PRn at the end of n years.

These form the terms of a geometric sequence, whose common ratio is R.

The compound interest formula then is

where P is the initial amount, An is the amount that P grows to after n periods of time,
interest .being charged at rftJo per period. The period need not necessarily be one year.
18ftJo per annum
9ftJo per 6 months
= 4!ftJo per 3 months
= l!ftJo per month etc.

EXAMPLE 16
At the beginning of each year a man invests $500. Calculate:
(i) the magnitude of his first investment at the end of 10 years,
(ii) the accumulated value of his investments at the end of 10 years, reckoning
compound interest at 8 per cent per annum.
(i) Number of dollars at end of first year

Number of dollars at end of second year


Number of dollars at end of tenth year

= 500 x 108

x (1'08)2

500 x (1'08)10

= 500 x 215892 (using a calculator).


107946
Magnitude of first investment at end of 10 years is $1079'46
= 500

(ii) Value

of last investment at end of tenth year = $500 x 1'08


Value of second last investment at end of tenth year = $500 x (1'08)2
Value of first investment at end of tenth year = $500 x (1'08)10
Thus the sum of his ten investments = 500(1'08) + 500(1'08)2 + ... + 500(108)10.
This is a finite geometric series of ten terms whose first term is 500( 1'08) and
common ratio 108.
a(RIO - 1)
.R - 1

1'08[1.08 10 -1]
108 - 1
500 x 1-08 x 1'15892
500

= 782271

He has $7822'71 at end of ten years.

(using a calculator)

11/SEQUENCES AND SERIES 251

Thus, if a person invests $P at the beginning of each period of time at r per cent per
period compound interest, the sum of his investments at the end of n periods of time
PR + PR2
= PR(1 + R
= PR(Rn -

+ PR3 + ... + PRn


+ R2 + .. _ + Rn-I)
1) where R = 1

R - 1

100

EXAMPLE 17
A person borrows $5000 and undertakes to repay $100 at the end of each month reckoned
from the date of the loan. Interest is charged on the unpaid debt at I! per cent per month.
How much does he owe after the 8th repayment?
.

R = 1

+~
=
100

1-015

Number of dollars owing after first repayment


= 5000(1-015) 100
Number of dollars owing after second repayment
= [5000(1'015) - 100]1'015 - 100
= 5000(1-015)2
100(1 + 1-015)
Number of dollars owing after third repayment
[5000(1-015)2 - 100(1 + 1-015)] 1015 - 100
= 5000(1-015)3
100(1 + 1015 + 1-0152)
If we observe the pattern established, then the number of dollars owing after the 8th
repayment
= 5000(1'015)8 - 100(1 + 1015 + 1'0152 + ... + 1-0157)
= 5000(1-015)8 _ 100(1-015 8 - I)
1-015 - 1

= 5000 x 1-12649 - 100 ~_~'~2649 (using a calculator)


= 563245 - 843-26
4789-19
He owes $4789 after 8th repayment.
Thus if a person borrows $P and repays $Q at the end of each period of time reckoned
from the date of the loan at r per cent per period, then:
Number of dollars owing after first repayment
= PR - Q
Number of dollars owing after second repayment

= (PR
= PR2

- Q)R -

Q(I + R)
Number of dollars owing after third repayment

[PR2 - Q(I+ R)]R = PR 3


Q( 1 + R + R2)

Thus, number of dollars owing after nth repayment


= PRn - Q(1 + R + R2 +
= PRn _ Q(Rn -

R - 1

1)

-+

Rn -1)

252 NEW SENIOR MATHS: TWO UNIT COURSE

11.12 Annuities _ _ _ _ _ _ _ _ _ _ _ _ __
An annuity (from the latin word annus: a year) is a series of payments made at equal
intervals of time; the payments may be made yearly, half-yearly, quarterly or at more
frequent intervals. Annuities may be considered in two ways:
(i) A person, as in example 17, borrows a certain sum of money and undertakes to repay

the debt in regular instalments and interest is charged on the part of the debt
outstanding. The instalments include repayment of part of the capital and the interest.
(ii) A person invests a lump sum of money with, say, an insurance company or other

investment agency and in return receives regular payments over a number of years. In
principle, (i) and (ii) are the same.

EXAMPLE 18
A person invests $5000 on condition that he is repaid the money in 10 equal quarterly
instalments. If interest is received at the rate of 4 per cent per quarter, what is the amount
of each instalment?
As in example 17, if $P is invested and $Q is repaid at the end of each quarter, interest
at the rate of r per cent per quarter, the amount remaining to his credit at the end of n
quarters is:
Q(Rn - 1)
R - 1

5000, R = 1'04, n = 10 and so:


5000 x 1'04 10

10

Q(I'04
0,04

1)
Q

0
5000 x 104' 0 x 004
1.0410 - 1
= 61649 (using a calculator)

:. A mount of each instalment is $61649.

EXERCISES 11 (e)
1. In 1976, 500 students entered for a particular examination. This number increased
each year by 20 per cent of the number who entered the previous year. Calculate
(a) the number who entered in 1981, (b) the total number who entered between 1976
and 1981 inclusive.

2. If the amount of wheat harvested in a certain area in 1982 is 42 thousand tonnes and
it is anticipated that this will increase annually by 25 per cent, estimate
(a) the amount to be harvested in 1990,
(b) the total amount to be harvested over the 9 years from 1982 to 1990 inclusive.
3. At the beginning of 1981, a mining town had a population of 15000. It was estimated
that this would increase each year by 8 per cent of its population at the beginning of
that year. What should the population be at the beginning of 1985?
4. The value of a motor car when new is $8000. Its value depreciates each year by 15 per
cent of its value at the beginning of that year. After how many years will its value be
$4000?

II/SEQUENCES AND SERIES 253

5. A man contributes towards providing a pension a sum of $500 annually on each of his
44' birthdays from his 21st to his 64th inclusive. Find the accumulated value of these
44 contributions on his 65th birthday,. assuming that the money is invested at 10 per
cent per annum compound interest.
6. John borrowed $2000 on 1st January 1980. He agreed to pay back $300 on 1st
January in each succeeding year and to add to the debt 6 per cent per annum interest
on the amount owing during the year just completed. Find (a) the amount still owing
immediately after 1st January 1985, (b) the number of years necessary to free himself
of the debt.
7. Jan borrows $8000 and agrees to repay it in equal instalments each year for 10 years.
If interest is charged at 7 per cent per annum on any part of the unpaid debt, calculate
the amount of each yearly instalment.
S. A man is to receive six payments of $500 each, the first payment in a year's time and
subsequent payments at two-yearly intervals. He invests each payment at 9 per cent
per annum. How much will he have immediately after the last payment is made?
9. What sum of money should be invested now to provide $2000 at the end of 10 years,
assuming interest at the rate of 8 per cent per annum for the first six years and 10 per
cent thereafter?

10. A married couple borrows $50000 in order to purchase a house. They agree to repay
the loan in equal quarterly instalments for 12 years. What is the amount of each
instalment, if interest is charged at the rate of 10 per cent per annum on any money
owing?
11. A person borrows $5000 at I!OJo per month reducible and pays it off in equal monthly
instalments. What should the instalments be in order to payoff the loan at the end of
3 years?
12. A firm is established with new equipment and creates a fund to provide for the
replacement of the equipment after 6 years at an estimated cost of $20000. How
much should be paid into the fund annually if interest at 11070 can be obtained?

13. A family borrows $30000 from a building society with interest at II!OJo p.a.
(a) How much must be repaid each year if the loan is to be repaid over 30 years?
(b) How much of the loan will be outstanding at the end of the 18th year of the loan?
(c) If repayments are made at the rate of $4500 per year, over what period will the
loan be repaid?
14. A person borrows $10000 and agrees to repay the loan in equal instalments over 20
years. Interest is 12070 per annum on any money owing
(i) What is the amount of each repayment, if the -repayment are made (a) annually
(b) quarterly (c) monthly (d) weekly.
(ii) Which is the cheapest of the four methods of repayment in (i)?

CHAPTER 12

Quadratic

Functions

12.1 Quadratic functions ~_ _ _ _ _ _ _ __


A function defined by the rule

where a, band c are constants, a

=1=

0, is called a quadratic function.

Its domain is R, the set of all real numbers, unless otherwise stated or implied.

y
2X2 - 3x - 5 for every real number x

for every real number t

y = f2 + 2t
y = 4p2 - 8
for every real number p

are examples of quadratic functions with domain R.


However, the area function A of a square of side length measure s, defined by the rule
A

S2

is a quadratic function whose domain is the set of all positive real numbers.

12.2 Graph of quadratic function _ _ _ _ _ __


The graph of a quadratic function has the characteristic shape of a parabola. At its vertex,
the parabola has a turning point and at this point, the function has a minimum value if
a > 0 or a maximum value if a < O.
y

Fig. 12-1 a > 0

'---

Fig. 12-2 a < 0

12/QUADRATIC FUNCTIONS 255

12.3 Maximum or minimum value of


quadratic function ____________
All quadratic polynomials can be expressed in the form
a(x

+ B)2 +

C,

where a, Band C are constants, a =1= 0. This form is often called 'the completing of the
square' form. Since (x + B)2 is a perfect square, it is non-negative for all real values of x
and the least value it can assume is zero, when x = -B. Hence
(i) if a > 0, the minimum value of a(x + B)2 + C is C;
(ii) if a < 0, the maximum value of a(x + B)2 + C is C.

EXAMPLE 1
Express 2X2
4x - 5 in the form a(x
and hence state its minimum value.

+ B)2 +

2X2 - 4x - 5

2 [X2 - 2x -

~ ] Take the coefficient of X2 outside the square bracket.

2 [(x 2 _ 2x

= 2[<X-l)2

1) _ ~ _ IJ Complet: ~he square by adding the square of half


2
the coeffIcIent of x.

;J

= 2(x -1)2 - 7

Hence the minimum value of 2X2 - 4x - 5 is -7 and it occurs when 2(x - 1)2 has its least
value, namely zero, when x = 1.
y

(1,-7)

Fig. 12-3
The graph of the function y
2X2 - 3x - 5 is shown in Fig. 12-3.
Hence the range of the function is y ~ -7. The graph is symmetrical about the line
1. This line is called the axis of symmetry. The coordinates of the
with equation x
vertex are (1, -7).

EXAMPLE 2
Find, by the method of completing the square, the maximum value of -3x2 - 12x - 7

256 NEW SENIOR MATHS: TWO UNIT COURSE

~3X2

-3

- 12x

G2 + 4x + ~ ] Take the coefficient of out~ide the square bracket.


X2

-3 [ (X2

[<x

-3

-3(x

+4; + 4) + ~ -

4] Complete the square.

+ 2)2 ~ ]
+ 2)2 + 5

Hence the maximum value of the function y = -3x2 - 12x - 7 is 5 and it occurs when
-3(x + 2)2 has its greatest value, namely zero, when x = -2.
The graph of the function is shown in Fig. 12-4.
The range of the function is y ~ 5. The axis of symmetry is the line x = -2. The
coordinates of the vertex are (-2, 5).
y

Fig. 12-4

EXAMPLE 3
A piece of wire of length 12 cm is bent in the shape of a rectangle. Find the maximum area
of the rectangle.
Let OI;le dimension of the rectangle be
x cm and hence the other dimension will be
(6 - x) cm. For any given x in the domain
o < x < 6, the area function is defined by
the rule
y
x(6 - x), 0 < x <6
x
6x - X2
Fig. 12-5
- [(X2 - 6x + 9) - 9]
= - (x - 3)2 + 9 .
3; in which case, the rectangle is a square.
Hence the maximum area is 9 cm2 when x
The graph of the area function is shown in
y
Fig. 12-6.
Domain is 0 < x < 6
9

Range is

Fig. 12-6

'--

12/QUADRATIC FUNCTIONS 257

EXERCISES 12(a)
Express each of the following functions in the form
y = a(x + B)2 + C
and hence find their maximum or minimum value and the range in each case (1. to 10.)
Y
2X2 - 4x
y = 7 + 16x
4X2
y = 8 - 2X2
y = 2X2
6x
10. y = -x2 + X + 11
2.
4.
6.
8.

Y
X2 - 2x +6
y = -2x2 + 8x - 3
y = 4X2 + 8x - 7
y = 7 - 2x - X2
y = 6
lOx - 5x2

1.
3.
S.
7.
9.

11. A stone is projected vertically upwards from the ground. The height, h(t) m, above
the ground is a function of time t s (t ~ 0), with rule
h(t) = 20t
5t2

Find (a) the domain of h,

(b) the greatest height reached.


12. The equation of the path of a cricket ball thrown at an angle of 45 with the
horizontal is
y

where x m and y m are the horizontal distance travelled and vertical height
respectively.
Calculate the greatest vertical height reached and the horizontal distance travelled.
13. The sum of two numbers is 20. Find the numbers and their product if their product is
a maximum.
14. A piece of wire of length 60 cm is bent in the shape of a rectangle. Find the maximum
area of the rectangle.
15. A man wishes to form a rectangular enclosure using his existing fence as one side. If
he has 20 metres of fencing material available to form the other three sides, find the
area of the largest enclosure he can form and its dimensions.
16. A piece of wire 6 metres long is cut into two parts, one of which is used to form a
square, and the other to form a rectangle whose length is three times its width. Find
the lengtnsof the two parts if the sum of the areas is a minimum.
17. A large open area is to have a portion surrounded by a rectangular fence. This
rectangle is then divided into six rectangles by one dividing fence parallel to its length,
and two parallel to its breadth. If the total length of fencing available is 1200 m, find
the maximum possible area.
18. A machine comes in two sections. The weights of the sections are x kg and bkg. The
cost, c, of the machine (in dollars) is given by c = 2x + b. The earning capacity, y, of
the machine is given by y = x(x + b). If c has the fixed value 10, express y as a
function of x, and hence find the value of x for which y is a maximum. Find the
maximum value of y.

258 NEW SENIOR MATHS: TWO UNIT COURSE

19. ABCD is a square of unit length and points E and F are taken on the sides AB and AD
respectively such that AE = AF = x. Show that the area, y, of the quadrilateral
CDFE is given by y

= ~(l + x

X2).

What is the greatest area the quadrilateral can

have?

12.4 Quadratic equations _ _ _ _ _ _ _ _ _ __

Let us solve the general quadratic equation and discuss the nature of its roots.
ax2 + bx + c
0
X2

bx

X2

+ c = 0 (dividing both sides by a)


a

+ bx
= _ ~ (adding
a
a

X2 + ~x + (:ci)2 (:0)2

(x + :a)2 b2 4ac

c to both sides)
a

(completing the square)

b
.Jb2 - 4ac
(taking square roots)
x + 2a =
-b + .Jb 2 - 4ac
-b - .Jb2 - 4ac
x
2a
or
2a
-b-+-.J8
-b
.J8
- or ---:::--
where 8 (delta); called the discriminant, is equal to b 2 - 4ac.
Thus, the roots of the equation ax 2 + bx + c
0 are
b +.J8
b - .../8
2a
Case 1
If 8 > 0, .J8 is a real number and so the roots consist of 2 real numbers.
Case 2
If 8 = 0, .J8
this case is - b

0 also, and so the roots consist of one real number only. The root in
We say that the equation has one root only or two equal roots.

Case 3
If 8 is a perfect square, .J8 is a rational number and so the roots consist of two rational
numbers.
Case 4
If 8 < 0, .J8 does not exist in the field of real numbers and so there are no roots.

12.5 Intersection of parabola and the X-axis - -


The roots of the equation
ax 2 + bx + c = 0
are the x-values of the points of intersection of the graph of the parabola
y = ax2 + bx + c

12/QUADRATIC FUNCTIONS 259

and the X-axis. In other words, at the points where the parabola cuts the X-axis, Y = O.
Hence the parabola
(j) cuts the X-axis at two distinct points, if d > 0

(ij) touches the X-axis at one point only, (or two coinciding points) if d
0
(iii) does not intersect the X-axis, if d < O.
y

x
6> 0

6 = 0

a > 0

a > 0

6 < 0
a > 0

Fig. 12-7
In the above parabolas, the coefficient of X2 is positive and so the vertex is downwards.
y

6> 0

a < 0

6=0

a < 0

6 < 0

a <0

Fig. 12-8

In the above parabolas, the coefficient of X2 is negative and so the vertex is upwards.

EXAMPLE 4
Write down the discriminant of each of the following quadratic equations and hence state
whether the associated parabola cuts the X-axis at two, one or no points.
(i) X2 - 5x + 6
0
y
(ii) X2 - 4x + 4 = 0
(iii) 2X2 - 3x + 3 = 0
b2

(i) d

25

4ac

24

= 1

i.e. d > 0, and so the equation


has two real roots. Hence the graph of

Y
X2 - 5x + 6 cuts the X-axis at two

distinct points (Fig. 12-9).

Fig.< 12-9
Y

X2 -

5x

260 NEW SENIOR MATHS: TWO UNIT COURSE

(ii) ~ = b 2

- 4ac
16 - 16
0, and so the equation has

one root only (or two coinciding roots)

and the graph of y = X2 - 4x + 4

touches the X-axis at one point only

(Fig. 12-10).

o
y
(iii) ~

b2

4ac

Fig. 12-10
4x + 4

= X2

9 - 24

= -15

i.e. .l < 0 and so the equation

has no real roots. The graph of y = 2X2

- 3x + 3 does not intersect the X-axis

(Fig. 12-11).

Fig. 12-11
2X2 - 3x

Note: The value of the discriminant merely determines whether the associated parabola
crosses the X-axis or not. To find the actual points of intersection (if they exist), it is
necessary to solve the quadratic equation.

EXAMPLE 5
Solve the following equations
(a) 3x 2 - llx - 4
0
(c) X2 = 2x + 5
(a) .l

20x

(b) 4X2

(d)

2X2

+ 25

= 0

3 = 0

169. Since the discriminant is a perfect square, 3x2


factors that can be found by trial.
3x 2
llx
4 = 0
(3x + 1)(x - 4) = 0

11x - 4 has rational

1
x = -- or 4
3

(b).l
O. Since the discriminant is zero, 4X2
20x + 25 has rational factors, both of
which are the same, and which can be found by trial.
4X2 - 20x + 25 = 0
(2x - 5)(2x - 5)
0
x = 2!

(c) .l = 24. Since the discriminant is positive but not a perfect square, X2
2x - 5 has
irrational factors. In this case, it is advisable to solve the equation either by using the
formula or by the completing of the square.

12/QUADRATIC FUNCTIONS ,261

X2 -

or

2x - 5 = 0

- b ",J b 2
x=
2a
2 ffi

4ac .

X2 X2

2x
(x -

2 2.J6

2x

+ I =5+
1)2

x-I=.J6
x

=1.J6

(d) Since Ll < 0, there are no real numbers for which 2X2

+x +

I .J6

O.

EXtRCISES 12(b)
1. Calculate the discriminant of each of the following equations and hence state whether
the\equations have two, one or no roots.
(a) ~2' + 6x + 2
0
(b) 2X2 + 3x + 4 = 0
(c) J4X2 - 12x + 9 = 0
(d) -3x2 + 2x
I = 0

~3 2X2 = 3x + 7
(f) 4X2 - 20x + 25
0

2. Without sketching the graphs of the following functions, determine whether they
crqss the X-axis or not.
(a); y = X2 - 5x + 2
(b) y = -4x2 + 2x - I
(c) Y
X2 - 6x + 9
(d) y = 8 - 3x - 2X2
(f) Y
-x 2 - X - I
(e) y = 3x 2 + 2x + 5
Calculate the discriminant of each of the following equations to help you to decide the
technique to use to solve the equations. Hence solve them (3. to 22.)

3. X2 + 2x
15
5. 12x2 = 25x - 12
7. 7x2 = 63

4. X2 - 9x - 5
0
6. 4X2 - 12x + 9 = 0
8. X2 - 6x = 0

9. (x + 1)2 = 4x
11. 2X2 - x = 5
13. 3x2 - 7x
3 = 0
15. 4X2 = 9x - 4
17. 3x2 + 4x = 5
19. X2 = 15x - 56
21. 2X2 + 5x + I
0

10.

4(x - 24)
2x + 2
(x + 6)2
x + 6
9x 2 + 24x + 16 = 0
2X2 + x - 4 = 0
x(2x
3) = 0
5x 2 - 7x + 2 = 0
X2

12. 3x2
14.
16.
18.
20.
22.

12.6 Equations reducible to


quadratic equations ___________
EXAMPLE 6
Solve the equations
(a) (X2 - 5X)2 - 2(X2 - 5x)
(b) X4
3x 2 - 10 = 0
x
(c) 4
12(2)X
32

(d)

24

~ + 1)' - 7 (x + 1) + 10

Each of these equations can be reduced to quadratic equations by a suitable substitution

262 NEW SENIOR MATHS: TWO UNIT COURSE

(a) Lety
Then

X2 - 5x
y2 - 2y - 24
(y - 6)(y + 4)
y
6
X2 - 5x - 6
(x 6)(x + 1)

i.e.

= 0
= 0

= 0 or
= 0 or

y+4=0

X2 - 5x + 4
0
or (x - 4)(x - 1) = 0

6,

1, 1, 4

(b) Lety

X2
y2 - 3y - 10 = 0
(y - 5)(y + 2) = 0

y - 5 = 0 or y + 2 = 0

X2 - 5 = 0 or X2 + 2 = 0
i.e.
:. (x
-vIs)(x + -vis) = 0 or X2 + 2 = 0 has no roots

Then

-vIs

x
(c) Lety = 2x
Then y2 - I2y + 32
(y - 8)(y - 4)

=
=
y - 8 =
x
2 = 8 =

i.e.

= 0

= 22

= x + x1

(d) Let y

Then

0
0

0 or y - 4
23 or 2 x = 4
3 or 2

y2
7y + 10 = 0

(y - 2)(y - 5) = 0

y - 2 = 0 or

x + 1 - 2

i.e.

x
X2 - 2x
(x

+1

I)(x - 1)

or

0 or

X2

y-5

1 5
x

5x + 1

or

1 or

X=

.J2I

.J2I

EXERCISES 12(c)
Solve the following equations:
1. (x - 2)2 - 2(x - 2) - 15
3. (x 2 + 2X)2 = I4(x2 + 2x)

5. (x 2 + 5X)2

6(x 2

5x)

2x
7. 3 - 28(3Y + 27 = 0
X
9. 4(2i - 5(2Y + 1 = 0
11. 5(5)2x
26(5Y + 5
0
13. 3(3)2x
28(3Y + 9
0
15. X4 - 2X2 - 8 == 0

17.

(x2x - t)' + 3(X t) = 0

19. 2

4(2t

+ 32

0
15

2. (x 2 2X)2 - II(x2 - 2x)


4. (x - 3)2 = 16
6.

(x +~y

24

12

=0

=0

16

8. 22x
- 6(2Y + 8 = 0
x
10. 4
I2(2Y + 32 = 0

12. 2(2)2X
9(2Y - 4
14. I6(2)2X - 33(2Y + 2
16. (2x - 5)2 = 2x
5

18. X4

5x 2

6 = 0

20. (x 2 - X)2 - 8(x2 - x)

12/QUADRATIC FUNCTIONS 263

21. x 6
23.

X4 x
-

25. 9

27. 7(7iX - 8(7Y


29. 4

31. 9x

22. x4 - 13x2 + 36 = 0
24. x4 - 2X2 - 15 = 0
26. (x + 1)2 = 4X2

28x3 + 27 = 0
5x2 + 6 = 0
4(3)X + 3 = 0

1 =. 0

(X + ~)' - 5 ~ + ! )+ 6 =

28.

8(2Y + 15 = 0
12(3Y + 27 = 0

30. x6 - 9x3 + 8 = 0
x
28(5Y + 75
-

32. 25

12.7 Relationship between roots


and coefficients _ _ _ _ _ _ _ _ _ _ __
The general quadratic equation is
ax2

bx

* 0)

X2

+ ~x + ~

0 (a

Dividing both sides by a we get

.. . . . . . . . . . . . . . . . . .. (1)

Let the roots of the equation be a and {3.

Then
(x
a)(x
(3)
0

i.e.
X2 - (a + (3)x + a{3 = 0 .................... (2)

X2 - (sum oj roots) x

i.e.

+ product oj roots

= 0

Equating coefficients of like powers of x in (1) and (2) we get

sum oj roots

product oj roots

a{3

b
a

{3
c

.............. (3)

................ (4)

(3) and (4) give us the relationship between roots a, (3 and coefficients a, b, c of a
quadratic equation.

EXAMPLE 7

Write down the quadratic equation whose roots are (a) 3 and -2
(a)

sum of roots = 1
product of roots = - 6
equation isx2 - x - 6 = 0

(b)

(b) 3

+ .J5 and 3

.J5.

sum of roots = 6
product of roots = 4
equation is X2 + 6x + 4 = 0

EXAMPLE 8
The quadratic equation 2X2
Find the values of m.
Let the roots be a and 2a.

(m

+ 2)x +

m = 0 has one root which is twice the other.

264 NEW SENIOR MATHS: TWO UNIT COURSE

3a = m

sum of roots
product of roots

2a 2 =

Substitute a
2(m

Then

+ 2)2

+2

.................. (1)

. . . . . . . . . . . . . . . . . . . . ..

(2)

m.+ 2 from (1) into (2).

36

+ 2)2
5m + 4

1)( m - 4)

(m
m2 (m -

9m
0
0
1 or 4

Test that these values of m are correct by putting them in the given equation. Then solve it
to see if one root is twice the other.

EXAMPLE 9
The parabola y = ax 2 + bx + c has vertex at (3, 18) and passes through the origin. Find

the values of a, b, c.

By symmetry, the parabola cuts the X-axis

again at x
6.

ax 2 + bx + c = 0 has roots 0 and 6.

b
sum of roots
-- = 6 :. b = -6a

product of roots = c

o :. c

When x = 3, y
18
18 = 9a + 3b
= 9a - 18a since b
-6a
a
-2 and b = 12

y = 12x
2X2 is the equation of the parabola

Fig. 12-12

Alternative method:
y = ax 2

When x
When x

+ bx + c

= 0, y = 0 and so

c =0

0 and so 36a + 6b
i.e. 6a + b
When x = 3, y
18 and so 9a + 3b
i.e. 3a + b
Solving (2) and (3) yields a = -2, b = "12.
=

6, y

=0

1. Write down the quadratic equation whose roots are

(a) 5 and -2
(c) 2

-J3

1
1

2
3
(d) p and 2p

(b) - and

.............. (2)

18
6

Can you think of any other approaches to solving this question?

EXERCISES 12(d)

. . . . . . . . . . . .. (1)

.............. (3)

12/QUADRATIC FUNCTIONS 265

2. The roots of the quadratic equation ax2


of a and b.

3. If a and {3 are the roots of the equation X2


(a) a

(b) a{3

{3

Use the fact that!


a

(c)

bx - 10

0 are -1 and 5. Find the values

= 0, what is the value of

4x

~+~

+ !{3 = a a{3
+ {3 for (c)

4. Write down the quadratic equation whose roots are p and !.


p

5. One root of the equation


3 q 2 = 16r.

X2

qx

6. The roots of the equation X2

px

7. The quadratic equation ax 2


Prove that 2b 2 = 9a.

bx

r = 0 is three times the other. Prove that

+ q = 0 are 2 and -3. Find the value of p and q.


+ 1 = 0 has one root which is twice the other.
+x +m

8. Given that x 2 is one root of the equation X2


the value of m.

9. If a and {3 are the roots of the equation X2


10. Find the value of k if for the equation X2
(a) the sum of the roots is 5
(b) the product of the roots is 12
(c) one root is two more than the other.

11. If a and {3 are the roots of the equation px2


1
1
1
1
(a) a

+ 73

Use the fact that a 2 + {32

(b) a 2

(a

O. Find the other root and

8x - 5 = 0, find the value of

(k

2)x

+ qx + r

1.

4k = 0

0 find, in terms of p, q and r,


(c) a 2{3

{32

~+

a{32

(3)2 - 2a{3 in (b)

-1

+ bx + c = 0 are
and -4. Find the values of band c.
13. If one root of the equation X2 + px + q = 0 is -2 show that q = 2p - 4.
12. The roots of the equation 2X2

14. Find the equation of the parabola which passes through the points (-1,0) and (0, -1)
and is symmetrical about the Y-axis.
15. Find the equation of the parabola through the points (2, 0), (4, 0) and (0, 8).
16. If one root of the equation X2

mx + 2

0 is double the other, find the values of m.

+ px 12 = 0 differ by 4.
18. If a and {3 are the roots of the equation X2 + 5x - 8 = 0 find the value of
1
1
(a) a + {3
(b) a{3
(c) a + 73
(d) a 2{3 + a{32
(e) (a + 2)({3 +
17. Find p if the roots of the equation 3x2

19. For what values of k will the equation X2 - (k


(a) the sum of the roots equal to 5
(b) roots that are equal but opposite in sign.

2)x

12 = 0 have

2)

266 NEW SENIOR MATHS: TWO UNIT COURSE

12.8 Sign of quadratic function _ _ _ _ _ _ __


EXAMPLE 10
For the graph of y = 2X2 - 3x
5, find the values of x for which
(i) y = 0
(ii) y > 0
(iii) y < 0
(i)

2X2 - 3x - 5
0
i.e. (2x - 5)(x + 1) = 0

.
5 or 1

l.e.
x = 2
Hence the graph of

y = 2X2
3x - 5

crosses the X-axis at x

-1 and x

= ~.

(ii) From Fig. 12-13 it can be seen that

y > 0 when x

< -1

or when x >

Fig. 12-13
2X2 - 3x - 5

(iii) From Fig. 12-13 it can be seen that

y < 0 when -1

<x <2

EXAMPLE 11
Prove that 2X2

4x + 5

> 0 for all values of x.

This is the same as asking to prove that the graph of y


X-axis.
Put 2X2
4x + 5 = 0
6. = b 2 - 4ac

= 16
40

2X2 - 4x + 5 does not cut the


y

<0

Since 6. < 0, 2X2 - 4x + 5 = 0 has no real


roots and so
2X2 - 4x + 5 > 0
for all x.

Alternatively
2X2

4x

= 2[X2

2x +

= 2 [<X2 =

+5

~J

2x + 1) - 1 +

2(x - 1)2

Fig. 12-14
f(x) = 2X2 - 4x + 5

3, which has a minimum value of 3 when x


so for all x, 2X2
4x + 5 ~ 3.

EXERCISES 12(e)
1. For what values of x is
(i) X2 - 3x - 10 > 0
(iii) 6

+ 11 x - 2X2

:E;;

(ii) 4
X2 ~ 0
(iv) 3x 2 + 14x < 5

1 and

12/QUADRATIC FUNCTIONS 267

2. For the parabola y


3. For the parabola y
y ~ O.

= 4X2
= X2 -

4. For the parabola y = X2

5. Prove that X2

+ 9 find the values of x for which y > O.


+ q)x + pq, (p > q) find the values of x for which

12x
(p

4x, find the values of x for which y > 5.

+ 2x + 7 > 0 for all real values of x.

6. What is the relation between a and b if the graph of y


(i) touches the X-axis
(ii) cuts the X-axis at two points?

X2

ax

+ b.

7. For each of the following functions find the values of x for which y > 0
(i) Y
X2 + 4x - 5
(ii) y = 4X2 - 3x - 7
(iii) y = X2 - 2x - 1
(iv) y = -3x2 + 2x + 1
(v) y = 1 + x - X2
(vi) y
9 - X2

8. For each of the following functions, find the values of x for which y < O.
(i) y
X2 - 4
(ii) y = X2 + 8x - 20
(iii) y = 2X2 + 4x + 1
(iv) y
-2x2 + 5x + 3
9. For each of the following functions, show that y > 0 for all x.
(i) y = X2 + 2x + 4
(ii) Y
X2 + 4
(iii) y = 3x2 + 2x + 1
(iv) y = X2 + X + 6
(v) y = 2X2 - 3x + 4
(vi) y
2X2 + 7

12.9 Further examples involving


discriminant _______________
EXAMPLE 12
Find the values of m for which the equation X2 + (m - 2)x
(i) one root
(ii) two roots
(iii) no roots.

4 = 0 has

m2

(i) For one root only, ~ = 0

~ = (m - 2)2 - 16

= m2

4m - 12
(m - 6)(m + 2)

= 0 when m = 6 or - 2

(ii) For two roots, ~ > 0


i.e. (m - 6)(m + 2) > 0

The graph of ~ = m 2
-

4m - 12
(Fig. 12-15) cuts the m-axis at m = 6
or -2.
Thus ~ > 0 when m > 6 or when
m < -2.

(iii) For no roots,

(Fig. 12-15).

< 0 when -2 < m < 6

Fig. 12-15
~

4m -

12

268 NEW SENIOR MATHS: TWO UNIT COURSE

EXAMPLE 13
Prove that the equation
X2 + (k - 3)x - k
0
has real roots for all values of k.
For real roots, .;l ~ 0
.;l = (k
3)2 + 4k
= k2
2k + 9

The graph of .;l = k 2


2k + 9 (Fig. 12-16)
does not cross the k-axis. Why?
Hence for all k, .;l > 0 and so the given
equation has real roots for all k.

Fig. 12-16
.;l

Alternatively
.;l

= k2 = k2

2k
2k
1)2

Hence

.;l ~

2k

+ 9

+ 1+ 8

+ 8
a perfect square + a positive number.

(k -

k2

8 for all k and so the equation has real roots for all k.

EXAMPLE 14
Show that the roots of the equation
X2

+ 2x

(m 2

1) = 0

are rational for all rational values of m.


For rational roots,

.;l

is a perfect square

4 + 4(m2
1)

.;l =

= 4m 2

a perfect square if in is a rational number.

EXERCISES 12(f)
1. Find the values of k for which the following quadratic equations have (i) one root;
(ii) two roots.
(a) X2 - 3x + k
0
(b) X2 + kx + 3
0
(c) X2 + (k
l)x - (2k + 1) = 0
(d) (k - l)x2 + (k + l)x
1 - k

2. For what values of k does the quadratic equation


(2k - 3)X2

+ (k + l)x - 1

have two roots?

3. For what values of m does the quadratic equation


(Sm

3)X2

4mx

+ m + 1 = 0,

have one root?

4. For what values of m are the roots of the following equations real?
(i) X2 + 2x + m 2
1 =0
(ii) (m - l)x2 + (m + l)x + m
1 = 0
(iii) X2 + 2mx + 2(m + 12)
0
5. If the roots of the equation X2
2mx
(ii) a =1= b
which (i) a = b

3 = 0 are a and b find the values of m for

12/QUADRATIC FUNCTIONS 269

6. Show that the roots of the equation


4(m

l)x - 3

l)x2 - 4(m

= 0, (m

are real for all real m.

'* -1)

7. Show that the roots of the equation


(3m - 5)X2 - 3m 2x

5m 2 = 0, ( m

'* ~ )

are rational if m is rational.


8. Show that the equation
X2 - (2a

b)x

ab = 0

has real roots for all values of a and b.

9. Find the values of p for which the equation


2X2

4x

+p

= 0

has (i) one root i.e. two equal roots

(ii) two roots?


10. For what values of m has the equation
X2 - 2mx
(i) one root i.e. two equal roots
(ii) two roots?

8m - 15 = 0

11. Show that the roots of the equation


ax2 - (a

b)x

0, (a

are rational, for all rational values of a and b.

12. For what values of m does the quadratic equation


X2 + mx + (m + 1)2
have two roots?

'* 0)

= 0

12.10 Identity of two quadratic


expressions .............................
An equation of the nth degree has n roots and no more. A quadratic equation (2nd degree)
has 2 roots e.g. the roots of X2 - 5x + 6 = 0 are x = 2 or x = 3. A cubic equation (3rd
degree) has 3 roots and so on. The roots may be real numbers or unreal and some of the
roots may be equal to one another.
If two polynomials of the nth degree are equal for more than n values of the variable,
they are identically equal i.e. equal for all values of the variable. The sign
is used to
indicate that the two expressions are identically equal. The coefficients of like powers of
the variable in the two expressions are equal.
Theorem: If a 1x 2
values of x, then

b1x

C1

a2x 2

b2x

C2

for more than two

It is a very common practice in mathematics to change an expression from one form to


another.
For example, we have changed a quadratic expression into the 'completing the square'
form.
In example 1, we saw that
2X2 - 4x - 5 == 2(x
1)2 - 7

270 NEW SENIOR MATHS: TWO UNIT COURSE

These two forms of the quadratic expression are equal for all values of x. We have
merely changed the appearance of 2X2 - 4x - 5.

EXAMPLE 15
Express X2

2x - 2 in the form ax(x

Method 1.

X2

+ 2x

- 2

==
==

1)

bx2 + c(x

1)

ax(x + 1) + bx2 + c(x + 1)


ax2 + ax + bx2 + cx + c
(a + b)x2 + (a + c)x + c

Equating coefficients of like powers of x, we get

a + b = 1

a+c=2

c
-2
Solving these linear equations, we get a = 4, b = -3, c = -2.
Thus
X2 + 2x - 2 == 4x(x + 1)
3x 2 - 2(x + 1)
Method 2. The identity is true for all values of x. Select 3 convenient values of x to form

three equations in a, band


Put x = 0
Put x = -1
Put x = 1
2a + b +
By solving, a = 4.

c.
c
-2
b = -3
2c

EXAMPLE 16
Express X2

2x - 2 in the form (x
X2

+ a)2 +

b i.e. the 'completing the square' form.

2x - 2 == (x
== X2

+ a)2 + b
+ 2ax + a2 +

Equating coefficients of like powers of x, we get


2a = 2
.'. a = 1
a2 + b
-2
:. b = -3
:. X2 + 2x - 2 == (x + 1)2 - 3

EXERCISES 12(g)
I .. Find the constants a, band c such that

n 2 == an(n + 1)

2. Express X2 + 5x in the form a(2x2

+ x)

q(x -

3. Express x(x - 1) in the form p

+ b(n + 1) + c

+ b(3x2 + x).
1) + r(x - 1)2.

4. Find the constants a and b such that

+ lOx +
+ 7X2 + 6x X2

5. Express X4 - 6x 3
hence solve the equation X4
6. Solve the equation X4
a(x2

+ 2X)2 +

b(X2

10

6x 3

4x3
2x) + c

== a(x + 2)2 + b(x +

8 in the form (X2


+ 7X2 + 6x 8

X2 -

lOx

+ ax)2 +
= O.

o by

1)

b(X2

+ c and

expressing it in the form

O.

Express each of the following quadratic polynomials in the form a(x


(7. to 10.)

7. X2 - 4x + 6

ax)

8. 2x 2

4x - 5

b)2

+ c

12/QUADRATIC FUNCTIONS 271

9. X2

10.

2X2

5x

= ax(x + 1) + bx 'for all values of x.


+ 2x - 2 in the form ax(x + 1) + bx2 + c(x + 1)
+ 2x in the form a(x - 1)(x + 2) + b(x + 2)

11. Find the constants a and b such that X2


12. Express X2
13. Express X2

12.11 Solution set of simultaneous equations _ _


The problem of finding the points of intersection of a straight line and parabola may be
solved algebraically by finding the solution set of the system composed of the equations of
the line and the parabola. The line may cut the parabola in two distinct points, touch it at
one point or not intersect it at all.
If the line is a tangent to the parabola, the solution set has only one ordered pair.

EXAMPLE 17
Find the points of intersection of the line y = x
At the points of intersection,
X2 - x - 2 = x + 6
i.e.
X2 - 2x - 8
0
(x - 4)(x + 2) = 0

x = 4 or-2

y = 10 or 4

They intersect at the points (4, 10) and


(-2, 4)

6 and the parabola y =

X2 -

X -

Fig. 12-17

EXAMPLE 18
Prove that the line y - x = 1 is a tangent to
the parabola y = X2 - 3x + 5.
At the points of intersection (if any)

X2 - 3x + 5 = x + 1

i.e.
X2 - 4x + 4
0

2,y

They have the point (2, 3) in common.


Since there is only one ordered pair, the
straight line is a tangent to the parabola at
the point (2, 3). (Fig. 12-18).

x
Fig. 12-18

2.

272 NEW SENIOR MATHS: TWO UNIT COURSE

EXAMPLE 19
For what values of m does the line y
(ii) intersect
the parabola y
X2 -.2x + 3?

mx - 6
(iii) not intersect

(i) touch

At the anticipated common points


X2 - 2x +3 = mx - 6
X2 - (m + 2)x + 9
0
The straight line and parabola will touch, intersect or not intersect according to whether
(i) Ll = 0
(ii) Ll > 0
(iii) Ll < 0
Ll = b 2
4ac
= (m + 2)2 - 36
m 2 + 4m - 32
(m + 8)(m - 4)

A+
-8
From the above sign diagram
(i) Ll
0, when m
-8 or 4
(ii) Ll > 0, when m < -8 or m
(iii) Ll < 0, when -8 < m < 4

m
y

>4
m<4

x
Fig. 12-19

Note: The principles involved in examples 17, 18 and 19 apply to the intersection of a
straight line and any curve whose defining equation is of second degree (e.g. a circle and

b)'

the rational function x :

EXERCISES 12(h)
Find algebraically the solution set of the following systems of equations (1. to 4.).
3x + 4
1. Y = 2X2
y = 12 - 3x

3.

y = X2
x+y=3

2. y
Y

3x - 2

.4.

2 - x - 3x 2

-7x + 2

x - y
X2 + xy

= 1

Find algebraically the coordinates of the points of intersection of: (5. to 7.)
5. the straight line y

6. the straight line y

2x

7. the straight line y

3 and the circle X2 + y2

=9

= X2

- 3x

3 - 2x and the parabola y = (x

2)2

1 and the parabola y

12/QUADRATIC FUNCTIONS 273

8. For what value of c is the line y

= 2x + c a tangent to the parabola y

X2 - X - 2?

9. For what values of c is the line y


x + c a tangent to the circle X2 + y2
Find the co-ordinates of the point of contact.

4?

10. For what values of a is the line y = ax a tangent to the circle


X2 + y2 + 20x - lOy + 100 = O?
11. For what values of m is the line y

= mx a tangent to the parabolay

X2 - 8x

+ 25?

12. For what values of m does the line y


mx - 6 (i) touch; (ii) intersect; (iii) not
intersect the parabola y = X2 - 2x + 3?

13. For what values of m does the line y


mx + 5 (i) touch; (ii) intersect; (iii) not
intersect the parabola y = 3 + 5x
2X2?
14. For what values of m does the line y
mx - 12 (i) touch; (ii) intersect; (iii) not
intersect the parabola y
2X2 - x - I O?
15. For what values of a does the line y = ax intersect the curve with equation y =
16. For what values of a does the liney

= ax not meet the rectangular hyperbolay =

~3?

x
3

17. Find the equation of the two lines which contain the point (1, 3) and are tangents to
the parabola y = X2 - 2x + 5.
18. Prove that the parabolas y
2X2
6x + 7 and y
and find the co-ordinates of the point of contact.

= X2

- 2x

+ 3 touch each other

12.12 The parabola as a locus _ _ _ _ _ _ __


A parabola may be defined as the locus ofa point P(x, y) which moves in
a plane so that its distance from a fixed point S is equal to its distance
from a fixed straight line.
We shall consider only simple cases where the fixed straight line is parallel to the X-axis
or the Y-axis.

EXAMPLE 20
Find the locus of a point P(x, y) which moves in a plane so that its distance from the point
S(O, a) is equal to its distance from the line y = -a.
y

x
Fig. 12-20

-8

274 NEW SENIOR MATHS: TWO UNIT COURSE

Using the formula for the distance between 2 points


PS = .J(x - 0)2 + (y - a)2 = ..Jx2 + (y - a)2
PM = ..J(x - x)2 + (y + a)2 = ..J(y + a)2
PS= PM
2 -+-("-y---a~)2
.J'-x-

-v(y-+-a): -: -2
r7

Squaring both sides, we get


X2
X2

+ y2

(y - a)2 = (y
2ay + a 2 y2

+ a)2

+ 2ay +

X2 = 4ay or y

a2

X2

4a

The equation of the locus is

X'

= 4ay

From your knowledge of quadratic functions, you can appreciate that this equation
defines a parabola with its vertex at (0, 0) and the Y-ax.is is the axis of symmetry. The fixed
point S is called the focus, and the fixed straight line is called the directrix.
The focal length is a and this is the distance of the focus from the vertex.
8y defines a parabola with a = 2:

For example, the equation X2


vertex (0, 0), focus (0, 2), directrix the line y = -2, axis of symmetry the Y-axis and focal

length 2 units.

EXAMPLE 21
Find the locus of a point P(x, y) which moves in a plane so that its distance from the point
S(a, 0) is equal to its distance from the line x = -a.
Comparing this example with example 20, you will notice that the point S(a, 0) is on the

X-axis, and the line x

-a is parallel to the Y-axis. In other words, we have interchanged

x and y. Should the equation of the locus be y2


y

4 ax?

M(-a, y)

x
Fig. 12-21
x

=-8

PS = ..J(x - a)2 + y2
PM
..J(x + a)2, and so
..J(x - a'-)2-+-y-2 = ..J(x

a)2

Square both sides of this equation:


(x - a)2 + y2
(x
X2 - 2ax + a 2 + y2 = X2

i.e.,
I,

+ a)2
+ 2ax +

a2

12/QUADRATIC FUNCTIONS 275

This equation defines a parabola with its vertex at (0, 0) and the X-axis is the axis of

symmetry. The/oeus is the point (a, 0), the/oeallength is a units, and the directrix is the

line with equation x = -a.

The equation y2
12x defines a parabola with a = 3:

vertex (0, 0), focus (3, 0), directrix the line x


-3, axis of symmetry is X-axis, focal length

3 units.

We will consider now situations where the fixed point is not necessarily on the X- or
Y-axis and the vertex is not at the origin.

EXAMPLE 22
Find the locus of a point which moves in a plane so that its distance from the point S(2, 1)
is equal to its distance from the line y = -3.
y
I

I
I

I
I

(2 1)S.-----
,

'"---

x
1(2
-1)
I
I
I
I
I
I

Fig. 12-22

-3

M(x, -3)

PS =
PM =
PS=
-J-c-(x----=-2--=-)2-+-(-:-y----:-"1):--2 =
Squaring both sides we get
(x - 2)2 + (y - 1)2 =
(x - 2)2 + y2 - 2y + 1 =
(x - 2)2
This is of the form

~2)2

+ (y - 1)2

-J(Y + 3)2
PM
.Jrc-(y-+--=3C-:-)2
(y + 3)2
y2 + 6y + 9
8(y + 1)

X2 = 4a Y where X
x - 2, Y = Y + 1 and a
2.
Hence the locus isa parabola with vertex (2, -1), axis ofsymmetry the line x = 2, focus (2, 1),
focal length 2 units.
In general, if the parabola X2 = 4ay undergoes a translation so that its vertex is at the
point (h, k), then its equation becomes

(x - h)2

= 4a(y

- k)

The focus is (h, k + a), directrix the line y = k - a, focal length a units.
Compare this translation with that of the circle X2 + y2 = r2 which has its centre at the
origin. If the circle undergoes a translation so that its centre is (h, k), then the equation of
the circle is (x - h)2 + (y - k)2 = r2.

276 NEW SENIOR MATHS: TWO UNIT COURSE

EXAMPLE 23
Find the equation of the parabola with focus (4, 1) and directrix the line x
Y

2.

x
Fig. 12..,23
x=2

-V(x - 4)2 + (y 1)2


-V(x - 2)2, and so
-V(X - 2)2

PS
PM
~r;-(x---4=-)2=--+-(:--y------=-I-:-=)2

Square both sides of this equation:


(x - 4)2 + (y
X2 - 8x + 16 + (y

1)2 = (x - 2)2
1)2
X2 - 4x + 4
(y - 1)2
4x - 12
4(x - 3)
Hence the equation of the locus is (y - 1)2
4(x - 3). This is of the form y2
4 aX,
where Y = Y - 1, X = x - 3 and a
1.
Thus the vertex is at (3, 1), the axis of symmetry has equation y = 1, and S is 1 unit to
the right of the vertex.
In general, the parabola with equation y2
4ax has its vertex at (0, 0), S is a units to the
right of the vertex, and the X-axis is the axis of symmetry.
If this parabola undergoes a translation so that its vertex is at the point (h, k), then its
equation becomes
(y - k)2

4a(x - h)

S is a units to the right of the vertex, and the axis of symmetry is the liney

k.

EXAMPLE 24
Show that the following equations represent parabolas, and find the vertex, focus and the
equation of the directrix of each. :
(a) y2 - 2y - 2x - 4
(a)

(b)x 2 - 6x

0
y2

2y - 2x
y2 - 2y

i.e.,

(y -

This equation is of the form (y - k)2


The vertex is at the point

(-~,

2x

1)2

+ 5

2(X + ~)

4a(x - h).

1).

4a

4y + 15

12/QUADRATIC FUNCTIONS 277


y

X=

I
I

:3

x
y

Fig. 12-24
Hence the focus is at the point (-2, 1) and the directrix has equation x
The axis of symmetry has equation y = 1.

(b)

X2 -

+ 4y + 15 = 0
X2
6x + 9 ~ -4y (x - 3)2

This equation is of the form (x - h)2


Note the negative sign in front of 4a.

-4

(Y + ~)

-4a(y - k).

(3, -~).

4a

The focus is at the point

-3 (fig. 12-24).

6x

i.e.,

The vertex is at the point

=4

(3, -~), the directrix has equation y -1 and the axis of

symmetry has equation x

3 (fig. 12-25).

You have been accustomed to writing the general quadratic function in the form
y = ax 2 + bx + c
With the aid of 'completing the square' technique we can easily change it to the locus form
(x - h)2

4a(y - k)

as seen in the previous example,


~

EXAMPLE 25
Express each of the following in the form (x
h)2 = 4a(y - k) and hence state the
vertex, focus and directrix
(i) y = X2
2x + 5
(ii) y = -2x2 - 8x +
Y

(i)

2x + 5

2x + 1 + 4

1)2 + 4
l(y - 4)

X2 -

= X2 (x

(x - 1)2

The vertex is (1,4), focal length


the line x
(ii)

unit and hence focus is (1,

4~), directrix y

31, axis

1.

_2X2 -

_2[X2

8x

+ 4x -

~J
----

-~--~-~---

278 NEW SENIOR MATHS: TWO UNIT COURSE

2(x
(x

The vertex is (-2, 9), a


the line x

+ 2)2 =

2)2 =

_2[(X2 + 4x + 4) - 4!]
-2(x + 2)2 + 9
9
y

-y -

9)

Note the negative sign before

= ~ and hence focus is (-2, 8~), directrix the line y =

~.

9l, axis

-2.

EXERCISES 120)
Sketch the following parabolas and write down (a) the coordinates of the vertex and focus
and (b) the equation of the directrix and axis of symmetry (1. to 12.)

2 Y -~2
- 12 x

8Y

3. X2
5. X2
7. y2

4y
-2y
=x
9. (x - 1)2
11. (y - 2)2

4.
6.
8.
10.
12.

4(y + 2)
4(x + 1)

X2
X2
y2
(x
(y

=
+
+

-8y
-6y
-2x
3)2
-2y
4)2 = -3(x

1)

In numbers 13. to 17., find the equations of the locus of a point P(x, y) that moves so that
its distance from
13. the point (0, 2) is equal to its distance from the line y = -2,
14. the point (0, -4) is equal to its distance from the line y

4,

15. the point (0, 1) is equal to its distance from the line y

-1,

= 2,
17. the point (-2, 4) is equal to its distance from the line y = 6.
16. the point (0, -2) is equal to its distance from the line y

18. Show that the following equations represent parabolas, and find the vertex, focus and
equation of the directrix in each case.
(a) y2 - 4x + 2y - 3 = 0

(b) y2 + 8x
4y - 8
(c) X2 - 2x + 4y + 15 =
(d) X2 + 6x - 5y
16 = 0.

Find the equation of the following parabolas and sketch each (19. to 26.)
19. vertex at (0, 0), focal length 2 units, axis of symmetry the Y-axis
20. vertex at (0, 0), focus (0, -1)
21. focus at (0, 7), directrix the line y = -7
22. vertex (2, -1), focus (2, 3)
23. the line x
24. the line x

2 as axis, vertex (2, 5) and crosses Y-axis at y = 9

= -1 as axis, focal length 2 units and crosses Y-axis at y = 25

25. vertex (2, -1), axis the line x

2, focus (2, 0)

12/QUADRATIC FUNCTIONS 279

26. the line x

= 0 as axis, (0, 0) as vertex and passing through the point (-4, 2)

27. Find the equation of the family of parabolas having


(i) the line x = 2 as axis and vertex (2, 1)
(ii) the Hne

x = -2 as axis and focal length ~ unit.

28. Rewrite each of the following equations in the form (x - h)2


state the vertex, focus, directrix and axis of the parabola.
(i) y = X2 - 4x + 4
(ii) y = X2 + 6x + 6
(iii) y
4
X2
(iv) 4y
X2 + 2x - 3.

= 4a(y

k) and hence

CHAPTER 13

Indices and

Logarithms

This chapter may be studied in conjunction with Chapter 18.


Let us assume that a biological organism reproduces by dividing every hour. The
following table shows the population of organisms (y) after x h, assuming that all
organisms are still alive.
Number of hours

Population

16

Original population y
After 1 h,
y
2 x (1)
2
After 2 h,
y = 2 x (2)
22
After 3 h,
y
2 X (22) = 2 3
After 4 h,
y = 2 X (23) = 24 , and so on.
Hence y defines a function whose rule is y
2x. This is an example of an index
(exponential) function.
Comparing the form of this function with a typical quadratic function, we note the
following:
Function
Quadratic
Index (exponential)

Rule

Base

Index

X2

x
2

2
x

2x

In the function given by y = x 2 , the index, 2, is fixed, and the variable base, x, takes all
values in the domain.
In the function given by y = 2x , the base, 2, is fixed, and the index, x, varies. In the
preceding example, x takes positive integer values. (See fig. 13-1).
Just as algebra of polynomials is needed to manipulate polynomial functions, so
algebra of indices is needed to manipulate index (exponential) functions.
In elementary work, an is defined as
an a x a x a x a x ... to n factors, where n is a natural number.
This definition will not do for other kinds of indices, e.g. a 112, for it makes no sense to talk
about a multiplied by itself half a time. We must review the meaning of aX as x takes
values successively in the integers, J, the rationals, Q, and the reals, R.

13/INDICES AND LOGARITHMS 281

16

12

Fig. 13-1
We must ensure that the index laws (first derived for natural number indices) are
conserved for each new extension of the meaning of aX. We start by reviewing the
properties of aX for x a natural number.
The symbol, an, n EN denotes the product of n factors each of which is a; i.e.
an
a X a X a x ... to n factors.
The letter n is called an index or an exponent or apower. We say that 'a is raised to the nth
power' or that 'n is the index or exponent of a' because n indicates the power to which
base a is raised.

13.1 Index laws _ _ _ _ _ _ _ _ _ _ _ _ __


You are familiar with the following index laws:

2. am

am - n
an
3. (am)n = a mn
4. (ab)n = an. b n

We now consider the proof of these laws, but only in the case where m and n are natural
numbers and a and b are real numbers.
1.

am X an

(a.a.a.a ... to m factors) x (a.a.a ... to n factors)


a. a. a . .. to (m + n) factors

= am + n

e.g.,

a3

a 2

a.a.a x a.a

a3+ 2

= as

2. The proof of this law will be left until we consider negative exponents.
3.

e.g.,

(am)n = am. am. am ... to n factors


(a. a. a .. , to m factors) ... n times
= a. a. a . .. to mn factors

= a
mn

3
(a )2 = a 3 a 3

a.a.a X a.a.a
= a6

282 NEW SENIOR MATHS: TWO UNIT COURSE

4.

ab. ab. ab . .. to h factors


(a. a. a ... to n factors) x (b. b. b ... to n factors)
= an. b n
3
6 = (2. 3)3 = 23. 33

(ab)n

e.g.,

13.2 Zero and negative indices _ _ _ _ _ _ __


We now extend the definition of an to allow n to be zero or a negative integer. We define
aO and a-n to permit us to extend the first index law established for n E N.
for m = 0,
If
am x an = a m+ n
aO x an == an
then
an
and so
aO
provided a ::f::: 0
an
We define the symbol aO to be equal to 1 and give no meaning to 0.
If
am x an
a m+ n
for m = -n,
n
a- x an = a n- n
then
= aO

a-n = an

and so

We define the symbol a-n to be equal to

(a

::f:::

0)

~.
an

We can now give a simple proof for the second index law:
am
-an = am - n
am
= am. a- n
Proof'
an
=

am - n

e.g.,

EXAMPLE 1
Simplify the following, expressing them with positive indices:
(a) 4

(a)
(b)

(c)

(b)

(!}3

(c) 8-3

4-2

(!)

-3

42
=

1
1

(22)2
= 23

8-3 = 83 = (23)3

24

13/INDICES AND LOGARITHMS 283

(d)

Thus
24.3 9
= 24.34

= 35

EXAMPLE 2
X-I
l'f
SImp
I y X-I

+ x'
1 expressmg
. t h e answer WIt
' h positIve
,. m
. d'Ices.
X-I

1 = 1
X

1 - X2
= -1 - x =
x
X
X-I + 1
1 + x
x
~--- = ---- x ----~
-x
x

X-I -

l+x x =#:0
1 +x
(1 - x)(l

x)

x =#: -1

EXERCISES 13(a)
1. Simplify the following:
(a) X2.X5.X3
(c) (P2q)4 X (q2p)5
(e) (2X2)5 x (4X3)2

(b) 2 3 x 42 X 82
(d) a 3b-2 X (a 2b 2)4
(f) m2p3 x (m3n2)3

(b) 2 n

(P-1)2

2ln X 23n

(2m2n)3
(d) (mn3)2 x (4ml)l
(alb)l. (ab)4
(f)
(a l b)3

(b) m2n2p-l

x (mnp2t3

plq3,-3
(d) p3q- I r
(f) 23 (2n)l .2-n

284 NEW SENIOR MATHS: TWO UNIT COURSE

4. Simplify the following, expressing them with positive indices:


(a)
(c)

X2 .X3 .X4
x5
2n.2n-3

q
(b) ap +q aP

3-2x- 1

(d)

23

(e) 2n. 4n . 8n

a 2b 3c-4

(f)

xm+n y3m-n

x-n~y3m

(g)

(3Xy2 )-3
4x4y

(h)

5. Write down each of the following as a negative power of 2:

1
1
1
1
1

(a) 4
(b) 16
(c)
(d) 0'125
(e) 64
(f)
(g) 025
6. Write down each of the following as a power of 10:
(a) 100
(b) 10
(c) 1
(d) 01
(f) 0'001

10~0

(g)

7. Write down the value of:


(a) 92 x 3-4
(d) a-5 x a- 3 x aO x a6

(h) 8-3

(e) 001

1
(i) 100,000

(h) 00001

(b) 2 0 x 50
(e) 6-3 x 25 X 33

(c) 8-3 X 27

8. Simplify the following, expressing them with positive indices:


32n X 25 2n - 1
(a)
(b) (x- I + y-I)(X- I - y-I)
15 n - 1
(c)

(e)

2n.4n+1
8n- 2
x - 5

+ X- I)3
X2 + 8x- 1
1 + 2x- 1

(d) (x-2

6x- 1

(f)

1 - 2x- 1

9. Simplify the following, expressing them with positive indices:


(a) 4-2 x 63 X 84 X 12-2

15 n + 1 x 25 X 53n - 4

(b)
9n-1 x 25n-2
(c) (-4>-2 x (-2)3
10. Show that, for any positive integer n,
(-1)n
1 when n is even, and
(-1)n
-1 when n is odd.
11. Find the value of x that makes each of the following true:
(a) 2x = 8
(b) 3x - 1
27
(c) x 3
x- 3

2
(d) x-2
81
(f) 4x
(e)
-x
(g) 9x

(h) 3x

27

(j) 3x

+ 5. 3x = 54

(k) 2- x =

64

(i)

-125

32

3. 5x

(I) 5x

12. Expand and simplify the following, expressing the results with positive indices:

(a) (a- I
(d) (a 2

b)(a- I

b)

2b- I )(a-2

b)

(b) (X-I

(e)

+ y)(x + y-I)
I
+ b-

(c) (x-2
(f)

+ y-2)(X-2 +

y-2)

13/INDICES AND LOGARITHMS 285

13.3 Rational indices _ _ _ _ _ _ _ _ _ _ __


The symbol a llq , where q is a positive integer and a > 0, is defined as the positive qth root
of a. That is,

We agree that !fJii, q = 2, 3, 4, 5, ... , shall mean that number which, when multiplied by
itself q times, gives a. That is
!fJii x !fJii x !fJii ... to q factors = a.
For example, ~ = .Ja means .Ja x.Ja
a i.e. a ll2 x a ll2 = a.
ifa means ifax ifa x ifa = aLe. a ll3 x a ll3 x a l/3
a.
.Ja or a ll2 is called the positive square root of a.
ifa or a ll3 is called the cube root of a or a raised to the power Of~.

J.;a or a ll5 is called the fifth

root of a or a raised to the power of ~.

4112 = ..[4 = the positive square root of 4 = 2


27 1/3 = !J27
the cube root of 27 = 3
In order to avoid difficulties at this stage and to confine the treatment to the field of
real numbers, we will consider a as a positive real number only. If a is negative and q an
even integer, a 1lq does not exist in the field of real numbers. For example, (-4)112
.J=4
and (-81)1/4
~ do not exist.
However, if a is negative and q an odd integer, a llq does exist. For example,
(-8)113 = !J=8 = -2.
plq
The symbol a , where p and q are positive integers and a > 0, is defined as the pth
power of a 1lq . That is,

e.g.,

Ia

P/ q

e.g.,

(!fJiit I

82/3 = (!JS)2

22 = 4

Furthermore, we define a-plq as a;/q'

e.g.,

4 5/2

1
(..[4)5

1
32

Earlier we proved these index laws:


am x an
am+ n
(am)n = amn
and
(ab)n
an. b n for positive integers m and n.
It is possible to prove these laws when the indices are rational, but we shall omit such
proofs and assume that the laws are true. That is, if p, q~ u and v are integers, then
aplq x aulv = aplq + ulv
(aplq)ulv = aPulqv
(abylq = aplq . bPlq

286 NEW SENIOR MATHS: TWO UNIT COURSE

EXAMPLE 3
Simplify (a) 322/5 (b) 125-2/3 (c)
(a) 322/5

= (2 5)215 =

X 512 X- 3/4

(d)

22

9,\ -112
( 49)

or

(e) 8213 x 9-312

32215

-V32)2

X512. X- 3/4

(d)

9'\ -1/2
( 49)

X512-3/4

X 7/4

EXAMPLE 4
1/4

2
.JfO x ~ (b) 3n - 81
xn-I9 n + I (c)

51/4 X .JfO x ~

(a)

(XI/2

X-I 12)2

51/4 x (2 X 5)112 x 2114


(22 X 5)3/4
5 114 X 2112 X 5 112 X

2114

(b)

3n -

3n -

x (3 2)n+ 1

(34)n I
x 32n + 2

34 - n or
(X 1I2 -

(c)

X- 1I2 )2

= (X1I2)2

= Xl -

2x l12x- 1I2 +
2xO + X-I
1

=x-2+
X

EXAMPLE 5
Solve the equations
(i)

52x

125 112

25

(e)

' l'f
( ) 5
1 y a
SImp

(b) 125 -213 = (53) -2/3


(c)

22

(X- 1I2 )2

13/INDICES AND LOGARITHMS 287

52x = 125 1/2 = (5 3)112

(i)

2x

=~

5312

2
3

3x

(ii)

3x

i.e.

1
16

or 22x - -

0
=

22x

16

2x = -4

x = -2

x=O
EXERCISES 13(b)
1. Evaluate the following:
(a) 64 2/3

(d)(

1)-2

( 1 )-312

(g) \16

(j) ~ x

Ys

(b) 49- 112

(c) (93)112

(e) 2213 x 4 116

(f)

(1 ~5 JII3

(h) 243-2/5

(i)

(k) ~

x ~

2. Express each of the following in simplest indicial form:


(a) ~
(b)
(c) ~.m
2
(d) .J3
(e) a X ;.;a
(f) ~2312
a 2/3 x ~
!J(j4
(h) ~X3y2
(i) 8- 2/3 X 4312
(g) ~

rs-r

(j)

(5 112)3

-\jX2y3
(k) 16 3/4 x 4- 112

x -J5

3. Simplify the following, expressing the results with positive indices:

x (b~2 )112

(a) X2l3. X312

(b) (a-IbF

(c) (XII2)2 - (X-2)I12


(e) (X1I2 + yll2)(XII2 - y1l2)
(g) (9X2)312 x (8x 3t 2/3

(d) (XI/3)2 x (X- 1y 3t l X x-S13y2


(f) (X1l2 + yll2)2
(h) a3/4 X a S/4 X a- 2

4. Simplify the following:


X

6-

(a) ~x2y3 x -

I/3

(b) (y213)3/4

y-112

(c) ~x2y +

-JXii

54114

(e)

63/4

12-112

(d) ..Ja 3b 2

(9xy) 112

X-3~

y-S x3y

x ~

(f) (8X 6)1/3 X (2xt3

(g) 3x X 9yl/2

5. Simplify

x (y1/St 5l3

and evaluate for x = ..j2, y = ...)6.

6. If (3xn)3 x (3x)n-6 = ax2, find the value of nand a.

288 NEW SENIOR MATHS: TWO UNIT COURSE

7. Evaluate the following:


(a)

64)213
(125

(b) 0-001-213

(c) 0-81 112

(d)

9,\ -312
( 16)

(e) 2 x 4- 112

(f)

3- 112 x

27112

8. Simplify the following, expressing the results with positive indices:


(a) (a 2b- 1t 2 + (a- 1b 2)2
(b) (X1l2 _ y-1I2)2

(c) (X 1/3

yll3)(XIl3 -

yll3)

9. Simplify the following:

1 - x- 2

(a) - - -

(c)

(Xl - 2x

;,;x4

(d) $2 X

(b) (a 2

when a

+ 1)-112 when x = 3

2ab

b 2 )112

4 and b

10. Simplify the following:


(a) 2n x 4 n x 8 n

(c)
(e)

3n x 9 n + 1

(d)

11. Simplify the following:


a l/2 x b 3/4 x (a 2 )]/4
(ab)1/4

(a)
(c)

2Y3

62n

32n

-:-=--

4n + 1

22n

(b) _25_2n-::-,-X--,-5n_-_l

3-J2

x 6 112

(b) 10-1

(d)

.J6 x

(b)

ax - 3

100005 X 1000
4 X 3 112

12. Find the value of x for which


(a) 9 x

(c) 2x
(e) 3x . 2x

13. Solve the following for x:


(a) 5x = 125

(c) 8-x
(e) (3 X

9)(5 X

(b) 16x = 128

(d)

1) = 0

(2x - 1) (3 x-

(f) 32x + 1

b)

27

13.4 Exponential and logarithmic functions _ __


The functionj(x) = ax, is called an exponential junction, where the base a is a positive
real number other than 1_
Its domain is the set of real numbers, and its range the set of positive real numbers.
Each ordered pair is of the form (x, aX). Some ordered pairs are (-3, a-3 ), (-2, a-2 ), (0, 1),
(1, a), (2, a 2), etc_

H a > 1, aX increases as x increases_

.1

13/INDICES AND LOGARITHMS 289

For x large negative, aX is small but positive.


For all values of x, aX > O.
For all values of x < 0,0 < ax < 1.
When x
0, aX = 1.
For all values of x > 0, aX > 1 (fig. 13-2).

x
Fig. 13-2: Graph ofj(x)

aX, a

> 1

Fig. 13-3: Graph ofj(x)

aX, 0

<a<

If 0 < a < 1, aX decreases as x increases. For x large positive, aX is small but positive
(fig. 13-3).

x,

If, for example, a = ~, then (~)x = 2- and so we can consider fig. 13-3 as the graph of
a-x, a > 1.
An exponential function has another very important property. A function is a set of
ordered pairs such that no two of the ordered pairs have the same first element. However,
several ordered pairs could have the same second element. If a function j is such that no
two pairs have the same second element, then there exists a functionj-I called the inverse
function of j. The inverse function j-I is a set of ordered pairs obtained by interchanging
the first and second elements in each ordered pair.
A function that has an inverse is often called a one to one function.
An expon'cntial function has the property that no two ordered pairs have the same
second element, and hence has an inverse called the logarithmic junction, which is a set of
ordered pairs obtained by interchanging the first and second elements in each ordered pair
of the exponential function.
y = aX may be written as x = 10gaY. By interchanging x and y, we get Y = logax.
Thus the exponential functionj(x)
ax, has the logarithmic functionj-I, or g where
g(x) = log ,X, for its inverse.
The domain of the logarithmic function is R+ (the range of j) and its range is R (the
domain ofj). It is also an increasing and a one to one function. If we draw both graphs on
the same axes (fig. 13-4), we see that the resulting figure is symmetrical about the broken
line with equation y = x. This is a characteristic property of the graph of any function and
its inverse.
Consider the graph of the exponential function with equation y = 2x. Some typical
ordered pairs are (0, 1), (1, 2), (2, 4) and (3,8). Also consider the graph of the correspond
ing logarithmic function with equation y = log2 x, some typical ordered pairs of which are
(1,0), (2, 1), (4,2) and (8, 3). What do you observe? (Fig. 13-5.)

j(x) =

290 NEW SENIOR MATHS: TWO UNIT COURSE

Fig. 13-4

Fig. 13-5

Note: Since y = 2x has been defined for x a rational number, we can plot points on the
graph for all rational values of x. Since a continuous curve can be drawn through these
points, it is reasonable to assume that 2 x can be defined for all x i.e., values of 2x for which
x is an irrational number can be defined.
To obtain a meaning for a number like 2Y1, we can use our known rational approxi
mations to ..[3. Then we have
17 < ..[3 < 1'8, and so 2 1-7 < 2Y1 < 2 1-8 ,
1'73 < ..[3 < 1'74 and so 2 1-73 < 2.ff < 2 1-74, and so on.
Hence 2-1'f can be defined to any desired degree of approximation.
The function I-I, which is the inverse to the exponential function given by 2x , is called
the logarithmic function (or logarithm function) to base 2, and is derived as follows. We
present a comparison between the one-one function given by I(x) = x 3 , and the derivation
of the corresponding inverse function.
I(x)

x 3 is defined by
y = x 3,

I(x)

so I-I is defined by

x = y3
(interchanging x and y).
To express y in terms of x, we use a
special operation called 'taking the cube
root' , symbolized by if , and we
write
y=
This defines the inverse (cube root)
function.

2x is defined by
y
2x,
so I -I is defined by
x = 2y
(interchanging x and y).
To express y in terms of x, we use a
special operation called 'taking the
logarithm to base 2', symbolized by log2'
and we write
y
log2 x
This defines the inverse (logarithm)
function.

13.5 Logarithms _ _ _ _ _ _ _ _ _ _ _ _ __
=

8. This same statement may be written thus: log28 = 3.


23 = 8 <=> log2 8
3
32 = 9 <=> log3 9 = 2
104
10000 <=> loglo 10000
4
aO
1
<=>
loga 1 0
aX = y
<=>
loga y
x
In general, if a > 0, then the statements aX = y and loga y
x are equivalent
statements.

Consider the statement 23


That is,
Also

13/INDICES AND LOGARITHMS 291

Thus we see that a logarithm is an index or exponent,


Definition: The logarithm of a number n is the index x of the power to which a base a
must be raised in order to equal the given number.

EXAMPLE 6
Without the aid of tables, evaluate the following:
(a) logz 16
(b) log5 125
log2 16 = x,
2x = 16
x = 4

(a) Let

then
and so

= 24

log5 125
x,
5x = 125
x
3

(b) Let

then
and so

53

Earlier in this chapter we considered the index laws:


aX. aY
ax+Y
aX
= aX - Y
aY
(ax)y = a XY

(1)

(2)
(3)

From these laws, we can deduce the laws of logarithms,


Let aX = m and aY n, and so
mn
aX, aY = aX+Y
loga (mn) = x + y, and so

Hence

toga (mn)

For example,

toga m

toga n

(4)

loga 15 = loga (5 X 3)
= loga 5 + loga 3
loglo 20 + loglo 5
loglo 100 = 2
log a 4 + loga 3 + log a 2 = loga (4 X 3 X 2) = loga 24
m

Also,

Hence

aY

aX - Y

loga (:)

x - y, and so

I log. (!fI-)

log. m

log. n

7
toga (15 ) = toga 17 - loga 5

For example,

0
log2 20 - log2 5 = log2 ( 25 )
loglo 5

log2 4 = 2

loglo 4 - loglo 2 = loglo (5 X 4)


loglo 2
= loglo 20 - loglo 2

log!. (2~)

log)o 10

(5)

292 NEW SENIOR MATHS: TWO UNIT COURSE

If m = 1, then

loga

(~)=
=

loga n

loga 1
-loga n

From this, we can see the relation between the logarithm of a number and the logarithm
of its reciprocal.
Also,
mP = (ax)p
Hence

loga (m

= a XP
= xp, and so

(6)
For example,

loga 81 = loga (3 4)
= 4log a 3
loga 96 = loga (3 X 2 5)
= log a 3 + 5log a 2
3log1o 5 = log)O (53) = log)O 125
2loga 6 = loga 36

Summary:

loga (mn) = loga m


log"

(~)

IOga(~)

loga n

log" m - log" n

= -loga n

loga 1 = 0
loga (mp) = p loga m
Note that m and n are both positive real numbers.

EXAMPLE 7
Simplify the following without using tables or a calculator:

31og,o 2 + log,o 18 - 21og,o (~).


3log)O 2

log)O 18 - 2log 1o (~)= loglo 8

log)O 18 - log,o

= log,o [8 x 18 x

~~ ]

= log,o 100

=2
EXAMPLE 8
If log,o 2

03010, and loglo 3

04771, find the following:

(a) log,o 6

(b) log,o 24

(c) loglo (;

(d) log) 0 5

(e) log)O 54

(f) log,o

9"

(;~)

13/INDICES AND LOGARITHMS 293

(a) loglo 6 = loglo (2 x 3)

(b)

loglo 24

(d)

logio 5

= loglo 2 + loglo 3
03010 + 04771
=

logio

(c)

0-7781

6" = -iogio 6
-0'7781

loglo (23 x 3)
= loglo 2 3 + loglo 3
= 3 loglo 2 + loglo 3
= 09030 + 04771
13801
logio

e~)

= loglo 10 - loglo 2

=
(e) loglo 54 = loglo (2 X 33)

(~ )

(f) loglO

+ 31og lO 3
03010 + 14313

= loglo 2

=
=

= 17323

1 - 0'3010
0'6990
logio 2 - loglo 9
loglo 2 - 2log lo 3
03010 - 09542
-0'6532

Note that the logarithm of numbers less than 1 is a negative number. For example,
log, 0

~ and log) 0 ~.

Is this true for any base?

Also note that the logarithm of a negative number does not exist. For example,
log) 0 -2 does not exist.
log I 0 (-2) = x, if it exists.
Let
lOx = -2.
Then
But there is no real number value of x, small or large, positive or negative, for which
lOx
-2.

EXAMPLE 9
For what value of x is log2 (x

1)

log, (x

1) - log, (x - 1)

and so

Le.,

log2 (x - 1)

IOg,e

3?

IOg,[~ ~ ~J

:J= 3

x + 1
x - 1
x

+ 1
x

= 23

= 8x - 8
1~
7

EXERCISES 13(c)
1. Without using a calculator or tables, evaluate the following:
(b) log9 3 .
(a) log3 9
(d) loga a

(c) log2 128


1

(f) log4 025


(e) log3 27
(g) log)) 243
(i) log2 8
(k) log2 1

(h) logs 512

(j) log5 5
(I) log3 -J3

------------~

"~~~~

..

~------

294 NEW SENIOR MATHS: TWO UNIT COURSE

2. Without using a calculator or tables; evaluate the following:


(a) logs 625
(b) log3 81

(d) log4 128


(c) log9 243

1
(e) logs 4"

(f) loga a 3

(g) log) 0 0-0001

(h) log4 0-125

3. Find x, in each of these cases:


(a) log3 x
3
(c) logs 625 = x
(e) log6 x = 3
4. Solve for x:
(a) logs x
-3
(c) logx 128

(b) log3 81 = x

(d) logx 64 = -3

3-5

(e) log9 X = 1-5


(g) log3 27 -J3
x
(i) logz (logz x) = 2

(f) log9 x = 0-25


2-5
(h) log7 x
0) logz x = logz 8 + log4 8

5. Simplify the following:


(a) logz 16 + logz 8
(c) (logz 16)(logz 4)
(e) loga 8
loga 2
(g) log) 0 125 + log) 0 32 - log, 0 4
(i) log3 81 X logs 125
6. Simplify the following:
1
(a) 2loglo 16 + 2log)O 5

(e) log)o 125


(g) log) 0

loglo 25

(b) logx 81 = 2
(d) logx 27
1-5
(f) logx 343 = 3

(b) loglo2 + loglo5


(d) log3 54
log3 18
1
(f) loga 5 + loga 5
(h) log2 18 - 210gz 3

(b) logz (2x)

loglo 5

3 +
+ log) 0 -

(d) log)O 25
loglo 5
(f) log (X3)
log x

(h) log x

log .JX

7. Solve these for x, without using tables or a calculator:


(b) loglo x

(a) loglo x = loglo 4 + loglo 2


) I

log, 0 4
(C) Iog)O X = 1--2

(d oglo x
oglo
(e) 21og, 0 x + 3 = loglo (X5)
(f) log) 0 Xz
8. Find a relation between x and y that does not involve logarithms:
(a) log x + log y
log (x + y)
(b) 21og lo y - 3log 1o X
(c) 2log 3 y - 3log3 X = 2
(d) 21og 10 y + 3log 1o X
(e) logs y = 2 + logs x
(f) log y = log 5 + 3log
~ (g) 2log x + 3log y = 0
(h) loglo (l + y)
loglo

loglo (2x)
x
(1 - y) = x

13/INDICES AND LOGARITHMS 295

9. Solve the following for x:


(a) loglo 2 + loglO 5 + loglo X - loglo 3
2
(b) 21og 10 x + 3 = 5log) 0 X
(c) log)O 2 + 51og) 0 X - log)O 5 - log) 0 (X3)
(d) log) 0 x = 4log)o 2 - 2log) 0 x
(e) log)O x - loglo (x - 1) = 1
(f) log)O x
2log) 0 3 + log)o 5 - log) 0 2

10. If X

log) 02 and y
2
(a) log)O 3'

log) 0 40

= log)O 3, express the following in terms of x and y:


4
(b) log) 0 9

(c) log)o 15
(e) log) 0 5'4
(g) log) 0 150
0) log) 0 4'5
(k) log) 0 81

(d) loglo 54
(f) log) 0 75

(h) log) 0 027

(j) loglO 0'6

(I) log) 0 18

11. If log) 0 2 = 0'3010, log)o 5 = 06990 and log) 0 6 = 0'7782, write down the value of
loglo x for the following values of x: 3, 4, 8, 9, 10, 0'5,
of y
loglo x.

j-, 02 and 01. Plot the graph

12. If y = al0 bx , express x in terms of the other symbols.

+ log) 0 P, express A in terms of the other symbols.


log a + n log x, find an expression for y.

13. If loglo A = bt
14. If log y

log x
.
f
log 2' express x III terms 0 y.

15 If Y

16. Express log x in terms of log a, log b and log c, given that x = a 2 -Jli3C.

17_ If log x
18. If y

= 0-6 and log y

= ae4/ ,

0-2, evaluate log

(~)_

express t in terms of a and y.

19. If 10g b a = p and c = a2 , find, in terms of p,


(a) 10gb c,
(b) loge b.
20. If loga 2

= 10gb 16, show that b

= a4

21. Find the values of x for which:


(a) (log) 0 x)(log) 0 X2) + log)o x 3 - 5 = 0
(b) (loglo X)2 - log)o X2 + 1 = 0
(c) (log2 X2)2
log2 x 3 - 10 = 0
(d) (log3 x)2 = log3 x 5 - 6

Formula for converting from one base to another.


loga n
Let
Then

loga n = y

aY

296 NEW SENIOR MATHS: TWO UNIT COURSE

Take logarithms of both sides of (i) to base b,

Hence
y 10gb a
10g b n

10g
b n 'f 1
y = -1
- - , 1 0 gb a
O.
ogb a
. 1
- 10g b n
I.e. oga n - -1-
ogb a
The most useful application of this formula is for conversion from any base to base 10
because usually logarithm tables to base 10 are the only tables available; in fact other
tables, except possibly tables of natural logarithms, are not necessary. Also your
calculator has a 'log' key, base 10, Hence, for practical purposes, the conversion formula
may be written as:

EXAMPLE 10
Evaluate logz 9.
Let
Then
Take logarithms of both sides to base 10.
Hence
xlog] 0 2
log] 0 9
log]o 9
x
log]02
09542
0'3010' using base 10 tables.
logz 9

Thus

3'170,

(1)

dividing 09542 by 03010.

Using a calculator we proceed with (1) as follows:

Display

Operation sequence
9 /logl

2 iIogl

Note: Since 23
between 3 and 4.

EJ

8 and 24

31699249
16 and 9 lies between 8 and 16, its follows that logz 9 lies

EXAMPLE 11
Solve the inequations (a) 2x > 9

(b) 04 x

< 03

(a) From the working above, 2x = 9 when x = 3'170. The graph of y


to the right and so 2x > 9 when x > 3170,

Check the result approximately from the graph of y


2x ,

(b) Put O'4 x


Hence

2x slopes upwards

03, Take logarithms of both sides to base 10.


xlog j 0 0,4
log] 0 O' 3
log] 0 0,3
X
log] 0 0,4

Using a calculator, we then proceed as follows:

Operation sequence
C

3 jIog/

The graph of y

D 4/log/

Display
1'3139638

0'4 x slopes downwards to the right, and so 0'4 x < 03 when x > 1314,

13/INDICESAND LOGARITHMS 297

EXAMPLE 12
Using the same scale and axes, sketch the graphs of
(a) y = log2 x
(b) Y
log2 2x

(c) y

= log2 (x

- 1)

IOQ2X

= IOQ2 (x

1)

Fig. 13-6

(a) The graph of y = log2 x crosses the X-axis where x l , since log a 1 O. The domain
is x > 0, the range is any real number. As x - + 0 from the right, y -+ - 00.

= 1 i.e. x = ~.
the graph of y = log2

log2 2x crosses the X-axis where 2x

(b) The graph of y

Since log2 2x
log2 2 + log2 X = 1 + log2 x,
2x is 1 unit
above the graph of y = log2 x. The graph of y = log2 X is translated 1 unit parallel to
the Y-axis.
(c) The graph of y = log2 (x
1) crosses the X-axis where x - I
1 i.e. x
The graph of y = log2 X is translated 1 unit parallel to the X-axis.

2.

Note: The general shape of the log curve is not affected by the base a.
EXERCISES 13(d)
1. Use the change of base formula and a calculator to evaluate to four figures:
(i) log2 5
(ii) log3 12
(iii) logs 20
(v) log3 16
(vi) log6 4
(iv) log4 3
(ix) log2 10
(vii) logs 3
(viii) log3 5

2. Find the value of x for which


(ii) 3x = 18
(i) 2x = 7
(iv) 04x = 2
(v) 6 x = 21
(vii) 5x = 16
(viii) 4 x
5

(iii) 5x

(vi) 3-x
(ix) 2-x

3. Complete the following table and then draw the graph of y

01
5

logs x

4. Complete the following table and then draw the graph of y


x

log4 x

298 NEW SENIOR MATHS: TWO UNIT COURSE

5. Find the values of x (to 2 decimal places) for which


(a) 5x ~ 2
(b) l'6x ~ 05
(c) 3 x < 02
x

x
(e) 2 ~ 5
(f) 025- < 1-5
(g) 08 x < 3

(d) 3-x > 27

(h) 0.7 x ~ 03

6. Using the same scale and axes, sketch the family of curves y
7. Using the same scale and axes, sketch the family of curves y

aX for a

1
2,2,3, 10.

= logo x for a =

1, 2, 3, 10.

8. Sketch the graphs of the following, stating their largest possible domain and range in
each case.
(i) f(x) = loglo 2x
(ii) f(x) = loglo x 2 , X > 0
(iii) f(x) = log) 0 (2x + 3)
(iv) f(x)
log) 0 (3x
6)
(v) f(x)
log) 0 Ixl
9. Sketch the graphs of the following, stating their largest possible domain and range in
each case.
(i) f (x) = e 2x
(ii) f(x) = 3ex + 1

(iii) f(x) = e-2x

(iv) f(x) = 2e-2x


2

REVISION EXERCISES B

1. Calculate the cosine of the smallest angle of the triangle whose sides are 5 cm, 6 cm,
7cm.

2. Prove that the triangle whose sides are 5 cm, 6 cm and 8 cm is an obtuse angled
triangle. Find the size of the obtuse angle.
3. In DABC, LB

53, L C = 48, AC = 8cm, find the length of BC.

4. (a) A(O, 0), B(6, 0), C(8, 2) and D are the vertices of a parallelogram ABCD. Find the
coordinates of D.
(b) This parallelogram is rotated in the plane through 90 in an anticlockwise
direction about the origin. Find the equations of AD and AC.
5. (i) The line 3x
2y = 6 is rotated in the plane through an angle of 90 about its
point of intersection with (a) the X-axis (b) the Y-axis. Find the equation of the
line in these positions.
(ii) Find the coordinates of the points of intersection of the lines 3y
5x - 19,
3x + 5y + 9 = 0 and 4y = x + 3.
Show that the triangle with the points of intersection as vertices is right angled.
6. Find the equation of the line perpendicular to the line 3x + 4y = 5 and passing
through the mid-point of the lIne segment joining the points (3, -2) and (5, 8).
7. Find the perpendicular distance from the point (2, 3) to the line 6x

8y

5.

8. Find the equation of the locus of a point which moves so that:


(i) its distance from the point (4, 1) is equal to its distance from the point (2, -5).
(ii) its distance from the point (-2, 3) is 5 units.
23 = O.
9. The equation of a circle is X2 + y2 - 2x - 2y
(a) Find its centre and radius.
(b) Prove that the point (7, -2) lies outside the circle.
(c) Find the distance from the point (7, -2) to the centre.
(d) Use the Theorem of Pythagoras to find the length of the tangent to the circle from
the point (7, -2).
10. Find the coordinates of the point A on the line x = -3 such that the line joining A to B
(3, 5) is perpendicular to the line 2x + 5y = 12.
11. Find the sum of all the numbers between 20 and 200 that are divisible by 9.

12. (i) Evaluate 6 + 3 + 1'5 + ...


(ii) A ball is dropped from a height of 20 m and rebounds to a height of 18 m. If each
time it rebounds it rises to hine-tenths of the previous height, calculate the total
distance it could travel.

13. Three numbers whose sum is 15 are successive terms of an arithmetic sequence. If 1, 1
and 4 are added to these numbers respectively, the resulting numbers are successive
terms of a geometric sequence. Find the numbers.
14. If -J3 - 1 and 2
-J3 are consecutive terms of a geometric sequence, write down in
simplest surd form, the next two terms and the sum to infinity.

I~

300 NEW SENIOR MATHS: TWO UNIT COURSE

15. (i) Express 02333 ... in the form m where m and n are integers.

(ii) The first, third and ninth terms of an arithmetic sequence form the terms of a

geometric sequence. Find the common ratio of the geometric sequence.


16. Find the values of x for which
(a) 7x+2
343
(b) 4x - 2 < 128

(c) 3 x

12

17. (i) For the function defined by Sn


n 2 - 3n for n = 1, 2, 3, ... find In and hence
show that the sequence is arithmetic.
(ii) For a geometric sequence, the second term is 6 and the fifth term is 48. Find the
sum of the first five terms.
18. Simplify:
(a) log3 18 + 2log 3 9 - log3 54
(b) loga (xy2) + log a (yZ2) - log a (XZ2)

6 + 4-J6
2-J6 - 3

(c) loglo
+ loglo
2
(d) 2log (x

1) - log (x

1) - 2 log (y

+ 1) + log (y

1) given x

5, y

= 2.

19. Express y in terms of x in each of the following


(a) logaY
x
(b) logloY = 2 + loglox - loglo (X2)
20. At the beginning of each year $100 is placed in a fund. Calculate the accumulated
value at the end of 12 years, reckoning compound interest at 6 per cent per annum.
If, however, the $100 due at the beginning of the fifth year was not placed in the
fund, what then will be the accumulated value at the end of 12 years?
21. The population of a certain town is 24,000 in 1960. Each year, thereafter, its
population increased by 25070 of its population during the previous year. What would
the population be in 1980?
22. In 1975, 200 students enrolled for a certain course at Sydney University. This number
increased each year by 10 per cent of the number in the preceding year.
(a) How many students enrolled in 1980?
(b) What was the total enrolment over this six-year period?
23. Find the sum of the series

1
1
1

4 + 16 - 64 + ...

24. Find by 'completing the square' method


(a) the maximum value of 5 + 4x
X2,
(b) the minimum value of 2X2 + 6x
9.
25. Without actually solving the equations, determine whether the following quadratic
equations have two, one or no roots:
(b) 2X2 - 3x - 7 = 0
(a) X2
2x + 5 = 0
(d) 3x2 + 4x
1 = 0
(c) X2 - 20x + 100 = 0
26. Solve the following equations:
(a) (X2 - X)2
5(x2 - x) + 6 = 0
(b) X4 - 4X2 - 45 = 0

(c) 32x
- 12(3)x + 27 = 0

27. For what values of k does the quadratic equation X2


(a) two roots
(b) one root

5x

(k -

1)

(c) no roots?

o have:

REVISION EXERCISES B 301

28. Show that the roots of the equation mx2


rational values of m and n.

(m

n)x

n = 0 are rational for all

29. Find the coordinates of the points of intersection of the line y = 2x - 3 and the
parabola y = X2
4x + 5.
30. If 2X2

9x

+ 9 ==

(ax - b)(x - b) for all values of x, find the values of a and b.

31. For what values of m does the line y = mx - 5 (a) touch (b) intersect (c) not intersect
the parabola y = X2 - 5x + 4?
32. Find the equation of the locus of a point P(x, y) which moves so that its distance from
the point (-2, 1) is equal to its distance from the line y = -1. Express the locus in the
form (x - h)2 = 4a(y
k) and hence show that it is a parabola.
Find the vertex, focus, directrix, axis and focal length.
33. Express 4y
X2 - 6x + 5 in the form (x - h)2 = 4a(y - k) and hence find the
vertex, focus, directrix, axis and focal length of the parabola.
34. Complete the following table and then draw the graph of y = log2 x using graph
paper:
x

Using the same scale and axes indicate the position of the graphs of y = log2
log2 (x + 2).

X2,

> 0 andy

35. If a rectangle whose area is 192 cm2 has its length decreased by 6 cm and its width by
2 cm, it becomes a square. Find the dimensions of the rectangle.
36. The parabolay = ax2
values of a and b.

+ bx + 4 passes through the points (-2,

18) and (1, 3). Find the

37. Show, algebraically, that the line y


x - 4 is a tangent to the circle X2
find the coordinates of the point of contact.

+ y2

8 and

38. A ladder 8 m long rests against a wall and its foot makes an- angle of 60 with the
horizontal ground. The top of the ladder slips down the wall until its foot makes an
angle of 45 with the ground. Find, in simplest surd form, how far down the wall the
ladder slips.
39. From a point A, level with the foot of a vertical pole and 30 m from it, the angle of
elevation of the top is 40. Calculate
(a) the height of the pole,
(b) the direct distance from A to the top of the pole,
(c) the angle of elevation, from A, of a point halfway up the pole.
40. AB and CD are two vertical buildings with their bases A and C on horizontal ground.
The height of AB is 30 m. The angle of elevation of B as seen from Cis 25 and the
angle of elevation of D as seen from A is 40 .
Calculate:
(a) the horizontal distance between the buildings,
(b) the height of CD,
(c) the angle of depression of B as seen from D.

302 NEW SENIOR MATHS: TWO UNIT COURSE

41. Two yachts sail in a straight line from a buoy B. One sails 10 km in the direction
040 0 T and the other sails 20 km in the direction 160 0 T in the same time.
(a) How far apart are they?
(b) What is the bearing of the first yacht as seen from the second?
42. (i) Calculate the distance of the point (3, 4) from the line x + y = 6.
(ii) A(l, 4), B(O, 3), C(-4, 6) are the vertices of a triangle ABC. Calculate the distance
of A from the side BC and hence find the area of """ABC.
43. (a) Find the equation of the perpendicular bisector of the line joining the points
A(4, 2) and BC-6, 4).
(b) Prove that the point P(-I, 3) is on this perpendicular bisector.
(c) Find the distance of P from A and B. What do you conclude?
44. (a) Prove that the points A(-2, -3), B(6, 2), C(8, 7) and D(O, 2) are the vertices of a
parallelogram.
(b) Find the coordinates of the mid-points of the diagonals AC and BD.
(c) What property of parallelograms is illustrated in (b).

CHAPTER 14

Differential
calculus
The development of the calculus was centred around the solution of two classes of
problem that had puzzled mathematicians for centuries. These were finding the gradient
at a point on a curve (such as a parabola) and calculating the area enclosed by curved
boundaries. While several methods are available (e.g. coordinate geometry) for finding
the gradients of straight lines, and while areas of rectangles are given by simple formulae,
the challenge presented by general curves proved too great. /
Archimedes (287 - 212 RC.) made some progress with the area problem, but it was left
to Newton (England):and Leibniz (Germany) in the seventeenth century to facilitate the
breakthrough and, indeed, to show that both problems are related.
Since relationships in the real world can be represented by mathematical functions that
can, in turn, be depicted graphically, the development of the calculus has provided a
powerful tool for solving problems in science, engineering, industry, economics,
medicine, the social sciences and many other areas. Every such application is related in
some way to one of the two classes of problem mentioned previously.
An introductory problem
Suppose that a ball dropped from the top of a high building passes us (on one of the
middle floors) two seconds later. We are interested in knowing the speed with which it
passes us. The law of falling bodies tells us that the distance travelled (dm) is related to the
time in motion (ts) by the formula d = 4'9/2 , 1 ~ O. We shall assume d = 5[2 to simplify
the arithmetic.
The problem could be approached as follows:
(a) Measure the distance travelled during a small time interval, which ends as the ball
passes us.
(b) Calculate the average speed during this time interval using the formula
distance travelled
average speed
time taken
If we make our time interval very small, then we should obtain a good estimate of the
speed with which the ball goes by.
Let 11 and 12 denote the times the ball has been in motion
(i) just before it reaches us,
(ii) as it reaches us.
The corresponding distances that the ball has fallen are d 1 and d2
d - d
Hence average speed v = 2
1 1
12 -

304 NEW SENIOR MATHS: TWO UNIT COURSE

We suppose that our timing and distance measuring devices allow us to take measure
ments over successively smaller time intervals. The following table shows values of the
average speed calculated for time intervals that are made smaller and smaller.
Ij

1
15
1'9
199
1999

dl

t2

5
1125
1805
198005
19980005

2
2
2
2
2

d2

t2 -

20
20
20
20
20

1
05
01
001
0001

tl

d2

dl

15
8'75
195
01995
0019995

V =

d2

dl

t2 -

tl

15
175
195
1995
19995

The table shows that, as our time interval (t2 - t l) becomes smaller and smaller, the
value of the average speed approaches closer and closer to 20.
A similar result is obtained if we take corresponding measurements during small time
intervals as the ball passes from our level to just below us.
However, if we put tl = t 2 , then we obtain v = 0/0. Since division by 0 is not allowed
(the resulting expression is indeterminate) we cannot find the exact speed at t
2 by
substituting t = 2 into the formula. The solving of problems like that of finding the exact
2 is what the differential calculus is all about.
speed at t
The appearance of the right-hand column in the table has the characteristic form that
we met when considering the limits of geometric series. However, in the present context,
our function is continuous (unlike the earlier case) and the limit-like behaviour occurs as
t2 tl 0, instead of n - 00. Then, before proceeding further, we need to obtain
further insight into the fundamental concept of limit and limiting behaviour, which are
the building blocks on which the calculus rests.

14.1 Limits _ _ _ _ _ _ _ _ _ _ _ _ _ _ __
The concept of a limit is a very important one in mathematics, and students need to
understand it before they proceed to the study of calculus. The word 'limit' is used
frequently in our everyday language; e.g., the speed limit is 60 km h- 1 in the suburbs.
What is the reaction of the driver as his speed approaches the statutory limit?
In this chapter, we consider the word 'limit' in a mathematical sense. We will be
concerned with, say,f(x) as x approaches a value a. In some cases, the function will not be
defined when x
a.
The notation used for the limit of f(x) as x approaches a is lim f(x) where 'lim' is the
x-a
abbreviation for 'limit'.
X a means 'x approaches the value a'.
(a) We have already considered the concept of a limit of a function in relation to the
infinite geometric series in chapter 11.
(b) Consider regular polygons of n sides, either inscribed in a circle or circumscribing a
circle.
n=4
n=5
n=6
n=8

Fig. 14-1

14/DIFFERENTIAL CALCULUS 305

We can make the perimeter and area of the polygon as close to the circumference and
area of the circle as we please by making n sufficiently large. As n increases, the difference
between the perimeter, P n , of the polygon and the circumference, C, of the circle becomes
less and less. In the limit, i.e. as n - 00, this difference is zero_ That is, n_<:x>
lim P n
C.
Similarly, as n increases, the difference between the area, An' of the polygon and the
area, A, of the circle becomes less and less. In the limit, i.e. as n - 00, this difference is
zero. That is, n_o>
lim An = A.
(c) Consider a sequence of rational number approximations to .fl:

1'4,1-41,1-414,1-4142,1'41421, ...
By taking a sufficiently large number of terms, we can make our approximation as
close as we please to .fl. If tn denotes the nth term of this sequence, then lim tn = .fl,
although there is no rational number whose value is exactly .fl.
n_o>

EXAMPLE 1
For the functionfwheref(x)

=x +

2, find lim (x + 2).


x-2

The domain of this function is the set of real numbers_ The following table showsf(x)
for values of x in the neighbourhood of 2.

195

199

1995 - 2

2-005

2-01

205

f(x)

395

399

3995

4005

401

405

- 4

The preceding table shows that, as x approaches 2 from either below or above 2, f(x)
approaches 4; Le., we can makef(x) as close as we like to 4 by making x sufficiently close
to 2. We write
lim (x + 2)
4.
x-2

In this example, you will notice thatf(2) = 4. Thus limf(x)


x-2

= f(2).

In this case, we say that the function is continuous at x = 2. However, we are not
concerned with f(x) when x
2, but when x - 2, even though f(x) is defined when

2.

EXAMPLE 2
For the functionfwheref(x) =

X2 X

4 find lim

X2 -

x-2 X

If x = 2, this function is not defined. Why?

X2 - 4
(x - 2)(x + 2)

If x =1= 2,
2
x + 2.

x-

:. lim

X2 -

x-2 X -

If x

=1=

4 = lim (x
x-2

2, then f(x)

+ 2)
X2 -

x-

4.
4

is equivalent to f(x)

= x

+ 2, the graph of which is a

straight line with a missing point at x


2 (fig. 14-2). We say in this case that the function
is discontinuous at x = 2.
Note that, in this case, limf(x) =1= f(2).
x-2

I!

306 NEW SENIOR MATHS: TWO UNIT COURSE

Fig. 14-2

EXAMPLE 3
Consider the functions
(a) f(x)
(b) f(x)

{x +

= I~I,

~ :~:~ ~ ~

!:

x*"O

(a) The graph of this function is shown in Fig. 14-3.

The domain is the set of real numbers. What is lim f(x)?

x-I

If x - I from above, x-I+


lim f(x)

2. Test this by considering values of x

1'01,

1'001, ... remembering thatf(x) = x + 1 for these values of x. We use the symbol,
x - I + to indicate that x approaches 1 from above 1.

Y
Y

3
f(x)

[X+1'X~1
3, x < 1

:/
0

-2 -1

Fig. 14-3

If x - I from below, x-l


lim f(x)

X
1 f(x) = ~
x

Fig. 14-4

3. Test this by considering values of x = 0'99,

0'999, ... remembering thatf(x)


3 for these values of x.
However, as x - I without restriction from either below or above, lim f(x) does
not exist.

.,

14/DIFFERENTIAL CALCULUS 307

x
x

(b)

f(x)

1 when x > 0

~ (which is indeterminate)

when x

=0

- x = -I when x < 0
x
The graph is shown in Fig. 14-4.

O.

The function is defined for all values of x, except x


As x - 0 from above, f(x) - 1.

As x - 0 from below, f(x) - -1.

However, as x - 0 without restriction from either below or above, lim f(x) does

not exist. The function is discontinuous at x = O.

EXAMPLE 4
Find lim f(x
h-O

h) - f(x) where (a)f(x)


h

(a)

f(x
f(x
f(x

+ h)
+ h)

f(x

+ h)

f(x)
h)

- f(x)
- f(x)

X2

(b)f(x)

5x - 2X2.

= X2
= (x + h)2
= X2 + 2xh +
= 2xh + h 2
2xh + h 2

h2

/~!A

- f(x)

l' 2xh + h 2

h~
lim (2x + h) if h
h-O

"* 0

= 2x
(b)

f(x

hlEa

f(x)
h)

=
=

6
6

+ 5x - 2X2
+ 5(x + h) - 2(x + h)2
+ 5x + 5 h - 2X2 - 4xh -

=6
- f(x) = 5h - 4xh - 2h2
- f(x) _ 5h - 4xh - 2h2
h

f(x
f(x

+ h)
+ h)

f(x

+ h)

2h2

- f(x) = lim 5h - 4xh - 2h2

h-O

= h-O
lim (5
=5-

4x - 2h) if h

"* 0

4x

Theorems on Limits

The theorems on limits of functions will be stated but their proofs will be omitted.

Theorem 1
For the constant functionf, wheref(x) = c,
limf(x)
x-a

Theorem 2
If limf(x) = L and lim g(x)
x-a

x-a

= M,

then

lim (f(x) g(x

x-a

=c

= x-a
lim f(x)
=L M

lim g(x)
x-a

308 NEW SENIOR MATHS: TWO UNIT COURSE

lim (X2 - 3x

e.g.

x-2

5)

= x-2
lim (X2)

+ x-2
lim

- lim (3x)
x-2

=4-6+5
=3

Theorem 3

lim (f(x). g(x)) = lim f(x) .lim g(x)

x-a

x-a

x-a

= L.M.
lim 2x(x 2 - 4)

e.g.

x--J

= x--J
lim

(2x) .liin (X2 - 4)


x--J

= -2.-3
= 6
Theorem 4
. (f(X))
hm g-()
x-a
X

l~ f(x)

x-3

x-a

. X2 + 2
1l m - -

e.g.

L.

= r1m g ()
X = M If M *lim (X2

2)

lim (x

1)

x-3

+ 1

x-3

11
4
on limits are easily remembered in verbal form:
sum = the sum of the limits.
difference = the difference of the limits.
product = the product of the limits.
quotient = the quotient of the limits.

These theorems
1. Limit of a
2. Limit of a
3. Limit of a
4. Limit of a

EXERCISES 14(a)
Evaluate the following limits (1. to 22.):

2. lim (X2

3. lim (9 - X2)

4. lim (X2 - 2x

x--J

x-3

+ 2)

5. x--4
lim X2(X
7. lim (a

a--J

x~3

(x

11. lim x

x--2 X

13. lim

x-3 X

15. x-J
lim

6. lim (h 2

5)(x

+3

3)

10. lim X2
x-o

hl~

16. x-4
hm

+6
3
x - 5
- 9 x 5

x-I
X2

2X2h + 3h
18 /~
1
h
20

h)2 - 4

21. l~ f(x) where f(x)

5x

X -

14. x-5
lim 2 X 2

x-I

(2

x-3

17 1
X~2X2 - 4
19

12. lim X2 - 5x

3+X 2

+X
x + 2

+ 1)
+ 4)

. X2 - 5
8. x-3
hm
-+
2
X

+8
+2

X2

4h

h-2

3)(a - 4)

4x)

x--2

x-3

'''--.

1. lim (3x)

={

X2

+1
1

when x
when x

0,

< o.

hl~

(l

h)3
h

14/DIFFERENTIAL CALCULUS 309

J"()
h
fi()
22 11m
X were J X
x_I
!

23. Evaluate lim j(x

={

(c) j(x)

2x when x ~ 1,
+ 4 when x < 1.

2X

h) - j(x) where

h-O

(a) j(x) = X2 -

1,

(b) j(x)
2X2
3x
(d) j(x) = x(6 - x).

= x 3,

2,

Show that the following limits do not exist:


24. lim!
x-ox

25. xli_moj(x) wherej(x) = {01

when x < 0,
when x > 0.

26. limj(x) wherej(x)

= {xx +

27. !~j(x) wherej(x)

={

x-O

X2

when x > 0,
when x < 0.

+ 1 when x > 0,
2

when x < 0.

Definition: Continuity at a point


A functionj, which is defined in some neighbourhood of x
at c if
(a) the function has a definite value j(c) at c, and
(b) as x - + c,j(x) -+ j(c) as limit, i.e. limj(x) = j(c).

c, is said to be continuous

x-c

y
f (c)
((x)

Fig. 14-5
A functionjis said to be continuous in an interval (open or closed) 'if it is continuous at all
points of that interval.
For a closed interval [a, b], continuity at a and b implies that
lim j(x) = /(a), and lim_j(x) = j(b).
x-a+

x-b

In simple language, a function is continuous in the interval [a, b] if its graph can be
drawn from x = a to x = b without raising your pencil from the paper (fig. 14-5).
All the polynomial functions e.g. linear functions (straight line), quadratic functions
(parabola), are continuous. The sequence function, however, is not continuous. Why?

14.2 The gradient of a straight line _ _ _ _ __


You are familiar with the general linear relation of the form
ax + by + c = 0,

31U NEW SENIOR MATHS: TWO UNIT COURSE

where a, band c are constant; the graph of the relation is a straight line. Either a or b, but
not both, may be zero.
If b
0, this relation is a linear function and may be written in the form

ax

-Ii - b'

the graph of which is a straight line not perpendicular to the X-axis.


The gradient of the line is defined as the tangent of the angle that the straight line makes
with the positive direction of the X-axis.
i

Fig. 14-6

Fig. 14-7

Consider two points P(x] , YI) and Q(X2' Y2) to the right of P. The gradient, m, of the line
segment is defined by

m = Y2 - YI = '.i
.
X2 - Xl
h
Y2 - YI'

tan 0,

where h = x 2 Xl and k
(a) If h
0, i.e. x2 = Xl' the gradient is not defined.
(b) If k = 0, i.e. Y2 = Yl' the gradient is zero.
In fig. 14-6, the gradient is positive and the straight line makes an acute angle, 0, with
the positive direction of the X-axis; the line slopes upwards to the right.
In fig. 14-7, the gradient is negative and the straight line makes an obtuse angle, 0, with
the positive direction of the X-axis; the line slopes downwards to the right.
The gradient of a straight line is constant.

14.3 The gradient of a curve _ _ _ _ _ _ _ __


The gradient of a curve at any point on the curve is defined as the gradient of the tangent
to the curve at that point. Whereas the gradient of a straight line is constant, the gradient
of a curve is continually changing. It is the geometrical aspect of differential calculus that
is concerned with the gradient of a curve at any point.
In fig. 14-8, P(x,f(x is any point on the curve Y = f(x), which is continuous over the
interval [x, X + h], and Q is a neighbouring point (x + h,f(x + h. Then the gradient of
the secant PQ is
'.i _ f(x + h) - f(x) (h 0),

h h
wherek = f(x + h) - f(x).
Imagine the secant PQ revolving in a clockwise direction about the point P, taking up
the positions PQl' PQ2' and finally being the tangent PS at the point P. What happens to

"'-. -

14/DIFFERENTIAL CALCULUS 311

x
Fig. 14-8
the value of h? It obviously becomes smaller as Q gets closer and closer to P. The gradient
of the tangent at P is defined as the limit of the gradient of the secant PQ when h - O. It
is written thus:

Gradient

if

tangent at P =

/.

f(x

/~.

+ h)

. k
/lm

-f(x)

h--oh

EXAMPLE 5
For the graph of f, where f(x)
1.
where x

2X2, find the gradient of the tangent at P, the point


y

I
I

= ((1 + h)

((1)

.IzJ

x
Fig. 14-9
f(x)= 2X2
f(1)= 2
k

= f(1 +

f( 1 + h)
h)
f(1)

2( 1

h)2
2h2

4h

2h 2

4h +
~ = f( 1 + h) - f(1) (h =1= 0)
h
h
4h + 2h2
4 + 2h
h
= gradient of secant PQ

lim ~ = lim f(1


h-oh
h-O

+ h) - f(l)
h

312 NEW SENIOR MATHS: TWO UNIT COURSE

lim (4 + 2h)

h-O

= gradient. of tangent at P

EXAMPLE 6
For the graph of fwheref(x) = X2 - 5x

+ 6, find the gradient at any point x.

I
I
I
I

Ik

f (x + h) - f{x)

I
I

__ ...JI

Fig. 14-10

Let P(x,f(x be any point on the curvef(x) = X2 - 5x + 6 and Q(x+ h,f(x


a neighbouring point.
f(x) = X2 - 5x + 6
f(x + h)
(x + h)2 - 5(x + h) + 6
= X2 + 2xh + h 2 - 5x - 5h + 6
k
f(x + h) - f(x)
2xh + h 2 - 5h

~ = f(x

2xh

h)

f(x) (h

+ h be

*" 0)

h 2 - 5h

= 2x + h - 5
By definition, the gradient of the tangent at P
r f(x + h) - f(x)
hl~
h
= lim (2x + h - 5)

(1)

h-O

5
In evaluating the limit in (1), we have constructed a new function f' where
f'(X) = 2x - 5, from the original function f where f(x)
X2 - 5x + 6. This new
function f is called
(a) the gradient function, or
(b) the derivative off, denoted by f'.
The value of the derived function at any x is denoted by
d
dy
f'(X), dx (f(x, dx or y I where y
f(x).
= 2x -

The operation of obtaining f' (x) is called differentiation.


f '(X)

r k r
h1!!b It = l!!b

f(x + h) - f(x)
h

EXAMPLE 7
For the function in example 6, evaluate lim f(x) h-O

{(X - h).

_r

14/DIFFERENTIAL CALCULUS 313

We note that, this time, we are involved with secants drawn back from P the other way.
f(x) - f(i - h)

=
=

Thus

f(x) - f(x h

liE! f(x)

{(X

(x 2 - 5x + 6) - [(x - h)2 - 5(x - h) + 6]


X2
5x + 6 - X2 + 2xh - h 2 + 5x - 5h - 6
2xh
h 2 5h

hl

= 2x - 5

h)

2x - 5

(h

*- 0)

= gradient of tangent at P, as in example 6.

P(x, ((x))

I
I

:k

f(x) - f(x - h)

I
I

h ...JI
___

Q(x - h f(x - h))

o
Fig. 14-11

We note that the limit is the same whether we take Q to the right or left of P. This is a
property of all derivatives. If the left-hand and right-hand limits are different, then we say
the derivative at P does not exist. Derivatives always exist at points on a smoothly
continuous curve, i.e. a curve without any sharp points or breaks.

EXAMPLE 8
(a) Differentiate 5 - 3x - 2X2 from first principles.
(b) Find the value of f'(-2).
(c) For what value of x is f'(x)
01

5 - 3x - 2x 2

Fig. 14-12

314 NEW SENIOR MATHS: TWO UNIT COURSE

(a)

f(x
f(x+ h)

f(x) = 5 - 3x - 2X2
h) = 5 - 3(x + h)
2(x + h)2
= 5 - 3x - 3h - 2X2 - 4xh - 2h2
f(x) = -3h - 4xh - 2h2

f'(X)

liEA

=
=

(b)

f'(x)
f'(-2)

lim f(x
h-O

h) - f(x)
h
(-3 - 4x - 2h)

-3 - 4x

-3
-3

4x

+8

=5
= gradient of tangent at x

(c)

f'(x)

i.e.,

-3 - 4x

i.e.,

= -2

=0
=0
3

-4

At the point on the curve where x = -~, the gradient is zero; i.e., the tangent is
parallel to the X-axis.
Alternative Notation
So far we have used the letter h to denote the difference of two values of the variable x and
k for the corresponding difference of two values of the function, i.e. k = f(x + h)
f(x). The symbols ox or ax (delta x) and oy or ay (delta y) are frequently used in place of
hand k respectively. It must be understood that ox and oy are single numbers and do not
represent a product 0 X x or 0 X y. They merely represent a small change or increment in
the values of x and y. This increment may be positive or negative. If y = f(x) and ox is a
small increase in x, then
oy

Then, if y

= f(x +

ox) - f(x).

f(x),
dy

dx

f'(x) = lim f(x


ox-O

ox) - f(x)
ox

= lim

oy
ox-O ox

The following example will illustrate the use of these symbols.

EXAMPLE 9
Find, from first principles, the derivative of x 3
f(x)
f(x + ox)
oy = f(x
oy
ox

f(x

+ ox)
+ ox)

= x3
= (x +
= x3 +

OX)3
3X2(OX) + 3X(OX)2 + (OX)3
3X2(OX) + 3x(ox)2 + (OX)3

- f(x)
- f(x)

dy
dx

ox

3x2

3x(ox)

lim oy
ox-oox
= ox-O
lim [3x 2
= 3x2

(OX)2 if ox ::1= 0

3x(ox)

+ (OX)2]

14/DIFFERENTIAL CALCULUS 315

f(xl

Fig. 14-13
By looking at the method used in the previous examples, it becomes obvious that f'(x)
cannot be calculated in some instances.

f'(x) is given by the limit of the quotient f(x

hh - f(x) , and so depends upon the

computation of this quotient.


If f(x) is not known or is not finite, then the limit cannot be found. This means that a
function is not differentiable at points where it is undefined; perhaps a rather obvious
conclusion.
Thusf'(x) cannot be computed at x

= 0 whenf(x) = 1
4

X2 -

g'(x) cannot De computed at x = 2 when g(x) = --2-' The points x

x-

0 and x = 2

are not in the domains of f and g. In 14'5, we shall see that a function is not always
differentiable at points within its domain.
It is also evident thatf'(x) is not meaningful if y = f(x) defines a relation; for thenf(x)

h- f(x)

would have several different values for a given x and lim f(x + h
h-O

would be

meaningless.
Only whenf(x) is uniquely defined for each x does the calculation of the limit become
possible. We see then that the calculus is necessarily concerned with functions, and we
shall restrict the use of the symbols f andf(x) to functions.
In example 6, we found the gradient functionf'(x) = 2x - 5, to have the same domain
as f, from which it was derived, This is not always the case, as is illustrat~d by the
following example.

EXAMPLE 10
Find the gradient function of the function f, where f(x)
f'(x) = lim f(x

h-O

' 1 'lze t henumerator 0 f


W e now rahona

,
I1m

h-O

= ,.JX,

x ~ 0,

h) - f(x)

-JX+h

--~--

.Jx +

hh

-,.JX as f 0 11ows:

316 NEW SENIOR MATHS: TWO UNIT COURSE

___
-_vX_x x rx+h +.fie (h
h
-Jx + h + .fie
_
(x + h) - x
- h(-Jx + h + .fie)

* 0)
y

h
h(-Jx

.fie)

lim

f' (x)

h-O

--;==~_-=

1
2.fie

f' (x) is not defined for x = 0, and so 0 must be


excluded from the domain of f' , so that
f'(x) =

2~' x

Fig. 14-14

> O.

Clearly, the domain off'


domain offin this case.
The Y-axis is a tangent to the curve at x = 0 and the gradient of the Y-axis is not defined.
Why?

EXAMPLE 11
Find, from first principles, the derivative of!, x
x
f '(x) lim f(x

* O.

+ h)

- f(x)

h-O

-h
= lim -"-------"

(A/W)

h-O

-1
x(x + h)
1

lim

h-O

-"]>x
x

* o.

Fig. 14-15

Note that bothf(x) andf'(x) are not defined

when x = O.

14.4 Rules for differentiation _ _ _ _ _ _ _ __


1. Derivative of X"
We have seen that the derivative of X2 is 2x and that the derivative of x 3 is 3x2 Do you
observe a pattern? What are the derivatives of x4, x 5 and x 6?
Show, from first principles, that the derivative of X4 is 4x 3 and that the derivative of x 5
is 5x4.
i

Iff(x)

= xn,

thenf'(x)

nxn-I.

14/DIFFERENTIAL CALCULUS 317

I if f(x)

In general,

axn, thenf'(x)

d
dx (X4)

d
4x 3, dx (X5)

anxn- I.

5x4

A formal proof of the derivative of xn will not be considered at this stage, but our result
can be justified by finding the derivative of x4, x 5 , from first principles. Furthermore,
the result is true for all n E R. For example,

(X3/2)

= ~Xll2,

(X-I)

= - x-2(example 10).

2. Derivative of a constant
Iff(x) = a = axo, thenf'(x)
0 (using the derivative of xn where nO).
Geometrically, f(x)
a represents a straight line parallel to the X-axis and hence its
gradient is zero.

3. Derivative of a polynomial
In example 6, we saw that the derivative of x 2 5x + 6 was 2x - 5. That is,
d
d
d
d
dx(x 2 - 5x + 6) = dx(x 2) - dx(5x) + dx(6)
- 5
+ 0

- 5

In example 8, the derivative of 5 3x - 2X2 was -3 - 4x. That is,

d
d
d
d

dx(5
3x - 2X2) = dx(5) - dx(3x) - dx(2x 2)

= 2x

= 2x

-3
- 4x
From these examples, we observe that the derivative of a sum or difference of several
terms is equal to the sum or difference of the derivatives of the various terms. This is true for
all sums and differences.
If
f(x) = 3x4 - 5x2 + 2x + 4, then
f'(x) = 12x3 - lOx + 2

EXERCISES 14(b)
1. Find f(x

+ h~

- f(x), h

* 0, for the functions defined as follows:

(a) f(x) = 2X2 - 3x

2. Find f(x

0:; -

(a) f(x) = 4 - 3x

f(x), ox

(b) f(x) = 4x - 3x2

(c) f(x)

x3

* 0, for the following functions


(b) f(x) = 3x2

2x - 1 (c) f(x)

7x - 4X2

3. For the graph off(x)


6x - 2x2,
(a) find the gradient of the chord joining the points whose x-coordinates are 1 and
1 + h respectively;
(b) deduce the gradient of the curve at x = 1.

318 NEW SENIOR MATHS: TWO UNIT COURSE

4. For the functionf(x) = 2x2 - 4x, find the following:


(a) lim f(3 + h) - f(3)
(b) lim f(x + h) - f(x)

h-O
h
h-O
h

Interpret your results geometrically.


5. Find lim f(x
h-O

hh

(a) f(x) = 4X2 -

f(x) for the following:'

X2

= -2 -

(b) f(x)

2x - 3

(c) f(x)

= x3

2X2

6. Find, from first principles, the derivative of


(a) 3x2 - 4x + 1
(d) 3x2
2x 3

(b) 3 - 2x
(e) X4

7. For the graph of the equation y = x 3

(c) (x - 1)(x + 2)
(f) 4x( 1 - x 3)

4X2

6x, write down:

(a) dy
dx

(b) the gradient when x = -1


(c) the coordinates of the points at which the gradient is 6.
8. For the functionf(x) = 3x2
(a) f'(x)

2x

7, write down the value of

(b) f'(-2)

(c) f'(a)

X4
3x3
(b) f'(2)

9. For the functionfwheref(x)


(a) f'(a)

(d) f'(3)

+ 2x, write down the value of


(c) f'<-2)

10. For the following functions, find the value of x for which f'(x) is zero:
(a) f(x) = x 2 - 4x
. (c) f(x) = X2 - 6x

(b) f(x) = 2x 3
(d) f(x)
x3 -

11. Write down the derivatives of:


(a) 3x 3 - 2X2 + 5x + 4

6x

4X2

(b) 3x(x2 - 2)
(d) x(2x + 1)(3x

(c) (x - 2)(x + 1)(3x + 1)


2
3
(e) 3x + 2x + x, x =1= 0

+ bx2 +
+ a)3

(f) ax 3

3)3
(h) (x
Hint: Expand or simplify all expressions first.

(g) (2x -

2)
ex

12. Find the gradient of the parabola y = X2 - X - 6 at the points where it crosses the
X-axis.
13. For the graph off, where f(x)
f'(x) = -5.

x3

X2 - 6x

1, find the values of x for which

14. Show that the graph of y = X2 + 4x - 12 crosses the X-axis at two points. Find the
gradient of the curve at these points.

15. P(x, y) and Q(x + h, y + k) are two points on the curve with equationy = X2 - 3x.
(a) Find the gradient of PQ.
(b) Deduce the gradient of the tangent at P.
(c) Find the coordinates of the point on the curve at which the tangent is parallel to
(i) the X-axis,
(ii) the line with equation 3x + 2y - 5
O.
16. For the graph of f(x)
(a) f(x)( = 0

(x - 1)2 find the values of x for which:


(b) f'(x) = 0
(c) f'(x)

-1.

14/DIFFERENTIAL CALCULUS 319

.17. Find the coordinates of the points on the curve y = X2 5,X' + 6 at which the tangent:
(a) makes an angle .of 45 0 with the X-axis,
(b) is parallel to the line with equation 3x + y - 4 = 0,
(c) is perpendicular to the line with equation 2y - x + 3 = O.
18. Find the coordinates of the points on the curve y

= ~X3

~X2 + 2x +

1 at which the

tangent:
(a) is parallel to the X-axis,
(b) makes an angle with the X-axis whose tan is 2,
(c) is parallel to the line y - 6x
1 = O.
19. Find the coordinates of the points on the parabola y
(a) the gradient is zero,
(b) the tangent is parallel to the line 2x + y
7.

20. Find the coordinates of the points on the parabola y


tangent:
(a) is parallel to the X-axis,
(b) makes an angle of 45 with the X-axis,
(c) is parallel to the line y = 2x.

X2 -

2x - 8 at which:

2X2

4x

1 where the

21. For the parabolaj{x) = X2 + 6x + 8 find:


(a) j(2) (b) j'(2) (c) j'{c) (d) the value of c for which j'{c)

-3.

14.5 Conditions for differentiability _ _ _ _ __


EXAMPLE 12
Consider the functions whose graphs are shown

x
x

Fig. 14-16

Fig. 14-17

320 NEW SENIOR MATHS: TWO UNIT COURSE


y

~X) ~x+ 1
o

fix) = -1

Fig. 14-18

Fig. 14-19

I.

Fig. 14-16 shows the graph of f(x)


Fig. 14-17 shows the graph of f(x)

= -x
- 2 ' x '* 2

FIg. 14-18 shows the graph of f(x) =


.

Fig. 14-19 shows the graph of f(x)

x,*O

X2 -

{X

+ -11 if
x
'f
1 X

Ixl

~ 1.

< 1.

Remembering that the calculation of fl(a) involves the evaluation of


lim f(a + h) - f(a) we note that for

h
'

(a) fl(O) cannot be found since f(O) is undefined (fig. 14-16),


(b) fl(2) cannot be found sincef(2) is undefined (fig. 14-17),
(c) fl(1) cannot be found since the left-hand derivative does not exist at x
1, even
though f(l) is defined (fig. 14-18).
(d) f' (0) cannot be found since the left-hand derivative is -1 and the right-hand derivative
is 1 even though the function is continuous at x
0 (fig. 14-19).
h-O

In fig 14-20, try to draw one, and only one, tangent at each of the points x
a, band c.
At a and c, the function is continuous, and at b, discontinuous. At each of these points,
the function has no derivative.

I
I

I
c

Fig. 14-20

EXERCISES 14(c)
Write down the value of fl(a) in each case where it exists. Sketch the graph of f in each
case.

1. (a) f(x)
(c) f(x)

2, a = 2
4
--2,a=2

X2 -

x+

(b) f(x)
(d) f(x)

X2 X2

4, a
2x

14/DIFFERENTIAL CALCULUS 321

{X

I'.

,xX ~
id
3
> l
l' a
an a =

2. J (x) = (x _ 2)2,

=4

3. f(x) = Ix - 21, a = 2 and a

,x ~ 0

X2

4. f(x)

= { x + 1, x > 0' a = 0 an a = 1

5 ji()

2x, x ~ 2
2, x > 2' a

{X2
x

,x > 3
3, -3 ~ x
1, x < -3

6. f(x) =

x +

2 and a

3, a

=3

3 and a = -3

14.6 Further rules for differentiation _ _ _ _ __


4. Derivative of a product of two functions
If g and h are two differentiable functions, then

[5x

.=

!g(x). h(x)1

Or, if we denote g(x) and h(x) by

g '(x)h(x)

+ g(x)h '(x).

and v respectively, then

d
dx(uv)

du
v' dx

dv
u' dx

Let y = uv and let ox be a small increment in x and ou, OV and oy the corresponding
increments in u, v and y, caused by this increment in x.
Then
y + oy = (u + ou)(v + ov)
oy

As ox -

= uv + uov + vou + ou.ov


= uov + vou + ou.ov
uov + vou + ou. ov

oy
ox
ox
oX
_ dy OV _ dv ou _ du ou _ 0
dx' ox
dx' ox
dx'
dy
du
dv
dx = v' dx + u' dx

0,

Thus

EXAMPLE 13

+ 2)(2x2 - 3x + 4).
d
dx [(3x + 2)(2x2 - 3x

Differentiate (3x

4) I

+ 2) dx(2x 2 - 3x + 4) + (2X2 - 3x +
= (3x + 2)(4x - 3) + (2X2 - 3x + 4).3

18x2 - lOx + 6

= (3x

4) dx(3x

2)

When dealing with simple polynomial functions, the product rule for differentiation can
be avoided by expanding the product into a sum or difference. This is so in this example.

dx(3x

2)(2x2

3x

+ 4)}

dx(6x 3

5x2 + 6x

8)

18x2

However, not all products can be expanded into a sum or difference, e.g.

x sin x.

+6
x 2 -J.X+l;

lOx

322 NEW SENIOR MATHS: TWO UNIT COURSE

5. Derivative of composite functions -chain rule


(a) Consider the functionf(x) (x + 2)2. For any given value of x,f(x) is calculated by
adding 2 and then squaring the result. Thus we can consider f as being composed of
two functions, the first having the rule 'add 2' and the second having the rule 'square'.
More formally, we may write this using the function notation as
f(x)

g[h(x)]

where h(x) = x + 2. That is, h is the function defined by 'add 2' and g(x)
That is, g is defined by 'square'. We may also writef(x) in the form
f(x) = u 2, where u
x + 2.

f is sometimes referred to as a 'function of a function'.

x 2.

= g[h(x)], where the rule of the inner function, h, is


(b) Consider f(x)
'square and subtract l' and the rule of the outer function g is 'square root';
i.e., h(x)
X2 - 1 and g(x)
.JX. In other words,
f(x) = u 1l2 , where u = X2 - 1.

(c) Consider f(x)


may write

sin 3x

= g[h(x)], where h(x) = 3x and g is the sin function. Then we


f(x) = sin u, where u = 3x.

(d) Consider f(x) = cos2 x. For any given value of x, f(x) is calculated by finding cos x
and by then squaring the result. Then we may write

f(x) = u 2 , where u
cos x.

The derivatives of such composite functions can be found by using a simple rule,
frequently called the 'chain rule'.
If y .f(x) = g[h(x)]
g(u), where u = h(x), then
dy
dx

dy du
duo dx

Let ox be an increment in x and ou and oy the corresponding increments in u and y


respectively. Then, provided ou =1= 0,
oy
ox
As ox _ 0 ou _ 0 oy _ dy oy _ dy
,
, ox
dx' OU
du
dy
Hence
dx

oy ou
ou ox
and OU _ du
ox
dx

dy du

duo

Note: The chain rule holds even when OU

0, although it is trivial in such cases.

EXAMPLE 14
Find the derivatives of the following:

(a) f(x)

(2x

(c) f(x)

(2X2 - 3x

(a) Lety

(2x

(b) f(x)

1)3

1)3

1)4,

u 3 , where u
dy
dy du
dx
duo dx

(d) f(x)

2x

(X3 -

..J2x - 1, x ~

+ 1.

3u 2 .2

1)5

= 6(2x +

1)2

14/DIFFERENTIAL CALCULUS 323

(b) Lety

= (x 3

x3

1)5 = u 5 , where u

1.

dy _ dy du _ 5 4 3 2 - 15x2(x3 _ 1)4
dx - du' dx - u. x
(c) Let y

(2X2

u4, where u = 2X2 - 3x


dy du
3
du'dx
4u .(4x - 3)

+ 1)4

3x

dy
dx
=

(d) Let y

= ~2x

- 1

U 112,

4(4x

3)(2x2

where u

+ 1)3

3x

2x - 1, x

dy du _ !. -112 2 _
du . dx - 2 u
. -

dy
dx

+ 1

&
1

-;::;;====-

x>
2

After a little practice, the symbol u need not be used. The result can be obtained in one
step. For example,

(X2

3X)8 = 8(X2 - 3x)1(2x - 3).

We can use the derivative of )('/, n a positive integer, and the chain rule to find the
derivative of x lln and xm1n .
Let y = X
(xlln)n = un where u = X11n
dy dy du
dx du'dx
d
nun-I. dx(x lln )
.

e.g.
Let y

~(Xlln)

dx

_(XIl3)

dx
x m1n

(xlln)m

1
!.u1-n = !.(xlln)l-n
_1_
-X l1n - 1
- nun- 1
n
n
n
1
1
1 = -x-2I3
= -X Il3
3
3

u mwhere u = Xlin .
dy _ dy du
dx - duo dx
1 lln - 1
= mum-I. -X

mxmln-l

d
dX(X 3/4 )

e.g.

3
3
4X3l4-1 = 4X-1I4

Sometimes it will be necessary to use the chain rule in conjunction with the product rule
and other rules in differentiation.

EXAMPLE 15
Differentiate the following:
(a) (x 2 -4x)(3x2 - 2x + 1)5
(a)

!(X 2 - 4x)(3x2 - 2x
(X2 - 4x)

(3X2 - 2x

(b) X2

5x

!.ix2 - 4

1)5

+ 1)5 + (3X2

- 2x

+ 1)5

(X2 - 4x) (using product rule)

324 NEW SENIOR MATHS: TWO UNIT COURSE

= 5(x2 - 4x)(6x - 2)(3x2 - 2x + 1)4 + (3x 2 - 2x + 1)5(2x


(b)

d
dx(x 2 + 5x

4)

3~

"x 2 - 4)

d
d
d
= _(X2)
+ -(5x)
+ _(X2
- 4)113
dx

= 2x + 5 +

dx
2x

dx

4>-2/3,

"* -

2, 2

6. Derivative of a quotient
g(x)
If f(x) = h(x)' h(x)

"*

O,.-,_th_e_n_ _ _ _ _ _ _ _ _--.
f

'( )
x

h(x)g '(x) - g(x)h '(x)


Ih(x) J2

u where u

In other words, if y

g(x), v

h(x) and y

du

f(x), then

dv

v. dX - u. dX.

dy
dx

,2

Establish this formula as an exercise.


Consider y

= u

uv- I where u and v are differentiable functions of x, and use the

product rule in conjunction with the chain rule.

EXAMPLE 16
Differentiate the following, using the quotient rule.
(a) 2x

(a) Let

1 x*-

(b)

2x + 1
4x - 3

u
v

dy

dx

(4x - 3) dX(2x

2(4x - 3) - 4(2x
(4x - 3)2

-10

y
dy
dx

(l

+
+

1)

"* 4

(4x - 3)2'

(b) Let

1) - (2x

X2

::::: -

X2) ix(X) -

-,----- x

R.

x-(l
dx

X2)

l)dX(4x

3)

14/DIFFERENTIAL CALCULUS 325

Earlier we stated that

(xn) = nxn- 1 in the case where n EN, which, of course, covers the

case of positive integers.


The quotient rule enables us, by considering -n where n EN, to formally extend this
result to the case where the exponent is a negative integer.
Let y

= x p where p

-n, n EN. Then y

By the quotient rule,


dy
dx

(xn)O nxn -

~(x
xn

= x-n

1.(nxn-l)
x2n

"* 0 is assumed).

(using previous result for natural number


exponents)

( -n)x- n -

= -no

Butp

dy = pxp - 1
dx

Hence
d
dx

Thus

-(xP)

pxp - 1 for p E J

T.hroughout this chapter, certain 'rules' for differ~ntiation have been shown. These rules
have reduced the process of differentiation to a routine operation, and this is precisely the
meaning of the word 'calculus'. However, students are urged to give careful consideration
to domains of functions and their derivatives.

EXERCISES 14(d)
1. Using the product rule, find the derivatives of the following:
(x - 2)(6x + 7)
(3x + 4)(X2 - 2x)
(2X2 - 5x)(x - 2)
(x - 1)(3x + 5)
(4x - 1)(5x2 - 7)

(a)
(c)
(e)
(g)

(0

(b) (2x + 1)(x + 3)


(d) (x - 1)(x2 - 3x)
4X)(X2 + 3)
(f) (x 2
(h) (x 2 - 5x)(2x + 3)

G) (3x - 1)(3x2 + 1)

2. Show that the following are composite functions, and hence write down their
derivatives:
(a)
(c)
(e)
(g)

f(x) = (X2
4)5
Y
(X3
3X)4
y = .(riZ+3x
y = .JX2 - 2x

(b) f(x)
(d) y
(2x + 5)-1
(f) y
(2X2 + 5x - 4)4
(h) f(t) = (t2 + 4t2

3. Use the chain rule to differentiate:


(a)
(c)
(e)
(g)

(b) (X2


(1 + 2xtl
(2x - 1)5
(3x 2 - 2x -

+ 1)112

(d) (X2 + 25)2

rx+1

1)4

(f)
(h) .Jx2

2x

4. Use the quotient rule to differentiate:


(a) x - I
(c)

x+
2x + 5
x+

(b) 3x - 7
4x + 5

(d) - - : : - - - -

326 NEW SENIOR MATHS: TWO UNIT COURSE

(e)

+ 3x +

X2

4
(f)

4X2

(h) 4X2

+ 5

(g) 2x

x
x - 3
2

5. Differentiate:
(a) (x 2 - 4)(x + 2)
(c) -J(5x - 1)

x - 2

(e) x

+5

5x

(g) X2 -

(i)

(b) 4X512 - 2X 312 + 6x l12


(d) x 2l3 + (2x - 1)3
(f) -J(X2

x +

(h)

(j) (x 2

2)

7x)(x

1)

6. Differentiate:
(b) (x 2 + X 3)5
(d) (x + X- I)2

(a) (x - 2)3
(c) -J25 - X2

(e) (x

4)2

(f)

(g) (x

1)6(x

(i)

X -

x
2)

+ 3x +

(h) X2

(j) 2x - 7
2x + 3

7. Differentiate:

+ -J9 (5x + 7)312

(a) x 3

(c)

(e) ~X2
(g) 2x

+ 5x

+ 7

X2

(b) 1

+ x3

(d) (4X2
(f) (x 2

(h) (x

5x +

1)2/3

2)-2

3)rx=3"

CHAPTER 15

Geometrical
Applications of
Differentiation
15.1 Sign of the derivative _ _ _ _ _ _ _ _ __
You are familiar with sketching the graphs of quadratic functions by finding their points
of intersection with the X-axis, and by considering the sign of the function. This provides
a limited amount of information. Further information can be gained by the use of the
derivative, which enables us to find the gradient at any point and, in particular, the
coordinates of stationary points, i.e. points at which the tangent is parallel to the X-axis.
Y
Consider the graph of a continuous function in the
neighbourhood of a stationary point B at x = b
(fig. 15-1). At B, f'(x) = O.
For all x in the interval a to b (written as interval
(a, b)), the curve slopes upwards, the tangent to the
curve at any point in this interval makes an acute angle
x
with the X-axis, and so the gradient is positive; i.e.
f'(x) > O.
Fig. 15-1
Also, as x increases from a to b, f(x) increases, and so we say that f is an increasing
function in the interval (a, b).
For all x in the interval (b, c), the curve slopes downwards; the tangent to the curve at
any point in this interval makes an obtuse angle with the X-axis, and so the gradient is
negative; i.e. f'(x) < O.
Also, as x increases from b to c, f(x) decreases, and so we say that f is a decreasing
function in the interval (b, c).
The point B is called a local maximum turning point.
Y
Consider the graph of a continuous function in the
b
neighbourhood of a stationary point B at x
(fig. 15-2). At B, f'(x) = O.
For all x in the interval (a, b), the curve slopes down
wards, and so the gradient is negative; i.e. f'(x) < O.
x
For all x in the interval (b, c), the curve slopes
upwards and so the gradient is positive; i.e. f'(x) > O.
Fig. 15~2
The point B is called a local miniumum turning point.

328 NEW SENIOR MATHS: TWO UNIT COURSE

15.2 Turning points: local maximum


and minimum ______________
A turning point is a point on a curve at which the tangent is parallel to the X-axis andJ(x)
at that point is either greater than or less than J(x) in the immediate neighbourhood of
that point; a turning point may be defined as a point at whichJ'(x) = 0 andJ'(x) changes
sign fro'm positive to the left of the point to negative to the right of the point, or from
negative to the left of the point to positive to the right of the point.
Thus points B in figs. 15-1 and 15-2 are turning points.
A function has a local maximum value at a
x a
point x = a if its value at that point is greater
than its value at a point in the immediate neigh
+

bourhood; that is,

J'(x) > 0 if x
J'(x) = 0 at x
J'(x) < 0 if x

< a (but near a)


>

Fig. 15-3

a
a (but near a)

A function has a local minimum at x = a if


its value at that point is less than its value at a
point in the immediate neighbourhood; that is,
J'(x) < 0 if x < a (but near a)
J'(x)
0 at x
a
J'(x) > 0 if x > a (but near a)

Fig. 15-4

EXAMPLE 1
Sketch the graph of a continuous functionJ, given thatJ(l)
all x < 1, andJ'(x) < 0 for all x > 1.
Since J'(1) = 0, the tangent is parallel to the
X-axis at the point (l, 3).
Since J'(x) > 0 for all x < 1, the curve slopes
upwards when x < 1.
Since J'(x) < 0 for all x > 1, the curve slopes
downwards when x > 1.
Hence the graph of J is a curve with a local maxi
mum turning point at (1, 3) (fig. 15-5). Note that we
have not been given sufficient information to state
the rule of J.

3,J'(I)

= O,J'(x) > 0 for

x
Fig. 15-5
y

EXAMPLE 2
Sketch the graph of J(x)

X2 -

You are familiar with the fact


that the graph of J is a parabola
with its vertex downwards. Con
sider now a calculus approach to
sketching the curve.
J(x) = X2 - 6x + 8
J'(x) = 2x - 6
= 0 when x
3
J(3) = 9 - 18 + 8
- 1

6x

+ 8.

x
Figs 15-6 and 15-7

15/ EOMETRICAL APPLICATIONS OF DIFFERENTIATION 329

2x - 6 (fig. 15-7) provides us


H ce (3, -1) is a sta' ary point. The graph of f' (x)
w' h the information conc ing the gradients off. For all x < 3, ff (x) < 0, and so the
raph of f slopes downwards. For all x > 3, f' (x) > 0 and so the graph of f slopes
upwards.
Hence the point (3, -1) is a minimum turning point.
The quadratic function has a minimum value of ~1 when x = 3.
The graph off crosses the X-axis at x = 2 and x = 4, and the Y-axis at y = 8, where
y = f(x).
The following steps will be useful for curve sketching of polynomial functions:
(a) Find the turning points, indicating whether they are maximum or minimum points.
(b) Find the points, if possible, where the curve intersects the X-axis by finding the
solution set of the equation f(x)
O.
(c) Find the point where the curve intersects the Y-axis by finding the value of f(O).

EXAMPLE 3
Sketch the graph off(x) x 3 - 12x + 16, in the domain -4 ~ x ~ 3 locating the turning
points and stating whether they are maximum or minimum points. State the range of f(x)
and the coordinates of the extreme points.
f '(x)
f '(x)
3x2 _ 12
f(x) = x 3 - 12x + 16
f'(x) = 3x2
12

= 3(x - 2)(x + 2)

= 0 when x = 2 or x = - 2

f(2)
8 - 24 + 16 = 0
f( 2) = -8 + 24 + 16 = 32
The coordinates of the stationary points
are (2, 0) and (-2, 32).
From the graph of the gradient function
(fig. 15-8), we see that, in the neighbourhood of x = -2,

Fig. 15-8

'
.
when x < -2, f'(x)
'() > 0O} an d so (2
- , 32)'IS a 1ocal
maxImum
POInt.
w hen x > -,
2 f x <

In the neighbourhood of x = 2,

when x < 2, f'(x) < O} d


..
.

'() 0 an so (2 , 0)'IS a 1oca>1 mInImUm


POInt.
2 f x >

wh enx >,
f(O) = 16, and so the curve crosses the Y-axis at (0, 16).

f(-4) = 0 andf(3) = 7. The coordinates of the extreme points are (-4, 0) and (3, 7). The

range is 0 ~ f(x) ~ 32.

2,32)

y
12x + 16,

(3,7)
(-4,0)

Fig. 15-9

(2,0)

:3

x~ 3

330 NEW SENIOR MATHS: TWO UNIT COURSE

EXAMPLE 4
The function with rule y
3x2 - x 3 is defined on the domain -2 ~ x
(a) the stationary points and their nature,
(b) the greatest and least values of y in the domain. Ske~ch the graph.

4. Find

dy
dx

Y 1 = 6x - 3x2 where y ,

= 3x(2 - x)

= 0 when x = 0 or 2

When x = 0, y
0
When x = 2, y = 4
The stationary points are (0, 0) and (2, 4).
- 2 - 1
When x < 0, y , < O}
d
(0
0)'
I
I
..
.
IS a oca mImmum pomt.
0 an so ,
When x > 0 ,y >

When x < 2, y'' O


> O}
'
.

a nd so (2 , 4)'IS a IocaI
maXImum
pomt.
Wh en x > 2 ,y <

4X

Consider the extreme val1,les of the domain.


When x = -2, y = 20
When x = 4, y = -16
The greatest and least values or absolute maximum and mmImum are 20 and -16
respectively. Note thaty = 4 is only a local maximum andy = 0 is only a local minimum.

EXERCISES 15(a)
1. A functionjhas the following properties:j(3) = 5,j'(3) = 1. Sketch the graph ofj

near x = 3.
2. Sketch the graph of j with the following properties: j(l) = 0, j'(X) = 2 for all x.
State the rule that defines the function.
3. Sketch the graph of a function given thatj(2)
j'(X) > 0 for all x > 2.

0,j'(2)

= O,j'(x) <

0 for all x < 2,

4. Sketch the graph of j, such thatj(3)


3, j'(3)
0, j(l) = 5, j'(I) = 0, j'(x) > 0
for all x < 1 and for all x > 3,j'(x) < 0 when 1 < x < 3.
5. For the function j(x)
values of x for which:
(a) j'(x) < 0
(b) j'(X) = 0
(c) j'(X) > O.

X2 - 5x

+ 6, sketch the graph of j', and hence find the

6. Sketch the curve y = 4x - x 2. For what value of x is :

= O?

What is the sign of the gradient to the left and right of this point? Has the curve a
maximum or minimum turning point?
7. For the graph of j(x)
6 - 3x - x 2, find the values of x for which the function
(a) increases when x increases,
(b) decreases when x increases,
(c) changes from increasing to decreasing.

15/GEOMETRICAL APPLICATIONS OF DIFFERENTIATION 331

8. Find the maximum value of 5x - 2X2 ~


9. Find the minimum value of x(x - 2) + 3.
10. For the functionf(x) = 2x 3
15x2 + 36x, sketch the graph off' and hence find the
values of x for which
(a) f'(x) = 0
(b) f'(x) > 0
(c) f'(x) < O.

Sketch the graph of f.

11. Consider the functionfwheref(x) = x 3 - X2 - X + 1. Find the values of x for which


(a) f'(x) = 0
(b) f'(x) > 0
(c) f'(x) < O.

Sketch the graph offin the domain -2 ~ x ~ 2, locating the maximum and minimum

turning points.

12. Sketch the curve y

= x3

6x 2 in the domain -1

6, locating the turning points.

13. Sketch the curve y = 2x 3 + 3x2 - 12x + 7 after finding the local maximum and
minimum points, and the point where the curve intersects the Y-axis.
14. Sketch the curve y
(2 - x)(l + X2), locating the turning points and the points
where it crosses the coordinate axes in the domain -1 ~ x ~ 3.
15. Consider the functionf(x) = 9x(x - 2)2, -1 ~ x ~ 3. Find the values of x for which
(a) f'(x) = 0
(b) f'(x) > 0
(c) f'(x) < O.

Sketch the graph of f(x) and state its range and greatest and least values.

16. For the graph of f(x)


x 3 + X2 - 5x - 6, find the values of x for which the
function:
(a) increases when x increases
(b) decreases when x increases
(c) changes from increasing to decreasing.

Sketch the graph of f, locating its turning points.

17. Prove that the parabola y


18. Show that the hyperbola y =

ax 2 + bx

!x

+ c has a turning point at x

= - 2a'

has no turning point.

15.3 Second derivative _ _ _ _ _ _ _ _ _ __


If we differentiate the derived function f' of a function f, we obtain what is called the
second derived function of f, which is denoted by f". Thus f"(x) denotes the derivative

of f'(x) for any value x, or, if y = f(x), the first derivative is


derivative is

~ (dY) = d
y =
dx dx
dx 2
2

f"(x) =

"

= f'(x),

and the second

332 NEW SENIOR MATHS: TWO UNIT COURSE

If

Y = f(x) = x 3

then

y' or dy
dx

f'(x)

X2 - X

3x2 - 2x -

+ 1,
1

(1)

Differentiating (1), we get


2
d y2
dx

i.e.,

!.(dY ) = ~(3X2 _ 2x dx dx
dx
y" or f"(X) = 6x - 2

1)

6x - 2

Fig. 15-10 is the graph of f(x) = x 3 - X2 - X + 1; the + and - signs indicate positive
and negative gradiertts in the interval [-1, 2].
Fig. 15-11 is. the graph of f'(x) = 3x2 - 2x - 1, and it provides all the information
concerning the gradients of fig. 15-10.
y
A
f(x) = x 3

X2 -

+ 1

Fig. 15-10

11

)(

I
I
I
I

I
I

1
1
-3,

I
I

Fig. 15-11

I
I

1-1
I
I
I
I

f'1x)
I
I
I
I

,,f'(x)

)(

3x 2

2x - 1

I
I
I
I

,
I
I

I
I
I
I

f"x) = 6x - 2

)(

Fig. 15-12

15/GEOlVIETRICAL APPLICATIONS OF DIFFERENTIATION 333

For all x in the interval

(-1, -i), the curve slopes upwards to the right; the tangent to

the curve at any point in this interval makes an acute angle with the X-axis, and so the
gradient is positive, Le.f'(x) > O. Also, as x increases,f(x) increases, and so we say thatf
is an increasing function in the interval (-1, -}).
When x =

-k, the tangent is parallel to the X-axis and the gradient of the curve at this
(1) 0

. .IS zero; I.e.


. f' - 3"
pomt

For all x in the interval (

-k, 1), the curve slopes downwards to the right; the tangent

to the curve at any point in this interval makes an obtuse angle with the X-axis and so the
gradient is negative, Le.f'(x) < O. Also, as x increases,f(x) decreases, and so we say that

f is a decreasing function in the interval (-}, 1).


At the point x l , the tangent is parallel to the X-axis and the gradient of the curve at
O.
this point is zero; i.e. f'(I)
For all x in the interval (1,2), the curve slopes upwards,f'(x) > 0 andfis an increasing
function.

15.4 The second derivative and


turning points ______________
Consider the point D in fig. 15-11, which gives the value of f'(x) at A. The gradient
of the curvef'(x) is negative at this point. This means thatf"(x) < 0 atA, and this result
is observed from the graph of f"(X) in fig. 15-12.
In general, if f has a local maximum at x = a, then f"(a) < O.
Consider the point E in fig. 15-11, which gives the value off'(x) at B. The gradient of
the curvef'(x) is positive at this point. This means thatf"(x) > 0 at B, and, again, this is
represented pictorially in fig. 15-12.
a, then f"(a) > O.
In general, if f has a local minimum at x
The second derivative thus enables the nature of a turning point to be established.
Summary
(i) Local maximum at x
(ii) Local minimum at x

= a if f'(a) = 0 andf"(a) < 0


= a if f'(a) 0 andf"(a) > 0

Points of inflexion
Consider the point G in fig. 15-12. At this point,f"(x) O. The corresponding point C in
fig. 15-10 is called a point of inflexion. Hence points of inflexion may occur where the
second derivative is zero, but, unfortunately, this is not always so (as will be seen in
example 6).
The point corresponding to G in fig. 15-11 is F, and it is readily observed that F is a
point at which the gradient f'(x) has a minimum value. Of course, the purpose of
discussing points of inflexion is to determine their meaning on the graph of the original
function.
The point on the graph of f at which f"(X) = 0 is found from fig. 15-10 to be the
point C. This means that the slope of the graph of f is 'most negative' at C.
To the left of C, the slope becomes more and more negative as C is approached. Once C
is passed, the gradient increases until it reaches the value 0 at B. Thus C is the point at

, !

334 NEW SENIOR MATHS: TWO UNIT COURSE

which the largest negative gradient occurs. The tangent to the graph at C is as shown in
fig. 1513.
Since f"(x) = 6x
2 has only one zero, C is the only point of inflexion. Hence th.ere
are no points of maximum positive gradient.
A

Fig. 15-13

o and f"(x) = 0 at a particular point, as illustrated in figs


Sometimes both f'(x)
15-14 and 1515.
At the point P,f'(x) = 0 because the tangent is parallel to the X-axis. To the left of P,
the gradient is negative, but is increasing as x increases; to the right of P, the gradient
decreases as x increases, and so, at P, f'(x) has a maximum value; i.e. f"(x)
0 at P.
At Q, f'(x) = O. To the left of Q, the gradient is positive, but is decreasing as x
increases; to the right of Q, the gradient increases as x increases, and so, at Q, f'(x)
y

Fig. 15-14

Fig. 15-15

has a minimum value: i.e. fl/(x)


0 at Q. P and Q are called stationary points of
inflexion.
Returning to fig. 15-10, we can now calculate the maximum and minimum values of
f(x) and the coordinates of the point of inflexion.
f(x)
f'(x)

= x 3 - X2 X + 1
= 3x2 - 2x 1
= (3x + 1)(x - 1)
1

o when x = -3 or 1
When x

= -3'

(-t)' - (_t)2 - (-t) +

f(x)

1
1
1
27 - 9 + 3 + 1

When x

1,

1 - 1

f(x)

1 + 1

o
f"(X)

When x
When x

6x - 2

f"(x) < 0 and so f(x) = 1257 is a local maximum

=
1,

f"(x) > 0 and so f(x)

= 0 is a local minimum

(1)
(2)

IS/GEOMETRICAL APPLICATIONS OF DIFFERENTIATION 33S

Alternatively, consider the gradients at points slightly to the left and right of x

--and
3

x = 1.
When x <

-~, j'(x) >

0 (by substitution in (2) above).

When x >

-~, j'(x) <

O.
,

Hence j(x)

= 17

when x

When x < 1, j'(x) < O.


When x > 1, j'(x) > O.
Hence j(x) = 0 when x

1 is a local minimum.

At a point of inflexion, j"(x)


When x

= -1

3'

j(x)

1 is a local maximum

O. That is, 6x - 2 = 0, x

= (1)3
- - (1)2
- - -1 + 1
3

3'

16 C oord'mates 0 fpomt
' 0 f'm fl eXlOn
.
27'

1 16_\
are ( 3' 27)'

EXAMPLE 5
Sketch the graph of y = x 3( 4 - x), locating the turning points and the points of inflexion.
(1)

(2)

For a turning point, :

= O.

Thus

I2x2 - 4x 3
4X2(3
x)

=0
=0

x
and so
When x <

0, d.dy > O~}

d;

0 or 3

= 0 or 27 from (1) above.

Hence y has neither max nor min at x

o.

When x > 0, dx > O.


When x < 3,

dy

d; < O,}

d. >

When x > 3, dx

Hence y

27 (local maximum).

O.

24x - I2x2
I2x(2 - x)

=0
=0

Points of inflexion:
i.e.,

x = 0 or 2
When x

dy
0, both dx

d 2y
0 and dx2

2, y
flexion at (0, 0). When x
maximum positive gradient.

0, and hence there is a stationary point of in

16. Hence the point of inflexion at (2, 16) is a point of

336 NEW SENIOR MATHS: TWO UNIT COURSE

(3,27)

x
Fig. 15-16
Note: You may get the impression from our consideration of the second derivative and
points of inflexion that, if f"(X) = 0 at a point, then the curve has a point of inflexion at
that point. In fact, this condition is not sufficient. It is true that, if a curve has a point af
inflexion, then fl/(x)
0 at that point, but the converse is not necessarily true, namely
0 at a point, then the curve does not necessarily have a point of inflexion
that, if fl/(x)
at that point.

EXAMPLE 6

Sketch the graph of y


Y

X4

X4

= 4x 3
Owhenx

=0

(1)

= 12x2

owhen x =

(2)

Fig. 15-17

From (1), the curve has a stationary point at the origin and (2) suggests that it may have a
stationary point of inflexion there. However, this is not so, as can be seen if we consider,
by substitution in (1), the sign of the gradient at points slightly to the left and right of
x
O.
dy
When x < 0, d. <
Hence minimum value of y is 0 at (0, 0).
When x > 0, dx > O.

d;

O.}

It is interesting and instructive to reconsider the family of curvesf(x)


x n , n = 2, 3,4,
5, 6, ... , and note the nature of the stationary point at the origin in each case.

EXERCISES 15(b)
1. Sketch the graph of y = x 3 (x - 2) in the interval [-1, 3], locating the turning and
inflexion points in this interval.

2. Find the local maxima, minima and inflexions of f(x)


off.
3. Sketch the graph of 8y = 9 + 8x2

x2(3

x) and sketch a graph

x 4 , locating the turning points.

15/GEOMETRICAL APPLICATIONS OF DIFFERENTIATION 337


4. Prove that the graph of y = ax 3 + bx2 + cx + d has two distinct turning points if
b 2 > 3ac. Find the values of a, b, c and d for which a graph of this form has turning

points at (~, 1) and

(~, -1).

5. For the graph of f(x) = x 3


6x2 + 2, find the maxima, minima and points of
inflexion. Sketch the graph of f.
6. Sketch the graph of y
3x4
44x 3 + 144x2, finding the coordinates of all the
turning points and points of inflexion.
7. Sketch the curve whose equation isy = X4 - 29x2 + 100, -5 ~ x ~ 5 finding the co
ordinates of its stationary points and points of inflexion. Find also the values of x at
the points where the curve crosses the X-axis.
8. Find the coordinates of the points common to the curves whose equations are
3y
X2 - 18x and 9y
2x 3 - 33x2 + 108x.
Find the turning points and sketch the two curves on the same diagram.
9. Sketch the graph of y = X4
points of inflexion.

X2, -2

2, locating the turning points and the

10. Find the turning points and points of inflexion of y

-x 3

+ 3x2 - 3x and sketch its

graph. Prove that it crosses the X-axis at one point only_ Show that :
except x

< 0 for all x

1.

11. Find the greatest and least values of the function given by f(x)
domain -3 ~ x ~ 1.
12. Sketch the curve y = 2x 3
points. State the range.

3x2

X2

5x

+ 4 in the

12x in the domain -2 ~ x ~ 3 locating its turning

15.5 Rational Algebraic Functions _ _ _ _ _ __


A rational function may be defined as a function

"'(x ) =

J.

P(x)
Q(x)

and P(x) and Q(x) are polynomials with no common nonconstant factors. There is a
restriction on the domain, since a quotient is undefined when the denominator is zero, and
hence f(x) is not defined for those values of x for which Q(x) = O.
The following statements define rational functions.
1
x + 1
f(x) = for x =1= 0
Y =
for all x

f(x)

- - 2 for x

x-

=1=

x-I

for x

=1= -

3, 3

Asymptotes

Graphs of rational functions are characterized by asymptotic behaviour whenever the


curves are extended to large distances from the origin. Fig. 15-18 shows the graph of
y = 1 x
(y

0).

=1=

O. On this graph, the asymptotes are the Y-axis (x

= 0), and the X-axis

.,

338 NEW SENIOR MATHS: TWO UNIT COURSE

o
Y=1x
Fig. 15-18
Given f(x)

~,

we know

~ does not exist.

The following table shows the behaviour of l/x as x is given values successively closer
to O.
x
lIx

0-5

01

001

0001

00001

10

100

1000

10000

Clearly, as x is made closer and closer to 0 (written x - 0), the value of l/x becomes
correspondingly larger and larger-there is no end to this process. To signify that the
vahie of 1/x increases without bound, we write! x

00

as x -

O.

Note: The term infinity ( 00) does not describe a real number. So 00 does not stand for a
number.
The meaning here is that, no matter how large a number we can imagine, the value of
l/x can be made to exceed that number by making x sufficiently close to O. (The graph
never reaches the Y-axis.)
There is corresponding behaviour as x - 0 through negative values. For example,

-~'1

-10,

-0'~01

10000. This time, the values of lIx are unbounded and

negative as x - O.
To describe the total behaviour, the following notation may be used:
1
+00 as x 0+
X

'

- 00

as x -

0-.

Notice that there is a discontinuity in the graph at x = O. This is provided by the vertical
asymptote, x = O.
Conversely, as x is made larger and larger (both positively and negatively), the value of
lIx approaches zero. For example,
1
1
100
0'01, -10000
-0,0001.
In this case, we write:
1

1
x

0+

as x -

0- as

+ 00

'

x - -00

IS/GEOMETRICAL APPLICATIONS OF DIFFERENTIATION 339

Consequently, the X-axis (y = 0) is a horizontal asymptote.


From this discussion, it should be evident that the statements 1/0 = 0::> and 1/0::>
which are sometimes seen, are totally incorrect.

0,

Vertical asymptotes
Vertical asymptotes always occur at values of x where the numerator is non-zero and the
denominator is zero.
Horizontal asymptotes
Horizontal asymptotes are determined by examining the behaviour of the function as
Ixl- 0::>.
The graphs of rational functions are very interesting and quite simple to draw if we bear
in mind general properties of functions such as domain of definition, sign of the function,
sign of the derivative, turning points, asymptotes, symmetry, points of intersection with
the axes, behaviour of function for large values 'of X, etc.

EXAMPLE 7'
Sketch the graph of f(x)

-
x - 2'
This function is not defined when x = 2, but as x - 2 from the left of 2, x - 2 is a
small negative number and so f(x) - - 0::>. Similarly as x - 2 from the right of 2, x - 2 is
a small positive number and so f(x) - 0::>. The line with equation x = 2 is a vertical
asymptote.
To save time and space we shall write statements like the above in the form:
f(x) -

0::> as x -

We further see that as x -

2+, and f(x) - -0::> as x- 2

0::>, f(x) -

0+ and as x -

-0::>, f(x) - 0- so that the line

y = 0, viz. the X-axis, is a horizontal asymptote.

Axial Intercepts
1

x-2
1

o has no solution and so there is no intercept on the X-axis.


-

~ when x

0so the intersection with the Y-axis is at (0, -~).


y

Fig. 15-19

----~

..

. .- - ...

~~~----~~----

340 NEW SENIOR MATHS: TWO UNIT COURSE

Turning Points
f'(x)

= -

(x

2)2'

'* 2

so f'(x) < 0 wherever defined. Thus the gradient is always negative; there are no turning
points.
Sign of f(x)
f(x) < 0 when x < 2 andf(x) > 0 when x > 2

Actually f(x) =

is the image of f(x) = 1 under a translation of 2 units parallel

x-

to the X-axis.

EXAMPLE 8
x - 1

Sketch the graph of f(x)

I
I
I

I
I

--_I

- --1- - ------ - -
I

y=1

12
I

f(x)

I
1
I

Fig. 15-20

Axial intercepts

Graph cuts X-axis at x

1 and Y-axis at y

Asymptotic behaviour
f(x) - 00 as x - 2+ }

x
ji()

-+

ooasx-

By division, f(x)
f(x)
f(x)

-+
-+

1+ as x - +
1- as x --

2- so x

1
00 }
-00

so y

=
1

= ~.

2 is a vertical asymptote.

so
1 is a horizontal asymptote.

Turning points
f'(x) = - (x

2)2 wherever defined (i.e. x

'* 2).

:. f'(x) < 0 for all x in the domain of f, and the graph has no turning points.
Sign of f(x)
For

x < 1,f(x) > 0


-1 < x < 2, f(x) < 0
x > 2,f(x) > 0

15/GEOMETRICAL APPLICATIONS OF DIFFERENTIATION 341

Actoally,/(x)

= xx--

21

= 1 + x~2 is the image of/(x) = x


~2 under a translation of

l unit parallel to the Y-axis. Using this fact, it could well be drawn directly from the
former graph. (See fig. 15-20.)

EXAMPLE 9

Sketch the graph of y

1
= x + x-.

Y = x

+ x1 =

X2

- - - x =t=

x'

O.

Graph does not cross the X-axis, since X2 + 1 =t= 0 for any x.

Graph does not cross the Y-axis, since x =t= O.

Asymptotic behaviour
ASX-WY-oo}
_'
so x
A sx- 0 , Y - 00
Y = x

0 is a vertical asymptote.

+ a quantity equal to x1 which is > O' for all x > 0 and which -- 0 as x

quantity is < 0 for all x < 0 and -- 0 as x -- -00.


As x -- 00, Y -- x from above.
As x - -00, y -- x from below.
The straight line y = x is called an oblique asymptote.

--

00.

This

Turning points
dy

1 _

dx

7
/'
/~+

x=t=O

(x

I)(x

(',2)/ /

1)

X2

o when x

'"

1 or -1

y=x+
x

: I

Fig. 15-21
Since, when x =t= 0, the denominator is positive, the sign of :
Thus :

> 0 when x < -lor x > 1 and :

is the sign of (x - 1)(x

< 0 when -1 < x < 1, except x

+ 1).
=

O.

When x - I , y = -2, so the graph has a local maximum at (-1, -2). When x = 1, y = 2, so
the graph has a local minimum at (1, 2).
The range of the function is the set of all real numbers except those in the open interval
between -2 and 2 (fig. 15-21).

I,'

,i

342 NEW SENIOR MATHS: TWO UNIT COURSE

EXERCISES 15(c)
Sketch the graph of each of the following, locating any stationary points and asymptotes.
State the range in each case.
1. Y
if

2. Y

3. Y

5. Y

x -

7. Y = -
x-2

2x

+ x

= x + xx
1
13. Y = - + 4 x

11. Y

1
2x - 3
4

4. Y

x
1

x-I

9. Y

x +
x
1
2 +

6. Y

8. Y

= x + 2 + x

10. Y

x + 3 +

12. Y

2x - 1

14. Y

= Ixl + x

x
9

+ 2x+

15.6 Maximum and minimum problems _ _ __


EXAMPLE 10
A piece of wire of length 12 cm is bent in the shape of a rectangle. Find the maximum area
of the rectangle.
It is possible to have many rectangles with a
perimeter of 12 cm, all with different areas. Thus
the length and width of the rectangle are variable,
x
and so too is the area. To obtain the maximum
area, A, we must obtain an expression for A in
terms of one variable only.
Fig. 15-22
Let one dimension of the rectangle be xcm and hence the other dimension will be
(6 - x)cm.
A (x) = x( 6 - x), 0 < x < 6
6x - X2
A '(x) = 6 - 2x
The rectangle will have a maximum area when A '(x) = o.
6
2x = 0
X= 3
When x < 3, A '(x) >
When x > 3, A '(x) <
maXImum area when x
3
Hence both the dimensions will be 3 cm and the rectangle will be a square of area 9 cm2
The graph of the area function is shown in fig. 15-23.
The domain is 0 < x < 6.
The range is 0 < A(x) ~ 9.

D(6-X)

o} :. .

Note: This example is the same as example 3 in chapter 12. Since the area function is a
quadratic function, we can find maximum and minimum values by 'completing the
square' method (chapter 12) or by a calculus method.
Note also the importance of the domain in practical problems.

--

---~

I
I'

15/GEOMETRICAL APPLICATIONS OF DIFFERENTIATION 343

Alx}

Fig. 15-23

x
EXAMPLE 11

A sheet of cardboard measures 15 cm by 7 cm. Four equal squares are cut out of the
corners and the sides turned up to form an open rectangular box. Find the length of the
edge of the squares cut out, so that the box will have a maximum volume.

Fig. 15-24
Let the length of the edge of the square cut out be x cm (0 < x < 3D. The dimensions of
the box will then be (15 - 2x), (7 - 2x) and xcm.
Denote the volume of the box by V.
:. V = x(l5 - 2x)(7 - 2x),
= 4x 3 - 44X2 + 105x

For maximum volume, dV

0<

< 3!

O.

dV = 12x2 - 88x

dx

+ 105

(2x - 3)(6x - 35)

o when x = I! or 5i
We can reject x
When x <

I!,
1

dV

5~

in this practical situation. Why?

> O}
:. maximum volume occurs when x

dV

= I!.

When x > 12, dx < 0


Maximum volume

2" x 12 x 4 = 72cm 3

Fig. 15-25 shows the graph of V


Vis 0 < x < 3!.

x(15 - 2x)(7 - 2x). The domain of definition of

Fig. 15-25

.~~"-~~--------------~~~~~~~

344 NEW SENIOR MATHS: TWO UNIT COURSE

EXERCISES 15(d)
I

1. Find the maximum area of a rectangular plot of ground that can be enclosed by 160m
of fencing.

2. The sum of two numbers is 12. If one number is x, what is the other number? Find the
value of x for which the product of the two numbers is a maximum.
3. A man wishes to form a rectangular enclosure using his existing fence as one side. If
he has 20 metres of fencing material available to form the other three sides, find the
area of the largest enclosure he can form and its dimensions.
4. A piece of wire 6 metres long is cut into two parts, one of which is used to form a
square and the other to form a rectangle whose length is three times its width. Find
the lengths of the two parts if the sum of the areas is a minimum.
5. A rectangular area is to be fenced and divided into six rectangles by one dividing fence
parallel to its length and two fences parallel to its breadth. If the total length of
fencing available is 120 m, find the maximum possible area.
6. ABCD is a square of unit length and points E and F are taken on the sides AB and AD
respectively such that AE = AF = x.

(a) Express the area of the quadrilateral CDFE as a function of x.


(b) Find the greatest area the quadrilateral can have.
7. A rectangular sheet of cardboard measures 16 cm by 6 cm. Equal squares are cut out
at the corners and the sides are turned up to form an open rectangular box. What is
the maximum volume of the box?
8. A block of wood in the shape of a cuboid is to have square ends. What is the volume
of the largest block if the sum of the length of the block and the perimeter of the end
is 12 cm?
9. A box in the shape of a cuboid with a square base is to be made so that the sum of its
dimensions is 20 cm. Find the maximum volume.
10. A rectangular box has a square base of edge xcm. Its framework of 12 edges is
constructed from pieces of wire of total length 36 cm. Find
(a) the height of the box in terms of x,
(b) the volume of the box in terms of x,
(c) the value of x for which the volume is a maximum.
11. A closed box in the shape of a cuboid, the length of whose base is twice the width, has
a total surface area of 216cm2 If the width of the base is xcm, find:
(a) the length of the base and the height in terms of x,
(b) the volume of the box in terms of x,
(c) the maximum volume of the box.
12. A rectangular field is to be fenced around three sides with 300 m of fencing. Find the
dimensions of such a field if the area to be enclosed is as great as possible.

13. The diagonal of the base of a box in the shape of a cuboid has a length of 10 cm. If
one edge of the base has a length of xcm, express, in terms of x, the length of the
other edge of the base. If the height of the box is equal to the length of this other edge,
express the volume of the box in terms of x and find the maximum volume of the box.

.1

15/GEOMETRICAL APPLICATIONS OF DIFFERENTIATION 345

14. The slant edge of a right circular cone is 6 cm in length. Find the height of the cone
when the volume is a maximum.
15. A piece of wire of length 30 cm is cut into two sections. Each section is then bent into
the shape of a square. Find the smallest possible value of the sum of the areas of the
two squares.
16. A block of metal is to be cast into the shape of a right cylinder with a total surface
area of 207rcm2 If the radius-of the base is rcm and the height h cm,
(a) express h in terms of r,
(b) express the volume, V, in terms of r,
(c) find the value of r for which the volume is greatest.
17. A piece of wire of length 50 cm is cut into two sections. One section is used to
construct a rectangle whose dimensions are in the ratio 3: 1; the other section is used to
construct a square. Find the dimensions of the rectangle and the square so that the
total enclosed area is a minimum.
18. Find the maximum volume of a right circular cone whose slant edge has a constant
length measure a.
19. A rectangular block, the length of whose base is twice the width, has a total surface
area of 300 cm2 Find the dimensions of the block if it is of maximum volume.

20. A window frame has the shape of a rectangle surmounted by a semicircle. The
perimeter of the frame is constant. Show that, for maximum area, the height of the
rectangle is equal to the radius of the semicircle.
21. A piece of wire of length 50 cm is to be cut into two sections, one of which is used to
form a square, the other of which is used to form a circle. If the length of the edge of
the square is x cm, find, in terms of x,
(a) the radius of the circle,
(b) the area of the circle,
(c) the total area of the square and the circle.

22

Show that, when x = 7, the total area is least. Use 7r = T.


22. Find the area of the largest rectangle that can be inscribed in the region bounded by
the parabola with equation y = 4 - X2 and the X-axis.
23. A square sheet of cardboard measures 12 cm by 12 cm. Four equal squares of edge
x cm are cut out of the corners and the sides are turned up to form an open
rectangular box.
(a) State the values that x may take.
(b) Express the volume, V, of the box in terms of x.
(c) Find the value of x for a maximum volume, and hence find the maximum volume.
(d) Sketch a graph of the volume function.
(e) If x ~ 4, find the maximum volume.
(f) If 2 ~ x ~ 5, find the minimum volume.

24. Find two non-negative numbers whose sum is 12 and whose sum of their squares is a
maximum.
~.
I

A
farmer wishes to fence off a rectangular paddock using a straight section of a river
as one of the sides. Fencing material costs $10 per metre and the farmer spends $1200
on materials. Find the dimensions of the paddock for which the area is a maximum.

346 NEW SENIOR MATHS: TWO UNIT COURSE

EXAMPLE 12
The cost of running a truck at an average speed of v km h- 1 is 64

+ 1~~ dollars per hour.

Calculate the average speed for which the total cost of running a truck from Sydney to
Katoomba, a distance of 100 km, will be a minimum. What is the minimum cost?

Cost per hour

(64 +

1~) dollars

Number of hours = 100, v >


v
Total cost, $y, is given by
y
cost per hour X number of hours

,/
,/

,/
,/

,/

(64 + ~\ 100
\
100) v
6400
v+--,v>O
v
dy _ 1 _ 6400
dv
v2
2
v = 6400
v
80

c:;

When v > 80,

<

,/

160

-1'

...

,/

,/
,/
,/
,/
,/

80

O}

lv >

,/

,/

,/

When v < 80,

= v+ .400
v

Fig. 15-26
:.

minimum cost when v

80.

When v = 80, y
160.
The minimum cost is $160 when the average speed is 80 km h- 1
The graph of the cost function is shown in Fig. 15-26.
As v - 0, y - 00 and so the y-axis is a vertical asymptote.
As v - 00, y - v and so y = v is an oblique asymptote.
j

EXAMPLE 13
A man in a boat is 3 kilometres from the nearest point 0 of a straight beach. His
destination is 6 kilometres along the beach from O. If he can row at 4 km h- 1 and walk at
5 km h- 1, towards what point on the beach should he row to reach his destination in the
least possible time?
The man rows in a straight line from B
to C, and then walks from C to D. Let the
8
point C be x kilometres from 0 where
~ x ~ 6.
Applying the Theorem of Pythagoras,
he rows a distance -JX2 + 9 kilometres at
-JX2 + 9
4 km h- I , and hence rows for
4
D

3ts:c

0 .... ____ 6 _____ ....

hours. He walks along the beach a


distance (6 - x) kilometres at 5 km h- 1,

6-x

and hence walks for - 5 - hours.

Fig. 15-27

The total time, T(x) hours, is a function of x, defined by


T(x) =

-JX2+9 +
4

6 - x

the graph of which is shown in fig. 15-28.


}

15/GEOMETRICAL APPLICATIONS OF DIFFERENTIATION 347

The domain of this function is 0


T '(x) =

-:--;;:::=;;===;;;:

6.

-S

5x - 4.J.X2+9

20..Jx2 + 9

o when 5x - 4..JX2 + 9 = 0
i.e. 5x = 4..JX2 + 9
25x 2 = 16x2 + 144 (square both sides

of the preceding equation)

9x 2 = 144
x = 4, ignoring the negative root.
Fig. 15-28
When x < 4, T'(x) <
When x > 4, T'(x) > 0 Hence mlDlmum value of Toccurs when x = 4.

O}

..

The man thus rows to C, 4 kilometres from 0, and takes 1 h 39 min.

EXERCISES 15(e)
1. The area of a rectangle is 400 cm2 If the length of one of its sides is x cm, express the
length of the other side and hence the perimeter in terms of x. Find the value of x
which makes the perimeter a minimum.

2. Find the coordinates of the point on the graph of y

!,
x >
x

0 which is closest to the

origin.
3. The cost of running a ship at a constant speed of v km/h is 160

+ 1~O v3 dollars per

hour.
(a) Find the cost of a journey of 1000 km at a speed of 10 km/h.
(b) Find the cost, C dollars, of a journey of 1000 km at a speed of v km/h.
(c) Find the most economica! speed for the journey, and the minimum cost.
(d) If the ship were to have maximum speed of 16 kmlh, find what the minimum cost
would be.
4. A company manufactures items at $2 per item and sells them for $x per item. If the
number sold is 800 per month, find the value of x for which the company could
expect to maximize its monthly profit.
S. A man in a boat is 4 km from the nearest point 0 of a straight beach; his destination is
4 km along the beach from O. If he can row at 4 kmh- l and walk at 5 kmh- l , how
should he proceed in order to reach his destination in the least possible time?
6. A rectangular box whose base is a square is to be made so that its total surface area is
constant. Pr\>ve that the volume of the box is greatest if the box is a cube.
7. A traveller employs a man to drive him from Sydney to Wollongong for an hourly.
payment of P dollars. Running costs of the car, which are also paid by the traveller,
are kv 3 dollars per hour, where v km h- l is the speed, and k is a constant. Find the
uniform speed that will minimize the total cost of the journey.

ax + x!!. and if y = 13 when x = 1 and y = 20 when.


x = 2, find the values of
a and b and the value of x for which y is a minimum.

S. If y

i,

348 NEW SENIOR MATHS: TWO UNIT COURSE

9. Find the dimensions of the largest rectangle that can be inscribed in the semicircle
Y = "";4 - X2
10. Two straight roads intersect at right angles. Two men, A and B, are 100 kilometres
from the intersection, one on each road. They drive towards the intersection at
30 km h- I and 40 km h- I respectively. Find the distance of each driver from the
intersection as a function of t, the time in hours for which they are driving. Hence
find their distance apart, d(t), at any time. For what value of t is their distance apart
least?
11. The running cost (cost of fuel) for a certain ship is $3 per hour when the ship is not
moving, and this cost increases by an amount that is proportional to the cube of its
speed, Vkm h- 1 If the running cost per hour is $6'75 when the speed is 15 km h- ' ,
obtain a formula for the running cost per hour at speed V, and calculate the value of
V for which the total running cost for a journey of 450 km is a minimum.

12. The cost function, C(v), of running a train at an average speed, vkmh- ' , is
b
C(v) = av + -, where a and b are constants. If C(20) = 108 and C(25)
90, find
v

v for the least cost.

13. A closed cylindrical can is to have a capacity of 167rcm3 What are the radius of the
base and the height of the cylinder for the total surface area to be a minimum.
14. A printed page of a book is to have side margins of 1 cm, a top margin of 2 cm and a
bottom margin of 3 cm. It is to contain 200cm2 of printed matter. Find the
dimensions of the page if the area of paper used is to be a minimum.
A

15. A rectangle PQRS is placed inside the


scalene triangle ABC as shown. If the
area of DABC is constant, prove that
the maximum area of the rectangle is
one-half the area of DABC.

R C

15.7 Equation of tangent and


normal to a curve ____________
Fig. 15-29

EXAMPLE 14
Find the equation of the tangent and normal to the curve y
the curve where x = 3.
Y

dy

dx

x2

x2

4x

+ 4 at the point on

4x + 4

2x - 4

When x = 3, y = 9 - 12

+4

When x = 3 , dy
=2
dx
Hence, the point (3, 1) belongs to the curve and the
gradient of the tangent at this point is 2.
Y - YI
m(x - Xl)
Y - 1 = 2(x - 3)
Y = 2x
5 (equation of tangent)

Fig. 15-30

I5/GEOMETRICAL APPLICATIONS OF DIFFERENTIATION 349

The normal is a line perpendicular to the tangent. Since the gradient of the tangent is 2,
the gradient of the normal is

-1.

m(x

Yl

= -~X
- 3)
2
= -x + 3

Y - 1

2y
2y

Xl)

+X

5 (equation of normal)

EXAMPLE 15
Find the equation of the tangent and normal to the curve Y = ..JX at the point
where x = 4.

011

the curve

Y = x 1l2

dy
lx- l12 _ _1_
dx
2
- 2..JX
Whenx = 4,y
-14
dy
1
1
dx

2-14

X Fig. 15-31.

Hence, the point (4, 2) belongs to the curve and the gradient of the tangent at this point is

y - Yl = m(x - Xl)

1
j<x
-

Y - 2
4y

Since the gradient of the tangent is

l,

4)

= 4 (equation of tangent)

the gradient of the normal is -4 at the point (4, 2)

Y - Yl = m(x - Xl)
Y - 2
-4(x - 4)
Y + 4x = 18 (equation of normal)

EXAMPLE 16
Find the equation of the tangent to the curve Y

= ~X3

X2 -

curve where the tangent is parallel to the line Y - 7X

~X3

Y
:

= X2 -

The gradient of the line y - 7 X = 5 is 7.


:. X2 - 2x - 1
7
X2
2x - 8 = 0
(x
4)(x + 2)
0
x = -2 or 4
Y

= - 11
- or7

3
Y - Yl = m(x Y

+ 11

3

7(x

Xl)

2)

X2

2x - 1

+1

5.

1 at the points on the

350 NEW SENIOR MATHS: TWO UNIT COURSE

3Y
7

3"

3y

21x

+ 31 (eqUation of tangent at (-2, - 131


) )

= 7(x - 4)

21x - 77 (equation of tangent

at~, ~) )

EXAMPLE 17
Find the points of intersection of the parabolas Y = X2 - 2x and Y
4x - X2 and the
angle between the parabolas at the point, to the right of the origin, at which they intersect.
Yl

2x

X2 -

Y2

= 4x -

X2

At the points of intersection Yl


Y2
.'.
X2 - 2x = 4x - X2
2X2 - 6x = 0
x = 0 or 3
Y
0 or 3
.'. co-ordinates of their point of intersection
to the right of the origin are (3, 3)
Fig. 15-32
= X2 - 2x
m l = 2x - 2

For

YI

=4

:. tan 01
01
For

Y2

m2

when x

= 4x -x 2

4
2x
-2
when x
:. tan O2 = -2
O2
11634'
0:

=4
= 7558'
=

= O2

01

4036' .

EXERCISES 15(f)
In questions 1. to 8. find the equations of the tangent and normal to the curves at the
points indicated:

1. Y =
2. Y

X2

at (2, 4)

X2 -

3. Y = 3x 4. Y =

2X2

5. Y

2X2

6. Y
7. Y

5x

X2

at (0, 0)

+ 3x - 4x +

- where x

x'

6 at (3, 0)

4, where x = 0
1, where the gradient is 4
=

-2

= 3 - x - x 2 , where it crosses the Y-axis

I5/GEOMETRICAL APPLICATIONS OF DIFFERENTIATION 351

8. Y

3x 3

7x 2 + 2x, where x

9. The straight line y =

x+

2 cuts the parabola y

!x

2 -

2 at two points P and Q.

Find the coordinates of P and Q. Also find the equations of the tangents to the
parabola at P and Q and the coordinates of the point of intersection of these tangents.
10. Find the equations of the tangents to the parabola y = 4x - 3x 2 at the points where
the parabola cuts the X-axis.
11. Find the equations of the tangent and normal to the parabola y = 2X2 - 4x
the point of zero gradient.
12. Prove that the parabolas y
2X2 - 6x + 5 and y
and find the equation of the common tangent.

X2 - 2x

1 at

+ 1 touch each other

13. The line y = x + 4 cuts the parabola with equation y = X2 - 2x at two points A and
B. Find the magnitude of the angles that the tangents to the curve at A and B make
with the X-axis.
14. The line y = x + 1 cuts the parabola y = X2 - X - 2 at two points P and Q. Find:
(a) the coordinates of P and Q,
(b) the equation of the tangents at P and Q,
(c) the coordinates of the point of intersection of these two tangents.
15. Find the equations of the tangents to the \ parabola y
where
,
(a) the gradient is zero,
(b) the tangent is parallel to the line y = 2x + 1.

X2 - 2x

16. Find the equations of the tangent and normal to the parabola y
the point on the parabola where the gradient is 3.

4 at the points

2X2

5x

1 at

17. The normal to the curve y = (x + 2)2 at the point AC3, 1) meets the curve again at B.
Find
(a) the equation of the normal,
(b) the coordinates of B,
(c) the angle at B between the curve and the chord AB.
18. The line y = x
2 cuts the curve y = X3(X - 2) at two points A and B. Calculate the
angles which the tangents to the curve at A and B make with the X-axis and hence find
the angle between the tangents.
19. Find the equations of the tangents to the curve y = (x 2
where the curve crosses the X-axis.

I)(x - 2) at the points

20. Prove that 20x + y = 28 is a tangent to the curve y = x2(1 - 2x) and find the
coordinates of the point of contact.
21. Find the coordinates of the points on the curve y = x2(2x - 3) at which the tangents
are parallel to (a) the line y - 12x
1 (b) the X-axis.
22. Prove that the curve y
(x - 2)(X2 + 2x + 6) crosses the X-axis at one point only
and find the equation of the tangent at that point.

352 NEW SENIOR MATHS: TWO UNIT COURSE

15.8 Primitive functions _ _ _ _ _ _ _ _ _ __

I:

We will now consider the reverse problem of finding a function f(x) when its derivative
f'(x) is known. Such a problem is one of antidifferentiation. Thus if F(x) is such that
F'(x) = f(x), then F(x) is an antiderivative or primitive of f(x). Similarly, f(x) is an
anti derivative of f'(x).
The derivative of X2 is 2x and so X2 is a primitive of 2x.

The derivative of X2 + 3 is also 2x and so X2 + 3 is also a primitive of 2x.

The derivative of X2 + c is also 2x and so X2 + c for any real number c is the set of

primitives of 2x.
Graphically, some members of this set are depicted in fig. 15-33.
At any given x, the tangents to each of the curves are parallel.
It is always necessary to include the constant c when writing down a general primitive.
Which curve of a given family is appropriate to any given problem may be obtained by
specifying a set of conditions.
y

x
Fig. 15-33

EXAMPLE 18

Find the equation to the curve defined by f'(x)


(2, 8).
f'(x)
:. f(x)

=
=

point

~X5 + c

The primitive of x 3 + X2 is lX4


The primitive of 2X4

th~

2x

X2 + c

When x = 2, f(x)
8, so 8 = 4 + c, i.e. c
Hencef(x) = X2 + 4 is the required curve.
The primitive of X4 is

2x, which passes through

3x 3

+ ~X3 + c

4x

+ 2 is ~X5

[ The primitive of x,

-x4 - 2X2
4

= ~xn+1 + C, n

2x

+c

oF - \

These require no proof, but can be verified by differentiating n

+ c to give xn.

I5/GEOMETRICAL APPLICATIONS OF DIFFERENTIATION 353

Observe the pattern. The power of x is increased by 1 and the coefficient is the
reciprocal of this increased power. Note also that the primitive of a constant a is ax + c.
The primitive of X 312 is ~X5/2 + c
5

Since 3 = 3x-2 , the primitive of 3 is -3x-1 + c i.e.

-~ + c.

EXAMPLE 19
The gradient function of a curve is 3x2 - 2x and the curve passes through the point (2, 1).
Find its equation.

When x

dy = 3x2
2x

dx

:. y = x 3 - X2 + c
= 2, Y = 1 and so

1
8 - 4 + c
:. c
-3.
Equation is y = x 3 - X2 - 3.

EXAMPLE 20
If ~7z

= 3, express y in terms of x i f : = -3 and y = 0 when x = 1.


d2y
dx 2 = 3

: =

3x

- 3 =. 3

+c

dy
.
SInce dx

-3 when x

-6

dy = 3x - 6
dx

~X2

o = :23 -

+ d since y = 0 when x

:2

~X2

3x 2

i.e. 2y

6x

+~
I2x + 9

6x

EXERCISES 15(9)
1. Find primitives of:
(i) 5x2
(iii) X2
(v) (2x

3x
1
+ 1)2

(ii) 3 + 5x + X2 - 3x 3
(iv) (x - I)(x - 2)
(vi) 2X2 - 7x + 5

-354 NEW SENIOR MATHS: TWO UNIT COURSE

2. Find rules for f(x) given that:


(i) f'(x)
(iii) f'(x)
(v) f'(x)

3x - 5
X2 + 3x
4x3 - 6x2

(ii) f'(x)
(iv) f'(x)
(vi) f'(x)

=5
=
=

(2x - l)(x
(x + 3)2

+ 2)

3. Express y in terms of x, given that:


(i) dy

=3 +

dx

(iv) y'
I
<

2x - 3x2

(x - 3)(x

..) d y
( VII
dx 2

3x2 - 2x

dx

4)

= x3 +

(ii) dy

(V): = X4 -

~?z

1 (viii)

2X2

(iii) y' = X4 - x 3

~X3 + ~X2 (vi)

6x 3 - 4X2

(ix) y" = 0

4. In each of the following, find F(x), given that F '(x)


(a) f(x)
(c) f(x)
(e) f(x)
(g) f(x)

X2

6. Findf(x) given thatf'(x)

(d) f(x)

= X- 312 +

2x - 2 andf(l)
~

4X2

3x

7. At all points on a certain curve, :

x3

(f) f(x)

= ~+ $

5. Findf(x) given thatf'(x)

= f(x):

= -
5
= (X2 -

(b) f(x)

x2(1 - 3x)
3x4/3 - x l12

y' = Xll2

1)(x2

+ 1)

X- 512

4.

+ 1 andf(-l) = 3.
4x

6. The point (2, 4) belongs to the curve.

Find the equation of the curve.


8. Find the equation of a curve, given that the gradient at any point P(x, y) is
3x2 - 2x + 3 and that the point (3, 3) belongs to the curve.
9. For the function f(x) = (x + 2)2, find the rule that defines the function F, where
F'(x)
f(x) and F(l)
4.
10. Find the equation of a curve, given that :
dy
x = 3, dx

2 and y

= 2x

+ b at any point P and that when

= -3.

11. Find the rule that defines a functionf, given thatf'(x) = X2 - 2x + b for all x, and
f'(O) = 1 and f(O)
2.
12. A curve contains the point (0, 4) and its gradient is (x - 1)(x + 2) at any point on the
curve. Find the equation of the curve.
13. At all points on a certain curve,

= 2x.

The point (3, 6) belongs to the curve and

its tangent at this point is inclined at 45 0 to the X-axis. Find the equation of the curve.
14. Find primitives of:
(i) 3

+ 1

.. ) 3X3
2X2 + X-I
(III ---X-=-2- - -

(ii) X2
(iv) 1

+2

15/GEOMETRICAL APPLICATIONS OF DIFFERENTIATION 355

15. In each of the following find J(x) , given that


(i) J'(x) =

+2

(ii) J'(x) = 3x2

X2

X 312

(iii) J'(x)

16. Given ds

X2

= 12t 2

+
-

(iv) J'(x)

d 2s

ds

~~

= 24 and s

dx

(b) x

2X ll2

0 when t = 0, find

(b) s when

.
d 2x
dx
18. GIven dt 2 = 2 for all t ~ 0 and dt
(a) dt

2X-1I2

6t + I, find s in terms of t given that s = 4 when t = 1.

17. Given dP = 10 - 2t, t ~ 0 and dt


(a) t when

O.

-5 and x

~~

0
4 when t

= 0, find t when

CHAPTER 16

Integral

Calculus

16.1 Area _ _ _ _ _ _ _ _ _ _ _ _ _ _ __
In chapter 14, we considered one aspect of the calculus, namely the derivative of a
function, and we saw how it measures the gradient of a curve and in chapter 19, we shall
see how it measures rate of change.
Another very important aspect of the calculus is its use in finding the area measure of a
region of a plane surface bounded wholly or partly by curved lines. At first thought, there
may appear to be no connection between these two aspects of the calculus. However,
Newton, in the seventeenth century, discovered that these two apparently unrelated
concepts are in fact intimately connected by way of the primitive functions.
We shall be concerned with the area measure of the region of a plane surface bounded
by the graph of a continuous function, the X-axis and the lines x
a and x = b, where
b > a.
y

Fig. 16-1

Fig. 16-2

The method of finding the area of a rectangle is familiar. The region ABCD in figs. 16-1
and 16-2 is bounded partly by the curved line DC. The first estimate of its area would be
that it is greater than the area of the lower rectangle, ABED, but less than the area of the
upper rectangle, ABCF. This, of course, gives a very rough estimate (fig. 16-1).
In fig. 16-2, the interval AB has been divided into several parts, and with these parts as
base, upper and lower rectangles have been constructed. The area measure of the region
ABCD is greater than the sum of the areas of the lower rectangles but less than the sum of
the areas of the upper rectangles. This would give a better estimate.
As the number, n, of subdivisions of the interval AB increases, the difference between
the sum of the areas of the lower and upper rectangles decreases.
As n - 00, the difference approaches zero.

16/INTEGRAL CALCULUS 357

EXAMPLE 1
Consider the problem of calculating the area, A, enclosed between the parabola y
x 2,
the X-axis and the ordinates at x = 0, x = 5.
There is no commonly known formula of mensuration by means of which the area
can be evaluated. Partition the interval [0, 5] into 5 sections of equal width, as shown in
fig. 16-3.
y

234

Fig. 16-3
The actual area A lies between the sum of the areas of lower rectangles such as PQRS and
the sum of the areas of upper rectangles such as PQUT.
Denoting the lower sum by SL and the upper sum by Su we have
SL
0 + l(P) + 1(22) + 1(3 2) + 1(42) = 29
(Using Area =
breadth x height
l(P) + 1(22) + 1(3 2) + 1(42) + 1(5 2)
54 for each rectangle)
29 < A < 54.
Hence
Now partition the interval into 10 equal sections of width

~. In this case

SL

= (~y

Su

= (~)3[p + 22 + 32 + 42 + 52 + 62 + 72 + 82 + 92 + 102]

[P

+ 22 + 32 + 42 + 52 + 62 + 72 + 82 + 92] = 2~5
35~

Hence

35i
== 385

= 48k

< A < 48k

If 20 subdivisions are taken each of widthl, then


SL = (lY[p

Su

Hence

+ 22 + 32 + ... + 192] = 6~ x 2470 = 38g

(!)3[1 2 + 22
4

+ 32 + ... + 202]
3812
32 < A < 441.2
32

44~

Evidently as the number of subdivisions increases, the approximation to the true area
by the upper and lower rectangles improves. If we make 100 subdivisions of the above
interval and use a calculator to evaluate the sums, then it is found that
SL
4104 and Su = 42'29, so that 4104 < A < 4229.
It seems that A can be calculated to any desired degree of approximation by taking n
sufficiently large. We must return to the general case to determine a method for
calculating the exact value of A.

I.

358 NEW SENIOR MATHS: TWO UNIT COURSE

Letfbe a continuous and increasing function in the interval [a, b]. Divide this interval
into n parts, not necessarily equal, by the points XI' X2, x 3' ... xn such that a = Xo < XI <
x 2 < . . . < xn = b (fig. 16-4).
Y

= X O X,X2

+
Fig. 16-4
Using the subdivisions (XI - x o), (x 2 - XI) ... (xn - x n- I) as bases, construct upper and
lower rectangles as shown. If we denote the sum of the areas of the lower rectangles by Sv
then
SL

= f(xo)(x l

- xo)

+ f(x l)(X2

+ ... + f(xn-I)(xn -

XI)

x n- I)

and if we denote the sum of the areas of the upper rectangles by Su, then
Su

= f(xl)(x l

- xo)

+ f(x 2)(X2

+ ... + f(xn)(xn

XI)

- x n- 1)

It is obvious from fig. 16-4 that Su > SL and that Su - SL equals the sum of the areas of
the shaded rectangles. If we denote the area bounded by the curve, the X-axis and the
ordinates X = a and X = b by A~ then SL < A < SUo
If we divide the interval [a, b] into a very large number of parts, i.e. n - 00, the sum of
the areas of the shaded rectangles becomes very small; i.e.,
Su - SL - 0 and Su and SL approach the same limit, A.
y
8
feb)

A
f(a)

x x+

ox b

Fig. 16-5
Consider now what could be described as a typical rectangle of width ox at any point X
in the interval [a, b] (fig. 16-5). The area of a typical lower rectangle is f(x) ox and of a
typical upper rectangle is f(x + ox) ox.
b

Then

SL = I:f(x)ox

and

Su = I:f(x

ox) ox

16/INTEGRAL CALCULUS 359

The Greek letter E denotes the 'sum of' and so

.E ftx) ox denotes the sum of the areas


a

of the n lower rectangles in the interval [a, b].


b

.E f(x) ox < A

Thus

<

As n -

00,

0 and Su -

ox -

.E f(x + ox) ox.

SL and so
b

= ox_o
lim E f(x) ox.
a

This limit is written thus:

,
A

J.b f(x) dx,

and is called the integral off from a to b.


We have not, as yet, established how this integral measures the area A.
If we denote the area under the curve from A(a, f(a to P(x, f(x by A(x), then the
area under the curve from A to Q is A (x + ox). The area, then, under the curve from P to
Q is A(x + ox) - A(x) (fig. 16-5). Hence
f(x)ox < A(x

Dividing by ox

=1=

f(x) < A (x

As ox - 0, f(x

ox) - A(x) < f(x

ox) ox.

0;; - A(x) < f(x

ox)

0, we get

ox) - f(x) and


A(x + ox) - A(x) _ !i ( )
ox
dx A x.

Thus !A(X) = f(x), from which it follows that A(x) is a primitive of f(x).
That is, A(x)
F(x) + c, where F'(x) = f(x) and c is any real number.

When x
a, A(a)
0 and so 0 = F(a) + c.

Hence
A(x)
F(x) - F(a)

When x
b, A(b) = A and so A = F(b) - F(a). That is,

r f(x)dx = F(b) b

F(a)

" a

where F(x) is a primitive function of f(x).


This is a very important result and is called the Fundamental Theorem of the Calculus.
It is a remarkable result in that the differential calculus, which deals with gradients and
rates of change, is related to the integral calculus by way of the primitive function.
In the above discussion, we considered only an increasing function in the interval [a, b].
The theorem is equally true for a decreasing function or for a function which is sometimes
increasing and sometimes decreasing in the interval, provided that the function is
continuous.
In evaluating an integral, it is convenient to set it out as follows:

1:

f(x)dx

= [F(X)I
= F(b) - F(a)

, " '. . . _.. .


-.~

~- ~~-'== =--=~'=~~=~----------------~===

360 NEW SENIOR MATHS: TWO UNIT COURSE

EXAMPLE 2
Evaluate the following:

f,3(X 2-

(a)

(a)

3(X2

x)dx

(b)

[3

x) dx =

- X-

r:(X 2X2 + 3x 3

4)dx

2J3

(9 - ~) -(~ - ~)

= 4j

(b)

DX

EXAMPLE 3
Calculate the area of the region bounded by the graph of the straight line y = 2x + 3, the
X-axis and the ordinates x = 1 and x = 5.
y

The required area measure is

r'(2X + 3)dx = [X2

Fig. 16-6

+ 3x

(25 + 15) - (l + 3)
36
area = 36 sq. units.
Hence
This area can be checked arithmetically by calculating the area of the shaded trapezium.
=

EXAMPLE 4
Find, by integration the area of the region enclosed between the parabola y

X-axis and the ordinates x = 0 and x = 5. (See example 1.)

J:X dX =
2

GX J:

X 53 -

3
41~.

x2, the

Fig. 16-7

=!

Compare this exact area with that obtained in example 1.

x
1,,__ _.

16/INTEGRAL CALCULUS 361

EXAMPLE 5
Calculate the area of the region bounded by the graph of the parabola f(x)
7x + 10, the X-axis, and the ordinates x = 3 and x = 4.

X2 -

Fig. 16-8
X3 _ Zx 2
[ 32

(~

- 56

lOX] 4
3

40) - (9 -

6i + 30 )

-2~

The integral in this case is a negative number which represents the measure of the signed
area of the shaded portion in fig. 16-8. Since, however, we associate the measure of area
by a positive number, we say that the area is 2~ sq. units.

EXAMPLE 6

Find (a) the signed area, and (b) the area, bounded by the graph of
f(x)

and the X-axis.


The graph of f cuts the X-axis at

x(x - 2)(x

X =

1)

-1, 0 and 2.

y
f(x)=x(x- ~x+1)

-1

Fig. 16-9

r:X(X - 2)(x + J)dx.


r:X(X - 2)(x + J)dx ~ r:(X3- X2 - 2x)dx

(a) The signed area is obtained by evaluating

362 NEW SENIOR MATHS: TWO UNIT COURSE

The signed area is 2! sq. units and denotes the algebraic sum of Al and A 2
(b) It is necessary to evaluate

IA II

and A2 separately.

Jo\X 3- X2 -

Al

= {~ - x'

= 4

Thus

IAII
A2

2x)dx

X{

- 4

= -2j
2~.

= Jc:(X'
=

- X2 - 2x)dx

[X4 _ x 3 _ X2] 0
4

-1

o-O+~

1)

12
5
Total area = 2i + 12
3-& sq. units
Some properties oj the integral:
1.

J>(X)dX = - Jbf(X)dX

2. i.f(X)dx + I'f(X)dx

3. I:f(X)dX

1:

g(x)dx

1:

({(x) gx)dx

I:f(X)dX 4. J:cf(x)dx = cI:f(X)dX

No formal proof of these properties will be given, but we will verify them with a few
numerical examples.

EXAMPLE 7
Evaluate:
(a) I:X 2dx, I:X2dX and verify that I:X 2dx= - I,'X 2dx
(b)

2
X2 dX, J:X 2dx, I'X 2dX2 and verify that I,2x2dx + I2'X 2 dX =I'X2dX

(e) I'X2 dX, I,'2xdx, I,'<x 2 + 2x)dx and verify that

I:(X 2 + 2x)dx

I'X 2 dx + J:2XdX

(d) J,'(4X 2 + 8x)dx, 4I:(X2 + 2x)dx and verify that

J.'(4X 2 + 8x)dx

4J:(X2 + 2x)dx

16/INTEGRAL CALCULUS 363

J:X'dx = Ux'J: 9 fX'dx = [ix'J: = 0 -

(a)

(b)

=9
-9
1

3 - 3 = 21

(c)

8~

f2Xdx =[x,]: =9 1 =8

f<x' + 2x)dx = [ix' + x'I

=(9+9)-0+ 1)

= 16~

= 8j + 8

(d)

!.'<4X'

8x)dx [~x' + 4X'J:


=

= (36

+ 36) - 0

72

4f:<x' + 2x)dx 4[ ix' + x'I

4[(9

+ 9) - 0] = 72

EXERCISES 16(a)
Evaluate the integrals (1. to 21.):
1

1.

Jo (2X -

4.

3
f1 (2t

7.

f:<6 + 8x 3x')dx 8. r:X(1 - x')dx

5)dx

- 4)dt

r:<3X + f)dx

3.

S.

f-:

6. f-:(2x

(u

2
-

2u)du

10. fo (X

2)3dx

13. fo\aX2

bx

11. f_:X2(l - x)dx

c)dx

f_:(X

2.

14.f Q(X2 - 2x)dx


o

4)dx
1)2dx

9. f_:(2X 2 - 8x

Q
12. fb 3x2dx
15. f04X1l2dX

8)dx

364 NEW SENIOR MATHS: TWO UNIT COURSE

16.

f_~l/2( -x3)dx

19. fOl(Vi

18. fo'(XI/ 5

~)dx

20.

f 1!2(
114

1)

3"

21J(

dx

xl/3)dx

~)' dx

22. Calculate the area of the region bounded by the line y = 3x


lines X = 3 and x = 6.

+ 5, the X-axis and the

23. Find the area of the region bounded by the parabola y


2.
ordinates x = -1 and x

+ 2, the X-axis and the

X2

24. Find the area of the region bounded by the X-axis and the graph of y

2 - x - x 2

25. Find the area of the region bounded by the X-axis and the graph of y

x2(1 - x).

26. Calculate the area of the region bounded by the graph of f(x) = X2
X-axis, and the lines x
1 and x = 4.

4x

27. Find the area of the region bounded by the curve with equation y
and the X-axis.
28. If/0(4 - 2x)dx

= 4,

4, the

5x - 2X2

find the value of a.

29. If f_:XdX = 0, evaluate a.

30. Ifcf_:(X - 5)dx = 1, evaluatec.


31. Calculate the area of the region bounded by the graph of f(x)
and the ordinates x
2 and x = 3.
32. Calculate the area bounded by the curve y

= (x

- 2)3, the X-axis,

x2(3 - x) and the X-axis ..

33. Find the positive number, k, such that the area of the region bounded by the graph of
f(x) = kx(2 - X)2 and the X-axis is equal to 1 unit.
34. For the graph of f(x)

(x

1)(x

1)2 calculate:

(a) the area bounded by the curve, the X-axis and the ordinates x

o and x = :21

(b) the area bounded by the curve and the X-axis


(c) the area, right of the origin, bounded by the curve and the coordinate axes.

35. Find the area bounded by the parabola y

8x - X2 and the X-axis.

36. Find the area of the region bounded by the curve y = x(x - 2)2 and the X-axis.
37. Show, by integration, that the area of a
unit square is
(a) bisected by the line y = x
(b) trisected by the curves y = X2 and .

y = Vi

(c) divided into four equal parts by the

~ andy = x.

curves y = x 3 , Y

y
H

(0,1)1-------.(1,1)

16/INTEGRAL CALCULUS 365

16.2 Area between two curves _ _ _ _ _ _ __


If f and g are two continuous functions
whose graphs do not intersect in the interval
[a, b] on which f(x) > g(x), the area
meas e of the region bounded by the two
curves a
the ordinates x = a and x = b is
equal to

fa

(x)dx -

f:

fb g(x)dx
a

(x) - g(xdx

Fig. 16-10

EXAMPLE 8
Calculate the a ea of the region enclosed by the graphs of f(x) = x
g(x) = X2 - x
2.
It is necessar first to find the

x-values of the poin s of intersection of


the straight line an the parabola. At
the points of intersection,
X2 -

+ 1 and

f(x)=x+1

g(x)=x2-x-2
(3,4)

x - 2 = x +

2x - 3 = 0

(x - 3)(x + 1) = 0

x = 3 or -1

In Fig. 16-11 we may consider the


interval [-1, 3] as being divided into n
intervals. The width of a typical
interval is bx. The area bA of a typical
element is of length f(x) - g(x).
bA ::::; (f(x) - g(x))bx
X2 -

(-1,

x=3

and A ::::;

E (f(x)

- g(x)) bx

Fig. 16-11

x=-I

As n - 00, i.e. bx - 0, the limit of the sum of the areas of the approximating
rectangles approaches the area A.
x=3

Hence

A = lim
(f(x) - g(x) )bx
ox-o x=-I

= J~(f(X) =

f-:( -x

:\X

+
2

- X3
+
3

g(xdx
1) - (X2 - X - 2dx

2x

X2

3)dx
]

3x

-I

366 NEW SENIOR MATHS: TWO UNIT COURSE

( -9 + 9 + 9 )

-0

I - 3)

Area = 10~ sq. units.

EXAMPLE 9
Calculate the area of the region bounded by the X-axis and the curves with equations
y = .Jx and y = 6 - x.
At the point of intersection of the
Y
curves,
.Jx = 6 - x
x = 36 - 12x + X2
X2 - 13x + 36 = 0
(x - 9)(x - 4) = 0
Thus x = 4, since x = 9 does not satisfy
.Jx = 6 - x but corresponds to the point
of intersection of the curves defined by
x
y = - .Jx and y = 6 - x.
Fig. 16-12
The point of intersection P is (4, 2).
It will be necessary to divide the interval [0, 6] into two intervals [0, 4] and [4, 6] because
a typical approximating rectangle in the interval [0, 4] has length .Jx and in the interval
[4, 6] has length (6 - x). Hence the required area is given by

Now

6
/4 (6 -

and

x)dx

Thus the total area = 7~ sq. units.


Alternatively, we may consider the region
as being divided into approximating
rectangles of width oy by lines drawn
parallel to the Y-axis for values of y in the
interval [0, 2] (fig. 16-13).
The lengths of these rectangles would
be the difference in the x-coordinates
of the appropriate points, namely,
(6 - y) - y2.

[6X - ~X2J:

=2

x
1 - _y3
1 ]
= [ 6y - _y2
= 7~

23
(units2)

Fig. 16-13

16/INTEGRAL CALCULUS 367

EXERCISES 16(b),
1. Calculate the area ofthe region bounded by the line y
2. Calculate the area of the region enclosed by the line y
y =

X2

2x and the parabola y

= x2

x + 1 and the parabola

2.

3. Calculate the area bounded by the graphs off(x)

= x 2 , g(x) = x-;,
x > 0 and the line

x = 3.

+ 3 and the parabola y x 2.


the parabola y = -x2 and the line x = 2.

4. Find the area of the region enclosed by the line y = 2x


5. Find the area enclosed by the line y = 2x,

6. Calculate the area of the region enclosed by the graphs of:


(a) the parabola y = X2 + 4 and the line y = 5
(b) the parabola y = 2X2
5x
3 and the line y = 3x - 3
(c) the parabolasf(x) = 6x2
5x and g(x) = 5x - 4x 2
7. Find the area of the region corresponding to the graphs of the following relations:
(a) {(x, y) : y ~ xl n {(x, y) :,Y ~ 3x - x21
(b) I(x, y) : y ~ 4 - x 21 {(x, y) : y ~ X2 - 41
(c) {(x, y) : y ~ 6 - xl n I(x, y) : y ~ X2 - 5x

61

8. A straight line through the origin cuts the parabola y = 4x - X2 at the point where
x = 3.
(a) Find the equation of this line.
(b) Calculate the area of the region bounded by
(i) the parabola and the straight line,
(U) the parabola and the X-axis. ""
,~.

--

Find the equation of the tangent to the parabola y = X2


and find the area enclosed by:
(a) the parabola, the tangent and the Y-axis,
(b) the parabola, the tangent and the coordinate axes.

+ 1 at the point where x = 2

10. Calculate the area of the region bounded by the parabolas y = X2 and y = 4 - x 2
Find also the area of the region bounded by the parabolas and the X-axis.
11. In what ratio does the X-axis divide the area of the region bounded by the parabolas
y = 4x - X2 and y = X2 - x?
12. Part of a vertical section of an ore deposit takes the form illustrated in fig. 16-14,
which is not drawn to scale. Exploration by vertical drilling at P and Q, along with
y

Fig. 16-14

:1

368 NEW SENIOR MATHS: TWO UNIT COURSE

geological interpretation of the rock formations in the light of evidence from other
sections, have indicated the distances shown on the diagram. Taking the axes OX and
OYas indicated, find
(a) the equation of the straight line AB,
(b) the depth of muck at R,
(c) the equation of the curve COD, assuming it is of the form y = ax 2n ,
(d) the total area of the section, ABDC, of the ore deposit.

16.3 Volume of solids of revolution

I'

Consider a continuous function in the interval [a, b]. If the plane section ABDC is rotated
about the X-axis, we thereby generate a solid as shown in Fig. 16-15 the vertical cross
sections of which are circles. Such a solid is called a solid oj revolution.
y

Fig. 16-15
P(x, y) is a point on the curve y = J(x) and Q(x + ox, y + oy) is a point close to P. The
ordinate PM describes a circle of area 7ry2 and QN describes a circle of area 7r(Y + oy)2.
The typical lower rectangle PRNM describes a cylinder of volume 7ry2 ox and the typical
upper rectangle describes a cylinder of volume 7r(y + oy)2ox. If a typical layer PQNM
describes a solid of volume 0 V, then
7ry 2ox

< 0 V < 7r(Y + oy)20X.

E 7ry 2ox <

Thus

E(Y + oy)20X.

<
b

As ox -

0,

V = lim

ox -o

E 7ry 2ox
a

b
i 7ry 2dx
7r ibY2dX

16/INTEGRAL CALCULUS 369

EXAMPLE 10
Find the volume of a right circular cone height h and radius of the base r.
The cone may be considered as a solid of revolution generated by rotating the right
angled triangle GAB about the X-axis (Fig. 16-16)
The equation of GA is y = ~x.
h

= /)x-o
lim

E 7ry

ox

Fig. 16-16

EXAMPLE 11
Find the volume of a sphere of
radius r.
The volume of the sphere may be
considered as the volume generated
by rotating the semicircle defined by
y = ..Jr2 - x 2, -r ~ x ~ r
about the X-axis-.
Hence
V =

7r

f :!2 dX where y = ..Jr2 - X2

f~(r2

7r

- x2)dx sincey2

r2 - X2

Fig. 16-17

. - . ......,..........-"---. -~.~-=.~
.~.~~~~~=.-.~.=-,-=
-..=.. ~---=-~---~~---------~~~

370 NEW SENIOR MATHS: TWO UNIT COURSE

EXAMPLE 12
The part of the parabola y = X2
between x = 1 and x = 3 is rotated
about the Y-axis. Find the volume
generated.
A typical layer cut perpendicular
to the Y-axis would be approxi
mately cylindrical in shape with
radius x and width oy and volume
oV given by
oV Z 7rX2oy
and the total volume V would be
given by

v = 11:~'X'dY

-3

Fig. 16-18
11:

~9 ydy since x'

=y

= 407r

EXAMPLE 13

Find the volume of the solid formed when the area bounded by the parabola y = 4 - X2
and the X-axis is rotated about (a) the X-axis (b) the Y-axis.
y

4
y

-2

Fig. 16-19

Fig. 16-20

(a) V = 7rJ2y2dX where y = 4 - X2

(b) . V

-2

= 7r f04X 2dY where X2 = 4 4


7r fo (4 - y)dy

[ 8 + -x1]
[(32 - ~ + 3;) - (-32 + ~ - 3nJ

7r 16x - -x 3
3
11:

512

= If7r

5 -2

[4Y- ~y~:

11:

=
=

7r[(16 - 8) -:- 0]
87r

16/INTEGRAL CALCULUS 371

EXERCISES 16(c)
1. Find the volume of the solid of revolution formed by rotating the arc of the parabola
y = X2 between x = 0 and x = 3 about the X-axis.
2. Find the volume of the solid of revolution formed by rotating the line y
x = 0 and x
4 about the X-axis.

= 2x between

3. A cone is formed by rotating about the X-axis the segment of the line joining the
points (1, 0) and (3, 4). Find the volume of the cone.
4. The semicircle y
-J9 - x 2 is rotated about the X-axis. Calculate the volume of the
sphere generated.
5. The region bounded by the parabola y = x - X2 and the X-axis is rotated about the
X-axis. Find the volume of the solid so formed.
6. Find the volume of the solid formed when the region bounded by the parabola
y = 1 - X2 and the X-axis is rotated about (a) the X-axis (b) the Y-axis.

7. The region bounded by the parabola y


(x
2)2 and the coordinate axes is rotated
about the X-axis. Find the volume of the solid so formed.

:,

8. The segment of the line joining the points (0, 3) and (6, 0) is rotated about (a) the
X-axis (b) the Y-axis. Find the volume of the cone formed in each case.
9. A football has a volume approximately the same as the volume generated by rotating
the ellipse 9x2 + 16y2 = 144 about the X-axis. Find the volume.
10. Find the volume of the solid formed when the region bounded by the parabola
y
9 - X2 and the coordinate axes is rotated (a) about the X-axis (b) about the
Y-axis.
11. A drinking glass has the shape of a truncated cone. If the internal radii of the base
and the top are 3 cm and 4cm respectively and the depth is 10cm, find by integration,
its capacity. If the glass is filled with water to a depth of 5 cm, find the volume of the
water.

12. A hemispherical bowl of radius a units is filled with water to a depth of units. Find,
by integration, the volume of the water.
13. Find the volume of the solid formed when the region bounded by the parabola
y = 4 - X2 and the line y = 1 is rotated about the Y-axis.
14. Find the volume of the solid formed when the region bounded by the parabola
y = X2 - 2 is rotated about (a) the X-axis (b) the Y-axis.
I

15. Find, by integration, the volume of the sphere generated by rotating the circle
X2 + y2 = 16 about the X-axis.
2x-J 1 - X2 between x = 0 and x = 1 is rotated about
16. The area under the curve y
the X-axis. Find the volume of the solid so formed.

17. Find the volume formed when the ellipse 4X2


(b) about the Y-axis.

+ y2 =

16 is rotated (a) about the X-axis

18. The region bounded by the curve x l12 + yll2 = 2 and the coordinate axes is rotated
about the X-axis. Find the volume of the solid generated.

372 NEW SENIOR MATHS: TWO UNIT COURSE

19. The region bounded by the curve xy = 1, the X-axis and the lines x = 1 and x = a,
a > 1 is rotated about the X-axis. Find V, the volume generated. Hence find lim V.
a_<x>

20. The area bounded by the parabola y = 2x - x 2 , the Y-axis and the line y
rotated about the X-axis. Find the volume generated.

,'I

= 1 is

21. Find the volume of the solid generated by rotating the region bounded by the
parabola y = 1 - X2 and the lines x = 1, y = 1 about (a) the X-axis (b) the Y-axis.
22. A cone is formed by rotating the segment of the line x + 2y = 4 cut off by the axes
about (a) the X-axis (b) the Y-axis. Find, by integration, the volume of the cone in
each case.

16.4 Approximate integration _ _ _ _ _ _ _ __


If f(x) is a continuous function on the interval [a, b], then fabf(X)dX is a specific number.

However, we may be unable to find exactly what this number is because of our inability to
find the primitive of f(x) in many cases. The art of integration lies in our ability to find a
function F(x) such that F '(x) = f(x). This is not always possible. There is, for example, no
familiar function F(x) corresponding to f(x) = .Jf+X4 or f(x) = e-x2 . Sometimes we
have to invent a function as in the case of the natural logarithm function .
Although we may be unable to find the integral exactly, there are methods available to
find this number approximately. One simple method is to draw a graph of the function
using graph paper and then to count squares; but this is rather tedious . We shall consider
two simple methods.

(a) Trapezoidal rule

y
y = f(xl

f(al

y = f(xl

f(a}

f(bl

f(b)

Fig. 16-21

Fig. 16-22

The simple application of the trapezoidal rule substitutes a straight line in place of the
curve between x = a and x = b as shown in Fig. 16-21 . The area of the trapezium is used
as an approximation to the area under the curve.
Using the formula for the area of a trapezium as

'-<1__ _.___"_._______ ___~_ _'____~-------

4of the sum of the parallel sides x the


...---.- .. .., . r

o -

-,-.-

':

16/INTEGRAL CALCULUS 373

distance between them, then

(b

Ja f(x)dx

~ 2(b - a)[f(a)

+ f(b)]

The shaded area in Fig. 16-21 indicates the error involved. If the graph of f(x) were a
straight line, there would be no error.
The error can be reduced but not completely eliminated by subdividing the interval
[a, b] into n sub-intervals of equal width h = b - a and constructing a trapezium on each

n
sub-division as shown in Fig. 16-22. In general, the more subdivisions, the smaller the
error. In this case

f(x)dx ~ 2[f(a) + 2f(x 1) + 2f(x2) + - - - + f(b)]

EXAMPLE 14
Evaluate

f02

dx using the trapezoidal rule by dividing the interval [0, 2] into

(a) 2 sub-intervals

(b) 4 sub-intervals.

(8)

(b)

/
o

Fig. 16-23

Fig. 16-24

05

1'5

f(x)

1936

1732

1'323

(a) The width h of each strip is 1 unit


:.

r~4 -

1-5

0-5

x'dx '"

~[f(O) + 21(1) + 1(2)]

= 1[2 + 3464 + 0]
2'732

374 NEW SENIOR MATHS: TWO UNIT COURSE

(b) The width h of each strip is 0'5 units

:. f02.J4 - x 2dx ::::: 0~5 [1(0) +


=

4[2

2996

2/(0'5)

3872

2/(1)

3464

2/(1'5)

2646

+ /(2)]

0]

x 2 + y2 = 4 is the equation of a circle centre (0, 0) and radius 2 units. Transposing, we


get y = ..)4 -

X2

and hence f02.J4

- X2 dx denotes the area of a quarter of the circle in

the first quadrant as shown in Figures 16-23 and 16-24.


1
Area
47rr2 = 7r when r = 2.

II

Hence the exact value of f02.J4

- X2 dx

is

7r(:::::

3'142). The trapezoidal rule undere

stimates the area in (a) with an error of about 0'410 or 13070 and in (b) with an error of
about 0'146 or 4'6%.

(b) Simpson's rule


The trapezoidal rule approximates the curve with a straight line or a series of straight lines
on the interval a ~ x ~ b. Simpson's rule is based on the fact that we replace the given
function by a quadratic function i.e. we replace the graph of the given function from
x
a to x = b by an arc of a parabola which intersects the given curve at x
a, x = b
and the mid-point

x a;

b as shown in Fig. 16-25.


y

... ---. V

parabola

"'\\
f

(a

b)

-2

a+b
-2

f(x)

f(b)

Fig. 16-25
It can be proved that the area under the parabola through these three points is

b -

a~(a)

4f~

; b)+ f(b)]

Simpsons's rule then states that

fa/(x)dx ::::: b
b

We can, of course, break that interval up into sub-intervals and apply Simpson's rule to
each sub-interval.

16/INTEGRAL CALCULUS 375

EXAMPLE 15
Evaluate f02 ~4 - X2 dx (as in the previous example) using Simpson's rule with
(a) 3 function values

(b) 5 function values.

0'5

1'5

f(x)

1936

1732

1323

2 and a

(a) Taking a = 0, b
f02

~4 -

b = 1, we get

X2 dx

~ 2 ~ l/(O) + 4f(l) + f(2)]


1

= -[2
+ 6'928 + 0]
3
2976

(b) We break up the interval [0, 2] into two sub-intervals [0, 1] and [1, 2], then apply
Simpson's rule to each sub-interval and add the two areas.

Lvf4 -

x'dx =

I - 0 [/(0)

= ~[2 +
= 1913

7'744

+ 4/(0-5) + /(l)]
+

1'732]

2
1
~4 - X2 dx ~ -i-[f(l) + 4f(I'5) + f(2)]

= ~[1'732 + 5292 + 0]
= 1171

:_ fo'vf4 -

x'dx = 1-913 + 1-171

= 3084

This differs from the correct value of 7r by less than 2 per cent.

Simpson's rule gives an exact value whenf(x) is a quadratic or cubic polynomial just as
the trapezoidal rule will give an exact value when f(x) is a linear function.

EXAMPLE 16
Evaluate J'x,dx (a) by integration (b) by using Simpson's rule with 3 function values:
2
x 2dX

fI

(a)

(b) a

1, b = 2, a

= [!x312 = ~ _ ! = Z

+ b = 1 5

JI

,,

376 NEW SENIORMATHS: TWO UNIT COURSE

fX2dx '" b -

= 6[1 +

~(a)

4f& ;

b)+ f(b)]

4 x 225 + 4]

. (
3"7 as III
a)

When discussing primitive functions, we stated that the primitive of xn is

+ c,

n+

except when n = -1. Although the primitive does exist, it is not of the form _1-lxn+l.

Actually it is associated with logarithms as will be seen in a later chapter.

EXAMPLE 17
EvaluateJ 3 !dx using (a) the trapezoidal rule with 4 strips (b) Simpson's rule using 5
IX

function values.
X

1'5

f(x)

0'6667

2'5

05

04

03333

Fig. 16-26

Fig. 16-27

fIx)

15

25

15

~dX '"

25

Simpson's rule

Trapezium rule

(a)

0'5 [f(I)

i[1

+ 2f(I'5) + 2f(2) + 2f(2'5) + f(3)]

1'3333

08

0-3333]

= 1'1166
The exact value to 4 decimal places is 10986. The trapezium rule overestimates the
result by 0-0180 which is less than 2 per cent error_

(b) Applying Simpson's rule to the sub-intervals 1

J
f ~dX
2

!dx

:::=;

~ X ~

2 and 2

2 - 1 [f(1) + 4f(1-5) + f(2)]

~ X ~

3 we get

0-6945

IX

'" 3

J: -dx
IX

:::=;

2 [f(2)

0-6945

+ 4.f(25) + f(3)]

04056

0-4056

1'1001
Simpson's rule overestimates the result by 0-0015 which is less than 0-2 per cent error.

16/INTEGRAL CALCULUS 377

EXERCISES 16(d)
1. Evaluate

14

dx using the trapezoidal rule by dividing the interval [0, 4] into 2

sub-intervals.

2. Use Simpson's rule with three function values to give an estimate of

LX2 d:

f:..,;x' + 9 tix.

"I

3. Evaluate

1 (a) using the trapezoidal rule by dividing the interval [0, 1] into 2

sub-intervals (b) using Simpson's rule with three function values.


4. Evaluate

12x3dx (a) by integration (b) using Simpson's rule with three function

values. What do you observe?


5. Given that f(x) = 2 x , complete the following table:

With these five function values estimate

Jor~~x.dx using

(a) Simpson's rule (b) the trapezium rule.


6. Evaluate

1:

4X dx using (a) Simpson's rule with five function values

(b) the

trapezium rule with four sub-intervals.


7. Evaluate

1\/XdX using

(a) direct integration


(b) Simpson's rule with three function values
(c) the trapezium rule with three function values.

Also, calculate the percentage error with the approximations used in (b) and (c).

8. Evaluate

1'' ';1 + x'dx using Simpson's rule with

(a) three function values

(b) five function values.

9. Use Simpson's rule with five function values to evaluate

f.2!dX.
IX

0>3

10. Use Simpson's rule with nine function values to evaluate

-{l>S

places.

dx to 5 decimal

378 NEW SENIOR MATHS: TWO UNIT COURSE

11. Evaluate

f:.J1

+ x'dx using Simpson's rule with 5 function values.

12. Use Simpson's rule with five function values to estimate the volume of the solid
formed by rotating the curve y

X2

about the X-axis between x

-2 and x

= 2.

CHAPTER 17

Trigonometric
( Circular)
Functions
17.1 Radian measure of an angle _ _ _ _ _ __
We shall see later in this chapter, when dealing with the calculus of the trigonometric
functions, that degrees are not a satisfactory measure of angle size in many situations.
The measure of an angle commonly in use is the radian. Consider the unit circle.
Starting from the point A, mark off, in an anti-clockwise direction, an arc of length
measure 1. We define the magnitude of the angle which this arc sub tends at the centre of
the circle as 1 radian. If we mark off an arc of length measure (J, then the magnitude of the
y
angle that this arc subtends at the centre of
the circle is (J radian. The same number
measures the length of the arc and is also the
measure in radians of the angle.
If the magnitude of an angle is 8 radians,
we write this as (Jc, but frequently we omit
the symbol c

An arc of length measure 'Ir subtends a straight angle of magnitude 'lrC at the centre of
the circle, and, as we already know, the magnitude of a straight angle is 180 0 This gives
us a convenient and accurate relation between the radian and degree measure of an angle,
even though 'Ir is an irrational number and 180 a rational number.

I
i.e.,
and

lc

'lr

= (1!0)0

= 1800 I

5718'

1
= (1;0) and so
0'
C8~OY and 0 = (1~~)'
c

380 NEW SENIOR MATHS: TWO UNIT COURSE

The relations 7r

180 and 1

7rc
180 enable us to convert radians to degrees and

degrees to radians.
7r C

7r C

47r c

77r c

Thus - = 90' - = 60'


= -5 x 180 = 144'' 6
= -6 x 180 = 210 .
2
'3
'
However, angles measured in radians are not always presented in terms of 7r. It is
usually necessary to use a calculator to multiply by 180 to convert to degrees
7r

180
4.6 c = 4'6 x 180
7r
7r
= 137.509 ...
= 263.560 ...
= 13731'
26334'
sin 2'4c and, say, cos 46 c can be found, using a calculator, without first converting

2'4

2.4

radians to degrees. Press the mode key so that RAD appears on the display. Then proceed
as follows:
Display

Operation sequence
2 41 sin 1

0'67546 .. .

4[] 6
Note that cos 46c is a Qegative number. Why?

-0,11215 .. .

Likewise, angles in degree measure can be converted to radians by multiplying by 1;0'


160

=
=

160 x

279

1~~

7532'

75.533 ... x.

1~~

EXERCISES 17(a)
1. Express the following angles in radian measure, in terms of 7r.
(a) 30
(d) 210
(g) 330

(b) 225
(e) 315
(h) 144

(c) 72
(f) 11230'

2. Express, in degrees, the angles whose radian measures are as follows:


(a) 37r

(b) 77r
(c) 67r

4 8 5
57r
(d) 37r
(f) 117r

(e)
2
12

117r
(g) 1-87r
(h) 8

3. Express, in degrees, with the aid of a calculator, the angles whose radian measures are
as follows:
(a) 0-5
(d) 4-26
(g) 5-24

(b) 1'82
(e) 272
(h) 4'782

(c) 3
(f) 3'426

4. Use a calculator to find the radian equivalent of each of the following:


(a) 42
(d) 164
(g) 13812'

(b) 74
(e) 489'

(h) 728'

(c) 105
(f) 220

17/TRIGONOMETRIC (CIRCULAR) FUNCTIONS 381

5. Express, in degrees, with the aid of a calculator, the angles whose radian measures are
as follows:
(a) 06
(b) 042
(c) 2
(d) 172
(e) 43
(f) 6
(g) 286
(h) 54

17.2 Arc length and area of


a sector of a circle ____________
Let the arc AB (fig. 17-2) subtend an angle

of magnitude ()c at the centre of a circle with

radius r units.

The length of arc AB = I.

Let angle POQ = 1C, and so length of


arc PQ = r.
arc AB
LAOB
arc PQ
LPOQ'
1
()
i.e.,
1

1=

r()

The shaded area is the sector AOB.


area of sector AOB
area of circle

= rlJ

Fig. 17-2

LAOB
27r

()

7rr2

27r

17.3 Area of a segment of a circle _ _ _ _ _ __


The chord AB cuts the circle into two
segments: the minor segment, ACB (the
shaded portion in fig. 17-3), and the major
segment, ADB.
Area of segment A CB
= area of sector AOB - area of ,6.AOB

= !r2() - !r2sin ()

2
2
That is, area measure, A, of a segment is
given by
Fig. 17-3
A

= !r2() - sin ().

382 NEW SENIOR ~ATHS: TWO UNIT COURSE

EXAMPLE 1
Find the length of the arc of a circle of radius 12 cm if the arc subtends an angle of
(a) 15 radians
(b) 40 at the centre.
(b) I

(a) I = r()

= 12 x 15
= 18

r()
7r

= 12 x 40 x 180

Arc length is 18 cm

= 83776

Arc length is 84 cm

EXAMPLE 2
An arc AB of a circle, centre 0, of radius 10cm is 8 cm in length. Calculate (a) L AOB in
degrees and minutes (b) the area of the sector AOB.
I = r()
:. 8 = 10()
() = 08 c

(a)

x 180

= 0.8

7r

(b)

= ~r2()
= ~ x
= 40

100 x 08

Fig. 17-4

Area of sector is 40 cm2

EXAMPLE 3
An arc AB subtends an angle of 12652' at the centre of a circle, centre 0, of radius length
6 cm. Calculate
(a) the length of arc AB,
(b) length of chord AB,
(c) the area of the sector AOB,
(d) the area of the minor segment cut off by the chord AB.

(a)

12652' = 126'866 ... x 1;0

= 2'214 c
I = r()

= 6 x 2214

= 13'284

Hence the length of the arc is 1328 cm.

(b) AB = 2AX = 2 x 6 sin 6326'


= 12 x 08944
= 10733
Hence the length of the chord is 1073 cm.

Fig. 17-5

17/TRIGONOMETRIC (CIRCULAR) FUNCTIONS 383

(c)

Area of sector AOB = 2,r2 8

= 2,1 x

36 x 2214

39'85

Hence the area of the sector is 3985 cm2

(d) Area of minor segment = area of sector OAB - area of 60AB


= ~r2(8 - sin 8)

!2

x 36(2'214 - sin 2'214 c)

18(2,214 - 0'8)

= 18 x 1414

2545

Hence area is 25.45 cm2

EXERCISES 17(b)
1. Calculate, in degrees and minutes, the magnitude of the angle subtended at the centre
of a circle of radius length 8 cm by an arc whose length is 15 cm.

2. Find the length of an arc of a circle of radius 15 cm if the arc subtends an angle of 70
at the centre.
3. An arc of a circle subtends an angle of 100 at the centre. If the radius length of the
circle is 12 cm, calculate the arc length in terms of 'Ir.
4. The area of a sector OAB of a circle, centre 0, radius length 20cm is 240cm2 ,
Calculate
(a) the magnitude of the angle A OB,
(b) the length of arc AB,
(c) the length of the chord AB.
5. A chord PQ, 24 cm long, is 5 cm from the centre of the circle. Calculate the length of
the arc PQ.
6. A point Pis 8 cm distant from the centre of a circle of radius length 5 cm. Find the
length of the major arc between the points of contact of the tangents drawn from P to
the circle.
7. A chord AB of a circle with centre 0 has length 16cm. If the radius of the circle is
10 cm, calculate
(a) the magnitude of angle AOB,
(b) the length of the minor arc AB,
(c) the area of the minor segment formed by the chord AB.
8. The minute hand of a clock is 20 cm long. Calculate
(a) the arc length along which the tip of the hand travels in 16min,
(b) the shortest distance between the initial and final positions of the tip of the hand.
9. An arc AB subtends an angle of 60 at the centre of a circle of radius 15 cm. Calculate
the difference in the lengths of the chord AB and the minor arc AB in terms of 'Ir.
10. From a circular piece of metal 6 cm in diameter, a sector of angle 30 is removed.
Find the area remaining. Express your answer in terms of 'Ir.

384 NEW SENIOR MATHS: TWO UNIT COURSE

#.

11. A chord subtends an angle of 140 0 at the centre of a circle of radius 16cm. Find the
difference in length between the chord and the arc.
12. Calculate, in terms of 7r, (a) the area of
the shaded region between the two circular arcs, centre 0 and (b) the
perimeter of the shaded region.

,\0"

45

~~ ..

7SC;;;-:::;;:;

13. The minute hand of a clock is 3 cm in length. What area is swept out by the hand in an
interval of 40 min? Express your answer in terms of 7r.
14. A circular metal plate is cut into two segments along a chord equal in length to the
radius. What is the ratio of the areas of the two segments?
15. A sheep, grazing in a paddock, is tethered to a stake by a rope 20 m long. If the stake
is 10 m from a fence, find the area over which the sheep can graze.
16. A sector of angle 160 0 is cut out of a circular piece of thin cardboard of radius 10 cm.
The cut edges of the sector are brought together to form a cone. Find the
circumference of the circular base of the cone.
17. Find (a) the perimeter (b) the area of a piece of land in the shape of a circular sector of
radius 25 metres and angle 30 0

18. A pendulum 40 cm long swings through an angle of 25 0 Find (a) the length of arc
(b) the shortest distance between the extreme positions of the bob.

-i
t

19. Two circles, each of radius length 10 cm, have theircentres 16 cm apart. Calculate the
area common to each circle.
20. Three circles are drawn with radius length 5 cm. Their centres are at the vertices of an
equilateral triangle whose sides are of length 10 cm. Calculate the area between the
circles.

17.4 Definitions _ _ _ _ _ _ _ _ _ _ _ _ __
We can define circular or trigonometric functions as functions of real numbers and, for
this purpose, we consider a circle of unit radius defined by the equation X2 + y2 = 1.
Starting from the point A, we can mark
off an arc of length measure () for each real
number (). The arc AP subtends an angle of
()c at the centre of the circle. The same
number () measures the length of the arc and
the number of radians in angle AOP. If
() > 0, we mark off the arc in an anti
c
clockwise direction; if () < 0, in a clockwise
direction. If P() is the end-point of this arc,
and if P is the point whose coordinates are
(x, y), we define the functions cosine and
sine as

Fig. 17-6

17/TRIGONOMETRIC (CIRCULAR) FUNCTIONS 385

-1

These two functions have for domain the set of real numbers and,since -1 ~ x ~ 1 and
~ Y ~ 1, their range is [-1, 1].
The circumference of our unit circle is 21r units.
At A, 0 = 0; the coordinates of A are (1, 0), and so cos 0 = 1 and sin 0 = O.
At B, 0
At C, 0

~; the coordinates of Bare (0, 1), and so cos ~

0 and sin ~

1.

1r; the coordinates of Care (-1, 0), and so cos 1r = -1 and sin 1r = O.

What are the values of cos 31r sin 31r, cos 21r, sin 21r?
2

The points P(O), P(21r + 0), P(41r + 0), ... all coincide. In general, P(O) and
P(2k1r + 0), k E J, coincide, and so cos (2k1r + 0) = cos 0 and sin (2k1r + 0) = sin O.
Such functions, whose values recur at regular periods, in this case 21r, are called
periodic junctions.
There are four other circular or trigonometric functions-tangent, cotangent, secant
and cosecant (their abbreviations are tan, cot, sec and cosec respectively) that can be
defined in terms of cos and sin.

sin 0
--0' cos 0 =# O.
x = cos
cos 0 .
cot 0 = x = -'-0'
Y
sm sm 0 =# O.
1
1
sec 0
--0' cos 0 =# O.
x
cos

tan 0

y
-

cosec 0

1
y

: I

-J
sm O' sin 0 =# O.

Thus there is a restriction on the domain of each of these functions. Tan and sec
.
1r 31r 51r
are not defined when cos 0 0, I.e., when 0 = 2' 2' 2' ... ; cot and cosec are not
defined when sin 0 = 0, i.e' 1 when 0
0, 1r, 21r, ....
Observe that cot, sec and cosec are reciprocal functions of tan, cos and sin respectively.

17.5 Symmetry properties of


trigonometric functions - - - - - - - - -
The results established in chapter 7 for 90 - 0, 180 0 0, 360 0 and -0 are true when
we replace the degree measure by radian measure.

Degrees
sin (90 0

0)

Radians
cos 0

cos (90 - 0) = sin 0


tan (90

0
-

0)

cos (180 - 0)
0)
sin (180
tan (180
0)

(i" - OJ = cos 0
cos (f
~ sin 0

sin

(~ - 0)

cot 0

tan

-cos 0
sin 0
- tan 0

cos (1r - 0)
-cos 0
sin (1r - 0) = sin 0
tan (1r - 0)
-tan 0

cot

386 NEW SENIOR MATHS: TWO UNIT COURSE

cos (180 0 + 0)
sin (180 + 0)
tan (180 0 + 0)

-cos 0
-sin 0
tan 0

cos (11" + 0) = -cos 0


sin (11" + 0) = -sin 0
tan (1t + 0) = tan 0

0) = cos 0
0) = - sin 0
0) = - tan 0

cos (211" - 0) = cos 0


sin (211" - 0)
-sin 0
tan (211" - 0) = -tan 0

=
=
=

cos (360 0
sin (360 0
tan (360

cos (211" + "0) = cos 0


sin (211" + 0) = sin 0
tan (211" + 0)
tan 0

cos (360 0 + 0) = cos 0


sin (360 0 + 0) = sin 0
tan (360 0 + 0) = tan 0

17.6 Some exact values _ _ _ _ _ _ _ _ _ __


1I"C

1I"C

6 ,60 = :3

Since 0 = OC,30

,90

1I"C

"2 ' the exact values established in chapter 7 can

be rewritten as:
0

sin

-11"

-J3

11"
4

2:
-J3

cos
tan

11"
6

11"

-J2
1
2

-J2

-J3

cosec

2
2

sec

-J2

2
1

-J3

cot

-J2

-J3

-J3

These exact values can be extended to certain angles in the other quadrants"

EXAMPLE 4
Without the aid of tables or a calculator find the values of:
(.")

. 6
511"

SIn

(iii)

i)
tan 411" = tan ( 11" + j)

(iv)

sin 311"

(ii)

cos 5 11"

(v) cos ( - 5

(vi)
(vii)

sin (11"

sec ( 11" -

cosec (211" -

cosec 1111"

-cos

11"

tan !!
3
. 11"

+ ~)

cos 511"

sec 2311"

2:

cos (11"

1r) =

. ( 11" - 6
11")

SIn

-SIn

cos

j)

11"

-1
1
2

3"

-sec

i)

2:

11"

3"

-2

11"
-cosec 6 = -2

17/TRIGONOMETRIC (CIRCULAR) FUNCTIONS 387

EXAMPLE 5
Find all values of () between 0 and 211" for which:
(i) cos ()

(iii) tan ()

(v) sin ()

(ii) sin ()

(iv) sec () :;:: -

.J3

-1

(i) Since cos () is positive, () lies in the 1st and 4th quadrants

cos ()

:2

!! 211" - !!
3'
3
11" 511"

()

3'3
(ii) Since sin () is negative, () lies in the 3rd and 4th quadrants

. ()

sm

()

-~

11"

11"

+ 4' 211"

11"
4

511" 711"

4'4
(iii) Since tan () is positive, () lies in the 1st and 3rd quadrants

tan ()

1
11"

11"

() :;:: 4' 11" + 4

(iv) Since sec () is negative, ().1ies in the 2nd and 3rd quadrants

sec () =
. cos():;::

.J3
-T

() = 11" -

11"

11"

6' 11" + 6

511" 711"

6'6
(v) Since sin () is negative, () lies in the 3rd and 4th quadrants

sin () = -1

11"
2

EXERCISES 17(c)
1. Express each of the following as a trigonometric function of x:
(i) sin (11" -

x)

(ii) cos

(f - x)

(iii) tan (20-

x)

388 NEW SENIOR MATHS: TWO UNIT COURSE

(iv) cos (7r + x)

(v) sin (27r

x)

(vi) cot

(~ -

x)

2. If 0 is an angle in the 2nd quadrant, state whether the following are positive or
negative:
(a) cos (7r - 0)

(b) tan (7r

(d) sin (27r - 0)

(e)

+ 0)

(c) sin

(f)

cos (7r

0)

3. If sin x = O 2, write down the value of:


(i) sin (7r
x)
(ii) sin (27r -'x)

(~ -

(iv) cos

(vi) cosec

t, express in terms of t,

4. If tan 0

(a) cot

tan(~ -

0)
0)

(iii) sin (-x)

( v) sin (7r + x)

x)

(~

(b) cot

(d) tan (27r - 0)

(e)

(~

0)

cot (7r - 0)

(c)

tan (7r - 0)

(f)

tan (7r

0)

5. If cos x = c, express in terms of c:

(i) sec x
(iv) cos (27r - x)

.(ii) cos (-x)


(v) sec (-x)

(iii) cos (7r - x)

(vi) cos (7r + x)

6. If ~ < x < 7r, show with the aid of a diagram of a unit circle that:
(i) cos (7r

+ x)

-cos x

(ii) sin (27r - x) = -sin x.

Without the aid of tables or a calculator write down the exact values of: (7. to 10.)
7. (a) sin ~

27r

(b) cos

(f) cot

8. (a) sin 7r

(b) cos

(e) sec

37r

47r

(c) tan

57r

57r
T

(g) cosec

77r

(d) cot 47r

. 77r

( h) smT

457r

(g) tan 7r

9. (a) sin 3 7r
2

(b) tan

57r
T

(c) cosec

(e) cot 77r

(f) cos

10. (a) sin 27r


(e) sec

6137r

117r

117r

(g) sin 57r

. 13-7r
(b) sm6

57r

(f) cos T

(h) sin 27r

(c) tan 57r

(f) sec

(e) cos

2:7r

(d) cos 7r

(c) tan

97r

(g) sin 77r

(d) tan 77r


(h) cosec

(d) cot 77r


(h) tan 117r

Find all the values of x between 0 and 27r for which: (11. to 22.)

' x
11 . sm

.J3
= -T

14. cos x = -1

12 . tan x

= -1

15. cot x

.J3

13 . cosec x = 2
16. sec x

57r

-,.j2

17/TRIGONOMETRIC (CIRCULAR) FUNCTIONS 389

17. sin x = cos x


20. 2 sin x

18. sin x = 0

.J3

21. sin x

0
+
0 22. cosec x = sec x
19. 2 cos x

+ .J3 cos x

23. With the aid of a diagram, find equivalent expressions for


(a) cos (3 7r

X)

(b) tan (3 7r - x)
2

.e" )

(c) sm

(d) cot (3 7r
2

T-x

X)

17.7 Graphs of the trigonometric functions _ __


y

Fig. 17-7
In fig. 17-7, the graph of y = sin x is represented by the continuous curve and the graph
of y = cos x by the broken curve and the graphs are drawn in the domain -27r to 2 7r. The
graphs, however, for domain R are unlimited curves and their form from -27r to 0 is the
same as from 0 to 27r and from 27r to 47r and so on. In other words, as x increases, the
values of sin x and cos x repeat themselves after an interval or period of 27r; sin and cos
are therefore called periodic junctions, the period being 27r. We saw earlier in this chapter
that the points P(O), P(27r + 0), P(47r + 0) on our unit circle all coincide and hence
cos O.
sin (27r + 0) = sin 0 and cos (27r + 0)
The maximum and minimum values of sin x and cos x are 1 and -1 respectively; in other
words their amplitude is 1. If the cos curve is translated ~ units to the right, parallel
with the X-axis, it coincides with the sin curve. This follows from the fact that

cos

sin(~ +

x).
y

21r X

Fig. 17-8

Fig. 17-9

390 NEW SENIOR MATHS: TWO UNIT COURSE

Fig. 17-8 shows the graph of y = tan x for 0 ~ x ~ 211". As x increases, the values of
tan x repeat themselves after an interval or period oj 11". The tan function is undefined at
x

, 311",
... Fig. 17-8 shows the graph of y
2

= cot x in the interval 0 to 211". The values

of cot x also repeat themselves after a period of 11". The function is undefined at x = 0, 11",
211", ...
y

cosec x
:

Fig. 17-10

Fig. 17-11

Figs 17-10 and 17.:.11 show the graphs of y


cosec x and y
sec x respectively. They also
are periodic, with period 211". What restrictions are placed on their domain? What is the
range in each case?
The graphs of the six circular functions have been drawn in pairs of complementary
functions-sine and cosine, tangent and cotangent, secant and cosecant.
As an exercise, the student should draw the graphs in pairs of reciprocal functions-sin
and cosec, tan and cot, cos and sec.
Remember that! y

00

as y -

0 so that if y is increasing on the interval (0, 1], then

! is decreasing on the interval [1, 0). Thus, for example, since sin x increases from zero to

1 as x increases from 0 to

, it follows that

1 x decreases from a very large value to 1 in

the same interval. These facts enable us to deduce the graphs of cosec, cot and sec from
sin, tan and cos respectively.

17.8 Graphs of y

a sin bx and y

a cos bx __

We consider now the transformations on the sin and cos curves caused by the constants a
and b. Since the maximum and minimum values of sin x and cos x are 1 and -1
respectively, it follows that the maximum and minimum values of a sin x and a cos x are a
and -a respectively; in other words, their amplitude is lal. The effect of the transformation
caused by a is to dilate (i.e. either stretch or compress) the graph parallel to the Y-axis.
We have also seen that as x increases, the values of sin x and cos x repeat themselves
after a period of 211". In place of x we have bx and thus putting bx = 211" i.e. x = 211" the
values of sin bx and cos bx repeat themselves after a period of 2 1l". The effect of the
b

transformation caused by b is to dilate the graph parallel to the X-axis.

17/TRIGONOMETRIC (CIRCULAR) FUNCTIONS 391

EXAMPLE 6
Sketch the graphs of (i) y = 3 sin 2x
(i) Amplitude

Period

3
2'1r

(ii) y = 2 cos

3x, showing one complete cycle.

(ii) Amplitude = 2
=

2'1r

Period

'1r

3
2

o
-1

y = 3 sin 2x

...

2... X

"4

-1

-2

-2

-3

Fig. 17-13

Fig. 17-12

17.9 Graphs of y = a sin b (x + c) and


y = a cos b(x + c) _ _ _ _ _ _ _ _ _ __
We consider now the transformations caused by the constant c.
You will observe that in place of x we now have (x + c). The effect of the transfor
mation caused by c is to translate the graph c units along the X-axis. The number c is
called the phase.

EXAMPLE 7
Sketch the graphs of (i) y = 3 sin 2

(x - i )

(ii) Y

2 cos 3

(x + j) showing one

complete cycle.

i has not altered the amplitude or period. It merely ha~ the effect of
moving the graph of y = 3 sin 2x, i units parallel to the X-axis in a positive direction

(i) The constant

(fig. 17-14).

3
2

3 sin 2

(x . f)

y = 2 cos 3

(x -

~)

Fig. 17-14

Fig. 17-15

392 NEW SENIOR MATHS:TWO UNIT COURSE

(ii) The constant

f has moved the graph of Y = 2 cos 3x, f units parallel to the X-axis in

a negative direction (fig. 17-15).

EXAMPLE 8
Using the same scale and axes sketch the graphs of Y = 3 sin x and Y
sketch the graph of Y
3 sin x + 2 cos 2x for 0 ~ x ~ 27f.

= 2 cos 2x.

Hence

The graphs of YI
3 sinx andY2 = 2 cos 2x are drawn in fig. 17-16 by broken lines. To
obtain points on the graph of Y = 3 sin x + 2 cos 2x, we use the process of addition of
ordinates of the component curves, taking sign into account. Let Y
YI + Y2 where
YI = 3sinxandY2
2 cos 2x
7f
e.g. at x = 0, Y = 0 + 2 = 2
atx = 2'Y = 3
2 = 1

atx

27f, Y

at x

0+2 =2

7f,Y
=

+2

atx
2

-3

OA, AD = AB

atx

+ AC and so on.

3 sinx

Y1

37f

3 sin x + 2 cos 2x

-5
Fig. 17-16

EXERCISES 17(d)
1. Sketch the graph of each of the following, stating period and amplitude:
(i) Y = 4 sin x, 0 ~ x ~ 27f.
(ii) Y = 3 cos 2x, 0 ~ x ~ 27f.
(iii) Y
(v)

sin

~, 0 ~ x ~ 27f.

(iv) Y

7f
7f
3 tan 2x, - 2 ~ x ~ "2'

cos x, 0

2. Sketch the graph of Y


(i) Y
-cos x

= -5

cos 2x, -7f

(vi) Y = sin 7fX, -2 ~


~

~ X ~

7f.

2.

7f and from it sketch the graph of:


(ii) Y = 1 - cos x

3. Sketch the graph of Y = sin x, -7f ~ X ~ 7f and from it sketch the graph of:
(i) Y
2 sinx
(ii) Y = -2 sinx
(iii) Y = 3 - 2 sinx

4. Sketch the graph of each of the following, showing one complete cycle
(i) y

(iii) Y
(v) Y

2 sin

(0 -

(ii)

sin 2(0 - 7r)

(iv) y

(vi) Y

tan 2(&

7f)

3 cos

5 cos 3 (0

i)

"';2 sin (2&

j)
~).

17/TRIGONOMETRIC (CIRCULAR) FUNCTIONS 393

5. Sketch the graphs of


(i) y = sin 20 + 1
(iii) y =
(v) y

(ii) y = 3 cos 0 - 2

4sin (0 - i) + 3
4 sin 3

(0 -

*)-

(jv) y

= 2 cos 2 (0

(vi) y

=3

- i) +

sin(o - ~).

6. Using the process of addition of ordinates, sketch the graphs of


(i) y
sin 0 + cos 0
(ij) y = 3 sin 20 + 4 sin 0
(iii) y = 2 cos 30 + 3 sin 20
(iv) y = sin 20 - cos 0
(v) y

4cos 20 -

sin 0

(vi) y

sin 0

sin 20.

17.10 Graphical solution of equations _ _ _ __


There are usually standard techniques for solving equations associated with various
functions e.g. linear equations, quadratic equations, trigonometric equations and so on.
However, there is usually no standard technique for solving equations which combine two
or more different functions e.g.
sin x = 1 - 2x
cos x
log x
They transcend (go beyond) known equation-solving techniques and hence are called
transcendental equations. Approximate solutions can be found in various ways. One such
method is to sketch the graphs of the two functions and find roughly the x-values of their
point or points of intersection. Then, with the aid of a calculator or tables, we can refine
this solution.

EXAMPLE 9
Solve the equation sin x

1 - 2x.

g(x)

Fig. 17-17

1 -2x

The sketch shows that the graph of f(x) = sin x and g(x) = 1
2x intersect at
approximately x = 03 and hence this provides an approximate solution of the equation
sin x = 1 - 2x. Furthermore, it has only one solution.
x

03

04

0335

sin x
1 - 2x

02955
0-4

03894
0-2

0-3288
033

Using a calculator to find the sin values, the table above shows that the solution is between
x = 03 and x = 04. A better solution is approximately x = 0335 after a few trials.

, 394 NEW SENIOR MATHS: TWO UNIT COURSE

EXERCISES 17(e)
Use a graphical method to solve the following equations 1. to 4. to two decimal places:
. 2
1. SIn x

1
= 2"x

2. cos x
4. tan x

1 - x

5. A rectangular strip of metal 10 Gm wide is bent to form a water channel, the section
perpendicular to the length being a circular arc whose chord is 6 cm long. Show that,
if the circular arc subtends an angle of 2x radians at the centre of the circle,
5 sin x = 3x
Solve this equation graphically to three decimal places and find the area of the cross
section of the channel in cm2
6. A semicircle of radius r is divided into two parts of equal area by a chord parallel to
the base. If the chord subtends an angle of x radians at the centre, prove that
7r

x - 2" = smx
Solve this equation graphically, finding x correct to two decimal places.
7. The chord of a segment of a circle subtends an angle of ~

+x

radians at the centre,

and the area of the segment is one-quarter that of the circle. Prove that x
solve this equation graphically correct to three decimal places.
8. Show from a rough sketch that the equation 2 -

~x

= cos x and

tan x has a solution which is

approximately 1. With the aid of tables or a calculator, find the solution to 2 decimal
places.
9. Show graphically that the equation
x
810g lo (O'Ix + 0'5) = 2
has a solution between x = 2 and x = 4. Find the solution correct to 2 decimal places.
10. An arc AB of a circle, centre C, is 20 cm long. If the chord AB is 15 cm long and
subtends an angle of x radians at C, show that 8 sin

~x

3x. Find, graphically, the

solution to this equation.

17.11 Derivative of sin x and cos x _ _ _ _ _ __


Consider the graph of sin x from x

0 to

7r.

Its gradient at the origin looks to be about 1,

corresponding to the value of cos x being 1 when x

= O. As x

increases from 0 to~, the

gradient of the sine curve is positive, and decreases from 1 to 0, corresponding to the fact
that cos x is positive, and decreases from 1 to 0 in the same interval. As x increases from ~
to 7r, the gradient of sin x is negative (the curve slopes downwards to the right), and
decreases from 0 to -1, corresponding to the fact that cos x is negative, and decreases from
o to -1 in the same interval. You should now compare the gradient of the sine curve and
the values of cos x from 7r to 27r. We conclude from this that the derivative of sin x is
probably cos x. This is certainly not a mathematical proof.

17/TRIGONOMETRIC (CIRCULAR) FUNCTIONS 395

y
cos x,'"
/
I

I
2T X

Fig. 17-18
Likewise, consider the graph of cos x from x
corresponding to the value of sin x when x

0 to 7r. Its gradient at the origin is 0,

O. As x increases from 0 to

i, the gradient of

cos x is negative, and decreases from 0 to -1, in comparison with sin x, which is positive
. and increases from 0 to 1 in the same interval. As x increases from

i t07r, the gradient of

cos x is negative, but increases from -1 to 0, in contrast with sin x, which is positive, but
decreases from 1 to 0 in the same interval. It seems feasible to conclude that the derivative
of cos x is probably given by -sin x,
Consider the unit circle, fig, 17-19,
Starting from the point A, mark off
arcs AP and A Q of length measure x
and x + h respectively. Hence the
length measure of arc PQ is h,
By definition,
A
sin x
MP
sin (x + h) = NQ
Thus sin (x + h) - sinx = RQ
sin (x + h) - sinx
RQ
and

RQ PQ

PQ'h

cos LPQR, ~Q

Fig, 17-19

As h -+ 0, the point Q approaches P, the measure of the angle PQR approaches the
measure of angle POM, i.e. x, and the length measure of the chord PQ approaches h, the
length measure of the arc PQ, In fact, both -0, but in such a way that
PQ -+ 1.
h
, sin
- sin x
Thus I1m --'--------:.--- = cos X
j'(x), wherej(x) = sin x
11-0

l1x

(sin

xl =

cosx

396 NEW SENIOR MATHS: TWO UNIT COURSE

Figure 17-19 shows the situation for x between 0 and

i.

The result is valid for all values

of x and you are recommended to consider the situation with P in other quadrants,
making use of the symmetry properties of the sine function.
Consider again the unit circle (fig. 17-19).
OM
By definition
cos x
cos (x + h) = ON
-NM = -RP
cos (x + h) - cos x = ON - OM
Thus
cos (x + h) - cos x
RP
and so
h
RP PQ

'Il
-sin LPQR. PhQ
As before, when h Thus

/~

0, PhQ -

cos (x

1.

h) - cos x

-sin x

f'(x)

i.e.

-sin x wheref(x)

= cos x.

We can use the derivative of sin x and the chain rule to deduce the derivative of cos x.
Let
y = cos x
sin

(i - x)

sin u where u
dy _ dy du
dx - du .
cos u. (-1)
-cos

(i - x)

-sin x

Note: x represents a real number or an angle of x radians but not an angle of xO.

d (.
0)
dx sm x

d fc. 'il'"X )
dx ,sm 180
'il'"

'il'"X.

h'

= 180 cos 180 usmg t e c am ru e


_

'il'"

- 180 cos x

17/TRIGONOMETRIC (CIRCULAR) FUNCTIONS 397

17.12 Derivative of tan x _ _ _ _ _ _ _ _ _ __


Let

= tan x

sin x
cos x

where u

sin x and v

cos x.

du
dv
vlT-ucr
x
x (quotient rule)

Ix

+ sin2 x

cos 2 x

x
=

sec2 x.
J

17.13 Derivative of sin (ax + b)


and cos (ax + b) _ _ _ _ _ _ _ _ _ __
Let y = sin (ax + b)
sin u where u
dy
dx

Let y
ax

dy du
du dx
= cos
= a

b
dy
dx

cos (ax + b)
cos u where u

u. a

cos (ax

-sin u. a

= - a sin (ax +

b)

Ix

sin (ax

b) = a cos (ax

d
dx cos (ax

b)

-a sin (ax

d
dx tan (ax

b)

a sec 2 (ax

EXAMPLE 10
Differentiate: (i) sin 3x
(i) Let

(ii) Let

y = sin 3x
dy
dy du
dx = du . dx
y

cos2 x

dy = dy . du
dx
du dx
(iii) Let

ax

dy du
du dx

y = sin (X2)
dy
dy du
du . dx
dx

(iii) sin (X2)

(ii) cos2 X

sin u where u

cos u . 3 = 3 cos 3x

u 2 where u

3x.

cos x.

2u. _ sin x = - 2 cos x sin x

sin u where u

X2

cos u .2x = 2x cos (X2)

b)

b)

b)

b)

'

398 NEW SENIOR MATHS: TWO UNIT COURSE

EXAMPLE 11

Differentiate: (i) X2 sin x


(i) Let

(ii) sin x tan x

(iii) x(2 cos x

+ 3 sin x).

y = X2 sin x

= uv where u
X2 and v = sin x.

dy
du
dv

dx = v. dx + u. dx (product rule)

sin x . 2x + X2 cos X

= 2x sin x + X2 cos X

(ii) Let

sin x tan x

= uv where u
sin x and v = tan x.

dy
du
dv

dx = v. dx + u. dx (product rule)

tan x cos x + sin x sec 2 x

= sin x + sin x sec2 x

= sin x (1 + sec2 x)

(iii)

y = x(2 cos x + 3 sin x)


= uv where u = x and v
dy _
du +
dv
dx-v'dx
u.

= 2 cos x + 3 sin x.

= (2 cos x + 3 sin x) . 1 +
(2

3x) cos x

+ (3 -

x( - 2 sin x
2x) sin x

+ 3 cos x)

EXERCISES 17(f)
1. From the derivative of sin x and cos x deduce the derivatives of:
(a) cot x
(b) sec x
(c) cosec x

Differentiate with respect to x: (2. to 46.)


2. sin 3x
5. 2 cos x
8. sin x cos x
11. sin (x +

3. 3 sin x
6. sin x + ~ cos x
9. sin 2x - cos 2x

i)

12. 2x tan x

14. X2 cos' x

15.

4. cos 2x
7. tan 2x
10. x sin x

. 1
1

13. sm 2x + 2cOS x

16.

sin x

17. x 3 sin 2x

sin x
18. sin (X3)

20. cos (X2 + 1)

21. sin (2X

~)

22. X2

23. (X2 - 1) cos 3x

24. cos (2X

f)

25. cos (3x - 2)

26. cos2 2x

27. sin2 3x

28. cos3 x

29. cos (X3)

30. sin 2x cos 2x

31. 3x

32. 2 cos2 x

33.

35. tan (X2 - 1)

36. x cos 2x

37. sin x (1

39 . ..Jsin 2x

40. (sin x - cos X)2

38. 2"1 cos 2x

x sm x

19. cos 3 x

sin x

+ tan 2x

+ 2 cos x

. 1x
34. 3 sm
2

- sin

3
4 cos 2x

+ cos x)

~x

x
3

17/TRIGONONIETRIC (CIRCULAR) FUNCTIONS 399

42. sin2 x - cos2 x


45. tan (7r - x)

41. sin2 x + cos2 X


44. cos (sin x)

43. sin (co~ x)


46. tan XO

17.14 Functions defined by integrals


.(indefinite integrals) _ _ _ _ _ _ _ _ __
F(x)

= !axf(t) dt

defines a function.

Provided the integral exists (e.g. if f is


continuous) then F(x) defines the area
enclosed between the curve y = f( t), the
t-axis, the fixed ordinate t = a and a
variable ordinate t = x.
For fixed a, to each value of x there
corresponds just one area, and hence just
one value of F(x) as is required for the
correspondence to define a function.

((t)

Fig .. 17-20
e.g.

F(x) J:3t dt
2

In either case

~~ =

= [

tl]: x' - a ' define typical area functions.


=

3x 2, which is the integrand evaluated at t

x.

This property has lead to a traditional and widely used notation for an antiderivative. It
is readily observed that in each of the above cases the form of the integral value is x 3 + c
where c = -1 and -a 3 respectively, i.e. it can be regarded as specifying an antiderivative of
the integrand.
The integral is said to be 'indefinite' in that the upper terminal is not a pr~scribed
number but a variable.
In this case it has become customary to abbreviate the notation by omitting the
terminals, and writing the result in the form
f3X2dX = x 3

+ c where c may be any real constant.

i
!

This symbolism which depends directly upon the link between antidifferentiation and
integration afforded by the Fundamental Theorem is frequently divorced from the latter
and used purely in the context of antidifferention.
Thus
just as

!
!

f(x)dx is used as an instruction to 'Find an anti derivative (primitive) of f(x),


(f(x is an instruction to find the derivative of f(x).

Examples showing the use of the 'indefinite integral' notation for antiderivatives are

+ c for n *- n + 1
JXndX = ~
f cos x dx = sin x

+c

By contrast !abf(X)dX is called a 'Definite Integral' or 'Integral'.


)

--- -- - -----

400 NEW SENIOR MATHS: TWO UNIT COURSE

17.15 Primitives of trigonometric functions _ __


Since !(Sin x)

= cos x,

Since !

+ b)

sin (ax

= sin x +

then fcos x dx

c.

+ b), thenfcos (ax + b)dx

a cos (ax

+ b) + c etc.

sin (ax

The following table gives a summary of the derivatives and primitives of the
trigonometric functions.

f(x)

f'(x)

sin x

cos x

sin (ax

b)

a cos (ax

-cos x + c
1
cos (ax + b)
a
sin x + c

b)

-sin x

cos x
cos (ax

Jf(x)dx

b)

-a sin (ax

1 sin (ax
a

b)

sec 2 x
a sec 2 (ax + b)
-

tan x
tan (ax + b)
sec 2 x

b)

tan x + c

EXAMPLE 12

f
f
(..)f .
f
(i)

(sin x

+ sin 3x)dx =

(U)

(cos 2x

III

1 dx
sm 2"x

(iv)

-cos x

2 cos x)dx

= -

(2 sin x

2 cos 2"x
1

3 cos x) dx

~ sin 2x
+

2 sin

x + c.

- } cos 3x

+ c.

c.

= -

2 cos x

6 sin x

c.

EXAMPLE 13
7r12

Evaluate (i)

J
7r/3

cosxdx

(ii)

(cos 2x

3 sinx)dx

1.

sin 2xdx

(iii)

7r

(7r/3

(iv)

,_
~,

0)

7r/6

7r12

~3

cos x dx

[J
sin x

7r12
~3

7r

SIn - -

7r

SIn -

f.

7r/4

sec2 x dx

17/TRIGONOMETRIC (CIRCULAR) FUNCTIONS 401

f
1T

(ii)

sin 2x dx =

[-~ cos 2XJ:


. 1

- 2 cos 21r + 2 cos 0 =

f: sec x
14

(iii)

(iv)

dx

1T/3

= [ tan x] :/4 = tan

(cos 2x - 3 sin x)dx =

[1

-2 sin 2x

+2

r- tan

=1

- 0

+ 3 cos X

1T/6

1T/3
1T/6

. 21r + 3 cos 3"1r) (1.


21 smT
2 sm 3"1r +
= -J3 + ~ _ -J3 _ 3-J3

= (

424

1r)
3 cos 6

3 - 3-J3
2

f:!2cos (2X

(v)

- ~)dX = [~Sin (2X - ~)r


1 . 2
1r

= 2 SIn

- 21.sm

(- 2
1r)

=-+-=1
11

EXAMPLE 14
Calculate the area bounded by the curve y
1r
1r
X
3" and x = 2'
Area

f.1T12
1T/3

sin x, the X-axis and the ordinates

sin x dx
1T/2

-cos

1T/3

(-cos~) - ( -cos~)
=0+

11"

11"

3 2:

Fig, 17-21

EXERCISES 17(g)
Write down the primitive (indefinite integral) of (1. to 18.):
1. sin 2x
4. sin x

7. cos 3x

2. cos
cos x

2x

3. sec2 x

(x + r)

S. 2 sin x - 3 cos x

6. sin

8. 2 sin 2x

9. 3 cos

3"x

11"

402 NEW SENIOR MATHS: TWO UNIT COURSE

. 1
1
10. sm 4x + cos 4x

II

13. cos

(2X - I)

1
"
- -1sm
2x
16. cos -x

(2X +~)

11. sin x - cos 2x

12. sin

14. sec 2 3x

15.

17. X2 + sin 2x

18. Xl/2 -

sin 2x

cos x

21 cos x

Evaluate: (19. to 32.)

19.

.f

21.

7rsin x dx
71'

22.

71'/3

+ cos x)dx

24.

71'/2

25.

f
f
J
J

(cos x

sin 2x)dx

26.

71'/6

(2 sin x

71'/3 (

3 cos 3x -

7r12

31.

-71'12

_:sin xdx

71'/4

(sin 2x

7r

28. fo (sin

+ sin 2x)dx

-71'/4

29.

cos x dx

cos 2x)dx

71'/8

71'/4

27.

sec2 x dx

71'12

cos -2xdx

23. fo7r(Sin x

71'/3

20.

1)
2 sin 2x dx

30.

4 cos 2x) dx

32.

fJi
f

~x + cos ~X)dX

sin x - cos

X)dX

71'13

(3 sin 2x

cos (x +

~)dx

71'/4

33. The mean value of f(x) between the values x


b

a and x = b is defined as

~ a f.bf(X) dx

Find the mean value of f(x) = 2 cos x

+ 3 sin x between x =

-2~

2~"

and x

34. Find, by integration, the area bounded by the curve y =. cos x and the X-axis between

x =

and x =

2"

35. The region under the curve y = sec x, 0 ~ x ~

i, is rotated about the X-axis. What

is the volume of the solid of revolution?


36. The region under the curve y

= tan x between x =

i and x -f is rotated about

the X-axis. What is the volume of the solid of revolution. Use the fact that
tan 2 x = sec 2 x - 1.
37. Sketch the graph of f(x)

=1-

cos x, 0

.
(7r12
x ~ ~. Evaluate 0 f(x)dx and indicate

the area represented by this integral on the sketch.


38. Sketch the graph of f(x) =

.../1 - cos2 x, 0

Illustrate your result on the sketch.

~ x ~ 2~ and evaluate

37r/2

71'

f(x)dx.

17/TRIGONOMETRIC (CIRCULAR) FUNCTIONS 403

39. Evaluate

511"/3(1
11"/3

4 cos 2x + cos x) dx.

= ,""sin x between x = 0, x = 1r and the X-axis is


rotated about the X-axis. Find the volume of the solid of revolution.

40. The area bounded by the curve y

41. Find the volume of the solid generated by rotating the curve y

,""cos x between

x = 0, x = ~ and the X-axis about the X-axis.


42. Calculate the area of the region enclosed by the graph of y
the ordinates x

and x

= cos 2x, the X-axis and

i.

43. Calculate the measure of area bounded by the given curves, the X-axis and the given
ordinates:
(a) y
1 + cos x; x = 0, x = 1r
o,x
(b) y = 1 +~ sin 2x; x
2
1
1
1r
1r
1rX
b
(c) y = 2: + 2: cos x; x = -4:' x = 4:
(d) y = a sin 7J;x = O,x = 2:

= sin 2() and y = cos 2() for values of ()


and calculate the measure of the area between the two curves in

44. Find the points of intersection of the curves y

in the interval [0,


this interval.

1r)

45. The mean value of a functionf(x) in the interval [a, b) is defined as


1
b - a

Find the mean value of f(x)

= 2

cos 2x

fbf(x)dx
.,0

i, f].
cos () for

+ sin 2x in the interval [

46. Find the points of intersection of the curves y = sin () and y =


and calculate the area between the two curves.

E;; () E;; 21r

Approximate integration.
EXAMPLE 15
Evaluate

1l"/2

sin x dx:

(a) by direct integration


(b) using the trapezoidal rule with two strips
(c) using Simpson's rule with three function values.

(a)
(b)

11"/2

sin x dx

[J
-cos x

11"/2

x
sin x

Fig. 17-22

404 NEW SENIOR MATHS: TWO UNIT COURSE

The width of each strip is

i units

:. J:12 sin xdx ~ i [Sin 0 + 2 sin *+ sin ~ ]

Error is 1

0948

0948

0052 :::: 5070 (Fig. 17-19)

(c) Taking a
Jo'l2 sin x dx

b - a

~(a) + 4f(a

+ f( b)]

; b)

~
(sin 0 + 4 sin !! + sin !!) =
12
4'
2

1002

The error is 1 - 1'002 = -0,002 :::: - 02070.


The parabola approximating the sin curve passes through the points (0, 0),

(i,

1 )

1) and is visually indistinguishable from the sine curve.

and (7r

EXERCISES 17(h)
1

1. Given that

0
/.

+ X2 dx ==

7r, find approximate values of 7r using:

(a) the trapezoidal rule with two strips,


(b) Simpson's rule with three function values.
2. Use (a) the trapezoidal rule with two strips and (b) Simpson's rule with three function
values to give an estimate of:
(i) Jo 7f sin x dx

(ii)

107f~sin x dx

Find the exact answer to (i) and hence find the percentage error involved by using
Simpson's rule.
3. Calculate the value of 2

1
7r 7r 37r
7r, and use Simpson's rule with
+ cosx for x = 0, -4' -2'

.
/.7f 2
f unctIOn
. va Iues to estImate
these f lve
o

dx

+ cos x

4. Use Simpson's rule with three function values to estimate the value of:
57r/6

(i)

7f/3

sin2 xdx

(ii)

7f/6

1-7f/3

cos 2 x dx

S. Use Simpson's rule with three function values to estimate the volume of the solid
generated when the region bounded by the curve y = ~sin x between x
0, x = 7r,
and the X-axis is rotated about the X-axis. Find the exact volume by integration and
hence calculate the percentage error involved in using Simpson's approximation.
6. Complete the table, of values for f(x) = x sin x.

x
f(x)

7r
2

17/TRIGONOMETRIC (CIRCULAR) FUNCTIONS 405

Use (a) Simpson's rule with these five values,


(b) the trapezoidal rule with these five values to find the approximate value of:

J.'lrx sin x dx
{'lr/4

7. Evaluate J 0

sec2 x dx by

(a) direct integration,


(b) the trapezoidal rule using two strips,
(c) Simpson's rule using three function values.
8. The length of one arch of the curve y = sin x is given by

10 -J 1 + cos
'lr

2X

dx.

Find the approximate iength using Simpson's rule with three function values.

CHAPTER 18

Exponential
and
Logarithmic
Functions
This chapter may be studied in conjunction with Chapter 13.

18.1 Derivative of exponential functions _ _ __


Fig. 18-1 shows the graph of I(x) = lOx.
To find the gradient at any point P, we proceed as in our treatment of other functions:
Let/(x)
lOx and so/(x + h) = 10x+h
By definition,

I' (x) = lim I(x

h-O

lOX + h

h) - I(x)
h
-

lOx

hm ----::,----

h-O

= lim 10x(10h -

1)

h-O

10h - 1
= 10x h-O
1I m - -

We now investigate the value of

10 h -

f(x)

= 10"

hash - 0

with the aid of a calculator.


h
0.1
005
0'01
0005
0'001
00001
I

10h - 1

h
25893
2'4404
2'3293
2'3159
2'3052
2-3029

h
-0,1
-0'05
-0,01
-0,005
-0,001
-0-0001

10h - 1

h
20567
21750
22763
22894
22999
2-3023

Fig. 18-1

18/EXPONENTIAL AND LOGARITHMIC FUNCTIONS 407

As h -

0 (through both positive and negative

values), the value of

10 h

is seen to approach a

limit. The value of this limit is 23Q26 (to four decimal


places) and is identified as the gradient at the point
where the graph of y = lOx crosses the Y-axis.
I' (x) = 23026 x lOx
Hence
If we investigate the derivatives of functions given by I(x)
2x and I(x) = 3x, and
2h - 1
3h - 1
with the aid of a calculator evaluate - h - and
as h - 0, we obtain the following
derivatives:

I(x)

I' (x)

Illustration

"2 x
3x
lOx

06931 x 2x
10986 x 3x
23026 x lOX

Fig. 18-2
Fig. 18-3
Fig. 18-4

y
f(x) =

Fig. 18-2

In general, !(a X )

Fig. 18-3

10x

Fig. 18-4

= ka x for some k a positive number.

Obviously, the most convenient function to work with would be the one for which the
multiplier, k, has the value 1. From the preceding table of derivatives, and with reference
to fig. 18-2 and fig. 18-3, we see that the base a will have a value between 2 and 3, but we
need not know this information in order to find it.
We shall denote this particular base bye, so that

408 NEW SENIOR MATHS: TWO UNIT COURSE

18.2 Evaluation of e
Put

lOb, so that eX

10 bx ,

(1)

d
d
-(ex) = _(lObx)

and so

(2)

dx

dx
y = 10 bx = lOu, where u
dy
dy du
dx = du . dx

Let

Hence

bx.

2'3026 x lOu. b
= 23026b X 10 bx

= 23026bex (by (1.

(lObx)

Substituting in (2), we have

eX = 2'3026be x

so

Hence

1
2'3026

(four decimal places)

04343

(using a calculator)

= 10'4343 = 2 7183

ex

g(x)

This particular number is known as the Euler


number (after Leonhard Euler) and is, after 71",
the most famous transcendental irrational
number.
Notice that the graph of ex crosses the Y-axis
with a gradient of 1 (fig. 18-5).

/ Y

"

)(

"

"" /

"

),<

/'

Fig. 18-5 shows the graphs of ex and loge x.

/
/
/

/
/

Fig. 18-5

18.3 Natural (Napierian) logarithms _ _ _ _ __


Logarithms to base e are called natural or Napierian logarithms after Napier, a British
calculating genius. Books of mathematical tables contain logarithms to base e as well as
the more familiar base 10. Your calculator will have an Inx key e.g. to find loge 2, loge 10,
proceed as follows:
Operation sequence

Display

2 llnxl

06931472

10 1/nxl

2'3025851

loge 2 = 06931 and loge 10 = 23026 (to 4 decimal places)


eO'6931
2
and e2-3026 = 10
Note: The notation for logex varies. In some books In x is used for logex, while in
others, the base is omitted and log x then indicates that natural logarithms are intended.
(In Inx, I is for logarithm and n for natural.)

IS/EXPONENTIAL AND LOGARITHMIC FUNCTIONS 409

18.4 Derivative of e kx, k a constant _ _ _ _ _ __


y = e kx = e U where u

Let

kx

dy = dy du = eU k
ke kx
dx
duo dx
.
is then a function whose derivative is proportional to the

The function f(x) = e kx ,


function value itself, i.e.
f' (x) = kf(x), or

k = 2 , dx
.!f (e 2x ) = 2e 2x

Put

k = - 3 , dx
.!f (e- 3x )
k =

3e-3x

!L(e 1l2X)

2' dx

It follows then that the indefinite integral (primitive) of e kx is iekX

+c

Le.
Put

~e2x15 + c

k=

+c

EXAMPLE 1
Differentiate (a) ex2 (b) e sin x
(a) Let

= ex2 = e where u =
U

dy _ dy du
dx - du' dx
= eU .2x
x2
= 2x e

X2

(b) Let

(chain rule)

(a) Let

(a) (2X2

e sinx
: . du

e where u
U

+ I )e 3x

(b) eX cos x

y = (2X2 + l)e 3x
= uv where u = 2X2 + I and v =
e 3x
dy
du
dv

dx = v. dx + u. dx (product rule)

= e 3X4x + (2X2 + 1)3e3x


= (6x 2 + 4x + 3)e 3x

(chain rule)

e cos x
cos x e sin x

EXAMPLE 2
Differentiate

y
dy
dx

sin x

410 NEW SENIOR MATHS: TWO UNIT COURSE

(b) Let

= ex cos x
= uv where u = eX and v = cos x

dy
du
dv

dx = v' dx + u' dx

= cos xeX - eX sin x

(cos x - sin x)ex

EXAMPLE 3
Evaluate

re

lx

(a)

(b)

1'5

[!e J:
lx

dx
=

1 4
-e
2

~(54-60

26-80

1
-eo
2
- 1) since

[+J

(ex - e-X)dx =

e-x

ex

-{)-5

eO

1-5
0'5

= (e l -5 + e- I ' 5) - (e O' 5 + e-0'5)

= (4-4817 + 0'2231) - (1-6487 + 0-6065)

= 2-4496

The values of ex and e- X e.g. e4, e l ' 5 , e- I ' 5 etc can be found with the aid of a calculator.
Some calculators have an ex key; if not, use 2-7183 as its approximation and proceed as
follows using the yx key.

Operation sequence
e
e

l '5 :

207183
2

e- I ' 5 : 2
eO-

e-{)'5 :

7183

[2]
[2]

or4~

4
1

4'48173

50

D 7183 0 1 =:J 5

D
5
7183 ~ D 5 Ed

207183 ~
20

Display
54-5996

EJ

5E2J

or

0223128

.0

or[}]

164873
0

0'606529

EXAMPLE 4
For the function f(t) = 2te-0' 5t , find the value of t for which f(t) has a maximum, and
hence calculate this maximum value. Sketch the graph of f(t)

-te-0' 5t + 2e-0' 5t
= (2
t) e-0' 5t = 0
e-0' 5t =1= 0 for any t
2 - t = 0, when t
2
f' (t)

and

When t < 22' ff:


>
When t >,
t <

(product rule)

OO} Hence the maximum value of f(t) occurs when t


f(2) ;;: 4e- 1

1472 (using a calculator)

18/EXPONENTIAL AND LOGARITHMIC FUNCTIONS 411

2
(2, '-47)

Fig. 18-6
The curve in fig. 18-6 crosses the I-axis where 2Ie-O-5t = 0 and since e-O- 5t > 0 for all I, it

crosses the I-axis where I = O.

For all I < 0, f(t) < O.

For all I > 0, f(t) > O.

The curve slopes upwards to the right for all I < 2, sincef'(t) > 0 for these values of I.
The curve slopes downwards to the right for all I > 2, since f' (t) < 0 for these values
of I; however, it does not cross the I-axis again. The I-axis is an asymptote.

EXAMPLE 5
Calculate:
(i) the area bounded by the curve y = e'-5x, the coordinate axes and the line x
_ (ii) the volume obtained by rotating this area about the X-axis.

(i) Area

Jo e'-5Xdx
2

= 2

= [~el.'xJ:
2

= 3(e3 - eO)
:::::: 1272
x

Fig. 18-7
y
7f" J02y2dX

(ii) Volume

7f"

where y

e'-5x

fo2e3xdX since y2 = (e' -5XF = e 3x

7f"
6
= -(e
3

eO)

7f"

:::::: -(4034 - 1)
3
= 4214

Fig. 18-8

412 NEW SENIOR MATHS: TWO UNIT COURSE

EXERCISES 18(a)
1. Write down the derivative of:
(a) e4x
(d) e 4x - e 3x

2e-x

(g) 3ex

(c) 4e 3x - e-2x

(f) e 3' 2x e 1' 6x

(b) 2ex12
(e) 2e 3x + e- X

+ 'ie-2x

(h) 4e 2x

(i) e 2x (e x - e-x)

2. Differentiate:
(b)(2x + l)e- x
(e) e- xx 3

(c) (X2 + X
(f) sin xe x

(g) e 2xcos':!

(h) sin 3xe-x

(i) x 3 - xe 2x

(j) eXcos 4x

(k) x 2

(I)

(a) x 3e 3x
(d) xe-2x

x 3e- x

(cos x

+ l)e 2x

+ sin x)e- x

3. Differentiate:
(a) e2x + 3
(d) 2e 3x -

(b)
(e)

ex2 - 2x
e3x - 1 + e4x + 2

(c) 3e-x3
(f) e sin 2x

.(i) 3e 2x2

(g) e-x..JX

(h) e COSX

(j) xe~

(k) 3e 2x -

(I)

e sin x

+ cos x

e2x + e8x write down the value of:

4. If y

(ii) ~~ and show that

(i) dy

dx

d 2y
dy

- 2 - 10- + 16y

=0

d 2x
dt 2

= 0

dx

5. If x

(l

t)e 5 /, prove that

dx

dx

10dt

+ 25x

6. Find the equation of the tangent to the curve y


Y-axis.
7. Find the equation of the tangent to the curve y
Y-axis.

ex at the point where it crosses the


=

e- at the point where it crosses the


X

8. Find the minimum value of (x - 2)ex


9. Find the coordinates of the turning point of the curve y = xe-o' 5x stating whether it is
maximum or minimum. Find also the values of x for which:
(i) y

> 0

(ii) dy

dx

>0

10. Consider the function defined by the rulef(x) = 3 - e-x, x


(a) Find the value of f(O) and f' (0).
(b) Show that f' (x) > 0 for all values of x in the domain.
(c) What is lim f(x)?

O.

x-oo

(d) Sketch the graph of f(x).


11. Consider the function defined by the rule f(x)

e-~ for all values of x. Find

(a) f'(x)

(b) the values of x for which:


(i) f' (x)
0
(ii) ff (x) > 0
Sketch the graph of the function.

(iii) f'(x)

< 0

18/EXPONENTIAL AND LOGARITHMIC FUNCTIONS 413

12. The concentration of a certain drug in the blood at time I hours after taking the dose
is x units, where x = 03 le- I lt . Determine the maximum concentration and the time at
which it is reached.

Plot the graph of x = 03Ie- I lt for I = 0,01,05, 1, 2, 3 using graph paper.

The drug kills germs only if its concentration exceeds 006 units. Find, from the

graph, the length of time during which the drug is able to kill germs.
13. Find the equation of the tangent to the curve y = e 2x at the point where x = 1. Find
also the co-ordinates of the points where this tangent intersects (i) the X-axis (ii) the
Y-axis.
14. Find the value of I for which e t

+ 4e-t has a minimum value.

15. Write down the equation of the tangent and normal to the curve y = 2
point where x = O.
16. Sketch the graph of J(t) = 2

+ 3e- t '

+ e-

at the

I ~ O.

(a) Show that J' (t) > 0 for all values of I in the domain.
(b) Find t_oo
limJ( I).
(c) State the range of the function.
17. J(x) is defined by the ruleJ(x) = e- Xcos x in the domain 0

7r.

(a) Find the values J(O), J(~} J( 7r).


(b) Write down J' (x).
(c) Show that J' (0) = -1 and J'

(347r) = O.

(d) Sketch the graph of J(x).


i .

18. The rectangle PQRS has two vertices on


the X-axis and two on the curve
y = e- x2 . Find (a) the value of x for

which the rectangle has a maximum area

and (b) the maximum area of the


rectangle.

:1

Fig. 18-9

19. Write down the indefinite integral (primitive function) of:


(a) e 2x
(b) e 5x
(c) e-{)'4x
(e) eX

e-3x

(f) e-2x - e- X

(g) .!.(eO'5X

(d) 5e 2'5x

e-{)'5x)

20. Find:
(a)

f e-xdx

(d)

f (e t -

l)dl

il

(b)

(e)

f (e 2u + u 2)du

(f)

f (e-2'5x + eO'4X )dx

(b)

fo2e 2xdx

(c)

II

e l12x dx

(c)

3x

e- dx

21. Find the value of:


(a) Illexdx
(d)

1,

le l'5t dl

fO'5
(e)

-0'5

e-2t dl

e- l12x dx

(f) flO e-3u du

414 NEW SENIOR MATHS: TWO UNIT COURSE

(g)

10'

(e 2x

e-2X)

1'5

dx

(i)

(eO

0-5

e-3)dO

0) fI\e x - e-xpdx
e2x + e-2x , the X-axis and the lines

22. Calculate the area enclosed between the curve y


x = 1 and x - I .

23. (a) Calculate the area bounded by the curve y = ex, the coordinate axes and the line
x = 2.
(b) Write down the equation of the tangent to y = ex at the point where x = 2.
(c) Calculate the area bounded by y = ex, the coordinate axes and the tangent at
x = 2.
(d) Calculate the area bounded by y = eX, the Y-axis and the liney = e2.
24. Calculate the area bounded by the curves y = eX, y = e- Xand the ordinate x = 2.
25. Calculate the area bounded by the curve y
x = 1.

eO' 5x - e-Q'5x, the X-axis and the line

26. The area under the curve y


e- X between x
0 and x = 1 is rotated about the
X-axis. Find the volume of the solid of revolution.
27. Find the volume generated when the curve y = eX, O' 5
X-axis.

=E;;

28. Find the volume generated when the curve y = e-Q'5x, -2


X-axis.
29. Find the volume generated when the curve y
rotated about the X-axis.

x
=E;;

=E;;

1 5 is rotated about the


=E;;

2 is rotated about the

eX + e- Xbetween x

-1 and x

= 1 is

30. Using Simpson's rule with three function values, find approximately the value of

fllx2exdx
31. Plot the graph of y = xeO' 4x for integral values of x from 1 to 5 and evaluate, using
Simpson's rule with five function values,

5
fl xe o'4xdx
32. Use Simpson's rule with three function values to evaluate

33. Use the Trapezoidal rule with four strips to evaluate

ire"" dO

f."'xe-xdx

34. Write down the derivative of (x - 1)& and use your result to evaluate f_:xexdx
35. (a) Find the area of the region bounded by the curve y = e-X, the coordinate axes
and the line x = a, a > O.
(b) Find the limit of this area as a - 00.
(c) Find the volume of the solid generated by rotating the region in (a) about the
X-axis and find the limit of this volume as a - 00.

18/EXPONENTIAL AND LOGARITHMIC FUNCTIONS 415

18.5 Derivative of loge X

_ _ _ _ _ _ _ _ _ __

We have already seen that!(x) ex and g(x) = loge x define an inverse pair of functions
(See fig 18-5). One of the characteristic features of a pair of inverse functions is that the
graph of one of the functions is the reflection of the graph of the other in the line y = x
i.e. the rule for one function is obtained by writing x in place of y in the rule for the other
(their domains and ranges interchange). This leads to an important property relating their
gradients at any point.
If, in the chain rule,
dy _ dy du
dx - du . dx

we write x in place of y,
then

d(x)
dx

dx du
du'dx

du
dx

dx du
du' dx
dx
l/
du

i.e.
Hence

since !(X)

This important result is most frequently met in the form


dy
dx

Let
i.e.,

l/dx
dy

y == loge

x = eY

(1)

Differentiating (1) with respect to y, we get


dx
-=eY=x
dy

Hence

i.e.

It follows that loge x

i.e.

dy
dx

l/ dx
dy

=1
x

d
1
-(/ogex) = -, x > O.
x
dx

+ c is the indefinite integral of 1,


x

J~dX =

loge x

+c

416 NEW SENIOR MATHS: TWO UNIT COURSE

Earlier we stated that

_1_ xn - 1 + c

n + 1
'
-1. We have now filled in the exceptional case.

XndX

except when n

f(x)
f(x) =

log~x

Fig. 18-10
Graph of f(x) = logeX', x > O.
The slope of the curve is
I

'

1x which is positive for

all x in the domain. The graph rises

steadily with a very steep slope for x small but this slope decreases as x increases. When x
is large, the slope approaches zero. The function is thus one-one and so has an inverse.
The curve crosses the X-axis at x = 1 (since loge 1 = 0) at an angle of 45 0
For all
x > l,f(x) > 0

When 0 < x < 1, f(x) < O.

The fact that loge x =

-IOge(~) provides an interesting property,

and so the lengths of the ordinates at x = 2 and x =

e.g. loge 2 =

-IOge~

~ are equal. Similarly for any other

value of x and its reciprocal.


Frequently the natural logarithm of x, (x > 0) which is denoted by In x or loge x, is
defined by

i.e. by the measure of the area bounded by the graph of the rectangular hyperbola
f(l) =

~,

(I

> 0), the (-axis and the ordinates ( = x and ( = 1. (fig. 18-11)

RM

RM

f(t)

f(t) =

"

Fig. 18-11

2 e3

Fig. 18-12

18/EXPONENTIAL AND LOGARITHMIC FUNCTIONS 417

.r.X~dt

= [lOge tJ~ = loge X

- loge 1

= loge x

Similarly, the number e may be defined by

.r.
1

el
,dt = 1, t > O.

The number e is such that the area bounded by the graph off(t) =

~, t > 0, the t-axis and

the ordinates t = e and t = 1 is 1 squnit. (Fig 18-12).

18.6 Derivative of loge (ax), a


Let

loge (ax)
= loge a + loge

dy = 0
dx

> 0 _ _ _ _ _ __

+1
X

1
X

or, let

loge u where u
dy
1
dx = u a
a
Y

ax

ax

=1

> O.
Fig. 18-13

Note" that the derivatives of loge X and loge ax are both l/x. What does this tell us about
their graphs?
The graph of loge ax is loge a units above or below the graph of loge X depending on
whether a > 1 or 0 < a < 1. They are 'parallel' curves in the sense that they are a
common distance apart and their gradients are equal at each corresponding point.

EXAMPLE 6
Differentiate with respect to x:
(a) X2 loge 2x
(a) Let

(b) sin x loge X

y = X2 loge 2x

uv where u = X2 and v = loge 2x

dy
du
dv

dx = v. dx + u. dx (product rule)
1
= 2xloge2x + X2.-,X
>0
x

= x(210g e 2x + 1)
(b) Let

x loge x
uv where u = sin x and v = loge x

du
dv

v . dx + u. dx (product rule)

y = sin
=

dy
dx

cos x loge x

+ sinx x

i
I

418 NEW SENIOR MATHS: TWO UNIT COURSE

18.7 Derivative of 10ge (ax

b) _ _ _ _ _ _ __

y = loge (ax + b)
loge u where u = ax
dy
dy du
.
dx = du . dx (cham rule)

Let

a
u

a
ax

Hence

ax - b

dx

-loge (ax
a

b)

+ c.

EXAMPLE 7

+ 1), (ii) loge (x 2 + 2x + 1), (iii) loge (sin x)


loge (X3 + 1)
(ii) Let y = loge (x 2 + 2x + 1)
3
loge u where u = x + 1
loge u where u = X2 +
dy du
.
dy
dy du
.
du . dx (cham rule)
dx = du . dx (cham rule)

Differentiate (i) loge


(i) Let

y
=

dy _
dx -

(X3

u'1 3x
3x

u1 (2x

+ 2)
2x + 2

+
(iii) Let y

2x

X2

+ 2x + 1

loge (sin x)
= loge u where u = sin x

dy du
dy
dx = du . dx

= -. cos x
u

cosx

-sin x

= cot

You will notice that the derivative of these three logarithmic functions is a quotient in
each case in which the numerator is the derivative of the denominator i.e.
d
f'(x)
dx[logef(x)] = f(x)

From this it follows that

I
<

f j(~i dx = logef(x) + c
or

af ' (x) dx = a log f(x)


f(x)
e

+c

IS/EXPONENTIAL AND LOGARITHMIC FUNCTIONS 419

EXAMPLE 8
Find the indefinite integral of:
( 0)
I

2
2x - 3

2x

(i)

II

X2

Jj(~)

3 dx

( 000) 4x

(00)

+4

X2 :

4 dx

!Ilf (x) dx where /(x)

2 /(x)
1
= 210ge (x 2
(iii)

f_4X_-_6dX
3x

4)

X2

X2 -

2f2X - 3 dx
X2 - 3x

X2 -

II :x

2J~(~; dx where/(x)

(iv)

eX
(IV) 1 + eX

dx where f(x) = 2x - 3

loge (2x - 3)

(ii)

6
3x

X2 -

III

I~(~; d; where/(x)

eX

= loge (1

ex)

3x

eX

EXAMPLE 9
dy

I f dx =


x1 and
y

h I .
2' express y III terms of x.

w en x

dy _!
dx - x
y =

I~dX

= loge

When x

x + c

= 2' y = 0 and so

o=
=

2+

-loge 2

loge

c = loge 2
y = loge x

loge 2x

loge 2

il

420 NEW SENIOR MATHS: TWO UNIT COURSE

EXA"MPLE 10

The gradient of a curve at any point on it is X 24: 1 and the curve passes through the point
(0, 0)_ Find the equation of the curve_

f'(x)
f(x) = 2 JX2 2: 1dx
210g e (x 2 + 1) + c
o = 210g e 1 + c and so c
f(x)
210g e (x 2 + 1)

Since f(O) = 0,

EXAMPLE 11

(. )J

-J x + 1dx,

Evaluate
, I

(I)

(iii)

(iv)

--dx= 3
Jlx+l

(i)

2X

3X

- 1 2 dx
-X-

J:

1)

3 [loge 3 - loge 2]
310ge 1-5

3 X 0-4055 (using a calculator)

1-2165

[lOge (x 2

loge 10 - loge 4

loge 2-5

0-9163 (using a calculator)

(ii)

o 2

JIx+l
--dx

= 3 [lOge (x

X _

2)J 4

x'

.-") S'mce, b y d IVlSlon,


- ..
x-x. 11th
(III
= x en

X2

=[2

J4

loge x

loge 4) - (2 - loge 2)

6 - loge 2

6 - 0'6931 (using a calculator)

5-3069

= (8

=
~

=
(iv)

7r12

cos

+'
dx
sm x

10ge(2

sin x)

= loge 3 - loge 2

=
~

2x - 1 d

loge 1-5

0-4055

7r12

2 x,

3X 2 7r12

4X2 -

"

cos _x dx

sm x

I8/EXPONENTIAL AND LOGARITHMIC FUNCTIONS 421

EXAMPLE 12
Find

(a) the area bounded by the curve y = loge x, the X-axis and the ordinate x == 2,
(b) the volume of the solid of revolution formed by rotating the area bounded by
the curve y = loge x, the co-ordinate axes and the line y = loge 2.

(a) Area = fl\Oge xdx


y

The required area is the area of the shaded region


BeE in the diagram. Unfortunately, it is beyond
the scope of this course to be able to find the in
definite integral of loge x. It involves a technique
known as integration 'by parts'. However, we
can find, by integration, the area ABED and then
subtract this area from the area of the rectangle
ABeD.
Since
then

y = loge

I
I

Fig. 18-14

x = eY
Area ABED =

loge 2

eY dy

[eYJ :oge 2
=

eloge 2 -

=
=

2 - 1

eO

Area ABeD = 210g e 2


... Area BeE = 210g e 2 - 1
(b)

2 x 06931

1 = 03862 units 2

r10~2

Volume
=

7r

7r

x 2 dy where x
loge 2

e2ydy since X2

eY
(eY )2

= e2y

~(4 - 1)

37r

. 3
umts

EXAMPLE 13

Differentiate x loge x and hence find the primitive function of loge x and so evaluate
f1210geXdX

'\

422 NEW SENIOR MATHS: TWO UNIT COURSE

y = xlog e x
= uv where u

Let

=x

and v

loge x

d
1
-d (xloge x ) = 1. loge X + x. - (product rule)
x
x
= loge x + 1
loge x

IIOg,XdX

flOg,XdX

dx(xlog e x) - 1

= !<xlog, x)dx - II dx
= xlogex - x

+ c

= [ xlog, X

J:

- X

(2 loge 2 - 2) - (loge 1 - 1)

03862 (see example 12(a))

= 2log e 2 ~

EXAMPLE 14
Find the volume generated by rotating the area beneath the curve y

1
-JX
between x =

and x = 9 about the X-axis.


Volume

'Tr

[lOg, xJ :

y 2 dx where y

= -JX

'Tr(loge 9 - loge 4)
'Trlog e 225
2548 (using a calculator).

x
Fig. 18-15

EXERCISES 18(b)
1

1. Differentiate:
(i) loge 2x

(iv) loge (2x + 5)

(ii) 2log e X

2. State .the largest possible domain of each of the following functions and find their
derivatives:
(i) f(x) = loge (3x + 2)
(iii) f(x) = loge (X2 - 4x + 4)

3. Differentiate:
(i) x In x
I

(iv) (x 2 + 1) In 2x
(vii) cos-x In 2x
x
(x) .' loge x

(ii) f(x) = loge (x 2 + 1)


(iv) f(x) = loge (4x + 3)

(ii) x 3 In x
(v)
(2x - 5) In x
(viii) eX In x

(xi)

loge x
x

(iii) (x + 2) In (x
(vi) tan x In x
(ix) e 2x In 2x
00) loge x
(XII

eX

2)

18/EXPONENTIAL AND LOGARITHMIC FUNCTIONS 423

4. Differentiate:
(a) loge (x2 - 2x)
(d) loge (X3)

(g) loge (x 2 -

(j)

1)

sin (loge x)

(lit) loge (loge x)

(b) loge (cos x)

(c) loge (ex)

(e) loge (x
x +

(f) loge (tan x)

33)

(h) loge (X4)

(i) cos (loge x)

(k) e10geX

(I) X2 loge (X2)

(0) tan (loge x)

(0) loge

S. Find the gradient of the curve y

vX

= loge (x 2 + 1) at the point where x = 3.

l' (x) (b) f'(X) (c) f' (2) (d) f'(2).

6. If f(x) = loge x, find (a)

7. Find the equation of the tangent and normal to the curve y


it crosses the X-axis.

= loge x at the point where

8. Find all values of x between 0 and 27r for which loge (sin x) is defined and state the
maximum value of loge (sin x).
9. Using the same axes sketch the graphs of y = loge x andy
their gradients are equal for all values of x.
Write down the equation of the tangent to the curve y

10ge(}X) and show that

(1

~l

= loge 2,x)

at the point where

it crosses the X-axis.


10. Show that y
gradient.

= loge (ex) is equivalent to y =

x for all values ofx and hence state its

11. Sketch the graph of y


loge (x - 1), stating its largest possible domain. Find the
equation of the tangent at the point where x = 2.
12. Find primitives of:
2
(b)
(a) x

(f)

(e)

13. If ddY
x

(c)

2
2x +

(d)

sin x
- cos x

( ) C?S x

(h)

x+

= x1 and y = 0 when x

g smx

eX

2, express y in terms of x.

14. The gradient of a curve at any point on it is 2x ~ 1 and the curve passes through the
point (1, loge 3). Find the equation of the curve.
15. Find the rule which defines f(x) given that f' (x)

and f(O)

16. Evaluate:
(a)

J,

1
(3 2
--ldx
(b) )0 x + 3 dx
x

(c)

J
0

-25

dx

+ 2x

(5 2x
(e) )

X2 _ 1 dx

(4

(d)

)2 2t _ 3 dt

424 NEW SENIOR MATHS: TWO UNIT COURSE

J.

+ 3 dx

(b)

3
13

1dx

(d)

fU

17. Evaluate:
(a)

(c)

f.' (X2 + ~)dX


--dx

(e)

4X2 ;:

7r/6

(g)Jr

7r12

sin x

X2 :

~2)dX

(f)f(x+x l )dx

f (x - ;,)2 dx

~os x x dx
dx

(h)

- cos

2x

(h)

18. Sketch the graph of f(x) = 2x ~ 1 and find the area enclosed by the curve, the X-axis

= 1 and x = 3.

and the lines x

19. Find the area of the region enclosed by the curve y

ordinates x

the X-axis and the

2 and x = 4.

x + .x:-+l' find the area enclosed by the

20. For the curve whose equation is y

curve, the X-axis and the lines x


21. Sketch the curve y

0, x

2.

= x + x!, x > 0 showing the coordinates of the turning point and

its asymptotes. Find also the area enclosed by the curve, the X-axis and the lines

x = 1 and x = 2.
22. Show that

1.

3
0 X2

2x

+ 9 dx = loge 2.

23. Write down the derivative of loge (cos x) and hence find the primitive function of
tan x. Use this result to find the area enclosed by the curve y = tan x, the X-axis and

24.

the lines x

0 and x =

7r

3'

lex

+ exdx

loge c. Find the value of c.

25. Find the area of tht.; region enclosed by the curve y

the X-axis and the line

1.

26. Find the volume of the solid generated by rotating the region enclosed by the curve

y2 =

~,

the X-axis and the ordinates x

= 1 and x = 3 about the X-axis.

27. Find the volume of the solid generated by rotating the area beneath the curve
y = -==1== between x = 6 and x = 11 about the X-axis.

I8/EXPONENTIAL AND LOGARITHMIC FUNCTIONS 425

28. Given a > 1, sketch the curve y = loge x, 1 ~ x ~ a and find the area enclosed by the
curve and the lines y = 0, x = a.
The region enclosed by the curve and the lines x = 0, y = logea and y = 0 is
rotated about the Y-axis to form a solid of revolution. Find its volume.
29. Sketch the curve y

;2'

> 0 for values of x from x = ~ to x

1. This part of the

curve is rotated about the Y-axis to form a solid of revolution. Find its volume.
30. Sketch the curve y = --=== from x

+x

0 to x

curve, the X-axis and the ordinates X= 0 and x


the volume of the solid so formed.

5. The region enclosed by the


5 is rotated about the X-axis. Find

2
-=== and the X-axis between x

31. The region enclosed by the curve y

8 and

x = 10 is rotated about the X-axis. Find the volume of the solid so formed.

32. The region enclosed by the curve y

.JX+l and the X-axis between x


x

3 and

5 is rotated about the X-axis. Find the volume of the solid so formed.

33. Evaluate an estimate of

3X

logex using

.., I

(a) Simpson's rule with three function values


34. Find approximately the value of

(b) the Trapezoidal rule with two strips.

~C logexdx using

(a) Simpson's rule with five function values (b) the Trapezoidal rule using two strips.

CHAPTER 19

Applications of

Calculus to the

Physical World

19.1 Gradient as a rate measurer _ _ _ _ _ _ __


Consider the linear function y = f{x) = 3x + 2. An increase, h, in the value of x leads to
an increase of 3h in the value of y. For any value of h 0, y increases by an amount equal
to three times the increase in x. We say that the rate of change of y with respect to x is 3.
The number 3 denotes the gradient of the line. Since the gradient is constant in this case,
then y increases at a constant rate.
Consider the quadratic function y = f{x) = 2X2 at the point (l, 2)

'*

f{x) = 2X2
f{l)

f{l + h) = 2{1 + h)2


k = f{l + h) - f{l) = 4h + 2h2

+ 2h (h

+ 4h + 2h2

'* 0)

Thus, when x undergoes a change, h, from 1 to 1 + h, f{x) undergoes a change


k = 4h + 2h2 i.e. the value of the function changes by an amount (4 + 2h) times the
amount that x changes
The average rate of change of y with respect to x = 4 + 2h. This rate varies with h.
As h - 0, 4 + 2h - 4 i.e. f' (l) = 4.
The number 4 denotes
(i) the gradient of the tangent at x = 1,
(ii) the rate of change of y with respect to x at x

1.

This concept of the derivative is a very important one in the study of the differential
calculus. It was developed in the seventeenth century by Leibniz and Newton.
There are many practical situations in which the change in a physical quantity depends
on time. If a vessel is being filled with water, the volume V of water in the vessel is a
function of time.

19/APPLICATIONS OF CALCULUS TO THE PHYSICAL WORLD 427

c::: is the rate at which the volume changes. It mayor may not be a constant tate.
The population P of a town may increase or decrease with time and c::: is the rate of
change of population.
If a spherical balloon is being inflated, then

~~ is the instantaneous rate of change of

volume with respect to time. Its radius is also increasing and c:;; is the rate of change of the
radius.

The volume of a sphere is given by the formula

4
V = -7('r 3
3

dV

dr

~~ measures

47('r 2

the rate of change of volume with respect to radius.

Boyle's Law states: 'The pressure, P, of a given mass of gas varies inversely as the
volume, V' i.e. the pressure P is a function of V where
P

dP

dV

= kV-1

V2

rate of change of pressure with respect to the volume.

The negative sign indicates that an increase in V leads to a decrease in P.

EXAMPLE 1
A cubical block of ice has an edge of 10 cm. It melts so that its volume decreases at a
constant rate of 25 cm 3 per hour and the block remains cubical. Find:
(a) the volume V at any time t
(b) the time required to completely melt the ice.
(a) Since the volume decreases at a constant rate of 25 cm 3 per hour,

dV
dt
V

(b) When V

= 1000 and so c =
= 1000 - 25t
0 = 1000 - 25t

0, V
:. V

When t
=

0,

-25

= -

f25dt

-25t + c

1000

t
40
1000
The volume at any time t is given by
V = 1000 - 25t, 0 ~ t ~ 40.

Note that the domain of the volume

function is restricted to values of t from

o to 40. Why? The volume function is a


o
linear function, the graph of which has a
constant gradient of -25.

1000 - 25t

Fig. 19-1

40

428 NEW SENIOR MATHS: TWO UNIT COURSE

EXERCISES 19(a)
1. A cubical block of ice has an edge of 10 cm. It melts so that its volume decreases at a
constant rate and the block remains cubical. If its edge measures 5 cm after 70 minutes,
find (a) the rate at which the volume decreases (b) the volume at any time t.

2. A trench is being dug by a team of labourers who remove V cubic metres of soil in
t minutes, where
t2
V = lOt - 20

(a) State the domain of the function, Le. the values of t during which soil is being
removed.
(b) At what rate is the soil being removed at the end of 40 minutes?
(c) Are the labourers working at a constant rate?
(d) What is their initial rate of work i.e. when t = O?
(e) At what time are they removing soil at the rate of 5 m3 per minute?
3. A water tank is being emptied and the quantity of water, Q litres, remaining in the tank
at any time, t minutes, after it starts to empty is given by:

Q(t) = 1000(20 - t)2, t ~ 0

(a) At what rate is the tank being emptied at any time t?


(b) How long does it take to empty the tank?
(c) At what time is the water flowing out at the rate of 20,000 litres per minute?
(d) What is the average rate at which the water flows out in the first 5 minutes?
4. The following table shows the temperature, TO Celsius, of water in a vessel, initially at
100C, after t minutes.
t(min)

TO Celsius

0
100

10

15

20

25

30

85

74

64

56

48

44

Plot the graph showing the relation between temperature and time and from it estimate
the rate at which the temperature is falling
(a) after 15 minutes
(b) when the temperature is 80 C.
5. A machine manufactures items at a variable rate given by:

~9

2t

+ 1, t ~ 0

where Q is the number of items manufactured in time t minutes.


(a) At what rate is the machine working
(i) initially (ii) after 10 minutes?
(b) What is the total number of items manufactured in the first 10 minutes?
6. Show that the rate of change of the area of a circle with respect to the radius is
proportional to the radius and find this rate when the radius length is 2 cm.
7. A right circular cylinder of volume V has height h and radius of base r. Find:
(a) the rate of change of volume with respect to height, if the radius of the base is
constant,
(b) the rate of change of the volume with respect to the radius of the base, if the height
is constant.
S. The pressure P, of a given mass of gas, kept at constant temperature and its volume V
.
dP
are connected by the formula PV = 500. Find
when V = 10.

19/APPLICATIONS OF CALCULUS TO THE PHYSICAL WORLD 429

19.2 Exponential growth and decay _ _ _ _ __


A quantity exhibits exponential growth when it increases by a constant percentage of the
whole in a given time period. This means, the larger the quantity at the start, the bigger
the increase will be. For example, if two countries, whose populations are 100 million and
10 million respectively, are each growing at an annual rate of 10070, the first country's
population will increase by 10 million during the next year, but the second country's
population will increase by only 1 million. This causes population control problems in
countries like India.
The table shows values of X2 (quadratic growth) and 2x (exponential growth) for
selected values of x. Corresponding graphs for x ;:,: 0 are shown in fig. 19-2. Notice how
the exponential growth far exceeds the quadratic growth as x increases beyond the value
x = 4.

o
o
2

10

16

. 25

36

100

16

32

64

1024

100

1030 (approx.)

In 1968, the Club of Rome was formed by an international group of concerned people.
Their study on the 'predicament of mankind', as they called it, was published in 1972
under the title of The Limits to Growth. This book is all about exponential growth and
decline in the world system-growth of factors like population and pollution, and decline
of factors like natural resources and quantity of food per head of population.
The general principle behind their reasoning can be appreciated if we imagine the
vertical axis in fig. 19-2 to represent population or pollution, and the horizontal axis to
represent time.
An increase of 1 unit in x, from x
1 to x = 2, causes only a small absolute increase in
y, from 2 to 4. This is like world population growth in past centuries. However, an
increase in one unit of 'time', from x = 10 to x = 11, causes y to increase by more than
1000. This illustrates the catastrophic growth that can occur when we have a growing
variable that is large enough to be on the steeply rising part of the curve. Exponential
decline is equally dramatic and this accounts for the current concern about the use of non
renewable resources.
y
y = 2x

,,
I

48

40

I
I

32

I
I
I

24

I
I
I

16

Fig. 19-2

.I
I
I

430 NEW SENIOR MATHS: TWO UNIT COURSE

There are many examples of exponential growth and decay in the physical world, e.g.,
population growth, growth of bacteria, the increase in girth of a tree, the decrease in the
pressure of the atmosphere at higher altitudes, the cooling rate of substances, the decrease
in the intensity of light passing through water or glass, the dying away of vibrations,
radioactive decay.
In each case, they follow the exponential law:

y = A e kx for exponential gro wth


A e-kx for exponential decay
y
If

y = Aekx

dy
dx

then

k Aekx

ky

dy - k
dx - Y
This states that the rate of change of one quantity, y, with respect to another quantity, x,
is proportional to y.
The rate of growth of bacteria in a culture is proportional to the number of bacteria
present at any instant. That is,
dN
kN
dt
The rate of decay of a radioactive element is proportional to the mass of that element
present at any time. That is,

d::

= -kM (Note the negative sign.)

The rate of cooling of a body is proportional to the difference between the temperature
of the body and that of the surrounding medium (Newton's Law of Cooling). That is,
d{) = -k{)

dt
where () is the temperature difference at any instant.
The rate of decrease of the pressure of the atmosphere with respect to height above sea
level is proportional to the pressure at that height. That is,
dP
-kP
dh
Light passing through a transparent medium loses its intensity as it penetrates to greater
depths. The rate of loss of intensity with respect to the depth is proportional to its
intensity at that depth. That is,
dI = -kI
dx
In each of these cases, we have a differential equation of the form
dy - k
dy - Y
It is important to appreciate the role played by the constants A and k in the equation
y = Aekx

Whenx = 0, y
Aeo = A.

Hence A is the initial value of y if x


Since :

= ky, k

0.

is the growth rate and influences the slope of the curve.

19/APPLICATIONS OF CALCULUS TO THE PHYSICAL WORLD 431

Ify = Aekx

A is the value of y when x


k is the growth rate

EXAMPLE 2
The annual growth rate of the population of two towns P and Q are 10070 and 5070
respectively of their populations at any time. If the initial population of Pis 20,000 and of
Q is 10,000, find their populations 3 years later.
Let N denote the population at any time t (years).
P:

dN
dt
:. N

Q:

O'IN

AeOo 1t
Since
N = 20000 when t
then
A
20000
:. N = 20oo0eOo lt
When t = 3, N = 20000eOo 3
~ 27000

dN
dt
:. N

0,

Since
then

= 0'05N

= AeOo05t

N = 10000 when t = 0,

When t =

A
:. N
3, N

= 10000

=
~

100ooeoo05t
10000eOo 15
11600

To find 20000eOo 3 using a calculator proceed as follows: (Use 2718 in place of e if there is
no key for e).
Operation sequence

20000

718

G03

Display

or 20000

GJ 0 3 0 B

269964

Note: It should be remembered that exponential growth (or decay) is a continuous


growth and that k indicates the instantaneous rate of increase of the population. Contrast
this with growth of money invested at compound interest where the interest is calculated
only periodically such as monthly, quarterly or perhaps half-yearly or yearly.

The graphs indicate the roles played by A


and k. Since A is the value of N when t = 0,
it is the point where the curve crosses the
N-axis.
The graph for P is steeper than the graph for
Q. This indicates the role of k, the growth
rate. The population of P grows more
rapidly than the population of Q.

N('OOO)
p

30

20

a
10-1----- N

= 10 000 eO-os,

Fig. 19-3

EXAMPLE 3

t (years)

A vessel containing water is being emptied and the volume V(t) cubic metres remaining in
the vessel after t minutes is given by:
V(t) = Ae-kt
(a) If V(O) = 100, find the value of A.
(b) If V(5) = 90, find the value of k.
(c) Find V(20).

(a) When t = 0, V = 100 and so A

100.

(b) When t = 5, V = 90 and so


I .
/

i/

432 NEW SENIOR MATHS: TWO UNIT COURSE


90 = 100e-5k
09 =e-5k
5k = logeO'9
-0'1054 (using the In x key on a calculator)
k :::::: 002
v(t) = 100e-o,o2t

V(20) = 100e-Q'4 since t = 20

(c)

: : : 67 (using a calculator).

To find 100e-Q'4 using a calculator, proceed as follows:

Operation sequence

100 x 2

718 ~

Note the use of the

D4

Display

4EJ

or 100 x

670348

key.

EXAMPLE 4
The pressure of the atmosphere, P cm of mercury, decreases with the height, h km above
sea level, according to the law P = 76e-Q2h.
Find the rate at which the pressure falls with respect to height above sea level when
(a) h = 5,
(b) P = 20.
P
dP
dh

76e-Q'2h

-0'2 x 76e-Q'2h

-15'2e-o'2h

(1)

(2)

-0'2P

Note that (1) gives

~~ as a function of h,

while (2) gives

~~ as

a function of P.

(a) Substituting h = 5 in (1), we get

dP

dh
-15'2e- 1
-5,59

The pressure falls at the rate of 559 cm/km when h


(b) Substituting P
Pfcm)

5.

20 in (2), we get
dP
dh

-0,2 x 20

-4

The pressure falls at the rate of 4 cm/km when P = 20.

76

Fig. 19-4

10 h(km)

Estimate what the air pressure would be on Mount Everest, which is about 9 km high.

19/APPLICATIONS OF CALCULUS TO THE PHYSICAL WORLD 433


i

EXAMPLE 5
The mass M of a radioactive substance is initially 10 grams and 20 years later its mass is
9.6 grams.
(a) Find the annual decay rate given that the rate of decay of a radioactive substance is
proportional to the mass of that substance present at any time.
(b) In how many years will the mass be halved?
dM
kMand so
dl
M
Ae-kt
(a) When 1

0, M = 10 and so A
10.
When 1 = 20, M
9-6 and so

9'6 = 10e-20k

096
e-20k
- 20k = logeO-96
= -0,0408
k
0-002
M = 10e-Q,002t

d::

= _ 0.02e-QOO2t

0'002M

5 = 10e-Q,002t
0-5 = e-Q'OO2t
. -0-0021 = logeO'5
= -0'69315
1 = 347

The mass will be halved after about 347 years.

(b) When M

5,

EXERCISES 19(b)
1. If

= 2y and y =

5 when x = 0, express y as a function of x.

dN
2. If (jj = -0-5N and N

100 when 1

3. If ~~ = 0'4Q and Q
4 . If dy
dl
5 . If dM
dl

-3y and y

0, express Q as a function of I.

50 when 1
20 when 1

-O'OIM and M

0, express N as a function of I.

0, express y as a function of I.

10 when 1

0, express M as a function of I.

6. If y = Ae- kt and y = 1000 when 1 0 and y = 368 when 1 = 2, find the values of A
and k.
7. If N
Aekt and N = 200 when 1 = 0 and N = 1478 when 1 = 5, find the values of A
and k.

8. If P = Ae-kh and P = 76 when h


and k.

= 0 and P = 28 when h = 5, find the values of A

434 NEW SENIOR MATHS: TWO UNIT COURSE

9. If the population of a city increases at a rate which is proportional to the current


population, and if the population of a certain city was 100000 in 1970 and 120000 in
1980 express, in terms of t, the population t years after 1970.
10. In a certain bacterial culture, the rate of increase is proportional to the number of
bacteria present.
(a) If the number doubles in 3 hours, find the hourly growth rate.
(b) How many bacteria are there after 9 hours, if the original population is 104 ?
(c) After how many hours are there 4 x 104 bacteria?
11. A heated body is cooling and the excess of its temperature, 0, above that of its
surroundings is 0 = Ae- kt where t is time in minutes.
(a) At time t = 0, 0 = 80, find A.
(b) If the temperature of the surroundings is 20C and the body cools to 70C in 10
minutes find:
(i) its temperature after 20 minutes,
(ii) the time taken to reach 60C.
12. The rate of decay of a radioactive element is proportional to the amount of the
element present at any time. If one half of a given quantity of the element
disintegrates in 1600 years, what percentage will disintegrate in 100 years?
13. The number of bacteria N in a colony after t minutes is given by N = 10oo0eo,o5t
Find
(a) the number of bacteria after 10 minutes,
(b) the time required for the original number to double,
(c) the rate at which the colony increases when
(i) t
10 (ii) N
20000.
14. A vessel filled with liquid is being emptied and the volume V cubic metres remaining
after t minutes is given by V
Voe- kt
(a) Show that

~~

= -k V

(b) If one quarter of the vessel is emptied in the first 5 minutes, what fraction remains
after 10 minutes?
/
(c) At what rate is the liquid flowing out
(i) after 10 minutes
(U) when one quarter of the vessel is empty?
15. For a period of its life, the increase in the diameter of a tree follows approximately the
law:

D(t)

Aekt

where D(t) is the diameter of the tree t years after the beginning of this period.
(a) If initially the diameter is 50 cm, find the value of A.
(b) If D I (t)
01 D(t), find the value of k.
(c) After how many years is the diameter 61 cm?
16. The charge, Qunits, on the plate of a condenser t seconds after it starts to discharge is
given by the formula

Q = Ae:- kt

(a) If the original charge is 5000 units, find the value of A.


(b) If ~~

= - 2000 when Q =

1000, find the value of k.

(c) Find the rate of discharge when Q = 5000.

19/APPLICATIONS OF CALCULUS TO THE PHYSICAL WORLD 435

17. The rate of increase in the number N of bacteria in a certain culture is given by
dN
0-15Nwhere t is time in hours.
dt
(a) If the original number of bacteria is 1000, express N as a function of t.
(b) After how many hours has the original number of bacteria doubled and what is
the rate of increase at this time?

18. Sunlight transmitted into water loses intensity as it penetrates to greater depths
according to the law
I(d) = I(O)e- kd
where I(d) is the intensity at depth d metres below the surface. If 1(300) = 0-3 1(0)
find
(a) the value of k
(b) the depth at which the intensity would be decreased by one half.

19. The rate of increase in the population p(t) of a particular island is given by the
equation !P(t)

=k

p(t) where t is time in years. In 1970 the population was 1000

and in 1980 it had decreased to 800.


(a) Find k, the annual growth rate.
(b) In how many years will the population be half that in 1970?
20. A substance decomposes at a rate equal to k times the mass of the substance present.
If initially the mass is M, find the mass m at time t.
If k = 0'1, find the value of t for which m =

~M.

21. Newton's Law of Cooling states that the rate of cooling of a body is proportional to
the excess of its temperature above that of its surroundings i.e.

~;

= -kt where TO is

the excess of temperature above its surroundings at any time t. The temperature of a
room is 25 and the outdoor temperature is 5. A thermometer which has been kept in
the room is moved outdoors. In five minutes the thermometer reading is 15. What
would its reading be in another five minutes?
22. The number N of bacteria in a colony grows according to the law:
dN = kN.
dt
If the original number increases from 4000 to 8000 in 4 days, find the number after
another 4 days.

23. A population of size N is decreasing according to the law:


dN
N
(ji = - 100'
where t denotes the time in days. If initially the population is of size No, find to the
nearest day how long it takes for the size to be halved.
24. A radioactive substance decomposes at a rate that is proportional to the mass present
at any time. If 10 per cent decomposes in 200 years, what percentage of the original
mass will remain after 1000 years?

436 NEW SENIOR MATHS: TWO UNIT COURSE

19.3 Kinematics _ _ _ _ _ _ _ _ _ _ _ _ __

,I

Kinematics deals with the study of the motion of a particle without consideration of the
force or forces causing the motion. Associated with a moving particle there are certain
quantities such as position co-ordinate, displacement, distance, velocity, speed and
acceleration, all of which depend on time. Our discussion will be restricted to the motion
of a body in a straight line.
'Particle' is the term used to represent a body such that all applied forces acting on the
body may be regarded as acting through a point. This definition of particle means that
quite large bodies e.g. trains can be classified as particles provided the stated condition
applies.

Position coordinate
X'~~--+---~--~--~~~X

Consider a particle, which could be represented by a point P, moving in a straight line


X' OX. The position coordinate, x, is its position relative to the fixed point 0, and may be
a positive or negative number according to whether P is to the right or left of O. The
origin of motion is not necessarily at 0, i.e. when t
0, P may be at, say, the point A.
Displacement is defined as the change of its position but does not necessarily represent
the distance travelled.

Velocity
Consider the equation x = f(1) which gives the position coordinate, x, of a particle
moving in a straight line at time t.
X

t + h, fIt + h)

f( t

+ h)

f( t)

t+h

Fig. 19-5
At time t, the particle is at A and at time (t + h) it is at B (fig. 19-5). Thus in the small
time interval h, the particle has changed its position by an amount k = f(1 + h) - f(l).
The average velocity in the time interval [t, t + h] =

fs"t+

The instantaneous velocity of the particle at time t is defined by

r f(l + h) - f(l)

'l~--h

and is denoted by v(l), f' (t),

~~ or x

hh - f(l) (h

*' 0).

191APPLICATIONS OF CALCULUS TO THE PHYSICAL WORLD 437

Velocity is defined as the rate of change ofposition with respect to time, or as the time
rate of change of position in a given direction.

EXAMPLE 6
Consider the equation x = 5 + 4t - t2, which defines the position x metres from 0, at
time t seconds (t ~ 0), of a particle moving in a straight line.
(a) Find the average velocity in the interval [t, t + h],
(b) Deduce the velocity at time t.
(c) Discuss the sign of the velocity in the time interval [0, 6],

f(t) 5 +
f(t + h) = 5 +
= 5 +
f(t + h)
f(t)
4h

(a)

f(t + h) - f(t)
h

4t - t 2
4(t + h)
(t + h)2
4t + 4h - t2
2ht
2ht
h 2

4 - 2t

h, h

"*

h2

= average velocity in the time interval [t, t + h]


(b) Velocity at time t:

v(t)

lim Ji'--..!{_t_+_h-=-)_-----'Ji'--..!{~t)
h
4 - 2t.
h-O

v = 4 and the particle is at A (fig. 19-6).


When t
2, v
and the particle is stationary at B. For the first 2 s of motion, the
particle is moving in a positive direction with a positive velocity.
When t = 3, v = -2. This negative velocity means that the particle is moving in a
negative direction with a speed of 2 m/s.
When t = 6, v = -8; the particle is moving with a speed of 8 mls in the negative
direction and is at the point C.
In the time interval [0, 6], the particle moves from A to B to C. Its final displace
ment from 0 is -7m. The distance travelled in the 6 seconds is 20 m.
Note that, when the velocity is zero, i.e., when the particle is at B, the particle
reverses its direction after being instantaneously at rest.

(c) When t = 0,

X
8

(2.9)

Vlt)

x = 5 + 4t

c
Fig. 19-6

t2

10,4)

(6, -7)

Fig. 19-7

438 NEW SENIOR MATHS: TWO UNIT COURSE

Acceleration
Acceleration is defined as the rate of change of velocity. Acceleration, like velocity, may
be positive or negative or zero. A positive acceleration indicates that the velocity is
increasing, and a negative acceleration indicates that the velocity is decreasing. A negative
acceleration is frequently called a deceleration or retardation.
If we denote the velocity by v(t), then the average acceleration over the interval

[t, t

+ h]

v(t

+ hh - v(t).

The instantaneous acceleration at time t is defined by

lim v(t

and

.j

+ h) - v(t)
h

h-O

..

dv d 2x
..
IS denoted by v' (I), a(l), f"(t), dt' dt 2 or x

In example 6, we saw that the velocity at any time t was given by v(t) = 4 - 2t.
The velocity is continuously changing. The rate of change of the velocity, namely, the
acceleration, is v' (I) = -2 m/s2; i.e., the velocity is decreasing at a constant rate of 2 mls
every second. The velocity is decreasing continuously and uniformly from 4 mls to -8 mls
in the time interval [0, 6].

EXAMPLE 7
A particle moves in a straight line such that its position x m from a fixed point 0 on the
line at time t s (I ~ 0) is given by x = t 3 - 12t + 16. Find
(a) its initial position, velocity and acceleration,
(b) the time when its velocity is zero, and its position and acceleration at that time.
(a)

(1)
x= t 3 - 12t + 16
dx
v -- 3t 2 - 12
(2)
dt
dv
(3)
a = - = 6t
dt
When t = 0, x = 16, from (1). Hence, the initial position is 16 m from O.
When t = 0, v = -12, from (2). Hence, th~ initial velocity is -12 m/s.
When t = 0, a = 0, from (3). Hence, the initial acceleration is zero.

(b)

12 =
2) =
t = 2 (t ~ 0)
Hence its velocity is zero after 2 s. The particle changes direction after 2 s.
When t = 2, x = 0, from (1). The particle is at O.

When t = 2, = 12, from (3). The acceleration is 12 m/s2.

3t 2
3(t - 2)(t

You should draw the graphs of equations (1), (2) and (3) for t ~
In the examples considered, we have been given an equation defining the position
coordinate x of a particle at any time t, and have deduced equations for velocity and
acceleration by means of the derivative.
On the other hand, if we are given an equation defining the acceleration at any time, we
can deduce equations for velocity and position by means of the indefinite integral and
additional information.

19/APPLICATIONS OF CALCULUS TO THE PHYSICAL WORLD 439

EXAMPLE 8
A particle starts from rest 5 m from a fixed point 0 and moves in a straight line with an
acceleration a ml S2 where a 3 t - 4. Find the velocity and position of the particle at any
time t.
a = 3t - 4

J (3t - 4)dt

= ~t2
When t

= 0,

= 0 and so c =

- 4t

O.

Thus

v = ~t2 - 4t

and so,

When t

0, x

+c

J(~t2 - 4t) dt

1
2,t3 - 2t 2

+d

= 5 and so d = 5.

I.

x = 2,t3 - 2t 2 + 5

Hence,

Units and symbols


Physical quantity

Unit

Time
Position

s
cm,m

Velocity

cm/s, m/s

Symbol

t
x (or s)
dx .
v, dt' x
dv d 2x ..
a, f 'dt' dt 2' x

Acceleration

s is the abbreviation for second, cm for centimetre, m for metre.


Acceleration due to gravity = 98 m/s 2 (::::: 10 m/s2).

EXAMPLE 9
A ball is projected vertically upwards from the top of a building 30 m high with an upward
velocity of 25 m/s. Find:
(a) the time taken to reach the highest point,
(b) how long it will take the ball to reach the ground,
(c) the speed with which the ball strikes the ground.
Consider- acceleration due to gravity as 10 m/s 2 (approximately).
If we consider the upward direction as positive, then a
-10; v = 25 andx = 30 when
t = o. It is convenient to consider the origin of motion at 0, ground level.
a = = -10
(1)

= X = - J10dt
-lOt

+c

440 NEW SENIOR MATHS: TWO UNIT COURSE

When t = 0,

x=

25 and so c = 25

x=
x

25 - lOt

(2)

(25 - 10t)dt

25t - 5t 2 + d
When t = 0, x = 30 and so d = 30

= 30

+ 25t -

5(2

5(6 + 5t - (2)
(3)
Equations 0), (2) and (3) give the acceleration (constant), velocity and height x above the
ground at any time t ~ O.
(a) At the highest point, x
O.
Hence from (2),
o =.25 - lOt
t = 2!

It takes 2! s to reach the highest point.

(b) When it reaches the ground, x


O.
Hence from (3),
o = 5(6 + 5t - t 2)
o = 6 + 5t - t2

t 2 5t - 6
0

(t - 6) (t + 1) = 0

t = 6 since ( ~ 0

(c) Substitute t = 6 into (2)


x = 25 - 10 x 6
-35
I

30

25

Fig. 19-8

The ball strikes the ground with a speed of 35 m/s. What is the significance of the negative
sign?

EXERCISES 19(c)
1. A particle moves in a straight line so that its position x m from a fixed point 0 on the
line, at any time t s (t ~ 0) is given by x = t 2 5t + 6. Find:
(a) its initial position,
(b) its initial velocity,
(c) when it first passes through 0 and with what velocity,
(d) when it passes through 0 for the second time and with what velocity,
(e) when and where its velocity is zero.
2. The position x m at time t s (t ~ 0) of a particle moving in a straight line is given by
x = t 2 5t + 4.
(a) At what time is its velocity zero?
(b) What is the acceleration at this time?
(c) What is the distance travelled in the first 4 s?
(d) At what time is the velocity 8 m/s?
3. A point moving in a straight line is distant s m from the origin 0 at time (, where

2t 3
15t2 + 36t
(a) Find the velocity and acceleration at any time (.
(b) Find the initial velocity and acceleration.

191APPLICATIONS OF CALCULUS TO THE PHYSICAL WORLD 441

(c) At what times is the velocity zero?


(d) At what time is the acceleration zero and find its velocity and position then?
(e) During what interval of time is the velocity negative?
4. The position x m at time 1 s of a particle moving in a straight line is given by

x = 21 3 - 9/ 2 + 121 + 6

Find:
(a) when its acceleration is zero and its velocity then,
(b) when its velocity is zero and its acceleration then.
5. Two bodies, starting at the same time, move along a straight path, so that their
displacement s m from a fixed point 0 at any time 1 is given by s
1 + 6 and
s = 12 + 4 respectively. At what times are they (a) together (b) travelling with the
same velocity?
6. A body starts from 0 and moves in a straight line. After time 1 units, its velocity is
12 - 4/ 3 Find, in terms of I,
(a) the position x,
(b) the acceleration.
7. The velocity v ml s at time 1 s (t

0) of a body moving in a straight line is given by


6/ 2 + 61 - 12

Its initial position is 7 m from O. Find:

(a) its displacement and acceleration at any time I;


(b) the acceleration when the velocity is zero;
(c) its initial velocity and acceleration.
8. The acceleration of a body moving in a straight line is 10 - 21 m/s 2 at any time 1 s.
The body is initially at zero displacement and is given a velocity of 11 m/s. Find:
(a) the velocity and displacement at any time I;
(b) when the body has zero velocity and its displacement then.
9. A body is projected vertically upwards with an initial velocity of 30 m/s. It rises with
a deceleration of 10 m/s2. Find:
(a) its velocity at any time I;
(b) its height, h m, above the point of projection at any time I;
(c) the greatest height reached;
(d) the time taken to return to the point of projection.
10. A particle is projected vertically upwards from a point 0 with velocity 25 mls and has
a downward acceleration of 10 m/s2.
(a) Find its velocity and height. above 0 at any time I.
(b) To what height does it rise?
(c) At what time has its velocity been reduced to half the velocity of projection?
11. The velocity v mls of a body moving in a straight line is given by v = 3/ 2 - 21 - 1.
The body initially has a displacement 1 m from O. Find:
(a) the displacement and acceleration at any time I;
(b) the time when the body has zero displacement and its velocity and acceleration
then;
(c) the distance travelled in the first two seconds.
12. The velocity v(/) of a particle moving in a straight line at any time 1
v(/) = 12/2 - 61 + 1. Find its position s(/), given that s(l) = 4.

0, is

13. The acceleration of a particle moving in a straight line is 10 - 21 ml S2 at any time

442 NEW SENIOR MATHS: TWO UNIT COURSE

t ~ O. The particle starts from 0 with a velocity of 24 m/s. After what time is its
velocity zero and what is its position at this time?
14. Two cars, A and B, travel along a straight road and in the same direction. Their dis
tances, x kilometres, from a fixed point 0 at any time, t hours, are given by the rules
A: x = SOt - 20t 2
B: x = SOP + 20t
(a) Calculate their speeds at the point O.
(b) After what time are they travelling at the same speed?
(c) If both cars reach a point Q at the same time, calculate the distance from 0 to Q.
(d) A third car, travelling at uniform speed, is 2 km ahead of A and B when they pass
the point O. If it arrives at Q at the same time as A and B, find a rule connecting x
and t for this car.
15. The displacement x(t) at time t ~ 0 of a particle moving in a straight line is given by
x(t) = 2t3 .,.- 9t 2 + 12t + 6

Find:

(a) when its acceleration is zero and its velocity at that time.
(b) when its velocity is zero and its acceleration at that time.

16. A particle moves in a straight line so that its displacement x(t) from a fixed point in
~

the line at time t

0 is given by

x(t) = 3

+ 4t - 5~
Find the displacement when the particle comes to rest.

EXAMPLE 10
A particle moves in a straight line and at any time t seconds (t
x metres from a fixed point 0 in the line is given by:
x(t)

7r

2 cos 5t, 0

:E;

:E;

0), its displacement

20

Sketch the graph of the displacement and find when its velocity is zero.
Discuss the motion.

xtt)

=4 +

2 cos ~ t

10

Fig. 19-9

15

20

19/APPLICATIONS OF CALCULUS TO THE PHYSICAL WORLD 443

The graph is the ordinary cosine curve with amplitude 2 and period 10 i.e. 21r where

f The cosine curve is raised 4 units.

The velocity

i~

given by

v
:. v

st

. 1r = 0.
0 when sm

st1r = 0, 1r, 21r, 31r, 41r, . ..


t

0, 5, 10, 15, 20 .

The particle is initially at rest at A 6 metres from O. It then moves from A to B in 5


seconds, stops at B and then returns to A in the next 5 seconds. It repeats this motion in
the next 10 seconds. This shows the significance of the period. The particle oscillates
between x = 2 and x
6 every 10 seconds. You may learn later that this particle executes
Simple Harmonic Motion.

EXAMPLE 11
A particle moves in a straight line and at any time t seconds, its velocity v m/s is given by
v e- t If initially the particle is 2 metres from a fixed point 0 in the line, find its position
x at any time t. Sketch the graph of x as a function of t.
v

v = dx = e-t
dt

..

Je-tdt

= -e-t +

When t = 0, x
2
.'. 2
- 1 + c and so c = 3

:. x

3 -e-t

The velocity-time graph v = e-t shows that


initially the particle moves with a velocity of
1 m/s and as time goes by this velocity gets
smaller and smaller so that as t - 00, v - 0
from above (fig. 19-10).

Fig. 1911

The displacement-time graph shows that the particle starts at x = 2 and as time goes by
it moves closer and closer to x = 3 but never reaches it. As t -:+ 00, x - 3 from below
(fig. 19-11).

444 NEW SENIORMATHS: TWO UNIT COURSE

EXERCISES 19(d)
1. The position x of a particle at time t is given by x
6 cos 4t + 3. Find:
(a) the velocity and acceleration at any time t,
(b) the position of the particle when t = 0,
(c) the values that x can take,
(d) the time when the particle first reaches the position x = O.
2. The position x metres of a particle relative to a fixed point 0 at any time t is

x
4 - cos 2t

(a) Sketch the graph of x as a function of t in the domain 0 ~ t ~ 21r.


(b) Find the times when the particle is at rest.
(c) Express the acceleration in terms of (i) t (ii) x.
3. A motorist takes 3 hours to cover the distance between two points A and B on a
country road. At time t hours after passing A, his speed v km/hour is given by
v = 60 + 40e-t
(a) Calculate his speeds when he passes points A and B.
(b) Write down the acceleration in terms of (i) t (ii) v.
(c) Sketch the velocity-time curve and comment on the particle's motion for large t.
(d) Calculate the distance from A to B.
4. A particle moves in a straight line. At time, t seconds, its displacement x cm from a
fixed point 0 in the line is given by

5 sin ~t, 0 ~

~ 4

(a) Sketch the graph of its displacement at any time t.


(b) At what times is the particle at rest?
(c) Find the speed when x = 2!.
(d) Express the acceleration in terms of (i) t (ii) x.
5. A particle moves in a straight line so that at time t its displacement from a fixed origin
is x and its velocity is v. If its acceleration is 2 sin t and v
1 and x = 1 when t = 0,
find x as a function of t.

6. A particle moves in a straight line so that at any time

t ~

0 its velocity v is given by

(a) If its distance from a fixed point 0 in the line is x, show that x = 10ge(1
given that x = 0 when t = O.
(b) Show that the particle is moving away from 0 as t increases.
(c) Find the acceleration when t = O.

t),

7. A particle moves in a straight line so that at time t its displacement from a fixed origin
o is x where x = 2 + t - 2 cos t.
(a) Write down the velocity and acceleration at any time t.
(b) Find its initial displacement, velocity and acceleration.
8. A particle moves in a straight line and its displacement x from a fixed origin at any
time t is given by x(t) = 2(1 - e-t )
(a) Findx(O), X(O) andx(O).
(b) Sketch the graph of x(t).
(c) Findtwhenx(t)
1.

19/APPLICATIONS OF CALCULUS TO THE PHYSICAL WORLD 445

9. A particle moves in a straight line and, at time t, its displacement from a fixed origin

on the line is x where x = 2 - 2 sin 21, 0 ~ 1 ~ 211"

(a) Draw the graph of x as a function of I.


(b) Show that the particle oscillates between x = 0 and x
4.
(c) For what values of 1 is the velocity zero?
(d) Express the acceleration in terms of (i) 1 (ii) x.

10. A particle moves in a straight line and at time 1 its displacement from a fixed origin in
the line is x and its velocity is v where v = e-t
(a) Express x in terms of t given that x = 1 when 1 = 0 and draw the graph of the

function.

d 2x

(b) Show that dt 2

2.

11. A particle moves in a straight line and at time 1 its distance x from.a fixed point 0 in
the line is given by x
410g e (1 + t) - 2/, 1 ~ 0
(a) Find x when 1 = O.
(b) Find the initial velocity i.e. when 1 = o.
(c) Find the acceleration at any time 1 and show that it is always negative.
(d) At what time is the velocity zero?
12. A particle moves in a straight line so that at time 1 its displacement from a fixed origin
is x and its velocity is v. If its acceleration is:
(a) 2 cos 1 and v
1 and x= 0 when 1 = 0, find x in terms of I.
(b) - 3e-t and v = 0 when 1
0, find the time at which v 2.
13. A body falls from rest in a medium and its velocity v at any time t is given by
v = 49 (l - e-O5t )

Find:

(a) the acceleration at any time I,


(b) the initial velocity,
(c) the terminal velocity Le. its velocity as 1 - 00,
(d) the distance fallen in the first 5 seconds.
2

14. The acceleration of a particle moving in a straight line is given by d x

12 cos 2/.

Initially v = 0 and x = 6. Find its velocity v and displacement x at any time 1 seconds

and sketch the graph of the displacement. How many times does the particle change

direction in the first 10 seconds?

I!

REVISION EXERCISES C

- 1. (a) Find the following:


1 - 4X2
( ) I'
x 3 - 27
(I.) I'1m
II
lm----:-
x-112 1 - 2x
x-3
X - 3
(b) (i) Find the equation of the tangent to the parabola with equation y = 4x at the point where the gradient is -2.
(ii) Find the area bounded by this parabola and the X-axi.

X2

2. (a) Differentiate with respect to x:


(ii) .x (x -

(i) X4

(b) Evaluate the following:


(i)

r:

(x 3

3x2

+ 2x

5)dx

(ii)

1)

r:

(iii) x 3

(X2 - 2X)2 dx

(iii)

2x

r:

(5

2x)' dx

3. (a) A body starts from rest and moves in a straight line so that its velocity, v mis,
after t s is given by v = 2t + 6t 2 Calculate:
(i) its acceleration at the end of the first second,
(ii) its displacement after 5 s, given that the body is initially at zero displacement.
(b) For the graph of y
15x + 12x2 - 4x 3 for -1 ~ x ~ 3, find the values of x for
which
(i) y increases as x increases,
(ii) y decreases as x increases,
(iii) y is maximum,
(iv) y is minimum,
4. Consider I: R - R where I(x) = 2x 3 + 3x2 - 12x. Find:
(a) the values of x for which
(i) I' (x) = 0
(ii) I' (x) > 0
(iii) I' (x) < 0
(b) the area bounded by the graph of I, the X-axis and the ordinates x
x = O.

-2 and

5. A figure consists of a semicircle with a rectangle constructed on its diameter, If the


perimeter of the figure is 50 cm, find the dimensions of the rectangle in onier that the
area of the figure may be as large as possible. (Take 7r = 3~,)
6. Sketch the graphs of y = I(x) such that
, ~---(a) 1(3) = 5, I' (3)
0, I' (x) > 0 when x < 3 and If (x) < 0 when x > 3,
(b) 1(-1)
8, I' (-1) = 0,/(2) = 3, I' (2) = 0, If (x) < 0 when -1 < x < 2 and
I'(X) > 0 when x < -lor when x > 2.

REVISION EXERCISES C 447

7. (a) Differentiate the following with respect to x:


(i) (2 + x)(2 - x)
(ii) x (x - l)(x
(iii) (2x - 3)3

(iv) 6x (3x

1)

1)2

(b) Evaluate the following:


(i)

f_: 3x(2 -

x)dx

8. (a) Find the equation of the tangent to the parabola with equation y
X2 at the point
where x = 2 and find, by integration, the area measure of the region bounded by
the parabola, the tangent and the X-axis.
(b) Find the area measure of the region corresponding to the graph of
I(x,y):y

-51 n I(x,y):y

=s; 4x - x 21.

9. The acceleration of a particle moving in a straight line is 6-81 m/s 2 at any time 1 ~ O.
The particle starts from 0 with a velocity of 10 m/s. After what time is its velocity
zero and what is its position then?
10. A figure ABCDE consists of a rectangle ABCE surmounted by an equilateral triangle
CED. If the perimeter of the figure is 45 cm, find the dimensions of the rectangle
when the total area is a maximum.
4x
x 2.
11. (a) Consider the functionj(x)
(i) Find the value of a such thatjf(a)
O.
(ii) State the range of j.
(iii) Find the image j(4) of 4.
(iv) Find the values of c such that j( c)
c.
(b) Find the coordinates of the points on the following curves where the gradient is
zero. Determine whether these points are local maximum or minimum points.
(i) y

= 3x 3 -

2X2

(ii) Y

= x3

3x2 - 9x

12. A piece of wire, 8 m long, is cut into two parts. One part is bent into the shape of a
square, and the other into a rectangle whose length is twice its width. Calculate the
length of each part if the sum of the areas of the square and the rectangle is to be a
minimum.
13. For the graph of y

= j(x) wherej(x) = ~3

4x

3, find the following:

(a) the values of x for which (i) jf (x) = 0 (ii) jf (x) < 0,
(b) local maximum and minimum values of j(x).

Sketch the graph.

14. (a) Calculate the area of the region bounded by the curve with equatio ny = 2X2(4 x)
and the X-axis.
(b) Find the equations of the tangent and normal to the curve with equation
y = 2X2(4
x) at the point where the curve intersects the X-axis.
15. The velocity v mls of a partide moving in a straight line is given by
v = 6/ 2
41 + 1 (t ~ 0).
The particle initially ha a displacement -10 m from O. Find:
(a) the displacement nd acceleration at any time I,
(b) the acceleration when the velocity is 3 mis,
(c) the velocity when the acceleration is 20 m/s2.

448 NEW SENIOR MATHS: TWO UNIT COURSE

16. A single heating element in an electric pottery kiln is switched on at time 0 hours and
switched off at time 3 hours. The temperature of the kiln at various times is as
follows:
Time t (hours)

Temperature (OC)

20

I!

212

105

170

225

270

310

340

Illustrate the data by a graph. Estimate the gradient of the temperature


(a) at time H hours,
(b) immediately before the element is switched off,
Interpret your answers to (a) and (b).
A special control is installed later, whereby the rate of increase of the temperature is
maintained at a constant value. If the kiln reaches the same temperature in three
hours as it did before, give a formula for the temperature TOC at time t hours.
17. Sketch the graph of y

3x 3

5x 2 for values of x in the domain

-! ~

~ i!,

locating the turning points.


What is the equation of the straight line passing through the origin and the point
(1i, 3)1 What is the magnitude of the angle between this line and the tangent to the
curve at the origin1
Calculate the area bounded by this line, the curve and the ordinates x

.\

1.

18. The equation of the path of a shell fired with a velocity of v m S-1 at an angle

0:0

with

2 16~2 0' where x m and y m are the horizontal


v cos 0:
and vertical distances travelled respectively.
If v = 800 and 0: = 45, calculate the greatest vertical height of the shell.

the horizontal is y = x tan

0:0 -

19. For the curve whose equation is y

x + 82 , find the values of x for which


x

(a) y < 0,
(b) the gradient is negative.

Calculate the measure of the area enclosed by the curve, the X-axis, and the lines

x
2 and x
4.

20. Find the function f, defined for positive x only, for which f' (x)

(2

1
x

y,

and

f(2) = 6.

21. An arched window with base length 2 m


and height 2 m is to be set into a wall.
The arch is to be either an arc of a
parabola, or a half-cycle of a cosine
curve.
(a) If the arch is the arc of a parabola,
find the equation of the parabola
relative to the axes shown in fig.
C-l.
(b) If the arch is a half-cycle of a cosine
curve, find the equation of the
cosine curve, assuming it has the
form y = a cos bx.

(c) Calculate the area for each window


design, and hence decide which has
the greater area.
y

x
Fig. C-l

REVISION EXERCISES C 449

22. Evaluate:
(3)

J,

7f12

(3 cos 2x - "';2 sin x)dx

7f/4

23. Differentiate:
(i) (x 2 + 3x)e-3x
(iii) (x 2 - 4x + 1)112

(ii) 2e-x cos 3x


(iv) loge(sin 2x)

24. Find the maximum value of 2xe- I >Sx and the value of x for which this function has a
maximum value.
Complete the following table and plot the graph of y = 2xe-1>sx in the domain 0 :E;; x :E;; 1.

0.5

Using Simpson's rule with these three function values, evaluate approximately
2 Jolxe-1>sXdX.

25. The running cost (petrol, oil and maintenance) for a certain make of car when new is
$5.40 per 100 kilometres. This increases by an amount'which is proportional to its age
x years, becoming $6.90 per 100 kilometres at age 3 years. Obtain a formula for the
running cost at age x years.
The 'investment cost' I (dollars) of running the car is the interest which its value
would earn if invested, and this is given approximately by the formula
I
80 - 8x + 0'4x2
per month up to the age of about 10 years. If the car travels 1,000 km a month, obtain
a formula expressing its total cost (i.e. running and investment) per month as a
function of x.
Find (i) the age at which total cost per month is minimum,
(ii) the age at 'which the total cost per month starts to exceed that of a new car of
the same make.
26. A rectangular sheet of metal measures 6 cm by 4 cm. Four equal squares are cut out of
the corners and the sides turned up to form an open rectangular box. Find the length
of the edge of the squares cut, so that the box may have a maximum volume.
27. Prove that the curve y = x2(3 - x) has a horizontal tangent when x = 2 and crosses
the Y-axis at right angles at the origin. Calculate:
(a) the area between the curve y
X2 (3
x) and the X-axis,
(b) the area bounded by the curve y = x2(3 - x) and the lines y = 3 - x, x = 15
and x = 2.
28. A body which is at a higher temperature than its surroundings cools according to
Newton's Law of Cooling,
o = 0oe- kt
where 00 is the original excess of temperature and 0 is the excess of temperature after t
minutes.

dO

(a) Prove that dt

-kO.

'I

, I

450 NEW SENIOR MATHS: TWO UNIT COURSE

(b) A cup of coffee whose original temperature is 95C is placed in a room whose
temperature is 15C and it cools to 68'6C in 10 minutes; calculate the value of k.
(c) Calculate the rate of decrease of 0, when the temperature of the coffee is 40C.
29. The mean value of a function between the values x
1
b - a

(b) If x

fbf(x)dx
a

2 cos 2x + sin 2x between x =

. Find the mean value of f(x)

d 2y
dy
xe 5x , show that dx
- 2 - 10dx + 25y

30. (a) If y

= e- r sin 2/, simplify d

= a and x = b is defined as

+ 2~~ +

and x = ~.

5x

31. Find by integration, the volume of the solid of revolution formed by rotating the
region bounded by
(i) th,e circle X2 + y2 == 1 about the X-axis
(ii) the line y
x + 3, between x
0 and x = 2 about the X-axis
(iii) the parabola y = X2 + 3 between y
4 and y
12 and the Y-axis about the
Y-axis.
32. Write down the derivative of:
(a) cos (x 2 + 2x)
(c) "';x 2 + 2x

(b) loge(x 2 + 2x)


(d) (x 2 + 2x) eX

33. Evaluate:
(a)

f~(eX

'/r/4

-e-X)dx

(b)

(sin 2x - cos.2x)dx

'/r/8

34. Sketch the graphs of y = e- x , y = x + 1 and the line x


the area of the region bounded by them.

= 2 and find, by integration,

35. (a) Find the values of x for which 4x - x 3 > O.


(b) Find the area under the graph of y = 4x - x 3 between x = 1 and x
2.
(c) Find the angle between the X-axis and the tangent to the curve y = 4x x 3 at x
36. Sketch (he graph of y = x 3 (3
x) in the domain -1
the turning points and points of inflexion.

1.

3 giving the coordinates of

37. After 1 years, the value in dollars of a car worth $10,000 when new is given by the
formula

v = Ae-o'2t

(a) Find the value of A.


(b) Find the value of the car after 6 years.
(c) Find the rate in dollars per year at which the car is depreciating when
(i) 1 = 6

(ii) V

5000

(d) How long would it be before it is worth only $1000?

3x 2 - x 3
38. Sketch the graph of y
The tangent to the curve at the point A on the curve where x
another point B. Show that the coordinates of Bare (-1, 4).
Calculate the area enclosed by the graph and the line AB.

= 2 cuts the graph at

REVISION EXERCISES C 451

39. John is in the bush at A 3 km from the


nearest point C on a straight path CB.
He wishes to get to B, 4 km from C, as
quickly as possible. He can run at a
speed of 20 km/h along CB but only at
10.J2 km/h in the bush. He runs in a
straight line through the bush from A to
a point X on the path and then along the
path from X to B.

(a) Find,in terms of x, the time taken to


get from (i) A to X (ii) X to Band
hence the total time t hours to get
from A to B.
40. Sketch the graph of y
by the integral

11 e-

e-x12 , x

xI2 dx.

~-----

8
4

---~

(b) Find the position of the point X for


which t is a minimum and find this
minimum time.

and indicate the region whose area is represented

Evaluate this integral by

(a) direct integration,


(b) the trapezoidal rule using two strips,
(c) Simpson's rule using three function values.
41. The position.x of a particle moving along a straight line at any time t is given by

x = 3

6 cos

1r
f/

(a) Find the position of the particle for values of t = 0, 2, 4, 6, 8, 10, 12.
(b) Find the velocity and acceleration of the particle when it first reaches the position

0.

42. The diagram shows a container, open at the top, whose dimensions in cm are as
indicated. The cross-section AOB is a parabola whose vertex is at the lowest point O.
ABCD is a horizontal rectangle.

(a) Using the axes as. shown, find the equation of the curve AOB.
(b) Find, by integration, the area of the end AOB and hence the total volume of the
container.
43. A haulage company made frequent deliveries from Sydney to Cairns and calculated
that the overhead cost $ C depended on the average speed v km/h according to the
rule
C

= v + 3600

Find the average speed to minimize this cost.

i ~

TEST PAPERS

(Time: 3 hours)

Test paper 1 ............................. .

1. (i) Express as integers


(a) -J5 x "'125

(b) 9 112

x 27- 113

(c) log327

(ii) Express in simplest surd form with a rational denominator -=_2_-=_


(iii) Solve the quadratic equation 5x2

(iv) Evaluate

(a)

(4 dx

Jl X

2x

3.
'/r/3

(b)"

J.

sin x dx

'/r/6

2. (0 For what values of x is Ix - 41 > 2.


(ii) Differentiate with respect to x:
(a)
(b) loge(sin x)
(iii) Find the equation of the normal to the curve y

(c) x cos x

.JX at the point where y =

1.

3. (i) Factorize (a) a 3 + 1


(b) (x - 2)2 - 3 (x - 2)
(ii) The first term of an arithmetic sequence is 2 and the eighth term is four times the
fourth term. Find the common difference.
(iii) If cos ()

~ and 90 < () <


0

180 0 , write down the exact value of

(a) sin ()
(b) cot ()
(iv) Sketch the function f(x) defined by the rule
X
1, x ~ 1
f(x) = { (x - 1)2, x > 1
4. (i) ABCD is a parallelogram and E is a point on the diagonal BD. Through E a line
FG is drawn parallel to AD and it meets AB in F and DC in G.
(a) Prove that6FBE and 6DGE are similar

. FB
BE

(b) Complete the ratIos DG


EG
(ii) Write down the exact values of

(a) cos 210 0


(b) sin 330 0
(c) tan 135 0
(iii) Find the equation of the line through the mid-point of the interval joining
A (3, 5) and B (7, -3) and which is parallel to the line 3y + 2x = 7.
S. (I) A box of matches contains 6 'live' matches and 4 'dead' matches. I withdraw 3
matches (without replacement). What is the probability that:
(a) they are all dead,
(b) at least one is alive,
(c) not more than 2 are alive?
(ii) A man who is colour blind owns three two-piece suits (trousers and coat) each
suit being of a different colour. What is the probability that he will wear a
matching suit if he selects the trousers and the coat separately and at random?

TEST PAPERS 453

6. (i) Sketch the graph of y

5 sin 2

(x - ~) for 0 ..; x..; 2 .., stating its period and

amplitude.
(ii) The function f(x) is defined by the rule f(x) = x2(6 - x) in the domain
-1 E;; x E;; 6.
(a) Sketch the graph of the function after finding its turning points and their

nature.
(b) State the range of f(x).
(c) Find the area of the region bounded by the graph of y

f(x) and the X-axis.

7. (i) The region bounded by the curve y =


and the X-axis is rotated about
the X-axis. Find the volume of the solid generated using
(a) direct integration,
(b) Simpson's rule with 3 function values.
(ii) I f :

2y and y

10 when x

0,

(a) express y as a function of x,

8.

(b) find the value of xwheny

(i) A particle is moving in a straight line. At time

point 0 on the line is given by x =

= 1.

t its position x relative to a fixed

O.

(a) Find its velocity at any time t and show that it is positive for all values of t.
(b) Find its acceleration at any time t and show that it is negative for all values of

t.
t = 0 from the
3.
point x
If its acceleration at time t is 1 + t, find the position of the particle at time
t = 2.

(ii) A particle is moving along the X-axis. It started from rest at time

9. (i) If a and {3 are the roots of the quadratic equation X2


of

+ 3 = 0, find the value

(a) a + {3
(b) a{3
(c) a 2 + {32
(ii) Find the values of m for which the quadratic equation X2 + 4mx + 8 - 4m

=0

has equal roots (one root only).


(iii) Solve the quadratic equation (X2 + 5x)2 - 84 = 8(X2 + 5x).
10. (i) Show that the locus of a pointP (x, y) which moves so that its distance from the
8 is twice its distance from the point (2, 0) is 3x2 + 4y2
48.
line x
(ii) A man borrows $3000 at I!OJo per month reducible interest and pays it off in
equal monthly instalments. What should the instalments be in order to payoff
the loan at the end of 4 years?

454 NEW SENIOR MATHS: TWO UNIT COURSE

Test paper 2 ...............................

1. (i) Solve for x:


(a) x - 2 = 3 (2 - x)
(b) X2 = 3x
(ii) If v 2 = u 2 + 2as, find a when u = -3,
(iii) For what values of x is:
(a) X2 > 3x
(b) Ixl < 5

(c) Ixl

v = 8, s

10.
(c) 3x

+2

3!

(iv) Simplify:
(a) 2a - b _ a

+b

(b)

x-

....:...
X
X2 -

+1
3x - 4

2. (i) Complete the following statements:


(a) The diagonals of a parallelogram ....
(b) The sum of the exterior angles of a polygon is ....
(c) The straight fine joining the mid points of two sides of a triangle is ....
(ii) In the figure, AB = BC, AD II BC, (iii) In DABC, LA = 30, L B = 105,
LADC
90 and LABC = 80.
BC = 2 cm.
Calculate the size of I ACD.
Calculate the exact length of AB.
A

3. (i) Find the centre and radius of the circle X2 + y2 - 2x + 4y - 4 = O.


(ii) Indicate the region of the cartesian plane determined by the inequations:
(x
1)2 + y2 < 1, Y ~ x, Y ~ 0
(iii) Find the co-ordinates of the vertices of a triangle whose sides have equation
y = x + 3, y = 3 - x and x = 3.
Find also the area of this triangle.
4. (i) (a) A clerk is employed at an initial salary of $8320 per annum and after each
year of service he receives an increase of $20 per week. What is his salary in
the tenth year of service and what will be his total earnings for the first ten
years?
(b) Another clerk had the same initial salary of $8320 per annum and after each
year of service his salary increased by 10070 of the salary of the preceding
year. Calculate his salary in the tenth year and his total earnings for the first
ten years.
(ii) The sum of the first four terms of a geometric sequence is 15 and the sum of an
infinite number of terms is 16. Find the first term and the common ratio.
S. (i) Write down the derivative of:
(c)

(a) xe X

- 3 andJ(O) = 2. Sketch the graph ofJ


specifying the co-ordinates of the turning points and the intercepts with the axes.

(ii) Find the functionJ such thatJ' (x) = 3x 2

(iii) Evaluate

places.

(a)

fl21

x dx

(b)

Iol e-

x12

giving your answers to 3 decimal

TEST PAPERS 455

6. (i) Of the students who sat for a particular mathematics examination, 60 per cent
were science students and 40 per cent were non-science students. The
examination was passed by 85 per cent of the science students and by 60 per cent
of the non-science students.
Find the overall percentage who passed the examination. (Illustrate with a tree
diagram).
.
(ii) If P(A)
0'6, P(B)
0-2 and P(AB) = 0-3 find Pr(A U B)
(iii) A pair of dice is thrown. What is the probability that they show a total of seven?
7. (i) A spherical mothball evaporates and its radius decreases at a constant rate. If the
initial radius of the mothball is 1 em and the radius is
the radius rem at.any time t weeks.
(ii) A variable line PQ through the

point A (1, 2) intersects the X-axis


at P(u, 0) and the Y-axis at Q(O, v).

(a) Write down the gradients of PA

and QA.

(b) Hence, express v in terms of u.


(c) Write down the area of t:.,POQ

in terms of u.

(d) Find the value of u for which

the area of t:.,POQ is a

minimum.

1em after ten weeks, find

y
Q

P(U,o)

8. (i) Find the equation of the locus of a point P(x, y) which moves such that:
(a) it is equidistant from the axes,
(b) the sum of the squares of its distances from the axes is 16_
(ii) (a) Sketch the parabola 8y = X2 stating the co-ordinates of the focus.
(b) Find the equation of the focal chord PQ from the point P(4, 2).
(c) Write down the co-ordinates of Q and the equation of tangent to the
parabola at this point.
9. (i) For what values of m does the quadratic equation mx 2 - 4x + m = 0 have
(a) real roots
(b) one root only and find this root.
(ii) From a point P on the horizontal

ground, the angle of elevation of

the top of a building AB is 25 At a

point Q, 100 metres closer, and in


line with the bottom of the
building, the angle of elevation is
40 Calculate the height of the
P 100m
Q
B
building.
0

10.

(i) In a certain mining town, the population in 1970 was 10,000 and in 1980 it was

8,190. If P(t) is the

po~ulation in

any year after 1970 and if ~P(t)

= -kP(t),

find the value of k and hence predict the population in 1990.


(ii) A particle moves in a straight line. At time t seconds its distance x metres from a

fixed point 0 in the line is given by x

2 + 2 sin

it.

(a) Find its original position.


(b) What is its velocity after 3 seconds?
(c) Find its position when its velocity is zero for the first time.

456 NEW SENIOR MATHS: TWO UNIT COURSE

Test paper 3 ............................. .

1. (i) Express as a single fraction in simplest form:

1 + ~
4 1 1

(a) 1 ~ ~ : ~
(b) ..)5 + 2 + 4-J5 - 9
(c) -v' (05)2 + 02)2
(ii) If a = 4 2 x 10 5 and b = 31 x 10-2 evaluate a 2 b 3 in scientific notation correct
to 2 significant figures.
(iii) Find the exact value of a 4 + 2a 2 + 1 when a = 3.J2.
(iv) Factorize:

(b) 9a 2 -
(a) a 2 - b 2 + 2a - 2b
4

2.

2x + 3

(I) Solve for x: 5x _ 1

2x +
= 5x - 5

(ii) Find the values of x for which:


(a) 5x + 2 < 4~
(b) 2 ~ log2x ~ 4
(iii) If two angles of a pentagon (5 sides) are 92 0 and 100 0 and the other three angles
A
are equal, find the size of each of the equal angles.
(iv) ABC is an isosceles triangle with

AB = AC. BX is perpendicular to

A C and C Y is perpendicular to AB.

BX and CY intersect at o. Prove

that

(a) LBCX and


L CB Yare

congruent,

(b) LA 0 Y and
LA OX are

congruent,

(c) AO bisects L BAC.


8
C
3. (i) From a set of cards numbered 1, 2, 3, ... 10 a card is selected at random. What
is the probability that the number on it is even or less than 5?
(ii) A bag contains a large number of marbles, 40070 of which are red and the
remainder blue. If three marbles are selected, find the probability that
(a) all are red,
(b) at least one is blue,
(c) red, blue, red are drawn in that order.

4. (i) State the largest possible domain that the following functions can have:
(a) f(x)

= 1 - X2

(b) f(x) =

.JT--=--:xz

(c) f(x)

(d) f(x)

1
1 - X2

10geO - X2)

(ii) Find the locus of a point P(x, y) which is


(a) r units from the point (a, 0),
(b) equidistant from (a, 0) and (-a, 0),
(c) twice as far from (a, 0) as it is from (-a, 0),
(d) equidistant from (a, 0) and the ~. ~ -~~~5.
5. (i) Write down the exact values of:
(a) cos 300 0
(b) tan 210 0
(c) cosec 150 0
0
0
(ii) Find all values of x between 0 and 360 for which:
(a) sin x = 1
(b) -J3 tan x + 1 = 0

TEST PAPERS 457

(iii) A pendulum is 90 cm long and its bob swings through an arc 20 cm long. Find:

(a) in degrees and minutes the angle through which it swings,


(b) the shortest length between the extreme positions of the bob.
6. (i) An observer moving along an E-W road observes a monument 60 0 E of N. On
moving 100 m to the east, it is now 45E of N. Calculate to the nearest metre:
(a) its distance from the observer at the second observation,
(b) its distance from the road.
(ii) Differentiate with respect to x:

x-I

+3

(c) 2x
(iii) Find an indefinite integral (primitive function) of

(a) ...,-------".

(b) 2 sin x

sin 2x

(c) e3x

7. (i) Show that (8, 4), (6, 6) and (0, -2) are the vertices of an isosceles triangle.
(ii) Find the equation of the line passing through the point (3, -2) and the point of
intersection of the lines 2x + 3y = 4 and 2x + y
O.
(iii) A, Band C are the points (2, 0), (8, 4) and (4, 6) respectively. Find the co
ordinates of D such that ABCD is a parallelogram.
8. (i) Find the equation of the tangent to the parabola y = 2X2 + 5x - 3 at the points
where it crosses the X-axis.
(ii) Find, using two different methods, the minimum value of X2 + 8x + 9.
(iii) Find the values of m for which 15 + 2m
m 2 > O.
9. (i) Sketch the curve y
2x 2 - x 3 after finding its turning points and their nature.
Find also the area bounded by the curve and the X-axis.
(ii) Find the equation of the curve y = f(x) given that

~~

6x - 4 and :

= 4 and

= 2 when x = 1.

(iii) Sketch the curves y

ex, y = e- and the line x = 3 and find the area enclosed.


X

10. (i) Find the area of the region bounded by the graph of y = logex, the coordinate
axes and the line y = loge2.
Find also the volume generated by rotating this region about the Y-axis.
(ii) A body moves so that its velocity v km/ h at time t hours is given by:

v = 80 + (2t
l)e-005t

(a) Find its initial velocity i.e. when t = O.


(b) Find the time required to reach its maximum velocity and find this velocity.
(c) Find its initial acceleration.

.458 NEW SENIOR MATHS: TWO UNIT COURSE

Test paper 4 ............................. .

1. (i) Express in its simplest form:


(a)

1.2+3
~ - ~
5

(b) a:~ where


(ii) (a) Simplify

(b) Express

a= (~y,

=( iJ, c =( ~r2

.J32 + ..J8

~ ~ ~ in simplest form with rational denominator.

(iii) Express each of the following in terms of 0 only:


(a) cos (180 0 - 0)
(b) tan (360 0
0
(c) sin (90 + 0)
(d) sec (90 0 -

0)
0)

2. (i) Solve the equations:


(a) x(x - 2) = 2 (Leave answer in surd form)
(b) log2x + log2x2 = 6

(c) 32x
- 123 x + 27 = 0
(ii) log 4 + log 8 + log 16 + log 32 = a log 2. Find the value of a.
(iii) What is the equation of the circle which touches the X-axis at (1, 0) and the
Y-axis at (0, 1)?

I' X2 - 4
3 (I') F'In d x~x
_ 2'
(ii) Differentiate (a)

~2

(c) sin 2x

(b) e 3x

(iii) Write down primitive functions for (a), (b) and (c) of (ii).

(iv) Express in terms of 7f' the area of a sector of 30 0 cut from a circle of radius 12 cm.
4. (i) Evaluate
(a)

r'

(x

(b)

+ 2),dx

2(X

2)2 dx

(ii) Evaluate f24X logex dx using Simpson's rule with three function values (Answer

to 3

signifi~ant

figures).

(iii) Express the recurring decimal

5.

032 in the form ~ where a and b are integers.

(i) Find the equation of the normal to the curve y

= x 3 + X2 + X - 2 at the point

where x = -2.

+ X2 + X -

2 and y = x 3
(a) intersect
(b) have the same gradient?
(iii) A ball, dropped from a height of 32 m, rebounds each time it strikes the ground
one-half the height from which it falls.
(a) How high will it rise on the sixth rebound?
(b) How far will it have travelled before coming to rest?
(ii) For what values of x do the graphs of y = x 3

6. (i) For what values of x is (x - 2)2(X - 1) > O?


(ii) Find the least value of x
(iii) Sketch the graph of y =

+ x1. for x >

~x +

O.

2 for -2

~x ~

2, giving the coordinates of its

TEST PAPERS 459

end points A and B. The region enclosed by this line, the X-axis and the ordinates

x = -2 and x = 2 is rotated about the X-axis, so generating part of a solid cone.


Calculate the volume of this solid.
7. (i) Find the sum of the first n odd numbers, that is, 1 + 3 + 5 + ... to n terms.
(ii) A person takes out a personal loan of $3000 and is charged interest at the rate of

I!lJfo per month on any money outstanding. If he pays back $500 at the end of

each month, calculate the amount owing immediately after:

(a) the first repayment,


(b) the second repayment,
(c) the sixth repayment.
.
6 3n
8. (i) For what value of n is

9n + I

I?

(ii) For what values of x is


(a) (2x + 3) (x - 2) > 0
(b) 5 + 4x - X2 > O?
(iii) Find the values of m for which the quadratic equationx2 - (m - 3)x - (2m - 3)

=0

has real roots.

9. (i) ABC is a triangle right-angled at B. 0 is a point on AC such that AO = OB.

Prove that OC
OB.

(ii) A sealed tin rectangular box is to

have a square base and a volume of

hem

64 cm 3 If the length of the edge of


the base is x cm,

(a) express the height h in terms of x,


(b) show that the total surface area
. . b yy -256 + 2x,
2
ycm 2 lsglven
x
x em
(c) find the area of the smallest

amount of tin necessary to

fulfil these specifications.

10. (i) The acceleration of a particle moving in a straight line is given after t seconds by

x = -2t. Initially the particle is located at x

0 and its velocity is 4 m/s.


(a) Find the velocity v and displacement x at any time .

. (b) Find x when v = O.

(c) Find v when the particle returns to the origin.


(ii) The rate of increase in the number N of bacteria in a colony after t hours is

proportional to the number N present.

(a) If initially there are 1000 and 2 hours later there are 1080, find the growth
rate.
(b) If they continue to multiply at the same rate, how many would there be in a
further 2 hours?

iI

460 NEW SENIOR MATHS: TWO UNIT COURSE

Test paper 5 ............................. .

1.

(i) Find the exact value of:


(a) (81 )-3/4

(ii) If 10 112

(b)

x 100 4

( 8"1)113

x (0'1)-2

1000-5/6 = lOx, find the value of x.

(iii) If x = -=-_1_= find the value of x 2 as a surd with rational denominator.

2. (i) The second term of a geometric sequence is 13! and the fifth term is 4. Find the
common ratio and the first term.
Find the sum of n terms and hence find the sum to infinity
(ii) Find the number of terms of the series -2 + 1 + 4 + ... which add to 115.
3.

(i) Factorize
(a) 16x4y

+ 2xy

(b) a 2 - b 2 - 3a
3b

sin 3 ()
(II) Prove that
+ sm () cos () tan ().
cos
(iii) From a point A on the ground 50 metres from the foot of a building, the angle of
elevation of the top of the building is 30 and the angle of elevation of the top of
a vertical tower on top of the building is 45. Find, in simplest surd form, the
height of the tower.

..

4. (i) ABCD is a square. EAD and FBC are equilateral triangles drawn so that E and F
are outside the square. Prove that
(a) LABF and LEDC are congruent,
(b) AF = EC,
(c) AFCE is a parallelogram.
(ii) Solve graphically the equation cos 2x = x, correct to 2 decimal places.
S. (i) Find the equation of the locus of a point which moves in such a way that its
distance from the point (-2, 3) is twice its distance from the point (l, -1). Show
that the locus is a circle. Find its radius length and the coordinates of its centre.
(ii) For the parabola X2
4y
(a) State the co-ordinates of the focus and the equation of the directrix.
(b) Find the equation of the tangent at the point on the parabola where x
4.
(c) Find the volume of the solid generated when the region bounded by the
parabola, the Y-axis and the lines y = 1 and y = 9 is rotated about the
Y-axis.
6.

(i) Evaluate:
(a)

r'

(x2I'

xll3)dx

(b)

[5 dx

J2

(c)

(ii) Find the area of the triangle enclosed by the lines y

J"

,,/2

sinxdx

2x,y = -:2and4y

3x + 10.

Show that it is a right-angled triangle and find the co-ordinates of its vertices.
7.

(i) Find the distance of the point (2, 3) from the line 7x
24y + 8
(ii) Illustrate with a sketch the region of the plane represented by:

+ y + 1 ;at. 0, x - y + 3 ~ 0, 7x y + 3
(b) ((x, y): y > X2 + 11 n {(x, y): y ~ 2x + 11

(a) x

;at.

O.

TEST PAPERS 461

8. (i) Sketch the graph of y = 4 + 3x2 - X4 after finding


(a) the turning points and the points of inflexion
(b) the points where it intersects the axes.
(ii) Find also the area A enclosed by the curve and the X-axis.
(iii) Find also the equation of a parabola whose axis is parallel to the Y-axis which
passes through the points where the above graph cuts the X-axis and for which
the area of the region enclosed by it and the X-axis is equal to A.
9. (i) A box contains three coins, one'of which is 'double-headed'.
(a) If a coin is selected at random and tossed, what is the probability that it will
fall heads?
(b) If a coin is selected at random and tossed twice, what is the probability it will
fall heads twice?
(ii) A box contains six red and four blue balls. Find the probability of obtaining two
balls the same colour in a sample of two if:
(a) the sample is drawn without replacement,
(b) the sample is drawn with replacement.
10.

(i) The area under the curve y

= x1 between
x = 1 and x = a is 1 square unit. What is

the value of a?

(ii) The radioactive element, radium, decomposes and the amount Q present at any
time t years is given by

Q = Qo e-O
'OOO5t

where Qo is the original amount.


(a) How long would it take for 10 per cent to decompose?
(b) What percentage of the original amount will remain after 1000 years?

SUMMARY
Symbols and abbreviations
E

n
N
J
Q
R

<
>
::::: or
II
l

III

-J
(x, y)

f(x)
Ixl
[x]
[a, b]

notation for a set


is an element of
union of
intersection
set of natural numbers
set of integers
set of rational numbers
set of real numbers
therefore
because
is equivalent to
is equal to
is less than
is great than
is approximately equal to
is parallel to
is perpendicular to
is congruent to
is similar to
positive square root of
ordered pair; cartesian
co-ordinates of a point
such that
value of a function at x
absolute value of x
greatest integer not
exceeding x
closed interval a to b,
a:1!!;.x:1!!;.b

(a, b)

(a, b]
D

m
lim
limf(x)
x-a

open interval a to b,
a < x < b; co-ordinates of
a point
half-open interval a < x :1!!;. b
discriminant, b 2 - 4ac
triangle
gradient
limit
limit of value of f as x
approaches a

h or ox
k or oy
f'(x)

increment in x
increment in y

r1mf(x +

h-O

h) - f(x)
h

dy
dx

r1m
oy
ox-oox

sum of

J.bf(X)dX

integral of f from a to b

probability of event A
complement of event A
P(A U B) probability of A or B or
both
probability of A and B
P(AB)

P(BIA)
probability of B given A
nth term of a sequence
tn

sum of first n terms of a


Sn

sequence
00
infinity
second
s
h
hour
centimetre
cm
m
metre
km
kilometre
displacement
x
I . dx
v
ve OClty; dt' x
P(A)

acceleration;

dv d 2x
d't d'
t2

SUMMARY 463

Geometry
1. If two straight lines intersect,
(i) the opposite angles are equal,

(ii) the adjacent angles are supplementary.

2. If two straight lines are parallel and are intersected by a transversal,


(i) corresponding angles are equal,
(ii) alternate angles are equal.
3. The sum of the angles of a triangle is 180 0
4. An exterior angle of a triangle is equal to the sum of the two remote interior angles.
5. The sum of the interior angles of a polygon with n sides is (2n - 4) x 90 0
6. The sum of the exterior angles of a polygon is 360 0
7. A parallelogram is a quadrilateral with both pairs of opposite sides parallel. Its
opposite sides and angles are equal. Its diagonals bisect each other.
8. A rectangle is a parallelogram with one angle a right angle. Its diagonals are equal.
9. A rhombus is a parallelogram with adjacent sides equal. Its diagonals bisect each
other at right angles.
10. A square is a< rhombus with one angle a right angle.
11. A trapezium is a quadrilateral with one pair of opposite sides parallel.
12. Two triangles are congruent if:
(a) two sides and the included angle of one triangle are respectively equal to two sides
and the included angle of the other triangle (SAS).
(b) the three sides of one triangle are respectively equal to the three sides of the other
triangle (SSS).
(c) two angles and a side of one triangle are respectively equal to two angles and the
corresponding side of the other triangle (ASA).
(d) the hypotenuse and one side of a right-angled triangle are equal to the hypotenuse
and the corresponding side of the other right-angled triangle (RHS).
13. If the intercepts cut off on one transversal by three (or more) parallel lines are equal,
the intercepts cut off by them on any other transversal are also equal.
14. The straight line joining the mid-points of two sides of a triangle is:
(a) parallel to the third side
(b) one half of the length of the third side.
15. Two triangles are similar if:
(a) the angles of one triangle are respectively equal to the angles of the other triangle
(b) the corresponding sides are proportional
(c) two pairs of corresponding sides are proportional and their included angles are
equal.

16. Theorem 0/ Pythagoras: The square on the hypotenuse of a right-angled triangle is


equal to the sum of the squares on the other two sides.
Conversely: If the square on a side of a triangle is equal to the sum of the squares on
the other two sides, the angle contained by these sides must be a right angle.

464 NEW SENIOR MATHS: TWO UNIT COURSE


I

Metric measurement _____________

, I

(a) Length
(b) Area
10 millimetres (mm) = 1 centimetre (cm)
100 cm
1 metre (m)
1000 m = 1 kilometre (km)
(~)

Volume

104 cm 2
1 m2
4
2
10 m
hectare (ha)
100 ha = 1 km2

(d) Liquid volume


1000 millilitres (ml)
1000 I

(e) Mass

1 litre (I)
1 kilolitre (kl)

(f) Temperature

1000 grams (g)

1 kilogram (kg)

Fahrenheit to Celsius: C

1000 kg

1 tonne (t)

Celsius to Fahrenheit: F

-(F - 32)
9
=

~C +

32

Mensuration _ _ _ _ _ _ _ _ _ _ _ _ _ _ __
(a) Right-angled triangle

Theorem of Pythagoras: c 2
sin A

= .'!.,
c

~,

cos A

cosec A

= a'

tan A

sin A
cos A

sec A

a2 + b 2

tan A

= b'

a
b

b
a

cot A

sin 2 A + cos 2 A
1 + tan 2 A = sec 2 A
1 + cot 2 A = cosec 2 A

30

45

60

90

sin

1
2

1
~

-J3
-

cos

-J3
-

1
2

tan

-J3

sin A
cos A

= cos B =
= sin B =

I
cos (90 - A)
sin (90 - A)

. r

SUMMARY 465

(b) Scalene triangle


Cosine rule: a 2 = b 2 + e 2 - 2be cos A
S'
I
abc
me ru e: sin A = sin B = sin C
Area =

~bP

~be sin A

= ~ product of two sides

and the sin of the

included angle.

(c) Circle

Arc length: I = rO
Area of sector: A

Circumference: C = 27rr

Area of circle: A = 7rr2

Area of annulus: A = 7r(R2 - r2)

= 'ir20

Area of segment: A = !r2( 0 - sin 0)


2
(d) Quadrilaterals
Area of rectangle = lb
Perimeter of rectangle = 2 (l
Area of trapezium

= &(a +

Area of parallelogram

b)

b)h

bh
b

(e) Miscellaneous areas and volumes


V denotes volume, C denotes curved surface area and S denotes total surface area.
Cylinder: V = 7rr2h, C
27rrh, S = 27rr2 + 27rrh
Cylindrical pipe: V = 7r(R2 - r2)h

~h

Cone: V = 37rr2h

Sphere: V = 37rr3,
47rr2
4 C = S
Cuboid: V abc, S = 2(ab + be + ea)
Pyramid: V

~abh

&

C"", -_-r-I~~~~~~~c~IJ-"

&

b
B
Algebra _ _ _ _ _ _ _ _
_ _ _ _ _ _ __

(a) Factors
1. common term
: ab + ae = a(b + c)

b 2
= (a - b)(a + b)
2. difference of two squares: a2
2
3. sum of two squares
: a + b 2 (no real factors)

: a 3 - b 3
= (a - b)(a 2 + ab + b 2 )
4. difference of 2 cubes
5. sum of 2 cubes
: a 3 + b 3 = (a + b)(a 2 - ab + b 2 )

466 NEW SENIOR MATHS: TWO UNIT COURSE

(b) Expansions

1. a(b

c)

2. (a - b)(a
3. (a + b)(c
4. (a + b)2
5. (a - b)2

+ b)
+ d)

6. (a + b)3
7. (a - b)3

= ab + ac (distributive law)

= a2
b 2

= ac + ad + bc + bd
a 2 + 2ab + b 2
= a 2 - 2ab + b 2
= a 3 + 3a 2 b + 3ab 2 + b 3

= a 3 3a 2b + 3ab 2 b 3

(c) Quadratic equation

ax2 + bx + c = 0 has roots ex and {3 where

= -

1. ex

and {3

- b -

~~2

4ac.

2. ex + {3 =
c
ti;

3. ex{3 =

4. b 2 - 4ac > 0 two distinct roots,

b 2
- 4ac = 0 two equal roots,
b 2 - 4ac < 0 no real roots.
a m+ n
a m- n

am X an
am + an
(am)n

Logarithms (a > 0, a =1= 1)


= x ~ n :;;: log a X
log a X + loga Y = log a xy; x, Y >0

(e)

(d) Indices

an

a mn

loga X

aO = 1

a p1q

:;;:

logo 1
1
logOx
-

({fa(

10g b a

0
-loga x,

> 0

b
1, b > 0
log x
logax = l-_b- (change of base)
X logo

ogb a

(f) Absolute value

.JXi

loga -; X, Y > 0
Y
p logo X; X > 0

10gaY

=
log a a = 1
log a xP

.1
a- n =
an
an X b n
(ab)n
a llq

Ixl:;;: { ~, x

;;:r. 0

x, x < 0

Ix I

x2

XI -

x 2,

XI

;;:r.

X2

X2 -

XI' XI

<

X2

(g) Sequences and series


Arithmetic sequence: a, a + d, a
nth term: tn
a + (n - l)d

Sum of first n terms: Sn = ~[2a

+
+

2d,

(n

l)d]

If a, b, c are successive terms of an arithmetic sequence then 2b


Geometric sequence: a, ar, ar2 ,
nth term: tn
arn - 1
a(r n - 1)
Sum of first n terms: Sn
r _ 1

Sum to infinity: So> :;;: - a1 if


-r

Irl <

a(l - rn)
1 - r ;r=l=1
1

If a, b, c are successive terms in a geometric sequence then b 2

Compound interest: A = PRn where R = 1 + 100

ac

c.

SUMMARY 467

Coordinate geometry _ _ _ _ _ _ _ _ _ _-
(a) Straight line
A(x l , YI) and B(x 2, Y2) are any two points.
Y2 - YI
-+
Gradient: tan v = m = X - x I ,Xl""" X 2
fj

YI = m(x - XI)' gradient form


Y = mx + c, intercept form

Distance:
AB = .../(X2 - x l )2 + (Y2 - YI)2

Perpendicular distance of a point (XI' YI) from a line ax


Equation: Y

lax i +

bYI

by

+c

+ cl

.../(a 2 + b 2 )

Coordinates of mid-point of AB:

(Xl; X"

Yl ; y, )

Pairs of lines

Gradients are m l and m2

Parallel lines
: mI = m2

Perpendicular lines: m l m 2 = -1

The pair of lines alx + bly + c. = 0 and a 2x


(1) intersect if

a.
a2

* bb z

(2) are parallel if

a)

bl

(4) coincide if

al
bl
a2 = bz =

blY

blY

+ c z)

C2

*S

a2
bz
(3) are perpendicular if a l a2 = -b l b 2

Cz

C
z
Any line through their intersection has equation

a IX

b IY

+ cI +

k(a zX

= 0

(b) Circle

A circle may be defined as the locus of a point P(x, y) which moves in a plane so that
its distance r (radius) from a fixed point (the centre) is constant.
Equation:
x 2 + yZ = r Z, centre (0, 0), radius r

(x - h)Z + (y - k)2 = rZ, centre (h, k), radius r


, . . - - : : - - - - = - : : - -
X Z + yZ + 2ax + 2by + c = 0, centre (-a, -b), radius

(c) Parabola
A parabola may be defined as the locus of a point P(x, y) which moves in a plane so
that its distance from a fixed point (the focus) is equal to its distance from a fixed
straight line (the directrix).
Equation
(x

x 2 = 4ay
.h)2 = 4a(y k)

Vertex

Focus

Directrix

Axis'

Focal length

(0,0)
(h, k)

(0, a)
(h, k + a)

y = -a
y=k-a

x=O
x=h

a units
a units

The focal length is the distance from the vertex to the focus

468 NEW SENIOR MATHS: TWO UNIT COURSE

Probability _ _ _ _ _ _ _ _ _ _ _ _ _ _ __
number of outcomes favourable
to A 'f the ou t comes are equaIIy I'k
I
.'
. .- - 1
ley.
total number of possIble . outcomes '
P(A) = 1
P(A)

P(A U B)
P(A) + P(B) - P(AB)

= P(A) + P(B) if A and B are mutually exclusive

P(AB) = P(A).P(BiA)

P(A). P(B) if A and B are independent

P(A)

-_..

Function __________________
Domain is the set of real numbers xon which f is defined.

Range is the set of values of J(x) as x varies over the domain.

Even function : f(x) = f( - x)

Odd function: f(x)


-.t( x)
Trigonometric functions
sin (-0) = -sin 0
COS

sin 2 0, +. cos 2 0 = 1
sec 2 8
1 + tan 2 0
2
1. + cot 8 = cosec 2 8

(-8) = cos 8

sin (27r
8)
-sin 8
cos (27r - 8) = cos 8
tan(27r .- 8)
-tan 8

sin 8
sin (7r - 8)
cos (7r - 8) = -COS 8
tan (7r - 8) = - tan 8

sin

(~

tan

( - ,

8) cos 8
cos ( - 8) = sin 8
=

cot 8

sin (7r + 8) = -sin 8


cos (7r + 8)
-cos 8
tan (7r + 8)
tan 8

Functions take positive values in quadrants shown:

0
sin

cos

tan

7r

7r

7r
3

1
2
~
2
1
~

7r
2

-J2

~
2
1
2

7r

Calculus _ _ _ _ _ _ _ _ _ _ _ _ _ _ _ __
(a) Derivative

f '(x)

hl~

f(x

dy = lim

dx

+
.

oy

Ox-Oox

h) - f(x)

. I

SUMMARY 469

(b) Rules for differentiation


d
(i) Product rule : dx(uv)

du
v. dx

dv
u. dx

du

(") Q '
II

d (
uottent ru e : dx

vu) v. (]X =

dv

(]X

. dy _ dy du
. dx - du' dx

(iii) Chain rule

(c) Continuity
A function f is continuous at a point x = a if
(i) the function has a definite value f(a) at x
(ii) limf(x) = f(a)

x-a

(d) Approximate Integration


(i) Simpson's rule:

b- 6 -a{f(a) + 4f\-2(a + b) + f(b) }for 3 functIOn


. values.
J. f(x)dx
Trapezoidal rule:
a

(ii)

Ja(bf(x)dx

b-a{ f(a)

( a ++
bf(b)
) } for 2 equal sub-mtervals.
.
2f\-2-

(e) Kinematics
~

Position

Velocity

.5!

f(t)
=

dx
.

dt = x

I-<

~ Acceleration
;a

I(x)

dv

d 2x

..

a = dt = dt 2 =

J'(x)

jl(X)dX

a(constant)

ax

xn

nxn- I

_1_xn+' + en*- -1
n + 1
'
1
a(n + 1) (ax + b)n+' + c, n

(ax

b)n

na(ax

logex

1
x

1
x

-Xl

loge hex)

h '(x)
hex)

b)n-'

h'(x)
hex)

loge x

+C

loge hex)

ekx

ke kx

!e kX
k

sin kx

k cos kx

_!cos kx
k

coskx

-k sin kx

ksm kx

tan kx

. k sec 2 kx

1 .

+C

+c

1
-klog,., (cos kx)

*- -1

470 NEW SENIOR MATHS: TWO UNIT COURSE

Somey basic curves

Parabola: x 2 = 4ay

Absolute value: y

Pair of lines: x 2

I xI

y2

(-r.o)
(-f.O)

(f.O)

( - f,O)

(f ,0)

(r ,0) X

(O,-f)

Circle: x + y2

Semi-circle: y

\0

Semi-circle: y ==

Positive square root curve

y = .Jx, x ~ 0

Il

.\
Rectangular hyperbola

= x' x

y = x

:t= 0

x'1 x

o
Logarithmic CUfve
y == loge x, x > 0

X
Exponential curve
y = e'

Exponential curve
y = eX

11'1

211' X

21
1

I
I
I

Sine curve
y
sin x, x ~

211"

Cosine curve

y = cos x, 0 ~

211"

Tangent curve
y = tan x, 0 ~

x ~ 211",

X =1=

~, 311"
2

ANSWERS

CHAPTER 1
EXERCISES 1(a)

1. (a) (i) 0-6


(d) (i) 0-025
(g) (i) 00625
G) (i) 0'58

60070
2'5%
6'25%
58%

(b) (i)
(e) (i)
(h) (i)
(k) (i)

0-875
0'3125
0-09
36

(ii)
(ii)
(ii)
(ii)

87-5%
31'25%
9%
360%

116

(ii) 185%

0-72
0'65
0'175
4'25

(ii)
(ii)
(ii)
(ii)

(c) (i)

(ii) 37.5%

(c)
(f)
(i)
(I)

(i)
(i)
(i)
(i)

72%
65%
17'5%
425%

(ii) 25%

(b) (i)

(d) (i)

12~

(ii) 23%

(e) (i) 1~

(ii) 175%

(f) (i)

!~

(ii) 32.5%

(g) (i)

;~

(ii) 84%

(h) (i)

4~

(ii) 2'5%

(i) (i)

:z

(ii) 87.5%

G)

:~

(ii) 61.25%

(k) (i) 215

(ii) 265%

(I) (i) 1

C)
2
II 5

(b) (i) 0-01

(ii) 100

(c) (i) 0-1

C)
1
II 10

C) 13

(e) (i) 0-15

(ii)

(f) (i) 0-65

C) 13
II

20

(h) (i) 0-825

C) 33

(i) (i) 1-25

(ii)

I!

(k) (i) 0'144

C) 18

(I) (i) 1-125

(ii)

lk

2. (a) (i)

(i)

3. (a) (i) 0-4


(d) (i) 0-325

4_

(ii)
(ii)
(ii)
(ii)

40

II

(g) (i) 0'075

(ii) 40

ij) (i) 1'16

(ii)

Fraction

In

Decimal

7
20

27
200

28%

Percentage

0'3

30%

0-135
45%
0-475

400

3375%

0-0825

0-4

III
20

97-5%
0-08
2'92

172%
65%
5
8
102-5%
17-5%

1;25

40%

1'55

40

0-025

0-03125

47'5%

110%

0-85

0'175

(ii) 182%

Decimal

20

8%

0'65

1
32

Fraction

I'
:1
I

0-125

1~
25

1
40
1
14 0

125

70%

0-3375
17
20

212'5%

0-32

0'28
1
8
2
25

II

40

35%
2-125

8
25
7
10

Percentage

II

20

8%
62'5%

2'875
1-6

287-5%
302%

472 NEW SENIOR MATHS: TWO UNIT COURSE

EXERCISES 1(b)
5 2 7 4 2
1. 9' 3' 12' 11' 13' 15

2. 3

0-666. ..

4
11
5

5
12

= 0~83

0'833 ..

.
5
.
3. 0-5 = 9' 0-17

0-7

= 0-13

0'1333 .. -

13 = 0-692307 692 3 _..

.
0-416

04166 .. ,

0-777 ...

15 =

0-36

0-3636 ...

6=

~ =

0'6

11

18

== 0'6111 ...

8
..
68
.
45' 0-68 = 99' 0'8

0'692307

= 0'61
19

9' 0'21

..

= 90' 0'51

17
.
7
..
33' 0-583 = 12' 1'37

3~ci, 2-136

3-58
4. ~

0-285 714,

= 0-428 57i,

~=

0'571428,

0-714285,

= 0'857142. Same digits in

each and in the same cyclic order but starting with a different digit.
3
5. 13 = 0-i53846, 13
0-230769, 13 = Oj07692, 13 = 0-384615, 1~
-8
13 = 0'538461, 13

. . .
-10
-11
0'615384, 13 = 0'692307, 13 =0'769230, 13

0-461538,

0'846153,

12 = 0-923076
13
7. yes
EXERCISES 1(e)

1. 12
5. 24
9. 84
13. 60
17. 84
21. 120

2. 60
6. 30
10. 60
14. 60
18. 42
22.42

3.42
7. 60
11. 168
15. 78
19. 99
23.24

4.
8.
12.
16.
20.
24.

60
66
105
210
120
60

EXERCISES 1(d)

3
1. 4 =
5
2. 8" =

9
12
10
16

3. = 30
7
35

15 _ 30 _
20 - 40 15
25
24 = 40 =

42
49

4. 24 = ~
60
20
5. 12
18

6. (i)

66_

77 - 63

A- = 120
10

~ =
9

(ii)

13 17 7
7. 16' 20' S13 7 5 11
10. 18' 9' 6' 12
13 5 1
. 33' 6' 11 22

21
28

45

72

= ~

300

= ~ = 60
126

(iii)

90

(iv) 11

(v)

13 3 7

8.

18' 4' 9

17 13 9
11. 20' 15' 10
2 19 4 7
14. 5' 45' 9' 15

13
18

(vi)

( .. ) 2
VII

9
9. 14'
7
12. 16'
3
15. 10'

(viii) 11

5 3

7' 4
31 1 17
64' 2' 32
8 33 2
25' 100' 5

ANSWERS 473

EXERCISES 1(e)

1. Ii

2. 12

3. 24

5. 4~

6. 9:~

7. 1 1
9
6

5
9. 24

10.

3
13. 16

14.

17.

11

8
36

11. It

12. 24

23

90

15.

8. 20

16. 5~

18. 4/6

EXERCISES 1(f)

1
1. 5"

2.

31

3. 1

5. 1~

6. 1

24

7. 35

10.

9. 15

13.

15

17. q~

92

14. 2!

11.

2:

15. 1

12. 45

16. 2!

18. 2*

EXERCISES 1(g)
1. 26

2. 43

3. 223

4. 11

5. 2~~

6. 2!

7. 13

8.

9. ~

10.

11. 25

12. 2i~

3
13. 28

14. 1

15 16
. 65

16. 65

17. 117

18. 1 ~~

63

EXERCISES 1(h)

1. (i) 10-605
(v) 63-492
2. (i) 4-8
(v) 0-849
3_ (i) 25
(v) 0-301
4. (i) 0-0261
(v) 0-555
5. (i) 88
(v) 7
6. (i) 4132
(v)

7. 0) 4-5
(v) 960

(ii)
(vi)
(ii)
(vi)
(ii)
(vi)
(ii)
(vi)
(ii)
(vi)
(ii)
(vi)
(ii)
(vi)

430-68
5-286
3-62
0-944
2-628
0-175

0-066
0-423

1-4
200
0-2
0'3462
4800
120

(iii)
(vii)
(iii)
(vii)
(iii)

1'111
10'19
12-64
0-14
816

(iii) 2-5

(iii)
(vii)
(iii)
(vii)
(iii)
(vii)

16
30
600
0'0025
5'4
1280

(iv)
(viii)
(iv)
(viii)
(iv)

33-09

4-0515

5-787

3'59

21-014

(iv) 2840

(iv)
(viii)
(iv)
(viii)
(iv)
(viii)

40

360

4-55

183

100

3'5

EXERCISES 1(i)

1. (i) 3 X 103 (ii) 4-682 x 102 (iii) 1'8 x 104 (iv) 1-2 x 107 (v) 7-52
(vi) 3-8003 x 10 5 (vii) 3'8 x 10 5 (viii) 4-2 x 106 (ix) 8'62 x 106

10 5

I .

474 NEW SENIOR MATHS: TWO UNIT COURSE

2. (i) 10-4 (ii) 4-1 x 10-3 (iii) 7-25 x 10-6 (iv) 5-002 X 10-3
(V) 2-2 x 10-7 (vi) 2-7 X 10-2 (vii) 8-65 x 10-5 (viii) 6-2 x 10-5 (ix) 5 x 10-4
3. (i) 1-28 X 10 5 (ii) 2-41 x 10 9 (iii) 1-2 x 1013 (iv) 1-8 X 10 5
(V) 5 X 10- 10 (vi) 3-52 X 10-1 (vii) 1-031 x 10-2
(viii) 1'12 x 10-7
4. (i) 4 X 106 (ii) 3 X 10 6 (iii) 1-44 x 10 8 (iv) 8 X 10 7 (V) 5 X 10 8
5. 534 )( 10-23 g
6. 4'02 X 10 13 km
7. 1'488 x 1011 m
8. 25-6 h
9. 1'47 X 10 5 km/h
10. 4'22 x 109
EXERCISES 1(j)

1.
2.
3.
4.
5.
6.
7.
8.

(i)
(i)
(i)
(i)
(i)
(i)
(i)
(i)

3 (ii) 3 (iii) 4 (iv) 3 (v) 1 (vi) 4 (vii) 3 (viii) 4


1 (ii) 4 (iii) 4 (iv) 3
9730 (ii) 86000 (iii) 18'08 (iv) 0'007 (v) 5 (vi) 3'1
108,600 (ii) 109,000 (iii) 110,000
4-4 (ii) 4-3 (iii) 19-07 (iv) 4-00 (v) 8-60 (vi) 0-667
0-00604 (ii) 75-4 (iii) 0-610 (iv) 52400000
8 (ii) 0-5 (iii) 20 (a) 80 (b) 0-2 (c) 14
1 (ii) 27 (iii) 1 (iv) 0-02 (v) 0-2

10 3

EXERCISES 1(k)

1.400
6. 0-006
11. 200
16. 480
21. 50000
26.0-4

2.0-7
7.0-02
12.40
17. 2100
22. 120000
27. 10000

3. 3000
8. 2000
13. 0-07
18.80
23. 0-0018
28. 400

4.9
9. 0-2
14.60
19. 500
24. 100
29.4

5.
10.
15.
20.
25.
30.

50000
30
400
600
14
4500

4.
8.
12.
16.
20.
24.

89-14
9-956
19-80
4143
47-44
7-8

EXERCISES 1(1)

1.
5.
9.
13.
17.
21.

115-4
0-2356
73'22
0-9766
181-8
101-9

2.
6.
10.
14.
18.
22.

4-663
154-8
35-88
960'8
1-277
4-370

3.
7.
11.
15.
19.
23.

186-2
22-24
68'11
14900
1-812
17-45

EXERCISES 1(m)

1. (a) 121
(e) 16
2. (a) 8
(e) 10
3. (a) 0-04
(e) 0-0016
4. (a) I-I
(e) 3-2
5. (a) 16
(e) 5~

.
1
6. (a) 2
(e)

7. (a) 0'01
(e) 2-89
8. (a) 1-5
(e) 2~

289
144
3
30
2-25
2-56
0-3
5-5
25
(b) 144

400
1600
7
40
10-24
1-96
0-2
0-7
49
(c) 625

(d)
(h)
(d)
(h)
(d)
(h)
(d)
(h)

(f) 2k

(g) 11~

(h) 51-6

(b)
(f)
(b)
(f)
(b)
(f)
(b)
(f)

(b)

(f) 10
(b)
(f)
(b)
(f)

1-21
9-61
3!
0-2

(c)
(g)
(c)
(g)
(c)
(g)
(c)
(g)

(c)
(g)
(c)
(g)
(c)
(g)

5
-
6
57
6k
0-0049
4-5
1~

225
625
11
13

0-09
1-21
0-6
2-1

(d) 7

t6

(d) 1~
(h) 13
(d) lIb
(h) 0-0009
(d) 0-1
(h) 3!

ANSWERS 475

(b) 13

9. (8) 10
(e) 2-5
10. (8) 2-27
(e) 8-54
11. (8) 1-331
(e) 12g

nI

(c) 2-5

(d) 0-5

(b) 9-27
(f) 2-42

(b) 0-125

(c) 0-17

(d) 7-81

(c) 0'001

(f) 42~

(g) 0'00000 1

(d) 1~!
(h) 0-064

(b) 0-4

(c) 2!

(d) ~
5

(g) 0-1

(h) 0-6

(c) 0'03277
(g) 4'626

(d) 3-375 x 10-6

(h) 0-6300

(f) 2

27

64

12. (a) 5

(f)

(e) 2i

(i) 10

13. (a) 140-6


(e) 4-337
0) 0'4642

(b) 64-48
(f) 0-8072

EXERCISES 1(n)

1.
5.
9.
13.
17.
21.
25.
29.
33.
37.

2~
4~

2.
6.
10.
14.
18.
22.
26.
30.
34.
38..

2M
2.J2I
4.J7
6,j5
4.JIT
15~

12M
4,j5

2-../5
2.JfO
3.J7
4../6
5,j5

3.
7.
11.
15.
19.
23.
27.
31.
35.
39.

1O~
4O~

6.Jf3
20M
.J7

2../6
3,j5

4.
8.
12.
16.
20.
24.
28.
32.
36.
40.

6~
7~

7-J3
12.J2
80~

18m
49,j5
12-J3

3-J3
4-J3
5-J3
6-J3
9~

9../6
10../6
15~

25,j5
33~

EXERCISES 1(0)

1. 7-J3
4. 3~ +
7. 4-J3 10. 7-J3
13.0
16. 8-J3
19. 5.J7
22. 5../6 25. 15,j5 28. 9-J3

2."
5.
8.
11.
14.
17.
20.
23.
26.
29.

3-J3
3.J2

3.[5
1O~
8~

7.J7
4,j5 - 5.J2
3.[5
7-J3
2../5
3~
3.[5 + 3~
.J2 + -J3
6../6
7~ - 4,j5
-../6

2-J3

3. 6,j5
6. ~
9. 6~
12. ~
15. 4.[5 + 4.J7
18. 5.Jf5 - 2M
21. 12-J3
24 .J7 + 3.[5
27. 5.[5
30.0

EXERCISES 1(p)

1.
5.
9.
13.
17.
21.

2. 4
6. 3~
10. 2-Jf5
14. 4-J30
18.20
22. 12../6

-Jf5
5
2.[5
8../6
24~

6.f35

3.
7
11.
15.
19.
23.

2-J3
.f35
9
8
30../6
8../6

4. 7

8. .J66
12. 2.[6

16. 10M

20. 36

24.40

EXERCISES 1(q)

1.[6
. 2

2.2

3 .J2
. 2

4 3.J2
. 2

5.

6. 2

476 NEW SENIOR MATHS: TWO UNIT COURSE

7 2-v1
. 3

8. 3

13. 2
19.

25. 1

9. -v1

10 ..J5
5

11. ..J5

121

16.

17.

18.

24M
. 3

14. .J7

15.

20.

21. 4

22. -v1

23.

27. -v1

28. 3

29. M

26. 1

30. 2-;

EXERCISES 1(r)

1. M +
4. .[6 :- 3-./2
7.4
10. 12-v1 - 3M

2. 5 + M
5. 3-./2 - 2.[6
8. 6-./2 - 9
11.

a+

3.
6.
9.
12.

.Jaij

14..J5
14-./2

6.J7

.JXY

EXERCISES 1(s)

1.
4.
7.
10.
13.
16.
19.
22.
25.
28.
31.
34.
37.

.J35 - M +

-.[6

16 + 7..J5
9 - 3.J7
5.[6 - 8-./2
27 + 12-./2
1

21
67 - 12.J7
4 + 3-v1
11 + 4.[6
3 + 15-./2
82 + 12..J42
11

EXERCISES 1(t)
1. -v1 + -./2
4.
7

=-=-=-~:::-:...=

2
6.,[55

10.

+ 8M

67

~~---=-='----=-~=-=-----'-~

2. .[6 + 4 + v'2I + 2v'T4


5. -3 - 4-v1
8. 8 - 3.[6
11. 4 + 2-v1
14. 39 + 12-v1
17.2
20.4
23. 8 - 2-vT5
26. 1

29. 37

32. 21 - 1O-v1
35. 15 - 25-v1
38. 13 - 4-v1

2-J7

--J6

22

3M + 3.[6

8. -(7 + 4-v1)
11. 5 + 2.[6

18 + 4-v1
13


23

14. 12-v1 ;- 9-./2

16. -

17

19. 3 - 2-./2
22

11 - 2v'T4
13

18 - 3.[6

10

20. 11
-

/M

23 13-./2 + 3.J30

34

25. 34

28. 7..J5 4- 15

34.

4fO

37 98 + 25..J5

209

3. .[6 + 3-v1 6. .J3O + ..J5 +


9. 20 + 6..J5
12. 7 2M
15. 1

18.7
21. 3

24. 18 +
27. 1

30.
..J5 +
33. 34

36. 35 + 12.[6

-./2 - 3
3-./2 + -v1
2M - 3-./2

2m

3 . ..J5
6.

.J3O

4-v1 - 2-./2

+ 3-v1

11M + 38

18


17..JI5 + 60

12

49

15. 9 +5.JIT
18. - -./2 4+ 2

21. 6

+,s-./2

24

12..J3 + 2.J30 - 45.J2 - 15v5


39

27.

17j2
37-v1 - 31

30. Yes

33

35. 103 ~ 42.[6

36. 5M 9

38. 4-./2 - 1

39

22

14

51-v1 - 3-vT5 - 72
17

ANSWERS 477

41. -'-'=---=
2

40. -.[5 - 2../3 - 3-J2

CHAPTER 2
EXERCISES 2(a)

2. -25
7. -192
12. 32
17. -88
22.64
27. 256
32. -5
37.32
42. -10

1.20
6. 19
11. 128
16. 12
21.
26. -512
31. 5
36. -1
41. -4

5!

3. -15
8. -1
13. 64
18. -5
23.48
28. 288
33. -5
38.25
43.25

5.47
10. 16
15. 8
20. 320
25. 1
30.28
35.60
40.9
45.22

4.25
9.24
14. -27
19. -2
24. 131
29.80
34. 12
39. -7
44. 144

EXERCISES 2(b)

1. lOx + 15
4. 9ab + 9a
7. 5a 2b - 3ab 2
10. 6abc
13. 5x2 x
16. 3x2 + 2xy
19. 15n - 2m
22. 15a + 15b
25. 11x2
13xy
29. 5x2 - 5xy
33. -x2 - 8x - 12
37. a2 5a- 3

2. lOx

26.
30.
34.
38.

3.
6.
9.
12.
15.
18.
21.

3a
3b
6mn
x2y + 6X 2y 2
6mn + 2m
5y2 - 2X2
2x
4y
5x - 20
24. 2a 2 + 7a
28. 3y - 2x
27. a + 4b
32. 27x - 42
31. 9m 2 - 8mn
36. 4x + 9y - 12
35. x 2 + 14x 1

5.4xy
8. 8xy2 2xy
11. p2q - 5pq
14. 6a 2 8a
17. 2a - 3b
20. lOx + 11y
23. 11k + 1
7a 2 + 2ab
9x2 + 4x
3x2 - lOx + 4
m 2 - 7m - 10

EXERCISES 2(c)

1.
6.
11.
16.
21.

64
554-4
7-25
8
33

3. 10
2.96
7. (a) 25-6 (b) 3 8. 38
13. (a) 7 (b) 9i
12. 55
17.2
18.65
22. (a) 4 (b) 3S 23. 1

4. (a) 140 (b) 5


9. (a) 29-28 (b) 8
14.24
19. 92-4
24. (a) 11~ (b) 100

5. 38-5
10. 15
15. (a) 0 (b) 5
20. 3-63
25. 1210

EXERCISES 2(d)

1. x 2 + 6x + 5
+ a 12
c - 20
10. x 2 + 12x + 32
13. p2 - lOp + 25
16. 2X2 + 13x + 15
19. 12y2 + y
6
22. 6m 2 - 41m + 44
25. 6x2 - 85x + 14
28. 4p2 - 81
31. 16p2 - 40p + 25
34. 25y2 - 30y + 9
37. 25p2 - lOp + 1

3. x 2

2. x 2 + 6x + 8
5. a 2
+ a - 20
8. x 2 - 5x - 6
11. x 2 - 4x - 32
14. x 2 - 2x + 1
17. 3x2 - lOx + 8
20. 6x2 - 31x + 28
23. 3y2 + 13y - 10
26. 8p2 + 2p - 3
29. 6x 2 + 13x + 6
32. 9x2 + 24x + 16
35. 16y2 + 72y + 81
38. 9p2 + 42p + 49

4. a 2
7. c2

5x + 6

+ 21
(2 + 4( - 5
x 2 + 6x
27
y2 + 14y + 49
6m 2 + 11 m - 7
35x2 - 11x - 6
lOz 2 - 3z - 70
9x2 + 12x + 4
2X2 + x - I
4a 2 - 20a + 25
12x + 9
4x2
4z2 + 24z + 36

6. p2 - lOp

9.
12.
15.
18.
21.
24.
27.
30.
33.
36.
39.

EXERCISES 2(e)

1. 2x3

3x2 - 8x

2. 6x 3

22x2 + 24x - 8
-)

478 NEW SENIOR MATHS: TWO UNIT COURSE

3. x 3 ~ 4x
5. 2x3 - 12x2 + 22x - 12
7. 8a 3 + 12a 2b + 6ab 2 + b 3

4x2 + 5x - 2
12x2 + 6x
I
3
8. 8x - 36x2y + 54xy2
27y3
10. x 3 + 2X2 - 4x - 8
12. 8p 3 60p2q + 150pq2 - 125 q 3
14. x 3 + 3x2h + 3xh 2 + h 3
16. 125m 3
150m 2n + 60mn 2 - 8n 3
4
6
18. a + 3a b 2 + 3a 2b 4 + b 6
4. x 3

6. 8x3

9. X4 - 2x 3 + 2X2 - lOx - 15
11. -27a 3 + 108a 2b + 144ab 2 + 64b 3
13. 64m 3 + 48m 2n + 12mn 2 + n 3
15. a + 3a 2l3 bIl3,+ 3a 1/ 3b2l3 + b
17. x 6 - 3x4y 2 + 3X2y 4
y6

EXERCISES 2(f)

1. 2(a + b)

2. 5(4x - y)

3. a(l

5. mn(m + n)

6. r(r - 2s)

7. 5a2(a + 2b)

9. a(b

13. a 2b(8

10. p(x - Y

d)

z)
14. m 2n 2(9m 5n)
18. 4(a 2 + b 2
e2)

3b)

17. 54x 2y 2(1 - xy)

4. 2x(x - 3y)

a)

11. a 2b 2(5b - 18a)


15. p2rq(8r + 3)
19. mp(l + m - p)

8. -5x(3 + 2x)
12. 3ab(a + 2ab + 3e)
. 16. 3ab(3a - 2b)
20. 5a 2b(1 -

b 2)

EXERCISES 2(9)
1. (a
4. (e

b)(x

2. (x
4)(P - 1)
5. (4x + 5)(x - y)
8. (x - y - z)(y + z)

2)

+ d)(a
+ q -

- 3b)
7. (p
r)(a + b)
10. (a - b)(x + 2y)
13. (x + z)(x - y)
16. (5y + 2x)(2
5y)
19. (a
3)(b + x)
22. (a 2 - b)(a - b)
25. (a + y)(y - 2)
28. (2x
3)(2a + 3b)
31. (ax + by)(x - y)
34. (x 2 + 1)(y + 1)
37. (a 2 + b 2)(a + 3b)
40. 2(X2 + l)(x - 1)

11. (a + b)(x + y)
14. (2x + 1)(y + z)
17. (a 2 + b 2)(a + 3b)

20. (3y

7)(x

2)

3. (5a - 3)(P - 6)
6. (3a - m)(2b - 3e)
9. (x 2 + 4)(2x
1)
12. (a + b)(x + 4)
15. (a - e)(a - b)
18. (e - 2d)(a - 2b)

21. (x

z)(x

23. (3 + 2a)(a - b)

24. (2m

26. (x2

29. (pq

27.
30.
33.
36.
39.

4)(x + 3)
5)(P
q)
32. (a 2 + 1)(2a
I)
35. (a - 4)(b - 3)
38. (3m - l)(m - n)

2y)

+ pn)(n + p)

(2e - d)(ax
(m2 + n)(p

1)
1)

(x
4)(2y
z)
(2 - y)(x - 3y)
(w

y)(x -

y)

EXERCISES 2(h)

7. (ab - e)(ab

10.

(~

1)(~ ~

1) (m + 1)
5)(3a + 5)
r)(pq + r)

2. (m
5. (3a
8. (pq

1. (p - q)(P +q)

4. (8 - m)(8 + m)
e)

1)

11.

13. (x - 2)(x + 4)

(p !)(p + !)

14. (x

1)

yz)(x

yz)

3.

+ 4)
0'6) (x + 0'6)

(x

4)(x

6. (x
9. (3x

2y)(3x

2y)

12. (6e

7d)(6e

7d)

15. (2a

3b)(2a

3b)

18. (x

y - z)(x

16 . (~2
31)(~
2 + 3
19. 9800

20. 46000

22. 3a(2a - b)(2a + b)


25. 6(2x - 3y)(2x + 3y)
28. b(2a - b)
31. (x - 3)(x + 3)(x - y)

23. 3y(x
3)(x + 3)
26. 2(2a - b)(2a + b)
29. a(a - b)(a + b)
32. (x - 2)(x + 2)(x + 3)

+ b)
2)(3a + 2)
27. (x + y - 2)(x + y + 2)
30. 2(x - 0'2)(x + 0'2)
33. (p - 4)(P + 4)(q - 1)

35. 3(4a - 5b)(4a + 5b)

36.(~

34. x(a 37. 3y2(X -

3)(x

+
+

1)

2)(x

40. 5(1 - x)(1


43.8p
46. (x -

l)(a

38. h(2

2)

x)

3)(x -

+ 4)

17. a(a

41.
3)

(~

+y +

z)

21. ab(a - b)(a

h)

~)(~

+
+

~)

44. (8 - 9x)(8
9x)
47. 2a 2(2x - 3y)(2x + 3y)

24. 5x(3a

y)(~

39. z(z -

l)(z

42. n(m -

y)

+
+ 1)

n)(m

n)

45. 2n2(m
3n)(m + 3n)
48. (a - b)(a + b)(a + 2)

ANSWERS 479

EXERCISES 2m

1. (y - 5)(y2

5y

+ 25)

3. (2p + 3)(4p2 - 6p
5. (6 - a)(36 + 6a +
7. (x + 7)(7x2
X +

9)
a 2)

2. (z + 1)(Z2 - Z + 1)
4. 2(3r + 2)(9r2 - 6r + 4)
6. (2x + 3)(x2 + 3x + 39)

13)

8. (b - a)(b

9. 8(2a + b)(4a 2 - 2ab + b 2)


11. p4X4(p - X)(P2

13.

+ px +

(~ ~)(:2 + a~

21.
23.
25.
27.
29.

a)(b 2

10. 4311"(R - r)(R2

x 2)

12. (x 2

:2)

(3x + y)(9X2
12xy + 7y2)
4x(4x2 + 3)
(lOp - q2)(I00p2 + lOpq2 + q4)
9(a - b)(a 2
ab + b 2)
2
2
a b(ab - l)(a b 2 + ab + 1)

ab

+ Rr +

r2)

y4)

b 2)(m

a 2)(b 2 - ab

+ n)(m 2 -

mn

a 2)

+ n 2)

16. (x - 3)(x + 3)(2x + 1)(4x2 - 2x + 1)


18. (2a - b )(a2 - ab + b 2)

20. (2;

b 2)

y2)(X4 _ x2y2

14. (a - b)(a2 + ab

15. (2x 3)(2x + 3)(x - 1)(x2 + X + 1)


17. h(3x2 + 3xh + h 2)
19. 2b(3a2

22.
24.
26.
28.
30.

b)(

~~2 + 2~b +

b 2)

2(3ab + 2c)(9a 2b 2 - 6abc + 4c 2)


(mn - 1)(m 2n 2 + mn + 1)
x(x 2 - 6x + 12)
(5c - 7d)(25c 2 + 35cd + 49d2)
2(x - Y + 3)(x2 - 2xy + y2
3x + 3y

+ 9)

EXERCISES 2(j)
1. (x

l)(x

3)

4. (x + 5)(x + 1)
7. (m + 5)(m + 4)
10. (p + 1)(P + 18)
13. (x + 6)(x + 2)
16. (x - 2)(x - 6)
19. (m
20)(m - 1)
22. (p - 5)(P + 3)

25.
28.
31.
34.
37.
40.

(x
(a
(x
(x
(x
(y

7)(x + 5)
- 6)(a + 2)
+ 12)(x - 6)
- 7)(x + 6)
- 14)(x + 3)
- 11)(y + 5)

2. (x
5. (a
8. (p
11. (x

14.
17.
20.
23.
26.
29.
32.

+ 3)(x + 7)
+ 6)(a + 1)
+ 6)(P + 3)
+ 4)(x + 3)

(x
(m
(x (p +
(x
(x (x
35. (x 38. (x +
41. (x +

3)(x
4)
5)(m - 4)
13)(x
1)
15)(P - 1)
5)(x + 2)
6)(x - 1)
24)(x + 3)
21)(x + 2)
7)(x - 1)
3)(x + 11)

3. (x + 3)(x + 8)
6. (a + 4)(a + 8)
9. (P + 2)(P + 9)
12. (x + l)(x + 12)
15. (x - 12)(x - 1)
18. (m -' lO)(m
2)
21. (P + 5)(P
3)
24.
27.
30.
33.
36.
39.
42.

(P - 15)(P + 1)
(x + 9)(x + 8)
(x - 9)(x + 8)
(a + lO)(a + 3)
(x + 21)(x - 2)
(y + 11)(y - 5)
(x
3)(x
11)

EXERCISES 2(k)

4. (4a + l)(a + 3)
7. (4x - 1)(2x - 3)

2. (3x - l)(x + 4)
5. (4x + l)(x + 1)
8. (2x + l)(x - 5)

10. (4x + 5)(2x + 1)

11. (3x + 4)(2x + 3)

13. (3x

14.
17.
20.
23.
26.
29.
32.

1. (2x

16.
19.

22.
25.
28.
31.
34.
37.
40.
43.

l)(x

1)

2)(x - 5)
(5x - 8)(2x + 1)
(3x
2)(3 x
2)
(5m + 9)(3m - 2)
2(3y
1)(2y + 3)
(6x - 7)(x
2)
(p + 3)(6p + 7)
(8x - 9)(3x - 4)
(3x + 5)(3x
2)
(5x + 3)(x
1)
(2x + 3)2

35.
38.
41.
44.

(3a + 7) (2a
(2x - l)(x +
(x - 2)(2x (5a + 2) (2a (3x
2) (2x
2(3x - 7)(x (5a + 2) (2a
(5x - 3)(3x (2x - l)(x (3p - 1)(P (3x + 5)2

9)
2)

5)

9)
7)
1)
3)
2)

4)
2)

3. (2x + 3)(x + 2)
6. (3a - 2)(a - 1)
9. (13c + 6)(c - 1)
12. (3x - l)(x - 4)
15. (3x + 1)(x
18. (2x - 3)(2x

21.
24.
27.
30.
33.
36.
39.
42.
45

(6x
2(y
(3x
(4x
(6y
(3x
(5p
(4x
(2x

4)
3)

- l)(x - 14)
- 3)(y + 1)
- 4)(2x
7)
+ 3)(2x - 1)
1) (2y + 5)
+ l)(x - I)
+ 3)(2p + 1)
+ 3)(2x - 3)
- 7)2

480 NEW SENIOR MATHS: TWO UNIT COURSE

EXERCISES 2m

. I

1. x(x
3)
4. x(x - 8)
7. 3y(x - 2y)(x + 2y)
10. (lOx - l)(x + 1)
13. (a - 7)(a + 6)
16. 3a(a + 7)(a + 1)
19. (a - l)(a 2 + a + 1)
22. x(x + 1)
25. 4(x - 15)(x + 8)
28. 2y(3y + 1)(y + 4)
31. 5x(a - 5)(a + 5)
34. (x - 2y)(x
lOy)
37. (m + 6)(m
n)
40. (x - I)(mx - y)
43. (x + a)(x
2)

46. (p

48.
49.
52.
55.

5)(P2 - 5p

(x + h + l)(x2
(4 - 7r)r2
(8 - 7r)r2
4(4 - 7r)r2

2a(a - 2) (a + 2)
3. 3x(x + 3)
(x - 3)(x + 3)
6. (x - 9)(x + 1)
5xy(x
2y)(x + 2y)
9. (1 - b + c)(1 + b - c)
a(a + 2b)
12. 6(x - 2)(X2 + 2x + 4)
(a
b)(m + n)
15. 2x(x + 8)(x - 1)
(x + 2y - 2)(x + 2y + 2)18. (x - 5)(x + 2)
ab(b + c + d)
21. (x
6y)(x + 6y)
(x + 4)(x
3)
24. (x + y)(y - z)
b(x - 7y)2
27. 3y(2y - 1)(y + 1)
15(a
2)(a + 2)
30. mn(3 - 5mn)(3 + 5mn)
4xy
33. 5t(t + 9)(t
8)
5(1
5a)(1 + 5a)
36. (x + y)(a + b)
(x + 3y)(x 3y
3z)
39. (x - 2)(x + 2)(x + 3)
(1 - x)(3 + x)
42. 2x(3x + 4)(x - 7)
(x - 18)(x + 8)
45. (ax + 1)(2c - d)

2.
5.
8.
11.
14.
17.
20.
23.
26.
29.
32.
35.
38.
41.
44.

+ 25)
2xh

47.

h2 -

(~-

1)(a2

+ ~ +

1)

+ 1)
51. 7r(R - r) (R + r)
54. 7r(R
2r)(R + 2r)
57. a(a + b + c)

50. (7r - 2)r2


53. 7ra(a - b)
56. 2(6 - 7r)r2

EXERCISES 2(m)

1. 2a - b
5.

2(2a - b)
3a

2. 3x +
3

2~

4.

3. 7

6.2

7.

8.1 + m

10. p - q

11. x ; Y

12.

13.~

14. k

15. x - 3

16.

17.~

18.

19.

22.

9. m

x+y

x+y

21.

~+

2
5

25. x-

x+

23.

x + 1
4xy(x + 2)
26.
x+4

ba

27. m + 2
m

30 ~
b

31. 1

33. 1

34.

35.

37. 2x - 1

38.

2
y2

5:

24. x - 2
x-2

29. 1

+ xy +

r + s

20.

x2

2(s - 6)

x+4

28.

. 32. 2y
m

39. 3x2 + 3xh + h 2

x 2 x~ + ~2
x-y
3x 2 +
40.
y
2x

36.

EXERCISES 2(n)
x
1. 30

2.

3 29a
. 30

5.--15

6. ~
6

7 3x + 1
12
.

2x - y

9.

12

10.

-a - 8b

18

11.

4.

12

8.

lli
12

7m - 3n

12. 3x

10

ANSWERS 481

i1
a

13. 3a
17 .

2
- n2
15. m
----'-'mn

14 1 -ab 2a

5b - 2a

18. c

a2 b 2

+a

19. 4a

abc

+ 3x
xyz
2x + 5
20. 3(x + 1)
16. 4z

~ 11

21. 3x -;- 1

.EXERCISES 2(0)

1. (x - 3)(x + 3)
4. 6(x - 2)
7. x 2 - 1
10. x 2 - y2
13. x(x - 2)2
16. xy(x 2 - y2)
19.

x2 -

2.
25
5. x 2 4x
8. 2(x - 3)(x

+ 3)
11. (x - 2)(x + 2)(x
14. x(x - 3)(x + 3)
2a
b)(a + b)
17. (a
x 2 + 2xy - y2
20. (x
y)(x + y)

x + y
x-y

22. 3 - x
25

23.
4

(x - I)(x - 3)(x

7a - 15
28. 2(a2 _ 9)

-a 2

3)

x- 2

37. .

4x
(x - l)(x

40.
(x

18. (x

21. a2

27

2x - 13
l)(x - 2)

41

+ y)(x

- y)

30. (3x

5)

2)(4x

1)

2x - 4

33. x2 - 16

x - 7

36

(x - 2)(x

I2a
(3a - I)(3a

b2

22

1
x-y

38
1)

+ y)

-2y

(x

35. a + 2

y)(x
4a

2x - 1
24. (x
2)2

x + 20

32.

25

x + 5y

1
y

(x - 5) (x

1)

x(x
2)
x(x - 4)(x + 4)
x 2 + 4x + 4
xy(x2 - y2)

15. x 2

4x + 7
(x - 2) (x + 3)

29

31. (3a - 4)(2a


34.

26
1)

3.
6.
9.
12.

39.
1)

2)(x - 1)

5x + 13
(x - 2)(x + 2)

5x - 31
42. 3x(x - 5)

x + S

(x - 4) (x + 4)

CHAPTER 3
EXERCISES 3(a)

1. 4!
7. -I!
13. -6t
19. -1~

2.
8.
14.
20.

4~
-1
2!
-1

3. 11
9. -1
15. 6!
21. -I4~

5. -si
11.2!
17.4
23. -3

4. 5
10. 1
16. -4
22.

It

6. 1

12. Ii
18. 1
24. -I!

EXERCISES 3(b)
1

1. 5

3.6'

3
4

4.

5. IOi

6. 4~

11. -3!

12. 3!

17. -2!

18.

-;;

24.

3"

8. 4~

-5
9. 13

1
14. -4

15. 13

16.

7
19. - 34

20.0

21. 15

22. -4!

23. Il~

25.9!

26.

27. -9

28. -7

29.

33. 1

34. -26

7. 3

31. -I

-Ii
1

32. 2:

10. -2
1

-2

3"

30. 1

482 NEW SENIOR MATHS: TWO UNIT COURSE

EXERCISES 3(c)
1. x
5. x
9. x
, 13. x
17. x

> 12
> 12
> -2
< 30
21. -9 < x < -3
25. 10 .ex < 17, x

29. 5

2.
6.
10.
14.
18.
22.
J 26.
30.

x < 1
x ~ 2
x < I~
x < 32
x ~ 3
5 ~ x :E;; 7

4. x ~ -2
3. x < -5
7. x < 3
8. x ~ 10
12. x > -3
11. x ~ I!
15. x
4 or 5
16. x = 3 or 4
19. x < -6
20. x < 2l
23. -4 ~ x < 5
24.0 < x :E;; 3
8 < x < 16
27. 0 < x ~ 17, x J 28. 5 ~ x :E;; 9, x J
2 <X< 21, X
J 31. 2 ~ x :E;; 7, x J 32. 2 < x < 18

EXERCISES 3(d)

1. 9

2. 2'5

3. 6x - 4 if x

~ ~, 4
if x + y

5. x + y if x + y ~ 0; -(x + y)
6. x + y if x, y ~ 0; x - y if x ~ 0,
y
7. 2x if x ~ 0; 0 if x ~ 0
8. 2x if x
10. 2x

+ 3 if x ~ -~;

12. (a) 5, -1

(b)~,

-(2x

+ 3) if x

-I! (c) 4, 1

:E;;

(d)~,

~
~
-

-1

6x if x

:E;;

4. 2 -

.J3

0
0;
y - x if x ~ 0, y ~ 0; -(x + y) if x, y
5; 10 if -5 ~ x ~ 5; -2x if x ~ -5
9. 8

:E;;

11. x

(e) 3, -7

Show x < -4 or x > -2


14. Show x > -4 and x < -1
15. Show x ~ -2 or x
Show --J5 ~ x ~ -J5
17. Show -2 < x < -1
Show the real line except for the interval between -1 and +1 exclusive.
20. 1 for all x*,O
21. 1 when x > 0; -1 when x < 0

Show -2 ~ x ~ 2
x
4
4 - x

22. - - when x < 0; - - when 0 < x :E;; 4;


when x ~ 4
13.
16.
18.
19.

23. 1 if x < 1; -1 if x > 1


24~ x - 5 if x ~ 5; 5 - x if x
25. x + 1 if x ~ -1 orx > 1; -(x + 1) if -1 ~ x < 1

:E;;

EXERCISES 3{e)

1. 0,5
4.0,

-2

7.0,4!
2

10.0, 3"

13. a, b
16. 5!, -5!
19. 0, 1~

2. 2,3

3. -I, 2

5. 0, -6

6. 7, -2t

8. 0, -1

9. 3, -4

11.0,6

12. 0,4!

14. 3a, -2b


17. 1
1
1

20. 2' -2

15. 2, -2
18. -I!

2.
5.
8.
11.

3.
6.
9.
12.

EXERCISES 3{f)

1.
4.
7.
10.

1, -1
0'5, -0-5
8, -8
2!, -2!

5, -5
4, -4
I!,
I!, -I!
1
1

-It

7, -7
3, -3
2!,
I!,

13. 1, -1

14.

16. 1~,
19. 7, -7

17. 3, -1
20. 3, -3

18. 6, -2
21. I!, -I!

23. -1, -5

24. 3, -1

26. .J3,

27. .[2, -.[2

22. 1,
25.

-S3

-J5, --J5

4' -4

-.J3

15. 4, -4

ANSWERS 483

35. -..fj, --..fj

30. -1 + 2..J2, -1 _. 2..J2


33 . .JiQ,-.JiQ
36. y'ff, -y'ff

2.0,5
5.0,4
8. 0, -8
11.0,7
14.0,3
17. 0, 1
1
20. 0, 2

3. 0, -5

{i. 0, 1

9.0,7

12. 0, -10

15. 0, -4

18.0,2
1
21. 0, 3

23. 0, -3

24. 0,2i

1. 1, 2
4. 1, 3

2. 1, 5
5. 3

3.4, -2
6. 1,4

7. -1, -8

8. -1,

28. y'ff, -y'ff


31. 2 + 2..[3, 2 - 2..[3
34. 2..[3, -2..[3

29. ..[3, -..[3

32. 3..J2, -3..J2

EXERCISES 3(g)

1.0,6
4. 0, -10
7.0,2!
10.0,2
13.0,4
16. 0, 1!
19. 0, -2
22.0,

EXERCISES 3(h)

9. 1, -2~

:s1

10. -6,2

11.2,

13. 2!, -2
16. 3!, -11
19. -4, -I!

17. 1, 8t
20. 12, 1~
3
23. 1, :s

15. -1, -4
18.4
21. 3, -10
1

24. 6, 2

26. -9, 2~

27. 1,

14. -5

22. 2,5

28. 3,5

-21

12. 32,

29.4,

32

30. 1,. -:-:s

'27

32. -~ -1
5'

33. 9, -1

34. 2, -6

35.

~,

-11

36. 1

37. -5, -6

38.

~,

-21

39.4,

'7

40. 12, -8

EXERCISES 3(i)

1. 4

2.9

3.49

4.25

6. 36

7. 100

8. 81

9. 144

16. 201
21.

a2

41

12. 2!

13. 12!

14.

17. 561

18. a2

19. b 2

5. 1

10. 4
15. 3O!
20.

c2

484 NEW SENIOR MATHS: TWO UNIT COURSE

EXERCISES 3(j)

2.
7.
12.
17.

1.-1,5
6. -3,7
11. 3, -10
16. -1, 11

4, -2
1, 25

-1, 12

0, 10

3. 1, -5
8. 1, 2

13. -2, 5

18.0,3

4.2, -6
9. 3, -4
14.2,5

5. 4,6
10. 1,4
15. 8, -9

EXERCISES 3(k)

-J5;

1. 1

3-236, -1-236

3. 1 2.J2f; 2-792, -1 -792

2. -2 2.J2; 0-828, -4-828


4. 3

.J7; 0-354,

5-646

5. -=-'---- 4-792, 0-209

7. 3

-J5;

9. 3

M; 6-742, -0-742
..J6; 3-449, -1-449

11. 1

5-236, 0-764

13. - - - - 2-303, -1-303

8. --=--==---.:-=- 0-618, -1-618


2

10. -2 -J5; -4-236, 0-236

12. -3 2..J33; -4-373, 1-373

14. 3 23-J5; 4-854, -1-854

EXERCISES 3(1)

1_

-"---=--'---:

1-851, -1-351

3. -----'----'-- 0-781, -1-281


4

4.

5. 5 6ffi; 1-847, -0-181

6. -----'-- 0-787, -2'120


3

7.

-J'l_' 2-823 , 0-177

8. 3 2

-"--=-'---"-:

1-535, ':'0-869

0-281, -1-781

-"-----==--'--

9. --6- - 2-703, -0-370

10.

11. 5 4.J97; 3-712: -1-212

12.

13. -1 4..J33; 1-186, -1-686

14. 4 3 -J'l; 2-215, 0-451

o 610

16. 5 6 -J'l; 1-274, 0-392

1. -1, -5
3. -1, 7

.fl; 0-414, -2-414


5. -1

2. 2, -4
4. 2,5
6. 3 -J5; 5-236, 0-764

15.

=--=--'--=--:

1-640,

---==---.c.._'

0-725, -1-725
1-215, -0-549

EXERCISES 3(m)

7. 1
9. 1
11. 7,8

..J6; 3-449, -1-449


M; 4-162, -2-162

13. -5 4
15.

m; -0-219, -2-281

-k,-1

17. - - - -1-215, 0-549

8. -5 2
10. -2
12. 3, -5
14.

m; -5-193, 0-193

.fl, -0'586, -3-414

--=--'-'---'

16.0,1-5

3-581, 0-419

ANSWERS 485

2.J2.J2,

19. -3

-0-172, -5-828

20.4

21. 1-5,-1

22.

23.

1-643, -0-243

1
24. -3",4

1-640, 0-610

25. -1-5, 1

26. -3 2..ff7; 0-562, -3-562

27.

-2-823, -0-177

29. 1

~~..;;..-

-13; 2-732,

31.

28.

-0-732

.JIT;

30. 3

2-351, -0-851

3-436, -0-436
6-317, -0-317

32. ----'---'- -0-564, -4-437


2

33. 3, -20

34. -9 6 ffi; -0-736, -2-264

35. .J5; 2-236

36. --==----:.---- 0-6l8, -1'618

EXERCISES 3(n)

1.
2.
3.
6.
9.
12.
13.

(a) 6
(a) 4

(b) 4
(b) 5

(c) 3
(c) 9

3
(a) 2 s, 6 s
(b) 4 s
1m
12 em, 16 em, 20 em
(i) (20 - x) em

(d) 8

(e) 2

(d) 7

4. 8, 11 or -8, -11
7. 6
10. 4 em
(ii) 20x - x 2

5. 8, 9 or -8, -9
8.4 m
11. 4 em, 12 em
(iii) x 2

20x

+ 84

0; 6 em, 14 em

EXERCISES 3(o}

(~, -3!)

1. (-2, 1)

2. (14, 4)

3. (5, -2)

4.

7. (2, -3)

8. (1, 2)

9. (7,3)

10.

13. (1, 2)

14. (8, -1)

15. (2, -4)

16. (9, 3)

19. (-8, 3)

20. (5, 3)

21. (7, 4)

22. (2,

(~, 3)

5. (-2, 0)

6. (5, 2)

11. (5!, 1)

12. (4, It)

17. (8, 3)

18. (7, 2!)

k)

EXERCISES 3(p}

1. 250, 200
5. 600, 400
9. 12
13. (i) y

2x - 7

2. 6, 2
6. 30, 20

3. 5, 35
7. 2, 4

4.6,30
8. $25, $20

10. 120, 15

11. 6, 12

12. $280, $160

" y
(u)

= -4x +

(.,')
III Y

3 - 3
4"x

(iv) y

-x
2

=- +

EXERCISES 3(q)

1.
4.
7.
10.
13.
16.
19.

(-1, 1), (6, 36)


(-2, 17), (7, 8)
(2, 3), (3, 2)
(3, 1)
(I, 1), (0'4, -0'2)
(3, 2), (-1-4, 15-2)
<-4, -2)

2.
5.
8.
11.
14.
17.
20.

(1, 1), (2, 4)


(-2, -5), (5, 2)
(0, -2), (3, 4)
(2, -2), (4, 2)
(-2, 0)
(1, 1)
(-3, 6), (5, 14)

3.
6.
9.
12.
15.
18.

(-1, 4), (5, 10)


(1, 3), (-1-8, -2-6)
(2, 1), (-1, -2)
(3, 2), (-18, 23)
(2, 5), (-5, -2)
(1, 1), (-0'2, 0'4)

486 NEW SENIOR MATHS: TWO UNIT COURSE

EXERCISES 3(r)

1. (4, 1), (-1, -1)


4. (4, 5), (-~, -5~)

2. (3, 4), (-3, -4)


5. (3, -4), (-3j, 6)

3. (1, 1), (5, -1)


6. (5, 3), (-7, -5)

CHAPTER 4
EXERCISES 4(a)

2.48
6. 120, 60, 60
10. 70, 30, 70, 80

1. 120

5. 90
9. 35, 75

4. 36
8. 42, 30
12. 36, 36, 144

3.40
7.80
11. 40, 35, 145

EXERCISES 4(b)

1. 70, 110, 70, 110, 70, 110, 70 2. 65, 115,65, 115, 115,65, 115 3. 72, 108, 108, 72
4. 125, 55, 125, 55
5. 70, 65, 45
6. 100, 80, 100
7. 32, 78, 70
8. 36, 144, 54
9. 75, 40, 65
EXERCISES 4(c)

1.
5.
9.
20.

2.
6.
10.
21.

36
96, 84
36
(c) 100

4. 70, 70, 48
8. 34, 142
12. 35

3. 111
7. 22, 72, 50
11. 30

45
75, 30,35
22~

36, 60, 84

EXERCISES 4(d)

2.
3.
5.
6.

(a)
(a)
(a)
(a)

LACD, LACB, LDAC


108 (b) 120 (c) 144
4 (b) 5 (c) 24 (d) 18
8 (b) 15 (c) 16 (d) 36

(b) LABD, LBDC, LADB, L CBD


4. (a) 72 (b) 60 (c) 36
(e) 30 (f) 36
8. 18
9. 108

EXERCISES 4(e)

29. (i) (a), (b), (c), (d)

(ii) (a), (b), (c), (d)

(iii) (a), (b), (c), (d)

(iv) (b), (d), (v), (c), (d)

EXERCISES 4(f)
(b) AFD, FDE, BED, CEF

1. (a) ADEF, FDBE, FDEC


EXERCISES 4(9)

1. yes
2. (a) L PAQ = L BAC (same angle), L APQ = L ABC (given)
(b) L AXY = L ACB (given), L XA Y = L BAC (same angle)
3 (b) PS - PT -

4 (b) PS - ST
( ) 21 m
5 (b) AB _ AC
RQ - TR - TQ
.
PQ - QR
PR c 2 e
.
AE - AD - ED
6, (a) x = 3, Y
3! (b) x = 1, Y = 2-4 (c) x = 3, Y = 2'1 (d) x = 2-5, Y = 3'3

7.

~~

~;

8. (a) 2-4, 12

(b) 6, 3

(g) 7'5,9-6 (h) 9-6,3'75 (i) 2-1,2


(i) 9 em
(ii) 12 em
(iii) 13i em

10.
16.
20.
22.

(c) 18

G)

3,2-4

(d) 2, 4i

(e) 14, 12-5

(f) 18

9. ,6ABC III ,6AXY

24 m
11. 20 m
12. 5 m
13. 4-8 m
14. 4 m
15. 4-8 m
10 m
17. (ii) 1'6 m
(iii) 1-8 m
18. 8 m
19. (b) 30 m
(b) 9 em, 16 em, 20 em
21. (a) 8 m (c) 3-2 m, 3-6 m
3-24 m
23. (a) ,6XCF and ,6XBE (b) 4 units

ANSWERS 487

24. (i) 12 em
26. AD

(iii) (a) 3'75 em

(b) 2-5 em

25. (i) 12 m (ii) 2?7 m


CE BE

27. t::,RXY, t::,PQR, t::,PYZ

29. AE' DE

(iii) 16'8 m

30. 2 : 1

EXERCISES 4(h)
2. (a) 5.fi m (b) 3'6.fi em
3. (a) 6.fi (b) 6 em
4. 5-J3 em
5. (a) 10 em (b) 5'2 em
6. 32 em
7. 106 km
8. (a) 10 km (b) 25 km (c) 50 km (d) 60 km (e) 75 m (f) 100 km
9. 20 em
10. (i) 7 em (ii) 40 em
11. 12 em
12. (a) 8 em (b) 24 em (c) 4'8 em (d) 2-4 em
13. 4 em
14. 48 em
15. 3-v'I3 m
19. 2'4 em
1. (a) 13 em (b) 2.J4f m (c) 3'9 em

EXERCISES 4(i)
1. (a) 160 em 2 , 56 em (b) 50 m 2 , 30 m (c) 18'3 m2 , 27'4m (d) 216 em 2 , 60 em
2. (a) 1'35 (b) 36 (c) 100 (d) 0-72
3. (a) 40 em (b) 40-8 m (c) 2-1 km (d) 1780 m
4. (a) 6-25 em 2 (b) 2-56 m 2 (c) 36 em 2 (d) 1225 m 2
5. (a) 124 em (b) 28 em (c) 520 m (d) 64 em (e) 1-6 km (f) 600 m

6. 63 m 2
12. 1250
IS, (a) 3-25 em 2
16. (i) (a) 22 em 2
(ii)

17. 30-7

7. 336 m 2
13, 21 "10
(b) 3-85 em 2
(b) 114 em 2
(b) 76 em
18. 125

8. 1-05
14. 2 : 1
(c) 120 em 2
(c) 25 em 2
(c) 29 em

(d) 48 em 2

(e) 672 em 2
(e) 7-5 em 2
(e) 13-4 em

(d) 100 em 2
(d) 40 em

I
I

10. 72 em 2

9. 1200

(f) 43-2 em 2

(f)6-72 em 2
(f) 10-8 em

CHAPTER 5
EXERCISES 5(a)

4. (a) 0 (b) 0'5 (c) 0 (d) 1 (e) 1 (f) 0-5


6. 5, 10, 15, 20, 25

9. (i) } (ii) ~ (iii) 1 (iv) } (v) - (vi) }


5
5
5

1. no; no
2. yes
5, 7, 35, 70, 105, 140
7. (i)

(ii)}

8, (i)

(ii)

11. (i)

10. (i) 10 (ii)} (iii) ;0 (iv) ;0


12. (i)

k (ii) k (iii) 1

13. (i)

(ii)

(iii)

15. (i)

(ii)

(iii)

8
16 (I.) 25

(") 2
11. 25

III

1~

22. (i)

~ (ii) ~

(ii) 1 1

18 .(")
18
24 (I') 37
II 37

(b)

k (v) ~

(iv)

5
~

(v)

(vi)

(') 2
IV

(vii)

(c)

(d)

~ (iv) ~ (v) ~

(ii)

(viii)

1 (iii) 1 (iv) ~
6
2

C") 1
11

13

C..)
III

4
13

(iv) ;2 (v) 52

(Vll
.. ) 3"
2 (Vlll
''') 59"

17. (i)

(iii) 11

(iii)

(Vi)}

14. 0)

(.. ') 4

18. (i) 5 (ii) (a)

20. (i)

(iv)

19. (i) }

(iv) 10 (v) 10 (iii) 1.


5
5

2
C") 7 (iii) ~ (iv) 13

0 (v) -5
11 10
5

C") 3

21. (i) 0-96

II

(ii) 0'04 (iii) 0-9 (iv) 0-48

23. (i) 0'4 (ii) 0-6 (iii) 0-079

("')
12 (')
12 ()
1 $2 , $36 , $36, $36, $36 ,ose
1 $1
III 37
IV 37
v 37;

488 NEW SENIOR MATHS: TWO UNIT COURSE

EXERCISES 5(b)
1
1. (a) 4 (b) 52

7
(d) 26

(c) 13

(e) 13" (f)

1
13

Yes, no, no

5
3'
12
15
7
7
12. 15 .

2. (a) 17 (b) 17 (c) 17 (d) 17 (e) 17 (f) 17 (g) 17 (h) 17 (I) 17 ij) 1; A and B, Band C
3. (a) 1

1
2

(b)

5
6

(c)

3"

(d)

4. (a) 20

13

(b) 20

(c)

5"

1
1
1
1
5
5. (a) 2 (b) 8" (c) 2' (d) 2' (e) 1 (f) 8" (g) 1 A and C, Band C
4
6. (a) 25
"

3
(b) 10 (c) 50

(d) 50

33

(e) 50

(f)

5"

18
(h) 25

(g) 50

5 1 7
7. (a) 36 (b) 36 (c) 18 (d) 36 (e) 1 (f) 18 (g) 18 (h) 1
13
8. (a) 18
11. (a)

5"

(b) 12
(b) 10

(c) 36

(d) 1 No

(c) 10

11

10. 16

9. No

12. (a)

5"

3
(b) 10

(c)

3"

EXERCISES 5(c)

1. HI, H2, H3, H4, H5, H6, Tl, T2, T3, T4, T5, T6; yes

(i)! (ii)!

2. (i) ~ (ii) ~
9

~~33

'gR

RR

RW

R8

w~!8

WR

8~~

8R
8W
88

R~W

1st cube

ww
W8

3. 12 pairs: AJ, AH, AT, AL, KJ, KH, KT, KL, SJ, SH, ST, SL

4. (i)

(ii)

Roast

Sweets

L~~0

LA
LC

to

A 8A
8C

8~C0

80

A PA
PC
0 PO

P~C

!lC
Q)

Q)

~A

A CA

B
Roast

c~~

CC
CO

ANSWERS 489

5.

Shirts

Ties

Pullovers

S,T,P,
S,T,Pz
S,TzP,
S, TzPz
SzT,P,
Sz T,Pz
Sz TzP,
S2 TzPz
S3T,P,
S3 T,P2
S3 TzP,
S3 TzPz

6 ....---e-.......___

6. Yes

(i)

(ii)

1
9

(iii)

51--___........__

...

--4I~e_

--4I~

___...

4 1--..-.......___--.1--___...

2nd 3
'all

l - -___.........____-4I>-----1~.

2t---e--+__

--4I~

___...

1st,o/l
1st toss

7. (i)

(ii)

(iii)

2nd toss

3,d toss
HHH
HHT
HTH
HTT
THH
THT
TTH
TTT

(iv)

8. 10, 20, 30, 40, 1W, 2W, 3W, 4W, 1R, 2R, 3R, 4R

(i)

(ii)

9. Same tree as in 7. Replace Hand T by S (success) and F (failure)

t
(i)

(ii)

8
10. Same tree as in 7. Replace Hand T by Rand W; outcomes not equally likely,
11. Same tree as in 7.

(i)

(il)

(iii)

(il)

1
8

(iii)

1
8

8
8
2
12, (1, 1), (1, 2), (1, 3), (1, 4), (2, 1), (2, 2), (2, 3), (2, 4), (3, 1), (3, 2), (3, 3), (3, 4),
(4, 1), (4, 2), (4, 3), (4, 4)

13. (i)

~~

(ii)

(iii) 36

(iv)

(i)

(v) 12

(vi)

(vii)

9 ('. ') 1
14 (I') 16 (")
II 16
III 4"

HDCS

HDCS

HDCS

HD~

I'

490 NEW SENIOR MATHS: TWn UNIT COURSE

EXERCISES 5(d)
1. (a)

k (b) t

3. (a) 21

( c)

(b) 1 (c) 1

5. 12
7. (a) Yes

(b) No

(c) No

85

4. (i)

6. (i)

(b) ~ (c) 1
8
5
,1
(ii) 1 C..) 1 (iv)4 (v) 0
.
6 HI 36
(ii) 1

8. (i) 25

c.)
HI

(ii) 25

1
10 (iv) 15

C) 4

25

III

(' ) 12
IV 25

{o

10. (i) 10 (ii) ~ (iii) ~ (iv)


20
10
7
9 (c) -3
3
3
12. (a) 16 (b) 16
8 (d) 16 (e) 16 (f) 16
1
1
1
14. (a) 24 (b) 14 (c) 4- (d) 12 (e) 8
1
1
1
16. (a) 16 (b) 16 (c) 16 (d) 16 (e) 4

9. 12

1~

11. (a) 100 (b)


64
13. (a) 125

(c) 100

1
(b) 125

15. (a) 9 (b)

(d) 100

(c) 125

(c)

(d)

92

(e)

98

125
1
1
5
17. (a) 216 (b) 216 (c) 8 (d) 8 (e) 216 (f) 216
19. (a)

2. (a)

81

63
(b) 1000 (c) 100

20. (a) 27

(e)

4
(c) 27

(b) 27

22. (a) 0-08

3
(d) 40

18. (a) .27

87

27
9
(b) 50 (c) 50 (d) 200

11
23. 21
25. 0-89

26. (a) 0-0125

27. (ii) 60

(b) 0-336
(c)

(!~r

(d) 1 -

999
29. (a) 1000 (b) 1000
(b) 0-97

(d) 27

64

7 ("') 13
(') 2 (11.. ) 15
24,15
11115

31. (a) 0-27

(c) 27

(f) 125

21. (a) 200

(b) 0-48

28, (a): (b) (!~y

1
(b) 27

(!~y
30. (a) 208

(c) 0-03

1
(b) 78

(c) 120

(d) 9360

(d) 0-32 (e) 0-162

EXERCISES 5(e)

5
141
2
13
1. (a) 14 (b) 28
5. (a) 3" (b) 12
2. 15
4. 84
3. 273
'
4
1
6. (a) 17 (b) 17 (c) 102 (d) 11
7. (a) 22 (b) 11 (c) 33
17
5
4
4
8. (a) 22 (b) 33 (c) 35
9. (a) 35 (b) 35 (c) 35
66
6
3
7
12. (a) 10 (b) 15 (c) 15 (d) 11. 11
10. 45
15
13. 38

3
14. (a) "7 (b)

16. 26

17.

1
21. (a) 5525
23. (a) 0-001

3"

1
3

(c)

18. 504

. 1
(b) 2197
(b) 0-729

8
15

15. (a) 15
19. 425
22. (a) 125

(c) 0-028

24. 52

(b) 1
3

(c)

20. (a)
44
(b) 125

8
15

51

7
(b) 25
117

(c) 125

1
2

(c)

ANSWERS 491

.20
2
25. (a) 29 (b) 87

11
27. (a) 850 (b) 5525
29. (a) 0'343

. 135
31. (a) 512

(b) 15

3"

<

(b)

43

(b)

3"

13
(b) 125

30. (a) 0'729

(b) 0'972

(c) 0-028

32. (a) 0'343

(b) 0'784

(c) 0'784

36. (a)

92

(b)

97

39. 0'648

"

<2 w~.~

.Y

44
9 (c) 45

1312
34. (a) 25 (b). 25

_6 . A~:~.A

~8<

'X<B~=
t

(b)

124

92

28. (a) 125

~~

35. (a) 15

38. (a)

. 997

(c) 1700

387

(b) 512
(b)

41

26. (a)

(b) 0'973

33. (a) 49

37. (a)

(c) 29

0'4

0-4

. 0-6

0-6

A~

0-4

04

AA
ABA
ABB

B
A

BAA
BAB
BB

40. 0'305

CHAPTER 6
EXERCISES 6

1.
2.
3.
5.
6.
7.
8.
9.
10.
12.

13.

(b) 1-3, 3, 8, 9, Ill, 1-2, 1, 6, 7} (c) R, 15} (f) -2 ~ x ~ 2, 0 ~ y ~ 2 (g) R, R


(a) R, R (b) R, g(x) ~ 9 (c) -2 ~ x ~ 2, -2 ~ h(x) ~ 0 (d) t ;a-; 0, f(t) ;a-; 0
4. g(x)
2x -4,' g(x) ;a-; -4
(a) -3, -12, 3a - 6 (b) 3 (c) x > 3
(a) 8, 8 (b) a 2 - 1, b 2 '- 1, (a + b)2- r (c) No (d) f(x) ;a-; -1
(a) x, rx+2, x + 2 (b) g(x) ;a-; 0
(a) f(x) ;a-; 0 (b) g(x) ;a-; -2 (c) h(x) ;a-; 0 (d) f(x) ;a-; 0 (e) y ;a-; 0 (f) y ;a-; 0 (g) y ~ 4
No, -1 ~ ~ 1, -1 ~ Y ~ 1
(a) f(x) = x + 3 (b) f(x) = x 2 + 2x (c) f(x) = x 2 ...:. 2x + 1 (d) f(x) = 3x2 - 7
(i) 2, -2, 1, -1 (ii) 3, -1, 2, -1 (iii) 4, 0, 2'4, -0-6 (iv) 4, -4, 2, -3
(i) x ;a-; 2; f(x) ;a-; 0 (ii) x ~ 3; f(x) ;a-; 0 (iii) x ;a-; 3 or x ~ -3; f(x) ;a-; 0
(iv) 2 ~ x ~ 3 (v) all x except x = 0; -1 and 1
(a) odd (b) neither (c) even (d) odd (e) even (f)neither (g) e(ven (h) odd (i) neither

14. (a) 2 (b) does not exist

(c) -1

(d) 6

16. (a) 0 (b) -2 (c) -8 (d) 2


17. (a) a
18. (a) f(x) ;a-; 0 (b) f(x) ;a-; 0 (c) 0 ~ f(x)

15. (a) 0 (b) 2 (c)


~

2, b = -3 (b) 3, 17
4 (d) f(x) ~ 16

(d) a 2

(c) 3
19. 1 or 4

CHAPTER 7
EXERCISES 7(a)

1. (a) 1st, 2nd (b) 2nd, 4th (c) 2nd, 3rd (d) 4th (e) 2nd (f) 3rd (g) 1 st
2. (a) 1st (b) 2nd (c) 2nd (d) 3rd (e) 4th (f) 3rd (g) 1st (h) 3rd (i) 4th
3. (a) sin A (b) sin A (c) -tan A (d) -cos A (e) -sin A (f) tan A

(1) 2nd

492 NEW SENIOR MATHS: TWO UNIT COURSE

4. (a) positive (b) positive (c) negative


5. (a) (i) 0'8192, -0'5736, -1'4281,
(iii) 0'8910,
-0'4540, -1'9626,
(b) (i) -0-4226, -0'9063, 0-4663,
(iii) -0'9455, -0'3256, 2-9042,
(c) (i) -0'9781, 0'2079,
-4'7046,
(iii) -0-3090, 0'9511,
-0'3249,
(d) (i) -0-4226, 0'9063, -0'4663,
(iii) 0'5736,
-0'8192, -0'7002,

(d) negative (e) positive (f) negative


-0'7002 (ii) 0'4695, -0'8829, -0'5317, -1'8807
-0'5095
2-1445 (ii) -0'6018, -0'7986, 0.7536, 1'3270
0'3443
-0-2126 (ii) -0'8572, 0'5150, -1'6643, -0'6009
-3'0777
-2-1445 (ii) -0'8480, -0'5299, 1'6003, 0'6249
-1'4281

6. (a) 0'2 (b) -0'2 (c) -0'2 (d) 0'2 (e) -0-2 (f) 5
8. (a)!

(b)

9. (a) -1.4281
11. (a) 0'6428

(c)

-c

(d)

(b) -0'5299
(b) 0'4226

(e)

7. (a)

!
t

-! (f)

(b) t (c) -t (d) -t (e)

(f) -c

(c) -0'7660
(c) -0'3640

(d) -0'3420
(d) -0'9397

(f) -0'1736
(f) -0'8391

(e) 1'1106
(e) -0'7660

EXERCISES 7{b)
(b)

1
2

2. (a) 0

(b)

~
2

3. (a) -1

(b) -.../3

4. (a) 0

(b) ~.

1. (a) 1

1 .(d)-1
- .../3
1
(c) 1
(d) .../3
(c)

(e) -.../2

(f) -.../3

(g) 1

(h) .../3

(f) -.../2

(g) 0

(h)

(e)

(c) -2

(d) -1

(e) -1

(f)

(c) 1

1
(d) .../3

2
(e) .../3

(f) 0

(g)

(g)

1
2
2
(h) - .../3

Yf

(h) -1

()

30

90

150

180

210

270

330

360

sin ()

0'5

0'5

-0'5

-1

-0'5

()

60

900'~

120

180

240

270

300

360

cos ()

0-5

-0'5

0'5

5.

6.

-0'5

8. 135, 315
12. 135, 225
16. 60, 120

7. 240, 300
11. 30, 210
15. 120, 240

10. 180
14. 0, 180, 360
18. 45, 225

9. 30, 150
13. 45, 225
17. 120, 300

EXERCISES 7(c)

1. (a)

25

24
25

(b)

4. (a) cot 2 ()
(e)

2. (a)
(b) tan 2

3.
5
(c) 2 sin ex cos ex

(b):5

()

(f) sin ()

. A cos A

7.

8.

x2
y2
11. a 2 + b 2
13. (a) 1

(b) a sin ()

x 2)

12.
(b) 2 sec 2 v

6. (a)

4(..J5

6)

15
(d) tan ()
(h) cos 2

(g) 1

SIll

5. (a) a sec () cosec ()

(c)

8
17

(b) 17 (c)

()

15
8

{8!8

+ --;+-
10. a-b -b-a

2:

(c) 2 cosec ()

(d) cot 2 f)

(e) 1

(f) sin 2 A

(g) -

(h) cos 2 (}

ANSWERS 493

REVISION EXERCISES A
1. (a) 4x2 - 20x + 25

(b) 5'6 x 106


(c) (i) (9 - 2a)(9 + 2a) (ii) 5y{2x
3
4

2. (a) 2 (b) 0,4 (c) 2


3. (a) 4 (b) 11
4.6 X 10-7
5.5
6. (a) x 2

(b) 16x -9

7. (a) 5

12

(c) 10

(b) 1

(d) 6

8. (a) 5xy{x - 2y - 1)
(b) (8 - 3a){8 + 3a)
(c) (a - 4)(a - 14)
9. {x + 1)2 = x 2 + (x - 7)2; X = 12
10. (a) x > 4 (b) - 2 ~ x < 6 (c) x
11. (a) ,10
(b) 1660
12. (a) 9 - x
(b) x(9 - x)
(c) x(9 - x)

13. (a) 2n

(b) (i) 4 - 2x
x2

15. (a) 1'125 x


16. (a) LDEA
17. (a) (i)

(ii)

23. (a) AB = DC, L ABE

~3

+ c

25. 0,001

28. (a) -1

(b) -eos 0

(b) -

.
22. (a) 0.23

(b) 2-8-v'5

19

(b) 45

L CDF, L AEB = L CFD

(b) :';{a 3 + b 3 + c3)

26. (i)

27. (a) sin 0

5, 4

~ ill)

18. (a) 9-J3 - 9.../6

-4 .J2
2

21.

< 0 or x > 2

= 20;

7 y
21
=,
=2"

(b) 5 f , 5(3

(ii) 1-851 or -1-351

_
19. (a) 1, 4 (b) -1, -2, 4, 5

24. (a) a +

14 x

(b) 3-6 x
LAFB, LDAE LABF

-=--=::'-4-'--..;;;..

3x +4
x2+2x+l

10 19

108

1)

(ii)

(c) -sin 0

(c) -1

(iii)

27

(c)

(iv)

(d) tan 0

(d) 0

(e) 2

(e) -tan 0

(f) -

(f) -sin 0

29. (a) 3

-7.J2

(b) 15

30. (a) (2 - x - h){4 + 2x + 2h + x 2 + 2xh + h 2)


(b) -x(3x + 1)(2x
3)
In
1
3
31. (a) a - b (b) a - 2 b
+ b 32. (a) 3i (b) 0-04 (c) 2j (d) 0-3 33. (a) 8" (b) 289

(a)

34. (a) (4x - 1){4x + 1)

35. (a) 50 (b)


38. (a) x

2~

;;li!:;

(b) (3x

(c) 108

36. 4 em

sin 2

40. (a) 0-6 (b) 0-6 (c) -0'8


42. (a) 210

43. (a) 1

(d)

44. (i) 2x3 - 3x2y + 3xy2 - y3

45. (a)

~3

(e) -0-75

330 (b) 90 , 270 (c) 135


8
2
(b) 15
(c) 3"

(b)

37. (i) (a) 18 (b)

~~

(ii) (a)

(c) -2 ~ x ~ 2
3
5
(ii) (a) (b) -.J34

(d) (2a - 3){4a2 + 6a + 9)

(c) (a - b){a + b + 2)

(b) all x exeept x = 2

39. (i) (a) 1 (b) cos 0 (c)

+ 7)(x - 1)

315

(f) -0'6

15~

1~5

(d) all x_

41. (a)

1
t

(b) t

(c)

m - 4

l){m - 2)(m

+ 2)

2){X2 + 2x

46. (I') 7

2M

CHAPTER 8
EXERCISES 8(a)

1.
4.
6.
7.
8.

(ii) 1O-J3 em
2. (i) 10 em (ii) 1O~ em
3. (i) 15-J3 m
(i) s-J3 m (ii) 5 m
5. (i) 4 em (ii) 4..)3 em (iii) 4.J7 em
(ii) 1O-J3 em
(iii) s.J2 em
(iv) 5..fi em
(i) 10 em
(i) 6-J3 em
(ii) 6 em
(iii) (6..)3
6) em
(iv) 6.../6 em
9. -J3 m
10. 30-J3 em, 30 em
(40 + 8..fi + 8..)3) em
(i) 10 em

(d) -t

(d) 30, 300 (e) 30, 210 (f) 45, 135

(ii) (a) (4y + 3){2y - 3) (b) x(x


(c) (m -

(b)

(ii) 30

+ 4)

I1

494 NEW SENIOR MATHS: TWO UNIT COURSE

EXERCISES 8(b)

1.
4.
7.
10.

(i) SW (ii) N 60 0 W ,(iii) S


2. 50 m, N 30 0 W
3. 13m, 21023' T
(i) 25 km (ii) 24 (iii) 30652'T
5. 192 km, 3084O'T
6. 16-38 km, 11-47 km

8. 39'74 km, 2-54 h


(iii) 20247' T

13 km, 11237' T
(i) 41-45 km

9. 20 km, 2338' T

(ii) 17-41 km

EXERCISES 8(c)

1. 12-4 m
2. (i) 8396 m (ii) 504 kinlh
3. (i) 45 m (ii) 75 m (iii) 6723'
4. 615 m
5. (i) 137-4 m (ii) 96-2 m (iii) 1835'
6. (i) 23-32 m (ii) 1825'
8. 7 m
9. (i) 21-16 m (ii) 4637'
7. (i) 13-03 m (ii) 13-14 m
10. (i) 129 m (ii) 37 m

11. (i) 75-J3 m, 25-J3 m (ii) 50-J3 m


.

12. 80-J3 m
3 '

EXERCISES 8(d)

1. C
5. 15
11.
14.
17.
20.

75, b

15-45

7. 14-71 em

2. A

8. 8-428 em

15, a

3-67, C

9. (a) 0'3

12-2

(b) 4-8

3. 18-81 em

10. 3-26

4.

81 03',61 57'; 24 57',118 03' 12. 3-88 km 13. 16 32',163 28,;c2'07 em, 40-42 em
20-2 units
15. (a) 0-5 (b) 30, 150
16. (a) 2-146 m (b) 1-7S8 m
(a) 8'638 em (b) 1-929 em
18. 9-903 m
19. (a) 561 km (b) 116 09' T
74 km
21. 1578 m, 1149 m
22. 92-54 m

EXERCISES 8(e)

1.
6.
8.
10.
14.
17.
20.
23.
25.

0-7308
2. 6-083,43 40',21
3. 11'73 em, 24'5 em
4. 13-12,8
5. 24-19,23 23'
109 28'
7. (a) 73 24' (b) 7 em (c) 25 13'
(a) 24 37' (b) 3-33 em (c) 18-33 em 2
9. (a) 78 28', 101 32' (b) 11-49 em
18-97
11. (a) 14-34 (b) 52 45'
12. 4-67 em
13. 10-63 km
16. 8'62 km; E 55 7' N
44-9 km
15. (a) N 30 E (b) E 21 47' N
10 em; 29-39 em 2
18. 20-06 em; 122-5 em 2
19. 39-97 m, angle of depression 0 56'
212'3 m
21. 2'655 km
22. (a) 47 09' (b) 3-715 em (c) 9-165 em 2
(a) 7 em, 11-36 em (b) 20-/3 em 2
24. 12-5 em
(a) 103-4 m (b) 964-2 m 2
26. 3420 m 2
27. 418 km

CHAPTER 9
EXERCISES 9(a)
1. (a) -1

2. (a) 45

(b)

(c) 0

(b) 26 34'

(d) -1

(e) -1

(c) 159 27'

(f) 5:

(g) -

(d) 56 19'

(h)

(e) 153 26'

(f) 135

EXERCISES 9(b)

. 1. 3x - 4y + 38
0
4. 2x
y
12
0
7. x + y + 7 = 0
10. x = -2
2
3
_
13. -3 14. 2 15~ 4 16.
21. (a) 3x +. 2y

2.
5.
8.
11.
3

-2

x + 2y + 2 = 0
3.
x
y =0
6.
3x + 4y + 5 = 0
9.
5x -2y = 10
12.
4
2
17'"5 18'"5 19. (a) (6,0), (0,

0 (b) 2x - 3y - 13

=0

22. (a) 4x - 5y

8x - 7y - 3 = 0
4x - 3y + 17 = 0
y= 2
2x + 3y + 6 = 0

4) (b) -3 20. (b) 3

= 0 (b) 5x + 4y = 0

ANSWERS 495

= 0 (b) 5x + 2y - 14 = 0 24. (a) 4x - 3y = 24 (b) 4x - 3y = 21


(a:) x + 2y = 2 (b) x + 2y + 8 = O
26. x + 3y + 18 = 0
3x - 2y = 14
28. (a) 2x - 3y = 6 (b) 33 42' (c) 2 (d) 3x + 2y = "22
(a) 2x - 5y + 31 = 0 (b) x + 2y = 7 (c) <-3, 5)
(a) x + 2y = 9 (b) 2x - y + 2 = 0 (c) - 2 (d) 2x - y
8; x + 2y + 6 = 0 (e) (2, -4)
32. (a) x + 5y = 0 (b) 2x + y = 0 (c) 4y = x - 7
~
33. y = 4x - 5
34. (a) x + 6y = 17 (b) x + 6y
32
.
23. (a) 5x + 2y

25.
27.
JO.
31.

35. (b)

(-~,

-t), tl~, ~),

(I, I), (0, 2)

(e) x - 14y

13 = 0, llx - 4y

8 = 0

36. (a) 3y - 4x = 1 (b) y = 2x - 1 (c) x = 2


37. (-3, 5)
38. (a) (-9, -7) (b) (-2, 0)
39. (a) y = x -7 (b) + 2y = 12 (c) (8~, 1~) (d) 108 26'
40. (b) (4, 3)
\ 41. (a) coincide (b) intersect (c) intersect (d) are parallel

EXERCISES 9(d)

7. yes, no, yes


8.
11. yes, yes, yes
12.
13. y = x, y = 1
16. y = x + 1,
-2x
19. (2, 2), (2, 1), (1, 1)

22.(~, 2~'"(0, 2), (2, 0)

no,
no,
14.
17.
20.

no, yes
no, no

23.(9,12), (-2'2, 0'8),

no, yes, yes

15. x = 2, x + y = 1
18. x + y = 1, x + 2y = 2
21. (0, 1), (2, 2), (3, 1~)

y
2x + 2, x + y = 2
y = x-I, y = -1
(0, 0), (1, 2), (I!, I!)

24. (0'6, 5'8), (-1, 1), (l~, 3~)


26. (0, 0), .(1, 0), (1'5,\0'5), (0, 2)

10.

9. yes, no, yes

(-t, -6~)

25. (-0'6, 1'2), (-2, -3), (2'2, -0'9)


27. (2, 0), (0, 2), (-2, 0), (0, ""2)

EXERCISES 9(e)

1. la) 9 (b) 5 (c) 2-JPi" (d)


5. 3../2, 5../2
7. 5, 10, 5..[5

(e) 5a (0 .J2(a - b)2


2. (15 + 5..[5) units
11. (b) 12 units
14. (a) -v34 units" (b) (9../2 + -v'34) units

-v'2f8

EXERCISES 9(f)

i. 5 units

.2. 2 units

7~ 2..[5 units
12. 2..[5 units
14. 2'5 units

6. 2../2 units
11. 3.J2 units
13. 4 units

3. 8 units

4 -Jill umts
.

8. 0'4 units

9. 4 units

.T

(ii) (2, 7)

2. (-11, 14) 3. (0, 3),

5. 2x

+y +6 0
+ 2y + 6 = 0

(iii) (-4, -8)

(4,

1) 4. E =

6. 6x

10. 2 units

15. 0'5 units

EXERCISES 9(g)

1. (i) (I!, -1)

5. 13 units

8y

(iv) (1, 1)

0,!),

1= 0

(v)

= (3i,
7.

e; b; d)
c,

(vi)

(a, a ;

-3!), G = (-1, -3!), H

(!, !)

(1, !} diagonals bisect each other.

8. 5x
9. (b) (0, 2) (c) 1, -1 (d) Line drawn from vertex to mid-point of base is ..L to base.
10. (a) (2!,2) (b) Diagonals bisect each other.
11. (b)

(1~,

-!). Diagonals bisect each other (c) -

12. (b) (-I!, I!)


14. (a) x

=0

(c) 5f units

and 3. Diagonals at right angles.

13. (a) (l!, 5), (3,

(b) 2y - x = 1 (c)

(o,!)

b)

6!>, (-!~"6), (""2, 4!>

496 NEW SENIOR MATHS: TWO UNIT COURSE

CHAPTER 10
EXERCISES 10(a)
1. (a) y
5 (b) y
-1 (c) 3x + y = 7 (d) 3y - x
5
2. Y
3
3. x
4. Y = --2!
5. Y = x 2
6. x 3y = 0
7. x - y - 3 = 0, x + y - 5
8. 3x 2y 0
9. 3x - 4y - 26 = 0, 3x + 4y + 14 = 0
10. 5x - 2y + 13 = 0,5x + 2y + 17
0
11. 2x + 3y - 8 = 0
12. (a) 8x + 8y
3 = 0 (b) x + y - 1 = 0
13. x = 3a

-4
0

EXERCISES 10(b)
(b) x 2 + y2
6x + 6y + 13 = 0
1. (a) x 2 + y2 + 2x + 8y + 8
0
2
(d) 4x2 + 4y2 + 12y - 55
0
(c) x + y2 + 4x - 5y - 2 = 0
(e) x 2 + y2 - 8x + 7
0
2. (a) x 2 + y2
6x - 4y - 40 = 0 (b) x 2 + y2 + 2x - 8y = 0 (c) x 2 + y2 = 25
3. (a)

(li,

-lil,

~ units

(b) (-2, -1), .JfO units

(c) (3, 0), ..J3 units

(2, -

4.

5.
10.
12.
14.
15.

16.

(e)
~), ~ units (f) (-I!,~).~..J385 units
(d) (-a, b), 2..fi units
2
(a) x + y2 - 12x + 2y + 3 = 0
(b) x 2 + y2 - 5x + 3y = 0
2
(c) x + y2 - 3x - 11y + 14 = 0
Inside 6. Outside 8. (a) (-3, 4), 5 units (c) 4x + 3y = 0 9. x 2 + y2
8x - 8y
(-8, 1)
11. (a) x 2 + y2 + 2x - !4y - 20 = 0 (b) 2..J2f units (c) 2.J7 units
x 2 + y2 - 2x
6y + 1 = 0
13. (a) 5,units (b) 2..J2f units
(a) (2, 1), .v4f units
(b) 2~ units
(a) (2, 5) (b) x 2 + y2 - 4x
lOy + 16
0 (c) (0, 2), (0, 8)
(a) (2, 4), 5 units
(b).../5 units, 4.../5 units

+ 16 = 0

. EXERCISES 10(c)

1. x 2 + y2 3. 3x2 + 3y2
5. x 2 + y2 =
7. x 2 + y2
9. x 2 + y2 11. x 2 + y2 13. x 2 + y2 -

+ 2y - 11 0
14x + 6y - 2
0
9; (0, 0); 3 units
6x + y
3 = 0
16x + 4y + 28 = 0; (2, 0)
2y - 3 = 0
4x = 0
4x

2.
4.
6.
8.
10.
12.
14.

Centres lie on line x + y - 6 = 0


3x2 + 3y2 + 26x - 28y + 23 = 0
Arc of circle x 2 + y2 = 9,0 < x < 3,0 <y < 3
(a) x 2 + y2 = 25
(b) ( 5, 0), (0, 5)
x 2 + y2
2x + 6y
24 = 0
x 2 + y2 + X - 3y
0
x 2 + y2 - 4x + 2y
0

CHAPTER 11'
EXERCISES 11 (a)
1. 0, 30, 72, k 2 + k,2n

121

2. (a) 2, 2' 9' 8

(c) a

1 1
3'4

(d) 3, 9, 27, 81

(f)

+ b,2a + b, 3a + b, 4a + b

1 1 1
12' 20

2' 6'

3. 1, 1, 2, 3, 5, 8, 13, 21

4. (a) 11, 13, 2n

1 (b) 24, 35, n 2

1 11

1 (c) 25'36' n 2 (d) 17,31, t n -

EXERCISES 11(b)
1. (a), (b), (d)

2. 50,86

3. 14-8, 11-2

4. 9q

8p

tn -

tn -

ANSWERS 497

5. 11 - 3n

6. -3,2,7, ...

1'
8. 3(2m

7. 1'4

n)

10. 2p + 3
11. -3, 2, 7, 12, 17 12.-3
14. 198
15. 294
16. 345
(a) 176'4 m (b) 53-9 m
18. 3275
19. 157!
20. n 2
13
22. 24
23. 3, 5, 7
24. (a) 2550 (b) 1050 (c) 550 (d) 3050
(a) 2n 2
14n (b) 16
26. 80,94,108,122
27. 115
15
29. 16
30. 12; 276
31. 297 m
32. 6n - 14
33. 47
35. 9900
36
- b + c - 2a, (b + c - 2a)(c + a)
34. -3, 3
n b _ a '
2(b - a)

9.
13.
17.
21.
25.
28.

3
9

38. (a) 12 (b) 26

37. 1, 3, 5
42. 6, 10

43. (a)

45. (a) 21

n2

(b)

(b) 120

40. 9 em, 12 em

39. 24

n2

46. 2

(c) ~(n

1); 20

41. 20

44. 15

47. 105, 115, 125, 135, 145

EXERCISES 11 (c)

1.

(i) 12 + 22 + 32
(iii) 1 + 3 + 5 +

(v)

x +

2X2

+ 42
... + (2p - 1)
+ 3x3 + .. - + 8x8

(vii) 12 + 32 + 52
(ix) x + x 2 + ...

2.

Ek

(ii)

1)2

(viii) 1

E (5k -

(3n

1)

12

(vi)

k=2

k=1

3. (i) 95
(v) n 2

Xk

+ ... +

k=1

E k(k + 1)

(v)

E k(k + 2)

(iii)

4)

4
10

k=1

k=O
p

+ ~3 + ... + ~

10

(iv)

! +

(vi)

+ .. - + (2p +
+ xr

E xk

0)

(ii) 1 + 3 + 32 + 33 + 34 + 35
(iv) 1. 2 + 2. 3 + 3.4 + 4. 5 + 5.6

(vii)

E ar"-1
k=1

r=2

(iii) 2n 2

(ii) 152
(vi) 185

E rx
(iv) 3n 2

5n

EXERCISES 11(d)

1. (a), (b), (d)


6.2

7. (a)

2. (a) 30i (b) 831 3. 4, ~ 4. 27


64 (b)

5~!

10. (a) 162'-"3 (b) 242('-"3

9. 3, 3

i3x - - 1] ;

x[(~x)n
13.

8. (a) 255

1
14. 1, 1; 4'

189

5. (a) 5, 10, 20, (b) 64, - 32, 16

(b) 156f:ls

1)

-21

(c)

;;:~

(d) 15-94

11. 10ft

12. 176.5

V35

16. 5905

15. 6 -

(e) 569,9

17. $1131
22.

-6"1

1
18. 64

20. 6, 12

19. 810001

24. a

23. 9, 6, 4; 45, -30, 20

1 2 8 32
25. 2' 3' 9' 27

26. 9

27.

31. 16, 8, 4; 48, -24, 12

37. 4 -

vfis, 3.../5 ; 4'-"3

39. P

32, q = 99

40.

~~

29. 0, 10

a+ 1
33. - - ; a
a

38. (a) 2178


41.

If) 4~~

27~

5~

(b) 16

(c)

17~

(d) 6

4'-"3
3

5, b = -10, c = 20

28. 3

32. 36

21. (a)

<
(c)

42. 3t

30. 2

-2 or a > 0

35. 18'316

36. 0

498 NEW SENIOR MATHS:TWO UNIT COURSE

EXERCISES 11{e)

1.
3.
7.
11.
14.

2. (a) 5-1 thousand (b) 41-8 thousand


(b) 4965
5. $358952
6. (a) $983 (b) 9
4. 4 to 5 years
20407
9. $861
10. $1800
8. $4818
$1139
12. $2528
13. (a) $3587 (b) $22744 (c) 13-4 years
$180-76
(ii) weekly
(i) (a) $1338-79 (b) $311-12 (c) $110-11 (d) $25-39
(a) 1245

CHAPTER 12
EXERCISES 12(a)
1. (x
1)2 + 5; minimum 5; range y ~ 5
2. 2(x - 1)2 - 2; minimum -2; range y ~ -2
3. -2(x - 2)2 + 5; maximum 5; range y ~ 5
4. -4(x - 2)2 + 23; maximum 23; rangey ~ 23
5. 4(x + 1)2 - 11; minimum -11; range y ~ -11 6. -2x2 + 8; maximum 8; range y .~ 8

9. -5(x + 1)2 + 11; maximum 11; rangey

7. -(x + 1)2 + 8; maximum '8; rangey


.

11. (a) 0 .~ t ~ 4 (b) 20 m


16.
15. 50 m 2; 5 m, 10 m

8.

1110. -(x -

12. 12! m; 50 m

~ m, 3~ m

2(X ~r -~; minimum -~; rangey ~ -~

k) + IIi; maximum II!; rangey ~ IIi


14. 225 cm 2

13. 10, 10; 100

17. 30000 m 2

18. 5, 25

19.

EXERCISES 12(b)

1. (a) 28, two


2. (a) yes

(b) -23, none


(b) no

(c) 0, one
(c) touches

1O~;

(d) -8, norie


(d) yes

5.

(f) 0, one
(f) no

~, Ii

3. 64; 3, -5

4.

6. 0; I!

7. 1764; 3

9. 0; 1

10. -368; none

11.41; 1 4.J4f

13. 85; 7 6.J85

14. 1; -5, -6

12.28; -1

~~

.15. 17; 9 8M

16. 0;

9 2.JfOf

(e) 65, two


(e) no

8. 36; 0, 6

17. 76; -2 3 "ff9

-Ii

18. 33; -1 4.J33

19. 1; 7, 8

21. 17; -5 +4 M

2
22. 9; 1, 5"

49;

20. 9; 0,

I!

EXERCISES 12(c)

1.
5.
9.
13.
17.
21.

-1, 7
0, -5, 1, -6
0, -2
-1, 2
2, 1, -4
1, 3

25. 0, 1
29. log23, log25

2.
6.
10.
14.
18.
22.

-2, . . 1, 3, 4
1, 3
2, 3
1, ':'4
1
2, 3

26. 1,

-1

3
30. 1, 2

3. -1, 3, -5
7.0,3
11. -1, 1
15. 2
19. 3
23. li, -J3

4.
8.
12.
16.
20.
24.

27.. 0, -1

-1, 7
1, 2
-1, 2
2!, 3
2, -1, 3
.J5

.28. 1,

2
32. 2,log s 3

31. 1, 2

EXERCISES 12(d)

1. (a) x 2 - 3x - 10

=0

(b) 6x2 - 5x +1

=0

(c) x 2 4x + 1

=0

(d) x 2

3px + 2p2 = 0

ANSWERS 499

2.2,8
6. p

= -1, q = -6

11. (a) _
15. y =

4. px2 (P2

3. ,(a) '-4 (b) 1 (c) -4

'1

8. -3, -6

(b) q2 - 2rp

x 2
-

18. (a) -5

6x

(b) -8

r2

(c) =sJ!."
p2

16. 3

(c)

9.

l)x

+p =0

10. (a) 3 (b) 3 (c) 0 or 12

12. b = 9, c

14. Y

= x2

17. 0

(e) -14

(d) 40

19., (a) 7 (b) 2

EXERCISES 12(e)

1. (i) x > 5 or x < -2 (ii) -2


2.

7.

I x except x

8. (i) -2

:E;

I! 3. x ~ p or x

2 (iii) x

(ii) x' < -I' or x

3 <x < 1

(v) 1 -; .J5

<

<2

(ii) -10

<

<

21

<2

kor x

~6

(iv) -5

> ~

(iii) x

< X < 1 +2.J5

(iii) -2 - -Ji

<

< 1-

(vi) -3

< -2 +

<

6. (i) a2

4. x > 1 or x <: -5

:E;

:E;

< -5 or x > 1

(i) x

(iv)

:E;

-Ji

<

= 4b

t
(ii) a2

-Ji or x

> 4b

> 1 + -Ji

<x <3

(iv) x

< -

kor x > 3

EXERCISES 12(f)

= 21

1. (a) (i) k

(ii) k

(c) (i) k = -1, -5

2. k > 1 or k < -11


4. (i) --Ji

5. (i) m

:E;

:E;

< -5 or k > -1

(ii) k

-Ji

2.J3 (ii) k > 2.J3 or k

(b) (i) k

(d) (i) k =

t,

(ii)!

< -2.J3

< k < 3

3. m = 1 or -3
(ii)

:E;

:E;

.J3 (ii) all m except m

(iii) m

-../3

10. (i) m = 3 or 5 (ii) m > 5 or m < 3

:E;

-4 or m

9. (i) p

12. -2 < m

2 (ii) p

<

<2

EXERCISES 12(g)

1. a = 1, b = -1, c = 1; n(n + 1) - (n
2. a = 14, b = -9; 14(2x2 + x)
9(3x2
3. (x2 - 2x + 1) + (x - 1); p = 0, q
5. a = -3, b = -2, c = -8; (x 2 - 3X)2 6. a
1, b = -5, c
6; (x2 + 2X)2 -

~ 1
4. a = 1, b = 6
2(x2 - 3x) - 8; -1,1,2,4
5(x2 + 2x) + 6= 0; 1, -3, -1 .J3

7. (x - 2)2 +2

9. (x

8. 2(x - 1)2 - 7

+ 1) + 1

x)

1r

11. a = 1, b = -1
12. a = 4, b = -3, c =
13. a = 1, b = 1; (x - l)(x + 2) + (x + 2)

+ ~r

+~r

+ 31
8
-2; 4x(x + 1) - 3x2 - 2(x + 1)
10. 2(X

EXERCISES 12(h)

1. (2, 6), (-2, 18)

5.
9.
.12.
13.
14.

2. (0, 2), (2, -12)

(0, -3), (3, 0)

6. (2, 3), (3,


10. 0,
(i) 4 or -8
(ii) m > 40r m <
(i) 1 or 9
(ii) m > 9 or in <
(i) -5 or 3
(ii) m :> 3 or m <
8
15. a > 0 or a < - 9
16. -3 < a
2-Ji, ( -Ji, =F-Ji)

5)
-Ii
-8
1
-:-5

<0

3~ (1, 2), (-5, 8)

7. (1, 1)
11. 2,-18
(iii) -8 < m < 4
(iii) 1 < m < 9
(iii) -5 < m < 3
17. Y = 2x

4. (2, 1),

(-~, -~)

8. -4!

1; Y = -2x, + 5

18. (2, 3)

500 NEW SENIOR MATHS: TWO UNIT COURSE

EXERCISES 12m

=
=

(b) y
(b) y

1. (a) (0, 0), (0, 2)


3. (a) (0, 0), (0, 1)

k)

5. (a) (0, 0), (0, -

(b) x

9. (a) (1, -2), (1, -1)

(b) y

13. x 2 = 8y
14. x 2
18. (a) (y + 1)2 = 4(x

22.
25.
27.
28.

6. (a) (0, 0), (0, -I!)

= -~,y =

8. (a) (0,0),

-3, x
-2, y

= -16y
+ 1); (-1,

(b) y

-k)

16. x 2

4y

I!, x

0
0

(b) y

17. (x

-8y

=k, x =

-3

= -~, y = -4
+ 2)2 = -4(y - 5)

(b) x

12. (a) (-1, -4), (-Ii, -4)

15. x 2

=
=

(-k, 0 ) (b) x = k,y = 0

10. (a) (-3, 0), (-3,

-1), (0, -1); x = -2

(x -

(-k, x

= -8
~) (l!, 2),
2); = 3!
(c) (x - 1)2 = -4(y + 3!); (1, -3!), (1, -4!); y =- -2!
(d) (x + 3)2 = 5(y + 5); (-3, -5), (-3, -3i); y = -6!

x 2 = 8y
20. x 2 = -4y
(x - 2)2 = 16(y + 1)
23. (x - 2)2 =- Y - 5
(x - 2)2 = 4(y + 1)
26. x 2 = 8y
(i) (x - 2)2 = 4a(y - 1) (ii) (x + 2)2 = 2(y - k)
(i) (x - 2)2 = y; (2, 0), ( 2, ~) y = -~, x = 2
(ii) (x + 3)2 = Y + 3; (-3, -3), (-3, -2i), y = -3!, x = -3
(iii) x 2 = -(y - 4); (0, 4), (0, 3i), y = 4!, x .= 0
(iv) (x + 1)2 = 4(y + 1); (-1, -1), (-1, 0), y = -2, x = -1
(b) (y - 2)2

19.

2. (a) (0, 0), (0, 3) (b) y = -3, x


4. (a) (O~ 0), (0, -2) (b) y = 2, x

0
0

(b) x

11. (a) (-1, 2), (0, 2)

=
=

k, x =

(b) y

O),(~, 0)

7. (a) (0,

-2, x
-1, x

21. x 2
= 28y
24. (x +-1)2 = 8(y - 3)

CHAPTER 13
EXERCISES 13(a)
1. (a) x lO
5y

2. (a)

3. (a)

X ll y 2

(b) 2 13

(c) p13 q l4

(d) a ll b 6

(b) 2 6n

(c) a 3 b 4

(d)

2~3

(e)

(b) _1_

(c) a 3 b

(d)

!L

(e) 2x

X 9y 7

7. (a) 1 (b) 1

(c)

8. (a) 3n+153n-1

(b)

(d) a-2

x2

y2

52n +3

(b) 3n -

9. (a) 3.27

11. (a) 3 (b) 4


12. (a)

:2 -

(b) 2

(f)2 n + 3
64x 9

-'-
27y 3

10-4

0)

(e) 4
(c) 2 8

(d)

~ + l + l + ~3

x6

x5

X4

(e) x - 3

(f) x2-2x

.-----:-~--

(b)

(d)

+ xy + }y

(c) 27

+4

(e)

1~

(c)

(f) 2!

;4

(g) I!

(h) 2 (i) 1 (j) -2, (k) 6, (1)-3

(d) 3 - a 2b - a;b

(e)

a~

(0

EXERCISES 13(b)

1. (a) 16

1(i)-5

(c)-2

(c) -5

(ff'a 2b 3

pr4
y2 Z
4c
xm
+
2n
b
2p
4. (a) x4 (b) a
(c) 2 2n - 6 (d) 3x (e) 2 6n , (f) () - - (h)
a2
g
yn
5. (a) 2-2 (b) 2-4 (c) 2-5 (d) 2-3 (e) 2-6 (f) 2-7 (g) 2-2 (h) 2-9
6. (a) 102 (b) 10 1 (c) 10 (d) 10- 1 (e) 10-2 (f) 10-3 (g) 10-3 (h)
mnp8

(f)m ll n 6p

(e) 29 X l6

(d) 9

(e) 2

(f) 5 (g) 64

(h)

(i) 6

(j) 5

(k) 2

ANSWERS 501
2. (a) 6 112

(b) 25/8

(c) 24/3 (d) 3 11/6 (e) a17/12 (f) 2112 (g) a 1/2 (h) X516 (i) 2 (j) 52
(k) 22
3
13/6
1l2
3. (a) X
(b) b2 (c) x - 1 (d) 1 (e) x - y (f) x - 2x y 1l2 + y (g) 27 x (h) 1
a
x
y
4

(b) y1l6

y6

(y7
)

3"

+ y1

8. (a) a 21b 2 (b) x - 2 (X)ll2

x-x+1

(e) 6 112

(c) X2/3 - y2l3

1
(c) 2"

(b) 6

(d) all/6b4/3

(c)}

6. n= 2, a =

S .,y4' 54

9. (a)

1
(f) (g) 9X l/2 (h) 1
4x

16
64

7. (a) 25 (b) 100 (c) 0-9 (d) 27 (e) 1 (f) 3

116

4. (a) x 2l3 y

(d) x 2

(d)

10. (a) 26n (b) 53n - 2 (c) 9 (d) 18 n


11. (a) a3/4b1l2
(b) 1000
(c) 36
1
12. (a) - 2"
(b) 3
(c) -3
(d)

(f) a 2 b
(d) 12..J2

(e) 1

(e) 0

13. (a) 3 (b) 1~ (c) Ij (d) 0, -2 (e) 2, 0

(f)-3

(f)-2

EXERCISES 13(c)

1. (a) 2

(b)

2. (a) 4

(b) 4

3. (a) 27

(c) 7
(c) .

(d) 1

2!

(b) 9 (c) 4

4. (a) 125

(b) 4

(c) 4

(e) -3

(f) -1

(d) -3!

(e) -

(d) 9

(e) 216

(d) 4

(e) 27

(g) 10

(f) J

(h) 3

(i) 3

(g) -4

(h) -I!

(g) 3!

(h) 49.fj

G)

(f).J3

7. (a) 8

(b)2

8. (a) y

(b) y

(e) y

(e) 10

(h) 2

(f) 10

lOX 312

(f) y = 5x3

25x

(j) 16..J2

(i) 16

(b) 1 (c) 8 (d) 1 (e) 3 . (f) 0 (g) 3 (h) 1 (i) 12


(b) x (c) 3 (d) 2 (e) 6log lO 5 (f) 3 (g) loglO (2 +.J5)
(d)

(I)

(f) 7

S. (a) 7
6. (a) 2

(c) 100

1 (k) 0

= ..J2

(c) y = 3X3/2

(d) y

(g) X2y 3 = 1

lOX - 1
(h) y = lOX + 1

Ib

(d) m(e)
(f)2!
(b) 2x - 2y
(c) y
x + 1
(d) x + 3y
(e) x + 3y
(f) y
2x + 2
(g) y - x + 2
(h) 3y - 2
(i) 2y - x
(j) x + y - 1
(k) 4y
(I) x + 2y - 1
11. 0-4772, 0-6020, 0-9030, 0-9544, 1, -0'3010, -0'4772, -0-6990, -1
1
y
12. X= jJIOglO
13. A = PlO bt
14. Y = ax n
IS. x = 2Y
9. (a) 30 (b) 10
10. (a) x- y

(c) 10

16. logx

210g a

3
1
+ 2"log
b + 2"log c

2~

19. (a) 2p . (b)

17. 1'1

21. (a) 10, 10-512

(b) 10 (c) 4, 2-514

(d) 9, 27

EXERCJSES 13(d)
(i) 2-322
(vi) 07737
2. (i) 2-807
(vi),. 2~096

1.

3.

x
log5 x

(il)
(vii)
(ij)
(vii)

2'262
0'6826
2-631
1'723

0-2

-1

0
"

(iii)
(viii)
(iii)
(viii)

1-861
1'465
0-4307
1'161

(iv)
(ix)
(iv)
(ix)

0'43

0-86

0-7925
3-322
-0'7565
-2'322

(v) 2'524
(v) 1'699

10

1-29

1'43

502 NEW SENIOR MATHS: TWO UNIT COURSE

4.

0'5

-0'5

1'29

1-5

0'5

~ 0'43
(b) x ~ -1-47
(c) x < -1-46
(e) x ~ 2'32
(I) x < 0-29
(g) x > -4-92
8. (i) domain: x > 0, range all real numbers
(ii) domain: x > 0,
(iii) domain: x > -I!, range all real numbers (iv) domain: x > 2,
(v) domain: all x except 0, range all real numbers
9. (i) domain: all real numbers; range: all positive numbers

5. (a) x

(d) x < -3
(h) x ~ 3'38

range all real numbers


range all real numbers

(ti) domain: all real numbers; range: all numbers greater than 1

all positive numbers


all numbers greater than --Z

(iii) domain: all real numbers; range:


(iv) domain: all real numbers; range:

REVISION EXERCISES B
1. ~

2. 92 0 52'

3. 9-83 cm

4. (a) (2, 2)

(b) y

-x, y

-4x

5. (i) (a) 3y + 2x
4
(b) 3y + 2x + 9 = 0
(b) (2, -3), (5, 2), (-3, 0)
6. 3y = 4x - 7
7. 3-1 units
8. (i) 3y + x + 3 = 0 (ii) x 2 + y2 + 4x 6y
10. (-3, -10)
11. 2250
9. (a) (1, 1); 5 units (c) 3-J5 units (d) 2-J5 units
12. (i) 12 (ti)380 m

15. (i)

16. (a) 1 (b) x < 5!

18. (a) 3 (b) 3loga y


21. 2081668
25. (a) none

(c) IOglO 3

22. (a) 322 (b) 1543


(b) two

(c) one

(c)

~ 2-262

17. (i) tn

19. (a) aX (b) 100

(d) 0

(d) two

24. (a) 9

26. (a) -1, 2,

2n

4 (ti) 93

20. $1788, $1629'

23.

14. 3vi32- 5, 7 -24vi3, 2f

13. 2, 5, 8 or 11, 5, -1

(ii) 3

30

12

(b) -13!
(b) 3

(c) 1, 2

27. (a) k < 7* (b) k = 7* (c) k >71


29. (2, 1), (4,5)
30. a
2, b = 3
31. (a) m
1 or -11
(b) m < -11 or m > 1
(c) -11 < m < 1
32. (x + 2)2
4y; vertex (--Z' 0), focus (--Z' 1), directrix y = -1, axis x
--Z, focal length 1 unit
33. (x
3)2 4(y + 1); vertex (3, -1), focus (3, 0), directrix y = --Z, axis x
3, focal length 1 unit

34.

0-25

0'5

--Z

-1

2'58

I log2 x

35. 16 cm, 12 cm
36. a = 2, b = -3
37. (2, -2)
38. (4V3
...[2) m
40. (a) 64'3 m (b) 54 m (c) 20 28'
39. (a) 25'17 m (b) 39'16 m (c) 22 45'
41. (a)

1O~ km

43. (a) y

5x

(b) 359 0 6' T

+8

(c) PA

42. (i) ...[2 units

= PB =

...ffj, units

(ii) 1'4 units; 3'5 units 2

44. (b) (3, 2) (c) diagonals bisect each other

CHAPTER 14
EXERCISES 14(a)

1. 9

9. 2

2. -3
10. 5

3. 0
11. 12

4. 9
12. 1

5. -32
13. 1'8

6. 0
14. 11

7. -10

15.

:3

8. 0'8
16.

61

ANSWERS 503

18. 2X2
(b) 4x

17. -0-25
23. (a) 2x

3
3

19. 4
(e) 3x2

20. 3
21. 1
(d) 6 - 2x

22. 2

EXERCISES 14(b)
1.
2.
3.
6.
7.

4x - 3 + 2h
(b) 4 - 6x - 3h
(e) 3x2 + 3xh + h 2
-3
(b) 6x + 2 + 30x
(e) 7 - Sx - 40x
2 - 2h (b) 2
4. (a) S (b) 4x - 4
5. (a) Sx (b) x - 2 (e) 3x2 - 4x
6x - 4 (b) Sx - 2 (e) 2x + 1 (d) 6x - 6x2 (e) 4x3 (f) 4 - 16x3
3x2
6 (b) -3 (e) (2, -4), (-2, 4)
8. (a) 6x - 2, (b) -14 (e) 6a - 2 (d) 16

(a)
(a)
(a)
(a)
(a)

9. (a) 4a3 - 9a2


. 11. (a) 9}t'
4x
(e) 6x + 2 .

+2
+5

(b) -2

13. x

12. 5, -5

15. (a) 2x
16. (a) x

h - 3

18. (a) (1, 1~, (2, Ij)


(b)

-}.

10. (a) 2

(e) x

(d) 0,

(e) 9x2 - 4x - 7
(d) ISx2 - 2x - 2
(g) 24x2 72x + 54 (h) 3x2 + 6ax + 3a 2

(e) (i) (1!, -2})

(e)

(ii)

17. (a) (3, 0)

(~, -1 ~~)

(b) (1, 2)

(e) (-1, -2~), (4, 6!>

(b) (0, 1), (3, 2!>

(-~, -zt)

(b) -1, 1 (e) 3

14. S, -S

(b) 2x - 3

1 (b) x

20. (a) (-I, -3)

(e) -66

(b) 9x2- - 6
(f) 3ax2 + 2bx

(-~, -2!)

21. (a) 24

(~, ~)

(e)

19. (a) (1,""9)


(b) 10

(e) 2c

(b) (0, -S)

+ 6 (d)

'4

EXERCISES 14(c)

1. (a) 1 (b) 4 (e) 1 (d) ~oes not exist 2. 2 when a 3 3. 1 when a = 4


4. L when a = -1 5. 1 when a 3 6. 0 when a = 3
EXERCISES 14(d)

1. (a) 12x - 5
(d) 3x2 - Sx + 3
(g)6x+2
0) 27x2 - 6x + 3

(b) 4x + 7
(e) 6x 2 - ISx

10
(h) 6x2 - 14x - 15

(e) 9x2 - 4x - 8
(f) 4x 3
4x2 - 2x (i) 6Ox2 - lOx - 28

2. (a) lOx(x 2

(b)2~

(e) 12(x2 - l)(x 3

4)4

(e) !(2x

x-I
(g) .../x2 - 2x

3. (a)

~.
'\IX- -

(d) 4x(x2

25)

(h) -4t(t 2

(b) - -

3) (x 2

3xt2/3

(f) 4(4x

5)(2x2

4t3
-2

(e) (1

1)

+ 2X)2

(e) 1O(2x

(f) !(x

x + 1

.../x2 + 2x

43
(b) (4x + 5)2

(e) (x

2X2 - 2x - 11
(e)
(2x
1)2

(f) (x _

1)-2/3

(g) 8(3x - 1)(3x2 - 2x - 1)3 (h)

4. (a) (x

-(2x - 5)
(d) (x2 _ 5x + 6)2
.8x 2
(g) (2x

+ 40x
+ 5)2

5. (a) 3x2 + 4x - 4

2)2

44x

(h) (x 2

(b) lOx 3/2

5)2
3X ll2

3X- 1/2

(e) :::-;:::.~==;.

12

3X)3
5x -.4)3

504 NEW SENIOR MATHS: TWO UNIT COURSE

(d) ~X-113

(g) 2x -'5

6(2x

6. (a) 3(x

(g) (6x

-9

(c)

9)(x - 1)5

(e) (x

(h) 2x

(f)

---;:;:=::========i

x2

-2x

+ --

2)

.
- (2x - 2)
(I) (x2 - 2x + 2)2

(h) (2x _ 1)2

+ X-I)

.J (x 2 +

20
(2x

7. (a) 3x 2
(d)

(f)

2)2

(d) 2(1 - x-2)(x

(j )

(e) (x + 5)2

2 x - 2
12x - 7

(j) 3x2

1)2

3)2

-;::;;::==~

~(8x
3

(g) (2x

-2

(b)

x4 - 2x

(c) !1(5x

X4

5)(4x2 - 5x + Itl/3(e) !(2x

.3

7)112

5)(x 2 ,+ 5xf2/3

(h) 2~

7)2

CHAPTER 15
EXERCISES 15(a)

2. I(x) = 2x
2 5. (a) x < 2l, x
2l, x > 2i 6. x = 2; positive, negative, maximum
-Il
8. 3k
9. 2
7. (a) x < -1~ (b) x > -ll (c) x
10. (a) x
2, 3
(b) x < 2 or x > 3
(c) 2 < x < 3

11. (a) x = -}, 1 (b) x < -} or x > 1 (c) -}


12. (0, 0) max.; (4, -32) min.
14. (},

;~) min.; (1,

<E;;

x :E; ,I; ( -}, 117) max.; (1,0) min.

13. (1, 0) min.; <-2, 27) max.; (0, 7)

2) max.; (2, 0), (0, 2)

15. (a) ~, 2 (b) x <

~ or x >

(c)

~<

< 2; range: -81

<E;;

I(x)

:E;

27; greatest 27,

least -81
16. (a) x <

-~ or x >

1 (b)

-~ < x <

1 (c) x =

-~; (1, -9) min; (-~, ~~) max.

EXERCISES 15(b)

1. ~!,

- ;~)

min.; inflexions (0, 0), (1, -1).

2. (0, 0) min.; (2, 4) max.; (1, 2) inflexion

4. a = 4, b
-12, c = 9, d = -1
3. (0, Ik) min.; (2, 3k) max.; (-2, 3k) max.
5. (0, 2) max.; (4, -30) min.; (2, -14) inflexion
6. (0, 0) min.; (3, 351) max.; (8, -1024) min.; (11, 161 17) inflexion, (6, -432) inflexion
7 . .(0, 100) max.; ( JV,

-HOi) min.; ( J. -I6~:) inflexions; 2, 5

8. (0, 0) and (9, -27); (9, -27) min for 3y


9y = 2x3 - 33x2

9. (0, 0) max;

= x2

18x, (9, -27) min; (2, 11~) max for

I08x

( ~, - ~) min; ( ~, - i6) inflexions

10. (1, -1) inflexion

11. least value

-z.!, greatest value 10

12. (-1, 7) max; (2, -ZO), min.

-zo

<E;;

<E;;

ANSWERS 505

EXERCISES 15(c)

1.
2.
3.
4.
5.
6.
7.
8.

asymptotes:
asymptotes:
asymptotes:
asymptotes:
asymptotes:
asymptotes:
asymptotes:
asymptotes:

-2 and y = 0; range: all y except y = 0


Ii and y = 0; range: all y except y = 0
5 and y = 0; range: all y except y = 0
o and y = range: all y except -4 < y
o and y = x; range: all y
o and y = 2; range: all y except y = 2
2 and y
1; range: all y except y = 1
1 and y = x + 2; range: all y except -1

x
x
x
x
x

x;

x
x

< 4; (2, 4) min. (-2, -4) max.

<

< 7; (3, 7) min. (-1, -1) max.

o and y =

2x; range: all y except -8 < y < 8, (2, 8) min. (-2, -8) max.
7 and y = x + 3; range: all y except 8 < y < 12; (8, 12) min. (6, 8) max.
1 and y = x; range: all y except -3 < y < 5; (3, 5) min. (-1, -3) max.

9. asymptotes: x
10. asymptotes: x
11. asymptotes: x

and y

12. asymptotes: x

o and y
o and y

13. asymptotes: x
14. asymptotes: x

2x - 1; range: all y except -8 < y < 4; (1, 4) min. (-2,

~8)

max.

== ~; range: all y except -1 < y < 1; (2, 1) min. (-2, -1) max.

Ixl; range:

all y; (1, 2) min.

EXERCISES 15(d)

1. 1600 m 2

2.. 12

=6

4. ~ m, 39 m

3. 50 m 2 ; 5 m, 10 m

:kl + x - x 2 ) (b) ~
7. 59-l:r cm3
8. 16 cm3
2
8
2x) cm
(b) x 2(9 - 2x)
(c) x = 3

6. (a) A
10. (a) (9
11. (a) 2x,

3x

13

cm 2

.J 100 -

X2, loox - x 3 , 2000..J3/9 cm 3

(b) 1I"r(1O - r 2)

16. (a) 10 - r2

17. Rectangle:

3~ cm, 1O~ cm;

21. (a)

(c) X2

(b) V

9. 296 287 cm3

no

2~;a3
11"

144x - 48x2 + 4x3

14. 2..J3 cm

~3I

(25 - 2X)2

11"

23. (a) 0 <x< 6

(c)

5
square: 5 14
cm 18.

(b) (25 - 2X)2


11"

5. 300 m 2

(c) 144.J2 cm 3

(b) 72x - iX3

12. 75 m, 150 m
15. 28k

x; x

(c) X

19. 10 cm, 5 cm,

~ cm

j.

22. 32..J3 sq units


9

= 2, V

128

(e) 64

(f) 20

25. 30 m, 60 m

24.6,6

EXERCISES 15(e)

4. 4

3. (a) $17000

2. (1, 1)

1.20

5. row direct to destination

(b). lOv 2

7.

9. 2.J2,.J2
10. d(t) = 10"'200 .:..14Ot
11. 11
12. 50.J2
13. r = 2, h = 4

Vfi

160000 (c) 20 km/h; $12000


v

8. a = 9, b

4; x

+ 25t2; 2'8h
14. 5(.J5 + 1), 2(2.J5 + 1)

EXERCISES 15(f)

1. y

3. y
5. y
7. x

= 4x
3x; y
4x

+y

4; x

+
x

4y - 18
4. y

2. y

=x -

3; y

3 - x

+ 3y + 12 = 0
4y + x + 4 = 0; 2y = 8x +
= lOx - 20; x + lOy = 2

3x - 4; x

6.
7; x + 4y = 6
8. y
3; y = x + 3

15

2
3

(d) $12560

COUJ~.SE

506 NEW SENIOR MATHS: TWO UNIT

9. (4, 6), (-2, 0);. y


4x
10, y = -2x - 4; (1, -6)
Y == 4x; 12x + 3y = 16
11. Y = -1; x = 1 .
Y == 2x - 3
13. 80 32', 104 2'
(a) (-1, 0), (3, 4) (b) y = 5x - 11, y + 3x + 3 = 0 (c) (1, -6)
(a) y = 3 (b) .y = 2x
16. y = 3x - 7, 3y + x +1 = 0

10.
12.
14.
15.

=x

17. (a) 2y

(-~,~) (c)

+ 5 (b)

45

18. 116 34', 82 53', 33 41'

19. y + 2x = 2, y
6x + 6, y = 3x - 6
20. (2, -12)
21. (a) (2, 4), (-1, -5) (b) (0, 0), (1, -1)
22. y = 14(x - 2)

EXERCISES 15(g)

1.

(i)

~X3

~X2 +

(iii) Ix3 - x

(v)

+ e

~X2 - 5x + e

(iii) f(x)

(i) y

(ix) y

(vi) y

+e

41

e
1

TIfs - )"x4 + ex + d

(b) F(x)

/5X3

16x4

+ e

(c) F(x)

(e) F(x)

= ~X7/3

- ~X31Z
3

+e

(f) F(x)

6. f(x)

I + 1.- +
~

~+e

~XSI2 +

5
31 2

+e

+I +2

3
41

- iX2 + X + 6
Ix3
3

2X2

7. Y

+ 4x - 1
3

12. y = iX3 + ~X2 - 2x + 4

3
-x4
4

= -2X-IIZ

+e

~X-3/2 + e
3

2X2 - 6x + 8
10. y = x 2 - 4x
13. y

= x2

~+ e

5x + 12

(iii) ;!x2
2x - _1_ + e (iv) - 2x12 + e
x2
2~
(ii) f(x)
x 3 + 4X1l2 + 3x + e

e (ii) x -

= 4)"X3

9. F(x)

- x2 + X + 2

(iii) f(x) =

16. s

= 3X312 + 4.x4/3 + e

= iX3

+ Ix2 - 12x + e

3'

(0 f(x) = x

15.

= ~X31Z +

(viii) y =

432
ex + d

x 2 - 2x

. 14. (i) 3x -

lx4 + Ix3 + e

- x3 + e

1x4 + ~X3 + e
3

8. y = x 3 - x 2 + 3x - 24
11. f(x)

(iv) y = Ix3

12

~.x4

~X3 + ;ix2 - 2x + e

1x4 + e

(d)F(x) = !xs - x

5. f(x)

(ii) y =

1x4 - Ix3 + Ix2 + ex + d

4. (a) F(x)

(g) F(x)

Ixs
5

+e

(vi) J(x) = tx3 + 3x2 + 9x + e

+ Ix2 + e

+ x2 - x3 + e

3x
5

(vii) y =

(iv) f(x) =

(iii) y = Ixs (v) y

(ii) f(x) = 5x

+ ;ix2 + e

Ix3
3

(v) f(x) = x4 - 2x3

3.

234
(iv) Ix3 - ;ix2 + 2x + e
32'
(vi) ~X3 - lx2 + 5x + e
3
2

~X3 + 2X2 + X + e

(i) f(x) =

2.

Oi) 3x + ~X2 + Ix3 - ;ix4 + e

+e

4x

(iv) f(x)

+ ~X312 + 3x + e

17. (a) 12 (b) 432

18. (a) 2! (b) ~ or 4

-~--------

--

--~\

ANSWERS 507

CHAPTER 16
EXERCISES 16(a)

1. -4
6. 61j
2
11.3"

3. -wj
8.0
8a
13.
+ 2b

2. 27i
7. 5
12.a 3

b3

5. 6j
10.0

4. 0
9. 18j
a3
c 14. 3 - a2

15. 5t

17. 384

1
18. 12

19. 1 152

20. -4

21. 22g

22. 55i sq units

23. 9 sq units

24. 4i sq units

25. /2 sq units

26. 3 sq units

27. 14274 sq units

28.2

29. 1

30.

32. 6! sq units

3
33.4"

16

.li

64

31.

~ Sq units

1~2 sq units

34. (a)

1
20

5
(c) 12 sq units

(b) It sq units

"

36. It sq units

35. 85t sq units


EXERCISES 16(b)

1. It sq units
2. 21t sq units . 3. 8 sq units
4. loj sq units
5. 6i sq units
6. (a) It sq units (b) 21t sq units. (c) Ii sq units 7. (a) It sq units (b) 21t sq units (c) loj sq units
8. (a) y = x (b) (i) 4i sq units (ii) loj sq units
9. y = 4x - 3 (a) 2j (b) 1U
6
10. 16;2 sq units, 13 (2 12. (a) 20y

=x +

480

.J2) sq units

11. 121 : 4

(c) y

(b) 16 m

x2
= 200

(d) 466i m 2

EXERCISES 16(c)

2437[".
1 -5umts

2 2567[".
. -3- umts

1~57[" units

6. (a)

(b)

10. (a) 259-517[" units

3~7[" units

(b) 4(}1;217[" units

1287["
.
17 ()
a -3- umts

~;

7.

units

(b) 647[".

~;

units

(b) 2

18

-3- umts

21. (a)

4 36
.
.
7[" umts

(b)

7["
.
. 30 umts

8. (a) 187[" units (b) 367[" units

11 3707[" em 3 6357[" em 3
. 3
'12

14. ()
.
a 64...fi7["
-1-5- umts

13 97[".
2 umts

20.

i units

3 327[".
. -3- umts

1t

.
uruts

641t

~ruts

i units

12 57["a units
. 24

15 . 2567[".
-3- umts
..

19

22. (a)

9. 487[" units

16. 87["
.
15 umts

(11
1\
7[" \: - ~p 7["

1~7[" units

(b)

3~7[" units

EXERCISES 16(d)

2. 1495

1. 1521

3. (a) 0775

(b) 0783

4. (a) 375

(a) 2167

51 :

I :

I : 1 :

(b) 225

(b) 375

508 NEW SENIOR MATHS: TWO UNIT COURSE

6. (a) 2-708

2
7. (a) 3" (b) 0-638; 4'3070

(b) 2-8125

9. 0'6933

8. (a) 3-219 (b) 3'240

10. -0'15551

(e) 0'6035; 9.5%

11. 22'37

7r

12. 3i5

CHAPTER 17
EXERCISES 17(a)

1f'
1. (a) 6
(e) 71f'

2. (a)
(e)
3. (a)
(e)
4. (a)
(e)
5. (a)
(e)

4
135
100
28 39'
155 51'
0'7330
0'8404
34 23'
246 22'

(b) 51f'
4

(e) 21f'

(f) 51f'

(g)

(b)
(f)

(b)
(f)

(b)
(f)

(b)
(f)

(d) 71f'
6

8
157 30'
165
104 17'
196 18'
1'2915
3-8397
24 4'
343 46'

(e)
(g)
(e)
(g)
(e)
(g)
(e)
(g)

lli

(h) 41f'

(d) 270
(h) 247 30'
(d) 244 5'
(h) 273 59'
(d) 2'8623
(h) 1-2590
(d) 98 33'
(h) 309 24'

6
216
324
171 53'
300 14'
1-8326
2-4120
114 35'
163 52'

EXERCISES 17(b)

1. 107 26'

2. 18-33 em

5. 30'58 em

6. 22-5 em

8. (a) 33-51 em

12. (a)

3. 2~1f' em
7. (a) 106 16'

(b) 29-72 em

em 2 (b) (2!1f'

4. (a) 68 45 (b) 24 em

(b) 18'55 em (e) 44-73 em 2

10.

9. (51f' - 15) em

13. 61f' em 2

10)em

(e) 22-58 em

em 2

11. 9 em

14. 21f' - 3..J3 : 101f'

3..J3

15. 1011 m 2
16. 27-9 em
17. (a) 63 m (b) 163'6 m 2
18. (a) 17-45 em (b) 17-32 em
19. 32-69 em 2
20. 4-025 cm 2
EXERCISES 17(c)

x
(iii) -tan x
(b) negative (e) negative
-0-2
(iii) -0'2
(iv) 0'2

1. (i) sin x
2. (a) positive
(ii)
3. (i) 0'2
1
4. (a) (b)
t

(ii) sin

(e) -I

5. (i) !

(ii)

(iii)

7. (a) 1

(b)

-2

8. (a) 0

(b)

9. (a) -1

(b) -..J3

10. (a) 0
11 41f' 51f'
. 3' 3

15.

1f' 71f'

6'"6

(b)

1
..J3

:../3
2

-c

(d) -I
(iv)

(v) -sin x
(iv) -eos x
(d) negative (e) positive
(v) -0-2
(vi) 5

(e) - -

(f) 1

(vi)

(v) -

-..J2

-c

(e) - -

(d) -1

(e)

(e) 1

1
(d) ..J3

(e) - -

(f)

(e) -2

(d) -1

(e) -1

(f) ..J3

1
2

12 31r 71f'
4' 4

16 31f' 51f'
4' 4

17. ~,

(e) 1

(d)

1-

..J3

(f) -..J3

(g) 1

-..J2

(g) 0

2
(e) ..J3
13 :! 51f'
6' 6

(vi) tan x
(f) negative

(f) 0

(g) (g) ..J3

(h) ..J3
2
1
(h)
2
2
(h) -..J3
(h) -1

14. 1f'
18. 0, 1f', 21f'

ANSWERS 509

211" 411"
19. 3' 3
23. (a) sin x

20.

i,

511"

21. 3

(b) cot x

(c) -cos

22.

11"

4'

(d) -tan x

EXERCISES 17(d)

1.

(i) period 211", amplitude 4


(iii) period 411", amplitude 1

(8) period 11", amplitude 3


(iv) period 11", amplitude 5

(v) period~,

(vi) period 2, amplitude 1

EXERCISES 17(e)

1. 1-24
5. 1'659; 15-86 cm 2
9.2'84

2.0-74
6. 2-31
10. 2'5

3. 1-36
7. 0-739

4.0-48
8.0'98

EXERCISES 17(f)

1. (a) -cosec 2 x
2. 3 cos 3x
5. -2 sin x
8. cos 2 X - sin 2 x
11. cos

(b) sin x sec 2 x


3. 3 cos x
6. cos x - 4 sin x
9. 2 cos 2x + 2 sin 2x

(x + f)

14. 2x cos x - x 2 sin x

17. 2x 3 cos 2x
20. -2x sin (x 2

+
+

12. 2x sec 2 x

15. sin x :-

cos x

18. 3x2 cos (x 3 )

1)

21. 2 cos (2X

37. -sin 2 x + (1
39. cos 2x
,""sin 2x
41.0

22. 2x

24. -2 sin (2X

26. -4 cos 2x sin2x


29. -3x2 sin (x 3 )

~ cos! x

34.

27. 6 sin 3x cos 3x


30. 2 cos 2 2x
2 sin 2 2x

~ cos !x + 6 sin ~x

3x2

38. -sin 2x + x cos x + sinx

cos x) cos x

44. -cos x sin (sin x)

i)

1
1. x
36 . cos 2:x - 2:x sm 2:

1)

+ ~ sec 2 !x

32. -4 sin x cos x

x
-

+ ~)

33. x cos x ~ 2 sin x


35. 2x sec 2 (x 2

16. x cos x ;; sin x

19. -3 cos 2 x sin x

1) sin 3x

25. -3 sin (3x - 2)


28. -3 cos 2 x sin x
31. 3 - 2 sin x -

2 tan x

(c) -cos x cosec 2 x


-2 sin 2x
2sec 2 2x
sin x + x cos x

2:1 cos 2:1x 2:I_

sm x

~n2x

3x2 sin 2x

23. 2x cos 3x - 3(x 2

4.
7.
10.
13

40. 2(sin x -: cos x) (sin x

45.

cos x)

43. -sin x cos (cos x)

42. 4 sin x cos x


-sec 2

46 . 180
11" sec 2 x 0

EXERCISES 17(g)

1. -

2:1 cos 2x + c

4. -cos x + sin x + c
7. } sin 3x

2. 2 sin!x

5. -2 cos x - 3 sin x
8. -cos 2x

3. tan x

+c

6. -cos ( x
9. 9 sin}x

+
+

f)
c

510 NEW SENIOR MATHS: TWO UNIT COURSE

10. -4 cos

13.

~x + 4 sin ~x +

~) + c

ksin (2X

16. 2 sin kx
19. 2

+ ~ cos 2x + c

20..,fj

k~nits2

44.

11" 511", '"


8'
S' ,,2

ksin 2x + c

14. t tan 3x

+c

kcos 2x + c

17. txl 22. 1

43. (8) 11"

(b)

45. ~

23. 2

29. -~8

28.4

36. (4.,fj6- 11")11"

35. 11"

42.

ll. -cos x -

21. -.,fj

27.0

34.2

24. 0

JO.O

37. 11"

~ + 1)

-41 cos 2x

18.

~Xl/2

(d)

33. ~

.,fj

11"

9.,fj
8

39.

.J2 + !!

ksin x + c

.J2 ~

32

31.0

+c

sin x

1
4

25.

38. L
(c)

15.

40. 211"

41.

ab
11"

11" 511" "'2 . 2


46. 4'
4; ,,~ umts

11"

EXERCISES 17(h)

1. (8) 3-1
2. (i) (8)

3.

(b) 3'13

(b) 2311"; exact answer 2; 070 error in (b) is 4-7

1
12

11"
2

311"
4

11"

0'369

0-5

0-773

!!

0'333

+ cos x

4. (i) ~ (ii) ~

(ij) (8)

(b) 2311"

; 1-807

5. 6'58; 4'7070

!!

!!

311"
4

11"

x sin x

0-555

1-571

1-665

7. (8) 1 (b) 1'049 (c) 1-006

(a) 3-147

(b) 2'977

8. 3-575

CHAPTER 18
EXERCISES 18(a)

1. (a) 4e 4x
(d) 4e 4x - 3e lx
(g) 3eX + 2e-x
2. (8) (3xl + 3x2)e lx
(d) (1 - 2x)e-2x
(g),: ( 2 cos

i - ksin i)e2X .

0) (cos 4x - 4 sin 4x)eX

(b)
(e)
(h)
(b)
(e)

ex / 2
6e lx - e-X
8e 2x - e-2x
(1 - 2x)e-X
(3x 2 - xl)e-x

(c)
(0
(i)
(c)
(0

12e lx -+- 2e-2x


3'2e Nx - 1'6e)'6x
3'e lx - eX
(2x 2 + 4x + 3)e 2x
(sin x + cos x)e X

(h) (3 cos 3x - sin 3x)e-X

(i) 3x2 -(2x

(k) 2x - (3x 2 - xl)e-x

(I) -2 sin xe-X

l)e 2X

11"

ANSWERS 511

3. (a) 2e2x + 3
(d) 6e 3x -

(g) (1 - 2x)e-x

2..JX

4.
7.
10.
12.

(b) (2x - 2)e X"-2x


(e) 3e 3x - 1 + 4e4x + 2

(c) "-9x2 e-x3


(f) 2 cos 2x esin 2x

(h) -sin xe COS x

(i) 12xe2x2

(j) (1 + 2x2)eX2
(k) 6e2x - 1
(I) (cos x - sin x)esinx+cosx
(0 2e2x + 8e 8x
(ii) 4e2x + 64e 8x
6. y = x + 1
y = 1 - x
8. -e
9. (2, e- 1) max;
(i) x > 0 (ii) x < 2
(a) 2; 1 (c) 3
11. (a) -2xe-x2 (b) (i) x = 0 (ii) x < 0 (iii) x > 0
max 0-1003 when t = 0-909; from t
0-27 to t = 2-16

13. y

e2(2x - 1);

15. Y = 3 - x; y = 3
17. (a) 1, 0, -e- 7r

1
19. (a) 2e2X

(ii) y

+x

(b) -(sin x

(c) 1 ~

16. (b) 2-5


cos x)e-X

1ft) < 2-5

18. (a)

+c

+c

(e) eX - -e-3x
3

+c

(f) -

(b) 2e l12x

(e) !e 2U

+c

(b) ..J2e-1I2

(c) -2-5 e-o' 4x

(g) eO' 5x _ e-0' 5x


20. (a) -e-X + c

21. (a) e - e- 1 (b) 2!<e4 - 1) (c) 2(eO' 5


(g)

-e2

+c

(d) 2e2-5x

(d) e-t

=1

(i) x

~21 e2 + e-2 - 2) (h) 0

(i) e 1' S

e+ 5)

le-2x + e-X + c
2

(c) - le-3x

+c
+ 2u + c

+c

+c
+ 2-5eo' 4x + c

3
(f) -0-4e-2' 5x

~el'5

- 1) (e)!<e - e-1) (f) !<e 3


3
2
3
4
5
5
O
5
2
2
le- ' - e ' + !e+ 0) e - e- - 4
3
3
(d)

1)

23. (a) e2 - 1 (b) y = e2(x - 1) (c) !e 2 - 1 (d) e2 + 1


25. 2eO' 5

2e-o' 5 - 4

29. 1/"(e2

4 - e-2)

26. ~(1 - e-2)

30. }(e'

e- 1)

1-029

::=

28. 1/"(e2 - e-2)

31. 51-80

32. f(1

35. (a) 1 - e-a (b) 1 (c) ~(1

34. (a) xex; 2e- 1

33. 0-188

27. 1/"(e3 - e)

2e)

::=

6-74

e-2a ); ~

EXERCISES 18(b)

1. (i)

... ) 2
(m

") -2
(II

2. (i) x > -

3. (i) 1

2_

3' 3x +

Inx

e2x
(ix)

4. (a)
(f)

3x2 lnx

(vi) tan x

2e 2x In 2x

2x - 2

(x)

(b) -tan x
x 2 - 2X
2
2x
sec x
(g)x2 -1
tan x

(k) lelOgex

(I) 2x

(ii) all x; x 2 2~ 1

(ii) x 2

(v) 2x - 5 + 21nx

(iv) 2x

(iii) 1

+ sec 2 x Inx

.log ex - 1
(loge x )2

._-) 11
2
2x -4
(m
a x, except x = ; x2 _ 4x + 4

+ 2x loge (X2)

(iv)

2)

x
x

(vii) cos

Xl

X2

(e) -

x-3

(i) -

1 sin (log eX)


x

(m) x log eX

sin x In 2x

( .) 1 - log ex

(c) 1
(h)

In(x

2x In 2x

(viii) eX

eX Inx

(xii) _-_x--,,=-

xe X

x+

0) ! cos (log eX)


x

1
2x

(0)

,=;=~~"=,""""'="~====~=~--~=---~---

512 NEW SENIOR


UNIT COURSE
. MATHS: TWO
....

5. 0'6

6. (a)

(b) -

8. 0 < x < 'li"; 0

;2

(c)! (d)

9. 2y = x

7. Y = x-I; Y = 1 - x
11. x > 1; y = x - 2

10. 1

'

12. (a) 2 loge x + c (b) loge(x + 1) + c (c) loge(2x + 1) + c (d) 2loge(x2 - 4) + c

(e)

loge(2x - 1) + c (f) loge(l - cos x) + c (g) loge (sin x) + c (h) loge(4 + eX) + c

x
13. y = loge2

14. y

loge(2x

15. f(x)

1)

1
= 2loge(x2
+

9)

16. (a) loge2 (b) 2 loge 2 (C)! logeS (d) logeS


1
7
11
(e) loge3 (f) 3 loge 2 ' S (g) 2 loge 3 (h) 3 loge 3"

17. (a) 8j

loge3

1
18. 2 logeS

(c) 6 - loge 2

(b) 0

1
19. 21oge3'4

20. 2

21. Turning point (1, 2) min; asymptotes y


23.

loge(cos x) = -tan x;

1
24. 2(1

27. 'li" loge


30. 'li" loge

e)

tan x dx

4 loge 3

x and x = 0; I'S + loge 2

-loge(cos x) + c; loge 2

1
25. 2 loge 2

26. 6'li" loge 3

31. 4'li" loge3

32. 'li"(1 S

4'

33. (a) }(8 loge2

+ 3 loge3) "'" 2'947 (b) 2 loge2 +

34. (a) 4'041

I! loge3

IOge~)

== 3'034

(b) 3'807

CHAPTER 19
EXERCISES 19(a)
12'S cm 3 /min.
(b) V = 1000 - 12'SI,0 :ti;; 1 :ti;; 80
0 :ti;; 1 ~ 100
(b) 6 m 3 /min
(c) no
(d) 10 m 3 /min
(e) SO min
(b) 20 min
(c) 10 min
(d) 3S000 litres/min
2000(20 - I) litres/min
1'7So Imin
(b) 2'4 Imin
5. (a) (i) 1 item/min (ii) 21 items/min
(b) 110
dV
dV
2
6. 4'li" cm /cm
7. (a) dh = 'li"r2 (b) dr = 2'li"rh
8.-S

1.
2.
3.
4.

(a)
(a)
(a)
(a)

EXERCISES 19(b)
2.N = looe-{)'5t
3. Q = SOeO' 4t
4. y = 20e-3t
5. M = lOe-{)'Olt
6. A
1000, k = O'S
7. A = 200, k = 0'4
8. A = 76, k = 0'2
1. y

= Se 2x

9. P

loo000eo,o2t

10. (a)

~~

= 0'23 N

(b) 8 x 104 (c) 6

(b) (i) SI'2SoC (il) 14'8 min


12. 4'2S
11. (a) 80
13. (a) 16490
(b) 13'86 min
(c) (i) 824/min (ii) l000/min
14. (b)
16~

~" (c)

(i) 0'032 Vom 3 /min (ii) 0'043 Vom 3 /min

(a) 5000 (b) 2 (c) 10000 units/s

17. (a) N

15. (a) SO (b) 0-1

l000eO' 15t

(b) 4'6; 300/h

(c) 2 years

ANSWERS 513

18. (a) 0-004

(b) 173 m

22. 16000

19. (a) -0-02

23. 70 days

= Me- kt ; 6'9

20. m

(b) 35

21. 10

24. 59%

EXERCISES 19(c)
(b) -5 mls

1. (a) 6 m from 0
(e) After 2! s,

-~ m

(c) After 2 s, -1 mls

(d) After 3 s, 1 m/s

from 0

m/s 2

2. (a) 2! s
(b) 2
3. (a) V 6t 2 - 30t

(c) 8i m
(d) 6i s
a = 12t - 30 (b) 36 mis, -30 m/s 2 (c) 2 s, 3 s
(d) 2! s, -Ii mis, 27! m from 0 (e) 2 < t < 3
4. (a) I! s, -I! m/s (b) 1 or 2 s, -6 or 6 m/s 2
5. (a) 2 s (b) 0-5 s

t3

6. (a) x = 3"

7. (a) s

= 2t 3 +

8. (a) v

9. (a) v
10. (a) v
11. (a) x

11

+ 36,

t4

(b) a = 2t - 12t2

3t2 - 12t + 7, a = 12t + 6 (b) 18 m/s 2 (c) -12 mis, 6 m/s2


lOt - t 2, s = Ilt + 5t2 - ~
(b) 11 s, 282t m

= 30 - lOt (b) h =
= 25 - lOt; h = 25t = t 3 - t 2 t + 1, a =

3t2 + t + 2
14. (a) A: 50 km/h B: 20 km/h

12. s(t) = 4t 3

15. (a) 1-5 s, -1-5 cm/s

30t
5t2
(c) 45 m
(d) 6 s
5t 2
(b) 3O;t m
(c) I! s
6t - 2 (b) 1 s, 0, 4 m/s 2 (c) 4 m
13. 12 s, 432 m from 0
(b) 9 min (c) 13_2 km (d) x = I!2t

(b) 1 or 2 s, -6 or 6

cm/s 2

16. -3

EXERCISE~J91d)

1. (a)

2. (b) 0,

-24 sin 4t,

-96 cos 4t

i, 1f, 31f,2 21f

5. x
7. (a)

(c)

(b) (i) -40e- t

5~1f cm/s

(c) -3

(d) (j) -

4 sin

(d) (i) 8 sin 2t (ij) 4(2

(b) 2 (c) -1
(b)

:!
6

(d) t

+
(c) 49

it

10. (a) x

12. (a) x

(d) 155 units

(ii) -

~\

(c)-I
8. (a) 0, 2, -2

x)

(d) t

(d) 218 km

+ t)

13. (a) 24-5 e -o' 5t

(ii) 60 - v

-2 sin t + 3t + 1
6. (a) x = loge(l
1 + 2 sin t; = 2 cos t (b) 0, 1, 2

9. (c) ~, 31f, 51f, 741f


4 4
11. (a)

(c) (i) 4 cos 2t (ij) 4(4 - x)

3. (a) 100 km/h, 62 km/h


4. (b) 1 s, 3 s

(b) x

=2

-2 cos t

(c) t = loge2
- e-t

+t +

(b) t

loge O-6

6 sin 2t, x = 9 - 3 cos 2t; 6 times

14. v

REVISION EXERCISES C
1. (a) (i) 2 (ii) 27
(b) (i) y + 2x = 9
2. (a) (i) 4x3 (ii) 2x- 1 (iii) 3x2 - 2
3. (a) (i) 14m/s2 (ij) 275 m

(b) (i)

(ii) Ioj units 2

-! <

(ij) -26j

9. 2! s, 22~i m from 0

(iii) 48

8. (a) y
10 .

15(16

+ -/3)

11

(iii) 1820

x < 2! (ij) x >2!orx <

4. (a) (i) 1, -2 (ij) x < -2 or x > 1 (iii) -2 < x


7. (a) (i) -2x (ij) 3x2 - 1 (iii) 24x2 - 72x + 54
(b) (i)

(ii) 34,25

(b) (i) -20

< 1

(b) 24 units 2
(iv) 162x2 + 72x + 6

= 4x
cm,

-!

- 4;

45(5 22

(b) 36

-/3)

em

(iii) 2!

(iv)

5. 14 cm, 7 cm

514 NEW SENIOR MATHS: TWO UNIT COURSE

11. (a) (i)

2 (ii) f(x)

(iii) 0

(iv) c

(b) (i) (0, 0) max, (~, - ];;) min;

0 or 3

(ii) (-1, 5) max,(3, -27) min

12. 3:~ m,
m 13. (a) (i) x = -Z or 2 (ii) -2 < x < 2 (b) max 8t when x = -2; min -zt when x
14. (a) 42~ units 2
(b) y + 32x = 128; x - 32y = 4
15. (a) x == 2t 3 - 2t2 + t - 10; 0 = 12t - 4
(b) 8 m/s 2
(c) 17 m/s
4 14,

16. (a) l00C/h

(b) 60C/h; T

17 (0 , 0) max,. (10
9'

5(0)..
- 243
mm, y

289 um't s2
= 2x,. 63 .26 "'192

20

3iOt

31'\

19. (a) x < -2

(b) 0 < x < 2,,2; 8

21..(a) y

x 2)

2(1

~x

(c) 2i;

23. .(i) (3
7x - _ = x2)e-3x~ (ii) -2e- (3 sin 3x
(iii) (x - 2)(x2 - 4x + 1)-112
(iv) 2 cot 2x

= 3"2

24. 0-4905 when x

0'5

0'472

0'446

0'4x2

27. (a)

6i units 2

(b) 1 ~l units 2

31. 0)

~71" units 3

(ii) 32i7l"

32. (a)

(2x

33. (a) 0

+ 2)

sin (x2

(b)

(i)

3iv

; 0'389

5 - ..fj
26. -3-cm

7iY

Oi)

(c) IC/min.

28. (b) 0'04

(b) 3-254

22. (a) -2-5


cos 3x)

25. 5'4

0'5x; 134 - 3x

18 5000 m

35
1
x3
= 24
- 4x + x + "3

20. f(x)

(b) Y= 2 cos

4
.71"

29.

30. (b) 0

(iii) 4071"

(b) 2x + 2
x 2 +2x

2x)

-J x 2 +

(d) (x 2

1
2x

39.(a)0)(ii) 4 - x. J = .../9

20 '

e- I12 ) == 0'7868

40. (a) 2(1

42. (a) y

1x2
100 .

4x

2)eX

+ e-2) units 2

34. (3

(c)

(b) 2i units 2
(c) 45
35. (a) 0 < x < 2 or x < -2
36. (2i, 8H~) max; (0, 0) inflexion; (l!, 5-ft) inflexion
37. (a) 10000
(b) $3012
(c) 0) $602'4 per year (ii) $1000 per year
38'@.43 u nits 2

(b) 0'7910

+x +4 -

1O-v'2

(c) 0'7870

(b) 853~ cm 2 ; 25600 em 3

x 0

20 '

(d) 11! years

~x ~4

(b) x = 3,' 0'35 hours

41. (a) 9, 6, 0, -3, 0, 6, 9


43.60kmlh

TEST PAPERS
TEST PAPER 1
1. (i) (a) 25

(b) 1 (c) 3

2. (i) x < 2 or x > 6


3. (i) (a) (0
.~. (i) (a)

(a)
./

+ 1)(02 -

L FEB
2

(ii)

(ii) (a)

I.

V5 + ,J3

H=t
x-I

(b)

(iii) 1 or

(b) cot x

1) (b) (x - 2)(x - 5)

L DEG, L FBE
1
2

= L EDG

(c) -1

-~

(iv) (a) loge4 (b),J3 2 1

(c) cos x - x sin x

(ii) -1'2

(iii) (a)

(b) FB = BE = FE

(iii) 2x

DG

+ 3y =

ED

13

EG

(iii) y

V5
3

(b)

+ 2x

_.
V5

ANSWERS 515

5. (i) ;0

(b) 29
(c) ~
(ii) ~
30
amplitude 5
(ii) (a) (0,0) min, (4, 32) max

(a)

6. (i) period

7~

(i) (a)

11",

~11"

~11"

(b)

~ 1)2

8. (i) (a) (1

(ii) (a) y = lOelx

(b) - (l

9. (i) (a) 1, (b) 3 (c) -5

~ 1)3
(ii) 1

(b) -115

(ii) X

or "2

(c) 108 units 2

(b) 0 ~ f(x) ~ 32

= 6l

(iii) I, -3, "2, 2

10. (ii) $88-12

.TEST PAPER 2
1. (i) (a) 2

(b) 0 or 3

(b) -5 < x

< 5

(c) 5 or -5

(iii) (a) x

(ii) 275

< 0 or x > 3

~ 21

(iv) (a) 4a - 5b
(b) 1

12

2. (i) (a) bisect each other (b) equal to 360 (c) parallel to the third side and equal to half its length
(ii) 40 (iii) 2.Ji cm
3. (i) centre (1, -2), radius 3 units
(ii) ART WORK
(iii) (0, 3), (3, 0), (3, 6); 9 units 2
(c) x

4. (i) (a) $17680, $130000


5. (i) (a) (1

x)&'

(ii) f(x) = x 3

6. (i) 750/0

(b) $19618, $132599

sin

(b)

3x

(~gA

(iii) 8,

(c) - (x

or 24,

-!

1)2

+ 2; (1,0) min, (-1, 4) max; (0, 2), (-2, 0), (l, 0)

(ii) 05

(iii)

(iii) (a) 1193 (b) 0787

16

(ii) (a) _2_, 2 - v (b) v = ~ (c) Area = ~


1-u
1
u-1
u-1
8. (i) (a) y = x (b) x 2 + y2 = 16
(ii) (a) (0, 2) (b) Y = 2 (c) (-4, 2); x
9. (i) (a) -2 ~ m ~ 2 (b) m = 2; 1
(ii) 105m

7. (i) r

10. (i) k

1 - 210

002; 6700

(ii) (a) 2 m from 0

(b) -

(c) x

11".../2

(d) 2

+y +2 =0

TEST PAPER 3
1. (i) (a) 38

(iv) (a) (a - b)(a


2. (i) x

(ii) (aj- x

~~

(c)

(b) -17

(ii) 53 x 106

(iii) 361

2)

<!

(b) 4

~ x~

16

(iii) 116

(iv) (a) BC = BC, L BCX = L CBY, L BXC = L BYC (ASA)


(b) AO = AO, AY = AX, LAXB = LAYC(RHS)

3. (i) 10

(ii) (a) 0064

(b) 0936

(c) 0096

4. (i) (a) all real numbers (b) -1 ~ x .~ 1 (c) -1 < x


(ii) (a) (x - a)2 +y2 = r2
(b) the line x = 0
(c) 3x2

5. (i) (a)!

(b)

6. (i) (a) 193 m

(iii) (a) logi1

3y2

(c) 2

Wax

x 2)

3a 2 = 0 (circle)

(ii) (a) 90

(b) 137 m

(d) y2 =

(b) 150, 330

(ii) (a) -2 sin x cos x


(b) -2 cos x

7. (ii) x + y = 1
(iii) (-2, 2)
8. (i) 2y= 14x - 7; 7x + y + 21 = 0
9. (i) (0, 0) min, (It, 1 25 7 ) max; It units 2

< 1

(b)

+ ~ cos 2x +
(ii) -7

(d) all real numbers except 1

2a~

~)(parabOla)

(iii) (a) 12 44'

~+
c

&'logeX
(c)

~e3X +

(iii) -3 < m < 5


- 2X2 + 5x - 2

(ii) y = x 3

(c) (2x

(b) 1996 cm

3)2

(iii) 1812 units

516 NEW SENIOR MATHS: TWO UNIT COURSE

10. (i) 1 units 2, 3 7r units 3


2

(ii) (a) 79 km/h

(b) 2! h; 81'15 km/h

(c)

2! km/h 2

TEST PAPER 4
1. (i) (a)

~~ (b) (~y5

2. (i) (a) 1
3. (i) 4

.J3

(ii) (a)

(b) 4

(ii) (a) - (

4.

(i) (a) 12i

(b) 5!

5. (i) x + 9y = 14

7. (i)

(b)
(b)

te3x + c

(c)

(ii) 6'70

(iii) 32

4 loge2

(ii) (a) 1, -2

8. (i)

(ii) (a) x

9. (ii) (a) h =

+ y2

- 2x - 2y

(c) 2 cos 2x

-21 cos 2x
99

-!

(b)

(ii) 2

(ii) (a) $2545

(iii) x 2

(ii) 14
3e 3x

6. (i) all x > 1 except x = 2

n2

(b) 7 - 4.J3 (iii) (a) -cos f) (b) -tan f) (c) cos f) (d) cosec f)

(c) 1, 2

(iii) (a) loge(x - 2)

6-12

(iii) (a)

(iv) 127r cm2

(b) 96 m

(iii) (-2, 1), (2, 3); 5;7r

(b) $2083

(c) $165

> 2 or x < -I! (b) -1 <

< 5

(iii)

E;

-3 or m ~ 1

(c) 96 cm 2

4 - t2; x = 4t - tt 3 (b) 5i (c)-8

10. (i) (a) v

(ii)

N = l000eo'04t (a) 0'04 (b) 1170

TEST PAPER 5
1. (i) (a) 27

2. (i) r

(b) 50

Oi) 6

= ~,a = 2Ol; s.

~3 ~

i'

3. (i)(a) 2xy(2x

1)(4x2 - 2x

i.JIO

-nn

(iii) 7 -

1)

60i

(b) (a

(ii) 10
b)(a - b

3)

"') 50(3 - .J3)


(III

3

4. (i) (a) AB = DC, BF = ED, L ABF = L EDC


(ii) 0-51
5. (i) 3x2 + 3y2 - 12x + 14y - 5
0; centre (2, -2i); radius 3i
(ii) (a) focus (0, 1); directrix y =
1
(b) y
2x - 4
(c) 1607r
6. (~) (a) ~ 270 (b).. IOge2'~ (c) 1
(ii) 51 unit~2)' (0, 0), ~-2, 1), (2, 4)
7. (I) 2 umts

(II) (a) (2, 1), (0, 3), ( -

8. (I) (iI) (0, 4) min,


(ii) 19-2

9. (i) (a)

10. (I) e

(Jf. 21)

(iii) y = 1-8(4

(b)!

(ii) (a)

(ii) (a) 210 years

1~

max, (

2' - 2

(b) (0, 1), (2, 5)

~, 21) inflexions;

x 2)
(b) 25

(b) 60 per cent

(b) (0, 4),

(2, 0), (""2, 0)

You might also like